Вы находитесь на странице: 1из 865

BALOL

UNIVERSITY OF CAMBRIDGE INTERNATIONAL EXAMINATIONS


General Certificate of Education Ordinary Level

*8413371862*

5014/01

ENVIRONMENTAL MANAGEMENT
Paper 1

May/June 2007
2 hours 15 minutes

Candidates answer on the Question Paper.


Additional Materials:

Ruler

READ THESE INSTRUCTIONS FIRST


Write your Centre number, candidate number and name on all the work you hand in.
Write in dark blue or black pen.
You may use a soft pencil for any diagrams, graphs or rough working.
Do not use staples, paper clips, highlighters, glue or correction fluid.
DO NOT WRITE IN ANY BARCODES.
Answer all questions.
All questions in Section A carry 10 marks.
Both questions in Section B carry 40 marks.
At the end of the examination, fasten all your work securely together.
The number of marks is given in brackets [ ] at the end of each question or part question.

For Examiners Use


1
2
3
4
5
6
Total

This document consists of 23 printed pages and 1 blank page


SP (SM/CGW)
UCLES 2007

[Turn o

186

For
Examiners
Use

BALOL

Section A

(a) The photograph shows a soil formed on a rock made of sand and the diagram is a
profile of the layers of the soil from the surface vertically downwards.

scale x 0.5

(i)

Which layer, X or Y, has the most organic matter? State the evidence for this.
layer .....................
evidence ...............................................................................................................[2]

(ii)

Z is parent material which has broken down. What is the evidence for this?
..............................................................................................................................[1]

UCLES 2007

5014/01/M/J/07

187

BALOL
(b) The soil in the photograph contains a high percentage of sand and has a coarse texture.

For
Examiners
Use

It has a pH of 4. Explain:
(i)

why the soil has a coarse texture,


..............................................................................................................................[1]

(ii)

what a pH of 4 indicates,
..................................................................................................................................
..............................................................................................................................[2]

(iii)

how the texture of a soil and the size of its pore spaces are linked.
..................................................................................................................................
..............................................................................................................................[1]

(c) (i)

State one advantage and one disadvantage, for crop growth, of a soil with a coarse
texture.
advantage .................................................................................................................
disadvantage ........................................................................................................[2]

(ii)

State one disadvantage of a pH of 4.


..............................................................................................................................[1]

UCLES 2007

5014/01/M/J/07

[Turn over

188

For
Examiners
Use

4
2 BALOL
(a) The map shows some of the worlds main marine fishing grounds.
90W

90E

66N

66N

X
45N

45N

23N

23N

23S

23S

45S

45S
90W

90E

Key
fishing grounds
cold ocean current
warm ocean current

What three features of location do they all have in common?


..........................................................................................................................................
..........................................................................................................................................
......................................................................................................................................[3]
(b) Describe the conditions which favour large marine fish populations.
..........................................................................................................................................
..........................................................................................................................................
..........................................................................................................................................
..........................................................................................................................................
......................................................................................................................................[3]

UCLES 2007

5014/01/M/J/07

189

BALOL
(c) When fish stocks collapsed, in area X, measures were taken to encourage the recovery

For
Examiners
Use

fish stocks increase

of the industry. The graph below shows the timing of these measures and their effects
on the fish population.

20,000 km2 area


closed to mobile
fishing gear.

closed
areas

US 500 km limit
created in 1983.
Fishermen given
control of fisheries.

open
areas
0
1982 84

86

88

90

92

94

96

98 2000 02

04

06

Describe, and suggest reasons for, the different effects of each of the two measures.
..........................................................................................................................................
..........................................................................................................................................
..........................................................................................................................................
..........................................................................................................................................
..........................................................................................................................................
..........................................................................................................................................
......................................................................................................................................[4]

UCLES 2007

5014/01/M/J/07

[Turn over

190

6
3 BALOL
(a) (i)

The diagram shows the instruments used to measure precipitation at a weather


station.

For
Examiners
Use

funnel
metal
container

grass
ground

tapered glass
cylinder
with
scale

surface
jar

How would you take accurate measurements of the precipitation using these
instruments?
..................................................................................................................................
..................................................................................................................................
..................................................................................................................................
..............................................................................................................................[2]
(ii)

Explain why this instrument is sited as shown in the diagram to give accurate
readings of the precipitation.
..................................................................................................................................
..................................................................................................................................
..................................................................................................................................
..............................................................................................................................[2]

(iii)

Why is it sometimes more difficult to measure precipitation accurately in places


with a tundra climate?
..............................................................................................................................[1]

UCLES 2007

5014/01/M/J/07

191

For
Examiners
Use

BALOL
(b) Describe the amount, frequency and intensity of precipitation in a desert climate.
..........................................................................................................................................
..........................................................................................................................................
......................................................................................................................................[3]
(c) Explain why trickle drip irrigation is the least harmful type of irrigation to use in a desert
climate.
..........................................................................................................................................
..........................................................................................................................................
......................................................................................................................................[2]

UCLES 2007

5014/01/M/J/07

[Turn over

192

For
Examiners
Use

8
4 BALOL
(a) Look at the population pyramid of Mexico.
age group
80 +
75-79
70-74
65-69
60-64
55-59
50-54
45-49
40-44
35-39
30-34
25-29
20-24
15-19
10-14
5-9
0-4

Male

% 10
(i)

Female

What percentage of the countrys population is below five years old?


................ %

(ii)

[1]

The table shows data missing from the population pyramid.

age group

males

females

2529

3.2%

3.8%

Complete the population pyramid for the 2529 age group.


(iii)

10 %

[1]

Circle the words which describe how the countrys population is changing now.
Choose from:
decreasing slowly

decreasing rapidly

increasing slowly

increasing rapidly.

not changing
[1]

(b) Describe and explain the problems for a country with this type of population structure.
..........................................................................................................................................
..........................................................................................................................................
..........................................................................................................................................
..........................................................................................................................................
..........................................................................................................................................
..........................................................................................................................................
......................................................................................................................................[4]
UCLES 2007

5014/01/M/J/07

193

BALOL
(c) Why is it difficult for governments to introduce policies to change this population

For
Examiners
Use

structure?
..........................................................................................................................................
..........................................................................................................................................
..........................................................................................................................................
..........................................................................................................................................
......................................................................................................................................[3]

UCLES 2007

5014/01/M/J/07

[Turn over

194

For
Examiners
Use

10

BALOL

Section B

(a) Look at the cross-section.


Section from the mountains to the sea
high
mountains

A
river
A
river

B
D

sea
layer of
sandstone
rock
(i)

Name the two stores of fresh water labelled B and C on the section.
B ...............................................................................................................................
C ...........................................................................................................................[2]

(ii)

Why might it be possible for people to obtain fresh water, at point D, other than
from the river?
..................................................................................................................................
..................................................................................................................................
..................................................................................................................................

(iii)

How could people obtain this water?


..................................................................................................................................
..................................................................................................................................
..................................................................................................................................
..................................................................................................................................
[3]

UCLES 2007

5014/01/M/J/07

195

11

BALOL(iv) Choose one of the sources from AD which is likely to have water that will be safe

For
Examiners
Use

and clean for people to use. Explain your choice.


Letter ....................
Explanation ..............................................................................................................
..................................................................................................................................
..............................................................................................................................[2]
(v)

Which one of the four sources is least likely to give clean water? Explain your
choice.
Letter ....................
Explanation ..............................................................................................................
..................................................................................................................................
..................................................................................................................................
..................................................................................................................................
..............................................................................................................................[3]

(b) Dams are often built to hold back reservoirs for water supply.
(i)

Give the name and location of a dam.


..................................................................................................................................

(ii)

Explain why the dam was built and what advantages it has brought to the local
people.
..................................................................................................................................
..................................................................................................................................
..................................................................................................................................
..................................................................................................................................
..................................................................................................................................
..................................................................................................................................
..................................................................................................................................
..................................................................................................................................
..............................................................................................................................[5]

UCLES 2007

5014/01/M/J/07

[Turn over

196

12

BALOL
(c) The divided bar graphs show average amounts of water used, by a family each day, in

For
Examiners
Use

rural areas of the UK and Bangladesh. They also show how the water is used.
Average daily water use and uses by a family in rural areas of the UK and Bangladesh
(litres)
0

10 20 30 40 50 60 70 80 90 100 110 120 130 140 150 160 170 180


Washing Car

Washing
Clothes

Watering
Garden

Flushing Toilet

Washing
Dishes and
Cleaning

Washing

Cooking and
Drinking

Rural
UK

Others

(litres)
0

Others

Washing
Clothes

Washing Dishes
and Cleaning
Cooking and
Drinking

(i)

Washing

Rural
Bangladesh

10 20 30 40 50

How many more litres of water are used by a family in the UK than in Bangladesh?
..............................................................................................................................[1]

(ii)

How many times greater is the amount used by a family in the UK?
..............................................................................................................................[1]

(iii)

For which one of the uses named in the graphs is the daily consumption of water
almost the same in the UK and Bangladesh?
..............................................................................................................................[1]

(iv)

Suggest a reason for this.


..................................................................................................................................
..............................................................................................................................[1]

(v)

The UK is a developed country. Bangladesh is a developing country.


Explain why there are differences in use of water between countries with different
levels of economic development.
..................................................................................................................................
..................................................................................................................................
..................................................................................................................................
..................................................................................................................................
..................................................................................................................................
..................................................................................................................................
..................................................................................................................................
..............................................................................................................................[4]

UCLES 2007

5014/01/M/J/07

197

13

BALOL
(d) There are large differences in access to safe water supplies between urban and rural

For
Examiners
Use

areas in many developing countries in Africa. Two examples are shown in the table.

% of people with access to safe water supplies


Country

(i)

Urban areas

Rural areas

Kenya

70

50

Nigeria

85

40

On the graph paper below, draw a bar graph to show these percentages. Complete
your graph with a key.

[4]

(ii)

Give reasons why there are differences in access to safe water supplies between
urban and rural areas in many developing countries.
..................................................................................................................................
..................................................................................................................................
..................................................................................................................................
..................................................................................................................................
..................................................................................................................................
..............................................................................................................................[3]

UCLES 2007

5014/01/M/J/07

[Turn over

198

14

BALOL
(e) Look at the cartoon about a rural area in Africa.

For
Examiners
Use

Walking four hours to fetch water

(i)

What does it suggest about the views of some people in the area to change and
development?
..................................................................................................................................
..................................................................................................................................
..................................................................................................................................
..............................................................................................................................[2]

UCLES 2007

5014/01/M/J/07

199

15

BALOL(ii) Explain how sinking a new well can reduce disease, increase economic output

For
Examiners
Use

and improve the quality of life for everyone in rural areas in developing countries,
especially for the women and children.
..................................................................................................................................
..................................................................................................................................
..................................................................................................................................
..................................................................................................................................
..................................................................................................................................
..................................................................................................................................
..................................................................................................................................
..................................................................................................................................
..................................................................................................................................
..................................................................................................................................
..................................................................................................................................
..............................................................................................................................[5]
(iii)

Why is outside help from charities such as Water Aid often needed?
..................................................................................................................................
..................................................................................................................................
..................................................................................................................................
..................................................................................................................................
..................................................................................................................................
..............................................................................................................................[3]
[Total: 40]

UCLES 2007

5014/01/M/J/07

[Turn over

200

16
6 BALOL
(a) Look at the two plate boundaries shown in Diagrams A and B.

For
Examiners
Use

Diagram A

Mountain range

Volcano
Ocean
Continental
plate

Oceanic plate
Su

bd

Mantle

Magma

uct

ion

zon

Key

Movement of flow of magma


Direction of plate movement
Diagram B

Continent
Ocean

Ocean
Continent

Plate

Plate
Magma
Mantle

Key

(i)

Movement of flow of magma


Direction of plate movement

Describe what is happening to the two plates in A and the two plates in B.
A ...............................................................................................................................
..................................................................................................................................
B ...............................................................................................................................
..............................................................................................................................[2]

UCLES 2007

5014/01/M/J/07

201

17

BALOL(ii) The source of the magma is different in the two diagrams. Where has the magma

For
Examiners
Use

come from in A and B?


A ...............................................................................................................................
..................................................................................................................................
B ...............................................................................................................................
..............................................................................................................................[3]
(iii)

Why are volcanoes formed at plate boundaries?


..................................................................................................................................
..................................................................................................................................
..................................................................................................................................
..............................................................................................................................[2]

(iv)

State one difference between volcanoes along these two types of plate boundary.
..................................................................................................................................
..................................................................................................................................
..............................................................................................................................[2]

(v)

Some volcanic eruptions result in great loss of life; in others, no one is killed. Give
reasons for the large difference in numbers of people killed.
..................................................................................................................................
..................................................................................................................................
..................................................................................................................................
..................................................................................................................................
..................................................................................................................................
..................................................................................................................................
..................................................................................................................................
..............................................................................................................................[4]

UCLES 2007

5014/01/M/J/07

[Turn over

202

18

BALOL
(b) In some countries electricity is made from geothermal power. Look at the diagram of the

For
Examiners
Use

geothermal power station.


Geothermal power

water at 100C
kept as liquid
by pressure

cold water
pumped down

200C

(i)

hot mass of
igneous rocks

How is the electricity produced?


..................................................................................................................................
..................................................................................................................................
..................................................................................................................................
..................................................................................................................................
..................................................................................................................................

(ii)

Why are areas of active volcanic activity needed for its production?
..................................................................................................................................
..................................................................................................................................
..................................................................................................................................
[4]

UCLES 2007

5014/01/M/J/07

203

19

BALOL
(c) The graph shows average costs of producing electricity from different energy sources in

For
Examiners
Use

2003.
Costs of producing electricity (2003)
50

US cents per kw/hr

40

30

20

10

Solar

Wave

Biomass

Wind

Nuclear

Geothermal

Hydroelectric

Fossil
fuels

Energy source
(i)

Describe what the graph shows about the cost of producing geothermal electricity
compared with electricity from other energy sources.
..................................................................................................................................
..................................................................................................................................
..................................................................................................................................
..................................................................................................................................
..............................................................................................................................[3]

(ii)

Explain how likely it is that geothermal power will be used more in the future as an
alternative to fossil fuels.
..................................................................................................................................
..................................................................................................................................
..................................................................................................................................
..............................................................................................................................[2]

UCLES 2007

5014/01/M/J/07

[Turn over

204

For
Examiners
Use

20

BALOL
(d) Look at the information below about world supply and demand for oil.
Oil reserves are running out

In 2002, 25 billion
barrels of oil were used
worldwide. Only 8 billion
barrels of new reserves
were discovered

Oil reserves are


running out
Total global oil
994 billion barrels
extracted
to date

Discovery vs demand
764 billion barrels
remaining in
known fields

142 billion
barrels
yet to find

Billion barrels of oil per year


60
discovery
50
40
30

demand

20
10
0

1955 1965 1975 1985 1995 2005

= 100 billion barrels

(i)

Describe what happened to the demand for oil between 1955 and 2005.
..................................................................................................................................
..................................................................................................................................
..................................................................................................................................
..................................................................................................................................
..............................................................................................................................[3]

UCLES 2007

5014/01/M/J/07

205

For
Examiners
Use

21

BALOL(ii) Quote values and information which support the following statements.
1 Oil reserves are running out.
..................................................................................................................................
..................................................................................................................................
..................................................................................................................................
..................................................................................................................................
2 The present use of oil is not sustainable.
..................................................................................................................................
..................................................................................................................................
..................................................................................................................................
..............................................................................................................................[4]
(iii)

What is likely to happen to the demand for oil after 2005? Explain your answer.
..................................................................................................................................
..................................................................................................................................
..................................................................................................................................
..................................................................................................................................
..............................................................................................................................[3]

UCLES 2007

5014/01/M/J/07

[Turn over

206

22

BALOL
(e) A

One persons view of nuclear energy

For
Examiners
Use

Future shortages of electricity


can only be avoided by building
new nuclear stations. It is a clean
source, capable of producing
large amounts of energy.

Another persons view of nuclear energy.

I strongly object to any


increase in nuclear energy.
It is just too dangerous.

(i)

Explain why some people hold the view about nuclear energy stated in A.
..................................................................................................................................
..................................................................................................................................
..................................................................................................................................
..................................................................................................................................
..................................................................................................................................
..............................................................................................................................[3]

(ii)

State the different arguments which supporters of view B could use.


..................................................................................................................................
..................................................................................................................................
..................................................................................................................................
..................................................................................................................................
..................................................................................................................................
..............................................................................................................................[3]

UCLES 2007

5014/01/M/J/07

207

23

BALOL(iii) What is your view on nuclear power? How strong are the different arguments put

For
Examiners
Use

forward in part (ii)?


..................................................................................................................................
..................................................................................................................................
..................................................................................................................................
..................................................................................................................................
..............................................................................................................................[2]
[Total: 40]

UCLES 2007

5014/01/M/J/07

208

BALOL
UNIVERSITY OF CAMBRIDGE INTERNATIONAL EXAMINATIONS
GCE Ordinary Level

MARK SCHEME for the May/June 2007 question paper

5014 ENVIRONMENTAL MANAGEMENT


5014/01

Paper 1, maximum raw mark 120

This mark scheme is published as an aid to teachers and candidates, to indicate the requirements of
the examination. It shows the basis on which Examiners were instructed to award marks. It does not
indicate the details of the discussions that took place at an Examiners meeting before marking began.
All Examiners are instructed that alternative correct answers and unexpected approaches in
candidates scripts must be given marks that fairly reflect the relevant knowledge and skills
demonstrated.
Mark schemes must be read in conjunction with the question papers and the report on the
examination.

CIE will not enter into discussions or correspondence in connection with these mark schemes.

CIE is publishing the mark schemes for the May/June 2007 question papers for most IGCSE, GCE
Advanced Level and Advanced Subsidiary Level syllabuses and some Ordinary Level syllabuses.

209

Page 2

Mark Scheme
GCE O LEVEL May/June 2007

Syllabus
5014

Paper
01

BALOL
Section A
1

(a) (i) X (1)


dark coloured (1)

[2]

(ii) pebbles/rock fragments

[1]

(b) (i) the sand has a medium/large grain size


(ii) pH = proportion of hydrogen ions/measure of acidity
4 = the soil is acidic

[2]

(iii) the coarser the texture the larger the pore spaces/vv./positive relationship

[1]

(c) (i) (coarse texture)


advantage: roots can penetrate easily/well-drained/aerated/easy to till/earthworm
movement facilitated
disadvantage: plants can suffer from drought/needs watering/irrigation/infertile/few
nutrients/needs fertiliser/manure
(ii) (pH of 4)
too acidic for many crops
fertiliser quickly leached
2

[1]

[1]
[1]

[1]

(a) coastal areas


continental shelf
(mainly) temperate/northern hemisphere
ocean currents

[3]

(b) shallow water


penetration of light
cool water
large rivers enter sea
nutrients/mineral salts in the water
cold and warm currents meet
cold water up-wells
bring up nutrients
abundant plankton
a.v.p.

[3]

(c) 500 km limit/fishermen given control of fisheries had no/little effect as it was not in their
interest to control fishing area closed to mobile fishing gear had great impact/fish population
rose quickly because fish could breed over a large area without interference
[4]

UCLES 2007

210

Page 3

Mark Scheme
GCE O LEVEL May/June 2007

Syllabus
5014

Paper
01

BALOL
3

(a) (i) pour water from jar into measuring cylinder


at same time each day
stand cylinder on a horizontal surface/avoid parallax
eye at level of meniscus

[2]

(ii) on grass so no splash


part buried to reduce evaporation
part buried for stability

[2]

(iii) snow/ice (has to be melted) or very strong winds

[1]

(b) low amount


infrequent/irregular
heavy

[3]

(c) less danger of salinisation of soil


less water to evaporate (and leave salts)

[2]

(a) (i) 18 18.4

[1]

(ii) correct plots at 3.2% (male) and 3.8% (female)

[1]

(iii) increasing rapidly

[1]

(b) large dependent population


burden on the working population
need to increase food supply/food shortages
more housing needed
more schools needed
may lead to unemployment
may lead to overpopulation
a.v.p.

[4]

(c) culture
tradition of large families
desire for sons/children as labour/look after parents/etc.
religious beliefs
low GDP
poverty of the individual
difficulty of introducing birth control in rural areas
a.v.p.

[3]

UCLES 2007

211

Page 4

Mark Scheme
GCE O LEVEL May/June 2007

Syllabus
5014

Paper
01

BALOL
Section B
5

(a) (i) B lake/reservoir (dam)


C ice/glacier/snow

[2]

(ii)&(iii) dig into the ground to reach the layer of sandstone rock
sandstone is porous/permeable
layer of rock outcrops on surface in mountains
can take in and hold rainwater in an underground store
it is an aquifer
dig a vertical shaft for a well
use a pump to draw water up to the surface
mark both parts as one.
three points made along these lines.

[3]

(iv) the best choices are D or C. B is a better choice than A.


no mark for choice.
Explanation of choice is likely to be more successful if either D or C is chosen, because
there is less likelihood of the water having been polluted. Particles are filtered out as
water passes through pervious rock underground. The snow and ice are high in the
mountains where no one lives and are maintained as pure rainwater. Lakes are better
than rivers because there is a chance for impurities to settle out, but they are affected by
what flows into them. Rivers are almost impossible to justify since they tend to flow
through settled areas and are used both deliberately and accidentally as sewers.
[2]
(v) A or B = 1 mark for choice.
Why? See comments above = 2 marks for explanation.
Maximum 2 marks possible for A or B, but likely max 1 for the others (e.g. pesticides etc.
can seep into and affect groundwater supplies, from mineral workings).
[3]
(b) (i) Name and locate is used in order to encourage a precise location, especially if a
local/national rather than a well-known international example is used. For example,
Aswan High Dam in Egypt is an example of name and locate if taken to the letter. Aswan
High Dam would be enough because, being international, everyone knows it is an
appropriate choice. Mark according to the spirit as described here = 1 mark.
(ii) Further information could be more about location, size, why it was physically possible
to build it in that place.
Reasons for building it usually include multi-purpose uses such as water supply for
domestic and industry, irrigation water, increased food output, hydro-electric, tourism,
navigation, flood control = 4 marks.
Max. 3 marks for general answers about dams (if full of detail).
Max. 4 marks for a named dam, but without any information that is precise to it.

[5]

UCLES 2007

212

Page 5

Mark Scheme
GCE O LEVEL May/June 2007

Syllabus
5014

Paper
01

BALOL
(c) (i) 135

[1]

(ii) 4 times

[1]

(iii) Cooking and Drinking

[1]

(iv) Idea conveyed, however expressed, that they are essential for life/survival; they are not
luxuries or comforts like some of the others.
[1]
(v) * In developed countries, people are richer/have a better quality of life so that washing
machines and dishwashers do more of the work than people; in developing countries
hand washing uses less water and it is often done in rivers/streams instead of houses.
* Sanitation is almost 100% in homes in developed countries, whereas in developing
countries sanitation/flushing water is in much less than 50% of homes, especially in rural
areas.
* Piped water reaches houses in developed countries by the taps, whereas water supply
from pumps and wells is more commonly located in public places in developing
countries.
Two or more ideas such as these stated in a two-sided manner (i.e. with positive
references to different levels of development) = 3 or 4 marks.
One idea well stated for developed and developing, or two ideas stated only for
developed or developing = 1 or 2 marks.
(d) (i) Frame labelled and bars drawn = 1 mark.
All four accurately plotted = 2 marks.
(Two correct (e.g. for one country or for one type of area) = 1 mark).
Key to match way in which differences between rural and urban shown = 1 mark.

[4]

[4]

(ii) more wealthy people live in urban areas


places where administrators/politicians with powers live
cities have higher levels of economic development than in the countryside
more need to improve to stop spread of disease with high densities of population
more engineers/people with necessary skills live in towns
Positive points like these for urban areas can in general be reversed for negative points
for countryside. These are just suggestions many different approaches are possible. In
a three mark answer, there must be at least one definite point made for both rural and
urban areas.
[3]

UCLES 2007

213

Page 6

Mark Scheme
GCE O LEVEL May/June 2007

Syllabus
5014

Paper
01

BALOL
(e) (i) cartoon suggests lack of will among some people/the men to improve
walking miles to collect water is seen as a normal (female) activity
why change?
View expressed with some understanding = 1 mark.
Understood that maintenance of the status-quo is suggested = 2 marks.

[2]

(ii) dirty water is a major cause of water related diseases


examples of diseases and how they spread
many millions of people in developing countries are affected
reduces ability to work and produce
constant bouts of illness reduce quality of life
particularly severe for infants and elderly resulting in high death rates
women can engage in productive economic activities without water to collect
examples crafts, textiles, taking produce to market etc.
children are more healthy/have more time for education and study
greatly improves their future prospects/chances of employment
These are just some of the ideas that are relevant to the answer.
Mark according to the worth of the answer overall.
* One or two relevant ideas, but little developed towards the main question theme. Some
will be about disease and nothing else.
[12 marks]
* Wider range of points, with fuller statements about them, but without complete
coverage of all aspects of the question.
[34 marks]
* Good coverage including some reference to why women and children may be the ones
to benefit most.
[5 marks]
[5]
(iii) money and expertise are needed for sinking a proper well
it may need to be lined with cement/pump needed to bring water to the surface
most communities in rural areas lack both the resources and expertise,
also they need an infusion of new ideas/modern technology from outside
work often left to charities because governments are too poor/only interested in urban
areas
Three points which hold together made along these lines.

[3]
[Total: 40]

UCLES 2007

214

Page 7

Mark Scheme
GCE O LEVEL May/June 2007

Syllabus
5014

Paper
01

BALOL
6

(a) (i) A moving together/converging


B moving apart/diverging

[2]

(ii) A magma is formed from melting rock in the subduction zone


due to the great pressure created as the oceanic and continental plate meet
B magma is formed in/comes from the mantle
this is new material that reaches the surface where the plates move apart
minimum 1 for each of A and B
(iii) fractures/weaknesses are formed that enable the magma to reach the surface
pressure from Earth movements forces magma out of the vent

[3]
[2]

(iv) shape of cone tall and steep in A, gentle sides and wide base in B
materials erupted mixture of lava and rocks, ash and dust in A, lava only in B
lava is sticky in A but runny in B
granite a common rock in A, basalt in B
activity can be violent/often occasional in A, continuous non-violent lava flows in B
land volcanoes or island arcs in A, volcanoes rise from sea bed in B to form occasional
islands
any one difference
1 sided = 1 mark

2 sided = 2 marks

[2]

(v) * Type of activity (see above); some volcanoes erupt occasionally and with violence,
resulting in more deaths than those from which lava flows semi-permanently. Usually
people have plenty of time to get out of the way of lava flows, less easy to escape when
solid materials are being violently thrown out.
* Amount of warning; either not monitored, or suddenly erupts after many years without
activity (some were thought to be dead volcanoes).
* Massive size and scale of the eruption
* What is caused by the eruption e.g. mudflows, tsunamis can cause even greater loss of
life
* Density of population in surrounding area
Two or more reasons need to be referred to and explained for full marks. Credit
references to valid examples as well.

[4]

(b) (i) diagram shows that cold water is heated by the hot mass of igneous rocks
hot water goes into generating station/power station
it is the energy source to drive the turbines that produce electricity
(ii) only in volcanic areas is the heat sufficient to drive the turbines/is it sufficiently close to
the surface
it is a constant source of heat for non-stop electricity production
Four points made there is likely to be some natural overlap between the two parts.

[4]

UCLES 2007

215

Page 8

Mark Scheme
GCE O LEVEL May/June 2007

Syllabus
5014

Paper
01

BALOL
(c) (i) one of the cheaper sources/third cheapest energy source for electricity
still more than double the cost of using fossil fuels
but cheaper than all the other alternatives except hydro
a fraction of the price of some of the others (e.g. solar is 7 times more expensive)
Recognises relative cheapness (however expressed) = 1 mark.
Two other comparative statements = 2 marks.

[3]

(ii) Cost might suggest that they are good, but the problem is that it needs particular
physical conditions, which exist only in certain areas of the world.
Examples could be quoted to illustrate this e.g. Iceland and New Zealand.
Some understanding = 1 mark
Good understanding and effectively expressed = 2 marks

[2]

(d) (i) Steep rise from around 5 to 20 billion barrels from 1955 to 1975
more gentle rise with some fluctuations from 1975
however clear overall/persistent increase to 25 billion barrels by 2005
Description supported by use of values needed for full marks
(ii)

[3]

Mark both parts together


* 1000 billion barrels already used, but only 750 billion in reserves/only 900 billion
barrels thought to exist to be used (i.e. a non-renewable resource is being overconsumed).
* Demand for oil exceeded discovery by 1975 and the gap in 2005 is wider than ever
before.
* Statement summarises current state of non-sustainability that in 2002 25billion barrels
were used and new reserves were only 8 billion (i.e. about one third of the demand).
Even wider if value for 2005 is taken from the graph (discovery approx. one fifth of
demand/use).
Two statements needed = 2 marks.
At least one relevant quote using values = 1 or 2 marks.
If all 4 marks not claimed, allow one mark for clear comment about non-sustainability. [4]

(iii) All the evidence suggests that it will go on rising (even if it is at a more moderate rate
than from 1955 to 75) = 1 mark.
history shows that cutbacks in demand have never lasted for long
increased use of oil is associated with economic development
especially growth in car and air transport
particular references such as growing demand in China
Explanation = 2 marks
If the alternative suggestion of demand falling is made, there is 0 mark for describing
what is likely to happen, but up to 2 marks can be claimed for explanation in terms of
greater use of alternatives, technological breakthroughs in their development and
increased energy efficiency.
[3]

UCLES 2007

216

Page 9

Mark Scheme
GCE O LEVEL May/June 2007

Syllabus
5014

Paper
01

BALOL
(e) (i) About view A
* Much less air pollution than from fossil fuels power stations, virtually no carbon
dioxide/greenhouse gas emissions.
* Known technology not dependent on new technological breakthroughs as needed for
many renewables.
* Reasonably cheap (see graph in part (c) above).
* Uses only small amounts of uranium/low raw material needs compared with amount of
energy released.
* Not restricted by high physical demands as for hydro for example.
Points made along these lines with references to at least two for all three marks

[3]

(ii) About view B


* There may be no air pollution, but any radio-activity released is much more dangerous
for life on Earth responsible for leukaemia and cancers in people.
* Contaminated nuclear waste dangerous for thousands of years with no satisfactory
means of storage.
* Dangerous if used irresponsibly by nations/terrorist threats.
* Some disasters such as Chernobyl which shows that it is not as safe as scientists
claim.
* Many leaks into nearby seas/water courses.
Points made along these lines with references to at least two for all three marks.

[3]

(iii) Mark according to the strength with which the chosen view is supported (not for the view
expressed). Candidates need to give some idea of relative strengths of arguments
referred to in (i) and (ii).
Some idea of candidates own view with sound reasoning = 1 mark
Clear view supported by strength of argument (irrespective of view taken) = 2 marks

[2]

[Total: 40]

UCLES 2007

217

5014 Environmental Management June 2007

BALOL
ENVIRONMENTAL
MANAGEMENT

Paper 5014/01
Paper 1

General comments
Many candidates found the short questions in Section A more challenging than the two longer questions in
Section B. Marks for Questions 1 and 3 were often lower than those for Questions 2 and 4. Gaps in
answering parts of questions were few and far between, but the majority occurred within Questions 1 and 3.
One or two candidates had clearly mis-allocated their efforts within the time allowed and failed to finish all
parts of the final question. However, most had no time problems, giving full answers throughout, and on
occasions more than filling the lines provided for answering. Overall standards of answering the two
questions in Section B were similar; it was rare for there to be a wide divergence in candidate performance
between Questions 5 and 6.
One specific weakness from Question 1 was the failure to make direct use of the photograph. Already by
1(a)(ii) some candidates had stopped looking at the photograph, despite the question asking for evidence
that layer Z was derived from parent material. In 3(c) some candidates stopped using the graph after making
an initial statement and digressed into more general answers about measures to increase fish populations.
A general weakness was a desire by some to give reasons when description alone was required by the
question; this was noticed most in answers to Questions 6(c)(i) and 6(d)(i). These candidates were the
ones least likely to use values in support of their answers, as they quickly moved away from direct
references to the graph and diagram information. Another observed weakness was the reverse of this,
namely a failure to give reasons in Questions 5(c)(iv) about differences in water use and 5(d)(ii) about
differences in access to safe water supplies between rural and urban areas; instead extended description of
actual differences in water use and supply was given. Some candidates failed to make the switch from part
(a) to part (b) in Question 5. The many answers of B and D in (b)(i) showed that candidates were referring
back to the cross-section in 5(a) instead of answering in relation to a named example of an actual dam.
Candidates should be made aware that the lettering and numbering of question sub-sections is significant; a
new letter usually denotes a fresh start and a different theme, even though if often follows on from what went
before.

Comments on individual questions


Section A
Question 1
The correct answer X was the main one given in (a)(i). The dark colour of the layer on the photograph was
better evidence for most organic matter than roots and grasses. In many of the weaker answers it was clear
that candidates had not realised that the diagram and the photograph were related. Some others chose Z,
which showed that they did not have a clue. Part (a)(ii) was badly answered. One very popular answer was
dark coloured at the bottom, which suggested weak candidate understanding of the different colour shades
in a soil profile. Only a few noted the existence of rock fragments and pebbles. Of the three parts of (b), part
(i) proved to be the most difficult. Instead of explaining that sand has a medium or large grain size which
gives the coarse texture, many answers began and ended with sand. One mark answers were typical for
(b)(ii); although a majority of candidates stated that a soil with a pH of 4 was acidic, there was no attempt to
claim a second mark by indicating that pH is used as a measure of acidity, or by noting the significance in
relation to a pH of 7. In (b)(iii) most understood the coarser the texture, the larger the pore spaces or vice
versa. Answers to part (c) were more consistent. In the mark scheme at least five different advantages and
disadvantages were listed, which meant that candidates had plenty of choices for answering (c)(i). In (c)(ii),
although most recognised lack of fertility, some referred to the need to use pesticides when they probably
meant fertilisers.

218

5014 Environmental Management June 2007

Question
BALOL 2
The three most obvious features of location in part (a) were coastal areas, continental shelves and ocean
currents; mainly in temperate areas (or mainly outside the tropics) was another alternative regularly
mentioned and allowed. Few candidates failed to note at least two of these. Part (b) attracted many full
answers with frequent references to the abundance of plankton and nutrients, especially in relation to the
presence and meeting of ocean currents and shallow sea water. The weakest answers came from
candidates who repeated location features from part (a) without describing any of the favourable conditions
they provided for large marine populations. The strongest answers in part (c) came from candidates who
dealt with the effects of the two measures one at a time, which best addressed the question demand for
different effects. The weakest came from those who described the line diagram as one, which made it
impossible for them to focus on the question set.
Question 3
In (a)(i) many candidates took their time to start to answer. There was a lot about rain water collecting in the
jar first. Most, however, eventually showed that they understood that the rain water needed to be tipped from
the jar into the measuring cylinder, on a regular daily basis, and read at eye level with great care. There
were many references to the need to be sited in an open area in answers to (a)(ii), but these were ruled out
by the as shown in the diagram part of the question. Some candidates stated site factors such as above
surface level and half buried in the ground without explanation. However, for the remainder it was a
relatively straightforward question. There were frequent references to snow and hail, and occasional
mentions of ice and strong winds, in answers to (a)(iii). Of these, only for hail was there no reward. Nearly
all claimed the mark for low amount in part (b), often supported by use of an annual total below 250 mm. For
frequency, rain only on a few days a year was a more common answer than the other acceptable answer of
irregular occurrence. Mainly light showers was a frequent answer for intensity from those without a full
understanding of a desert climate. In (c) virtually all candidates understood the meaning of trickle drip
irrigation. Unfortunately, some focused on saving/not wasting water instead of reducing the harmful effects.
Occasionally their answers led to the acceptable point of reduced evaporation; more rarely did it lead to
salinisation. In fact, only a minority of candidates managed to establish the link between evaporation and
salinisation.
Question 4
Answers between 18.0 and 18.4 were accepted in (a)(i). The majority were correct, but some gave values
around 9.0 because they had used one bar only, either for male or female. Any mistakes in (a)(ii) usually
resulted from reversing male and female percentages. Some candidates circled two options in (a)(iii); often
these were the opposites deceasing slowly and increasing rapidly. Although increasing rapidly was the
most popular answer, decreasing slowly had much support among less strong candidates, who were
describing pyramid shape rather than interpreting what was being shown about a countrys population.
There were many different points that could be made in part (b); this made it relatively easy for candidates to
claim three or four marks. Many of those who had circled an incorrect option in (a)(iii) were still able to gain
one or two marks, as they referred to problems associated with the high percentages of young people shown
by the wide base of the pyramid. In part (c) references to religion, culture and tradition were more common
than those to low GDP, poverty and the difficulty of introducing birth control in rural areas. Full answers were
frequent.
Section B
Question 5
Part (i) was the best answered part of 1(a). Most candidates stated either lake or reservoir for B and ice or
snow for C. However, in (a)(ii) and (iii) many fewer than had been expected noted the closeness of the layer
of sandstone rock to the surface at D, and the resulting possibilities for obtaining a fresh ground water
supply. Instead a good number of candidates gave unsatisfactory and unlikely answers based on building
pipelines to bring water in from elsewhere or damming the river at D. The choice of C made explanation
easy in (a)(iv), although some were able to make good progress with B by concentrating on sediment
settlement in the bottom of a large surface water body. While A was perhaps the most obvious choice in
(a)(v), a good case could also be made for B as well. Some of the explanations, however, were too brief for
a question with three marks attached to it.

219

5014 Environmental Management June 2007

In
part (b) a few candidates used a letter from the cross-section in (a)(i) for their dam location; they could
BALOL
gain no more than three marks for a general answer. In the event many of their answers were too vague or
too limited in range to reach even three marks. Of a different standard were the majority of answers based
upon a named dam. Major dams such as Aswan, Three Gorges and Hoover were popular choices, but
many also used an example from their home country, especially those candidates from Centres in Pakistan.
This was likely to increase the amount of specific information included and thereby increase their chances of
reaching full marks.
The correct answer of 135 litres was the almost universal answer in (c)(i). However, answers of 3 times in
(c)(ii) were almost as common as those for the correct answer of 4 times. Although cooking and drinking
were almost invariably selected in (c)(iii), some answers to (c)(iv) suffered from a repeated statement
instead of a reason. The reason needed to relate to these as essential human activities, not dependent on a
persons wealth or place of residence. The worth of some answers to (c)(v) was restricted either by too
much description and too little explanation, or by over-concentration on one type of country so that real
differences between developed and developing were not explained. The strongest answers came from
candidates who referred to both types of countries and varied their language beyond the statement of direct
opposites, one positive and one negative.
The practical task in part (d)(i) posed few problems. However, for the diagram to be a visual success, the
key needed to separate out rural and urban areas and not the two countries. Some candidates drew their
key using a pen and then used a pencil for showing the same shading on the graph, which was accepted
provided that the type of shading was intended to be the same, but it was not the best way to do it. Strong
candidates answered part (d)(ii) well; weaker candidates concentrated on describing how people in urban
and rural areas actually gained access to water supplies. The best answers included references to the
concentration of wealth, economic development and government investment in cities, often to the exclusion
of rural areas.
Some candidates failed to give views about change and development in (e)(i). Part (e)(ii) was an easier
question; candidates explained why well water was likely to be cleaner than surface water, and how a better
water supply could help both peoples health and economic activities such as farming. In answers worth four
and five marks candidates typically named examples of water-related diseases, and made reference to
reasons for health improvements in young children and women in particular. In (e)(iii) not all candidates
concentrated upon referring to reasons why outside help from charities was vital, such as lack of money,
resources, know-how and technology, particularly in rural areas in a continent such as Africa. Instead, some
repeated answers already covered in earlier questions about how bad the water supply situation was, which
did not address the question effectively.
Question 6
Some candidates used a lot of words to reach the relatively short and simple answers of converging and
diverging (or their equivalents) in part (a)(i). Many were attempting to answer a much bigger question than
the one set, more like Explain what is happening at plate boundaries A and B. The majority possessed the
knowledge and understanding needed in (a)(ii) to claim at least two of the three marks. The best answers to
(a)(iii) concentrated on the presence of lines of weakness and fractures, as well as the pressure from earth
movements which allowed magma to reach the surface. Part (iv) was the least well answered part of (a).
Several headings were available to candidates for establishing one difference, such as cone shape,
materials erupted, nature of volcanic activity and location. Many relied upon location, for which there was
some help on the diagrams. Some of the answers given to (a)(v) were more appropriate for earthquakes
than volcanoes. Otherwise in well-focused answers candidates used a variety of reasons, with some useful
references to actual eruptions, notably those of Mount Etna, included as well.
How geothermal power was generated was well understood in part (b). Occasionally weaker candidates lost
sight of the geothermal power theme in (b)(ii) and somewhat unaccountably began to state advantages of
the use of volcanic areas for human settlement such as fertile soils, presence of minerals and tourism.

220

5014 Environmental Management June 2007

The
overall message that could be taken from the graph in part (c) was that the production of geothermal
BALOL
power was relatively cheap compared with most other ways of generating electricity, especially many of the
alternatives. A majority of candidates realised this. Those who quoted relative costs to elaborate upon this,
as well as upon the cheapness of fossil fuels, soon claimed all three marks in (c)(i). Part (c)(ii) was a good
discriminator. Many candidates used words to the effect that the relative cheapness of clean geothermal
power would make it a great alternative to dirty fossil fuels as they are used up. This had some merit, but
was not as good an answer as the one from candidates who showed awareness that opportunities for cheap
generation are restricted to areas with certain favourable natural conditions such as Iceland and New
Zealand.
Provided that candidates restricted themselves to description using only the demand line, it was
straightforward for them to gain at least two of the three marks in (d)(i). The most common reason for the
loss of one mark was lack of use of any values. For the loss of two or more marks, the causes were either
digression into explanation for increased demand, or attempts to relate variations in demand to discovery.
The worst type of answer in (d)(ii) was to do no more than list individual values without any attempt to
establish the context. Single quoted values such as Discovery of oil was 57 billion barrels in 1965 or
Demand for oil was 25 billion barrels in 2005 were not enough by themselves to indicate oil reserves
running out or unsustainable use. An answer of this type was never going to satisfy all the needs of a four
mark question. Much superior were answers from candidates who attempted to use several values to make
a relevant point. One of the most effective responses was adding up the totals from the second and third
columns under total global oil, in order to make the point that people have already used more oil than is
known or likely to be available for use in the future. In (d)(iii) all the evidence given suggested that demand
will keep rising. This was easy to explain by reference to growth in world population, continued economic
development especially in China and India and the important uses of oil in car and air transport. Almost
equal credit was given to the opposite suggestion of falling demand, provided that explanation was given in
terms of the greater use of alternatives, technological breakthroughs in their development, increased energy
efficiency and savings in use imposed by higher prices and lower availability.
In general candidates found it more difficult to expand upon what was already stated in the speech bubbles
for A than for B in part (e)(i). Most showed themselves to be familiar with the dangers from radio-activity,
contaminated waste, leaks, explosions and accidents in (e)(ii). References to Chernobyl were frequent. In
(e)(iii) the majority opinion among candidates was against nuclear power, but only slightly. One common
view was to increase the use of nuclear power as an already proven alternative, which would help to reduce
the frequency of power cuts in developing countries. Some candidates made the point that there have been
no major disasters since Chernobyl, which showed that nuclear can be safe. A frequently expressed
opposite view was that nothing could justify the use of something as dangerous as nuclear power, no matter
how great the energy need. As is always the case in questions which ask for a candidates view, the
strength of the argument is what is assessed and not the view held. The weakest answers came from
candidates who gave equal weight to both sides of the argument; often they had little new to add what had
already been written in (e)(i) and (ii).

221

BALOL
UNIVERSITY OF CAMBRIDGE INTERNATIONAL EXAMINATIONS
General Certificate of Education Ordinary Level

*8337170735*

5014/01

ENVIRONMENTAL MANAGEMENT
Paper 1

October/November 2007
2 hours 15 minutes

Candidates answer on the Question Paper.


Additional Materials:

Ruler
Protractor

READ THESE INSTRUCTIONS FIRST


Write your Centre number, candidate number and name on all the work you hand in.
Write in dark blue or black pen.
You may use a soft pencil for any diagrams, graphs or rough working.
Do not use staples, paper clips, highlighters, glue or correction fluid.
DO NOT WRITE IN ANY BARCODES.
Answer all questions.
At the end of the examination, fasten all your work securely together.
The number of marks is given in brackets [ ] at the end of each question or part question.

For Examiners Use


1
2
3
4
5
6
Total

This document consists of 27 printed pages and 1 blank page.


SP (SM/CGW)
UCLES 2007

[Turn o

222

BALOL

Section A

For
Examiners
Use

(a) The photograph shows a volcanic area in New Zealand used for developing geothermal
power.

(i)

What shows that this is a volcanic area?


..............................................................................................................................[1]

(ii)

What is being used for the transfer of the source of energy to the power station
down the valley?
..............................................................................................................................[1]

(iii)

Why does a valley location help the transfer?


..............................................................................................................................[1]

(iv)

What disadvantages does the transfer of this energy source have for the area?
..................................................................................................................................
..................................................................................................................................
..............................................................................................................................[2]

UCLES 2007

5014/01/O/N/07

223

BALOL
(b) Geothermal energy cannot be developed in all areas of the world. Describe the

For
Examiners
Use

underground conditions necessary for its development and how people use them.
..........................................................................................................................................
..........................................................................................................................................
..........................................................................................................................................
..........................................................................................................................................
......................................................................................................................................[3]
(c) What are the advantages of geothermal energy?
..........................................................................................................................................
......................................................................................................................................[2]

UCLES 2007

5014/01/O/N/07

[Turn over

224

4
2 BALOL
(a) Look at the diagram showing words that can be used to describe the cycle of the
waterborne disease, typhoid.

For
Examiners
Use

water with
typhoid bacteria
Q

P
faeces and urine

R
mouth
Key
P Q R points at
which the cycle
can be broken

intestines
typhoid illness

bloodstream

(i)

Complete the diagram by adding arrows to show the cycle of the disease.

(ii)

P, Q and R on the diagram show points at which the cycle of typhoid can be broken.
The table below shows methods used to break it. Match these methods with the
points by writing the correct letters in the table.

method

[1]

point on diagram

drugs
install sanitation
provide a clean water supply
[2]
(iii)

Explain why methods of breaking the typhoid cycle are not used everywhere in the
world.
..................................................................................................................................
..................................................................................................................................
..................................................................................................................................
..................................................................................................................................
..................................................................................................................................
..................................................................................................................................
..............................................................................................................................[4]

UCLES 2007

5014/01/O/N/07

225

BALOL
(b) The diagram shows a new device for supplying water that is being used in some African

For
Examiners
Use

villages. The Play Pump costs an average of one US dollar a day to install and maintain
for 15 years.

Villagers collect
clean water
without using
manual
pumps

The children push


the roundabout,
generating energy
to power a
water pump

2
Clean water
is pumped from
100 metres deep boreholes
to a special tank near the
roundabout
What are the advantages of this method of water supply?
..........................................................................................................................................
..........................................................................................................................................
..........................................................................................................................................
..........................................................................................................................................
......................................................................................................................................[3]

UCLES 2007

5014/01/O/N/07

[Turn over

226

6
3 BALOL
(a) The diagram shows part of a label designed for a pesticide container.

For
Examiners
Use

CAMSPRAY
Systemic insecticide for the control of aphids on fruit trees

FOR USE ONLY AS AN AGRICULTURAL PESTICIDE

Rates of use
100 cm3 CAMSPRAY in 200 litres of water. Apply to leaves
until run-off.

Timing
Apply when aphids are first seen. Repeat at 10-14 day intervals.

Harvesting Interval
Allow a minimum of two weeks between the last application
of CAMSPRAY and harvesting the crop.

PRECAUTIONS
1

Wear protective clothing

Keep livestock out of treated areas for 7 days.

Do not contaminate ponds and waterways.

10 Do not apply at flowering stage.

Which instructions indicate that the pesticide can harm the environment?
..........................................................................................................................................
..........................................................................................................................................
......................................................................................................................................[3]

UCLES 2007

5014/01/O/N/07

227

BALOL
(b) (i) What will be the consequences for the environment of over-using the pesticide?

For
Examiners
Use

..................................................................................................................................
..................................................................................................................................
..................................................................................................................................
..................................................................................................................................
..............................................................................................................................[3]
(ii)

Describe how farmers can control pests in less harmful ways.


..................................................................................................................................
..................................................................................................................................
..................................................................................................................................
..................................................................................................................................
..................................................................................................................................
..............................................................................................................................[4]

UCLES 2007

5014/01/O/N/07

[Turn over

228

For
Examiners
Use

8
4 BALOL
(a) (i)

The pie chart shows global fuelwood production in 1998.

Central America
and Caribbean

Former
USSR Europe
North
America

90

10

80

20

Key:
Asia

30

70

Africa
South
America

40

60
50

Complete the pie chart using the information in the table below. Use the key
provided.

continent

fuelwood production

Asia

50%

Africa

27%

South America

10%
[2]

(ii)

How does the amount of fuelwood used in the developing world differ from that in
the developed world?
..............................................................................................................................[1]

UCLES 2007

5014/01/O/N/07

229

BALOL
(b) What social and environmental problems will be caused by an increased use of

For
Examiners
Use

fuelwood?
..........................................................................................................................................
..........................................................................................................................................
..........................................................................................................................................
..........................................................................................................................................
..........................................................................................................................................
......................................................................................................................................[4]
(c) How could the use of fuelwood be made more sustainable?
..........................................................................................................................................
..........................................................................................................................................
..........................................................................................................................................
..........................................................................................................................................
......................................................................................................................................[3]

UCLES 2007

5014/01/O/N/07

[Turn over

230

10

BALOL

Section B

Look at the world map which shows total population in 2004 and expected population in 2050 for
five continents.
Population change 20042050

ASIA
6000

EUROPE
millions of people

millions of people

NORTH AMERICA
2000
1000
0

2004 2050

2000
1000
0

millions of people

5000
4000
3000
2000
1000

2004 2050

2004 2050

EUROPE

NORTH
AMERICA

ASIA

AFRICA
SOUTH
AMERICA

AFRICA

2000
1000

UCLES 2007

2004 2050

millions of people

millions of people

SOUTH AMERICA
2000
1000
0

2004 2050

5014/01/O/N/07

231

11

BALOL
(a) (i) By how much is the population of Asia expected to increase between 2004 and

For
Examiners
Use

2050?
..............................................................................................................................[1]
(ii)

Compared with the other continents, what is expected to be different about


population change in Europe from 2004 to 2050?
..............................................................................................................................[1]

(iii)

In which continent is the fastest rate of population growth expected between 2004
and 2050?
..............................................................................................................................[1]

UCLES 2007

5014/01/O/N/07

[Turn over

232

12

BALOL
(b) The two countries in the world with most people are China and India. The table shows

For
Examiners
Use

population data for them.


Total populations in China and India

(i)

2004
Population (millions)

2050
Population (millions)

China

1300

1400

India

1100

1530

Draw bar graphs to show the population data in the table.

[3]
(ii)

What significant change is shown between 2004 and 2050?


..................................................................................................................................
..............................................................................................................................[1]

UCLES 2007

5014/01/O/N/07

233

13

BALOL(iii) Give reasons why population growth is higher in some countries than in others.
1.

For
Examiners
Use

Reasons why population growth remains high in some countries.

..................................................................................................................................
..................................................................................................................................
..................................................................................................................................
..................................................................................................................................
..................................................................................................................................
..................................................................................................................................
2.

Reasons why population growth is much lower in other countries.

..................................................................................................................................
..................................................................................................................................
..................................................................................................................................
..................................................................................................................................
..................................................................................................................................
..............................................................................................................................[6]

UCLES 2007

5014/01/O/N/07

[Turn over

234

14

BALOL
(c) World population is expected to grow from 6.5 billion people today to 9 billion by 2050.
Population growth causes economic, social and environmental problems.

Economic problems in countries in sub-Saharan Africa


Income per head in Sub-Saharan
Africa and Asia

Location

Sub-Saharan Africa

South Asia

East Asia

US dollars

4000
3000
2000
1000
Sub-Saharan
Africa

0
1970

1980

1990

2000

Changes in % of people living on


US$1 per day

50

1990

2000

50
%

40

40

30

30

20

20

10

10

40
1990

1990
2000

30
2000

20
10
0

0
Sub-Saharan
Africa

UCLES 2007

50
%

East Asia

South Asia

5014/01/O/N/07

235

15

BALOL Describe what the graphs show about income in sub-Saharan countries compared with

For
Examiners
Use

other developing countries in Asia. Use values from both graphs to support your answer.
..........................................................................................................................................
..........................................................................................................................................
..........................................................................................................................................
..........................................................................................................................................
..........................................................................................................................................
..........................................................................................................................................
..........................................................................................................................................
......................................................................................................................................[5]

UCLES 2007

5014/01/O/N/07

[Turn over

236

For
Examiners
Use

16

BALOL
(d) (i)

Percentages in sub-Saharan Africa


Total world population
% living in sub-Saharan Africa
World population suffering from hunger % living in sub-Saharan Africa
World total of health workers
% working in sub-Saharan Africa

Total world population


sub-Saharan Africa

World population
suffering from hunger

15
25
1

World total of health


workers

Key:
sub-Saharan Africa
rest of the world

Complete the two pie graphs by showing the percentages in sub-Saharan Africa for
hunger and for health workers. Fill in the key.
[3]
(ii)

What are the likely effects of these percentages on levels of disease in countries in
sub-Saharan Africa? Explain your answer.
..................................................................................................................................
..................................................................................................................................
..................................................................................................................................
..................................................................................................................................
..................................................................................................................................
..............................................................................................................................[3]

UCLES 2007

5014/01/O/N/07

237

17

BALOL
(e) The diagram below shows one example of a poverty cycle in poor countries such as

For
Examiners
Use

those in sub-Saharan Africa.

Poverty cycle
Poor farming
family

Less food
grown

No access to clean
drinking water

Too ill to work


during wet
season

Water related
diseases common

Poverty cycles are often called poverty traps. Why is it difficult for poor people to break
out of a poverty cycle like the one shown here?
..........................................................................................................................................
..........................................................................................................................................
..........................................................................................................................................
......................................................................................................................................[2]

UCLES 2007

5014/01/O/N/07

[Turn over

238

18

BALOL
(f) Aid might be one way of helping people to break out of this poverty cycle. Three types of

For
Examiners
Use

aid are listed below.


A
B
C
(i)

Food aid basic foods supplied free


Development aid money and equipment given for sinking a well
Farm aid high yielding seeds and new machines provided

Which type of aid do you consider to be the best for people in this poverty trap?
Give reasons for your choice.
..................................................................................................................................
..................................................................................................................................
..................................................................................................................................
..................................................................................................................................

(ii)

Which type of aid might be the least useful? Why?


..................................................................................................................................
..................................................................................................................................
..................................................................................................................................
..............................................................................................................................[4]

(g) One environmental problem is soil erosion. Look at the photograph.


Colca Valley in the Andes mountains of Peru

UCLES 2007

5014/01/O/N/07

239

19

BALOL (i) Describe the natural features which show that there is a high risk of soil erosion in

For
Examiners
Use

this area.
..................................................................................................................................
..................................................................................................................................
..................................................................................................................................
..................................................................................................................................
..................................................................................................................................
..............................................................................................................................[4]
(ii)

Using the photograph, describe what has already been done in the area on the
photograph to reduce the likelihood of soil erosion occurring.
..................................................................................................................................
..................................................................................................................................
..............................................................................................................................[2]

(iii)

What else might farmers in this area do to prevent soil erosion? Describe two soil
conservation strategies which could be used by farmers in this area.
..................................................................................................................................
..................................................................................................................................
..................................................................................................................................
..................................................................................................................................
..................................................................................................................................
..................................................................................................................................
..................................................................................................................................
..............................................................................................................................[4]
[Total: 40 marks]

UCLES 2007

5014/01/O/N/07

[Turn over

240

For
Examiners
Use

20
6 BALOL
(a) Look at the diagram of an oil trap.
Oil trap

NE

TO
MES

LI

OIL

Y
CLA

E
TON
S
D
AN

Y
CLA

(i)

Which type of rocks are shown in the diagram? Circle one answer.
igneous

(ii)

sedimentary

metamorphic

[1]

What was oil formed from?


..............................................................................................................................[1]

(iii)

Why is oil trapped here?


..................................................................................................................................
..................................................................................................................................
..................................................................................................................................
..................................................................................................................................
..............................................................................................................................[3]

UCLES 2007

5014/01/O/N/07

241

21

BALOL(iv) Explain the methods used by oil companies to extract oil from underground traps

For
Examiners
Use

like the one shown.


..................................................................................................................................
..................................................................................................................................
..................................................................................................................................
..................................................................................................................................
..................................................................................................................................
..............................................................................................................................[3]
(v)

State one danger for people working in oilfields.


..................................................................................................................................
..............................................................................................................................[1]

UCLES 2007

5014/01/O/N/07

[Turn over

242

22

BALOL
(b) The graph below shows world oil supply and demand in developed and developing

For
Examiners
Use

regions.
World oil supply and demand (2004)

Developed World
North America
Demand
Supply

22.7
10.3
Europe

Demand
Supply

16.3
6.4

Japan
Demand
Supply 0

5.4

Developing World
Middle East
Demand
5.3
Supply

24.5

Central & South America


Demand
6.6
Supply
10.5
Africa
Demand
Supply

2.7
9.2

Calculate the difference between supply and demand in


(i)

North America
..................................................................................................................................
..................................................................................................................................

(ii)

The Middle East


..................................................................................................................................
..............................................................................................................................[3]

UCLES 2007

5014/01/O/N/07

243

23

BALOL(iii) How important is the Middle East as a supplier of oil to other regions of the world?

For
Examiners
Use

Explain your answer.


..................................................................................................................................
..................................................................................................................................
..................................................................................................................................
..................................................................................................................................
..................................................................................................................................
..............................................................................................................................[3]
(iv)

Describe the types of environmental damage that result from transporting oil by
pipelines and tankers. Refer to an example in your answer.
..................................................................................................................................
..................................................................................................................................
..................................................................................................................................
..................................................................................................................................
..................................................................................................................................
..................................................................................................................................
..............................................................................................................................[4]

(v)

Explain why some oil spills can be cleaned up more quickly and effectively than
others.
..................................................................................................................................
..................................................................................................................................
..................................................................................................................................
..................................................................................................................................
..................................................................................................................................
..................................................................................................................................
..................................................................................................................................
..............................................................................................................................[4]

UCLES 2007

5014/01/O/N/07

[Turn over

244

24

BALOL
(c) In 2005 the US government gave the go ahead for oil exploration and extraction to start in the
Arctic National Wildlife Refuge.
Arctic National Wildlife Refuge

Kaktovik
N

Beaufort Sea

Area 1002

Prudhoe Bay
oil field

100 km

CANADA

elin

Arctic National
Wildlife Refuge

-Ala

ska

pip

ALASKA

ircle

Tra
ns

Arctic C

Fact File
Arctic National Wildlife Refuge

ALASKA
Fairbanks
Anchorage

CANADA

Established
1960
Size
7.7 m hectares
Climate and vegetation
Tundra
Inhabitants
Less than 300 people, mainly Inuit
Way of life
Hunting, fishing and whaling
Wildlife
Polar bears, caribou, musk ox,
grizzly bears, wolves, arctic foxes,
snow geese and many migratory
birds and whales
Mineral resources
Oil in Area 1002 (0.7 m hectares of
land)
Estimated oil reserves 6 bn to 16 bn
barrels

UCLES 2007

5014/01/O/N/07

245

For
Examiners
Use

25

BALOL (i) Describe the characteristics of the tundra climate and vegetation.
..................................................................................................................................
..................................................................................................................................
..................................................................................................................................
..................................................................................................................................
..............................................................................................................................[3]
(ii)

The Arctic National Wildlife Refuge was set up because it is a wilderness.


A wilderness is an area of undeveloped land which is still natural.
Describe how the map and information in the Fact File show that at present the
Arctic Refuge is a wilderness.
..................................................................................................................................
..................................................................................................................................
..................................................................................................................................
..................................................................................................................................
..................................................................................................................................
..................................................................................................................................
..............................................................................................................................[4]

UCLES 2007

5014/01/O/N/07

[Turn over

246

26

BALOL
(d) People have widely different opinions about the decision to allow oil exploration in the Arctic
National Wildlife Range.
President of the USA

Politician from the opposition party

We will get some extra oil


reserves. It will make America less
dependent on oil from overseas.

Is it worth losing a natural


treasure for ever, one of our last
great wild places, for a few months
supply of oil? A 10 bn barrel oil field
is only about six months supply of
oil for the energy-hungry USA.

Politician from Alaska


President of a Wildlife Society
Modern methods of drilling
are far less damaging to the
environment and that is a fact.
It will replace our oil imports
from the Middle East for many
years. Only a tiny part of
Alaska will be affected.

There are certain places in the


world where oil drilling and industrial
development should never be allowed.
The Arctic Refuge is one of them.
Americans should unite to protect
our countrys most beautiful places.

Inuit living in the village of Kaktovik

Im all for it. I have a young daughter and


hunting and fishing are not enough to keep her
housed, clothed and educated. I need a job now
and oil is all that weve got. I would prefer to
get a job as a tourist guide, but when they tried
eco-tourism, very few tourists came. It is too
remote and the climate is too harsh for them.

UCLES 2007

5014/01/O/N/07

247

27

BALOL (i) Describe the economic and environmental arguments made by those people who

For
Examiners
Use

support the decision to allow oil extraction in Area 1002.


Economic
..................................................................................................................................
..................................................................................................................................
..................................................................................................................................
..................................................................................................................................
..................................................................................................................................
Environmental
..................................................................................................................................
..................................................................................................................................
..................................................................................................................................
..................................................................................................................................
..............................................................................................................................[5]
(ii)

Explain your opinion about whether new oil extraction in Alaska should be allowed.
..................................................................................................................................
..................................................................................................................................
..................................................................................................................................
..................................................................................................................................
..................................................................................................................................
..................................................................................................................................
..................................................................................................................................
..................................................................................................................................
..................................................................................................................................
..................................................................................................................................
..................................................................................................................................
..............................................................................................................................[5]
[Total: 40 marks]

UCLES 2007

5014/01/O/N/07

248

5014 Environmental Management November 2007

BALOL
ENVIRONMENTAL
MANAGEMENT

Paper 5014/01
Paper 1

General comments
Questions 1 to 4 in Section A appeared to be of roughly equal difficulty. Although there were wide
variations in individual candidate performances within and between questions, no consistent pattern was
discernible to suggest that one of the questions was significantly more or less difficult than the others. For
the majority of candidates, the total mark for these questions was usually similar to the mark from one (or
even both) of the questions in Section B.
In most Centres, candidate performance in Section B was similar between Questions 5 and 6; in a few,
Question 5 was significantly better answered. Question 6 included the least well answered individual
question on this years paper, which was 6(a)(iii). Other parts of Question 6 that were not particularly well
answered by many were (a)(iv) and (b)(iii), (iv) and (v), for a variety of different reasons. As usual, the
stronger the candidate, the less noticeable was the difference in answer quality between Questions 1-4,
5 and 6. For a few candidates, another reason why their answers began to fall away towards the end of
Question 6 appeared to be mis-allocation of time. There was still the tendency, noted in previous
examinations, for candidates to write answers in Section A beyond the length expected for the low number
of marks available. Whilst it was essential to compose the written answer to a question such as 5(b)(iii) in
Section B, in order to cover and elaborate upon a sufficiently wide range of factors, all the questions in
Section A required short answers only. Begin with the answer from the first written word is the best advice
for tackling questions in Section A. Filling the lines with writing does not guarantee full question coverage,
particularly when the first two or three lines of each answer are filled with non-mark earning introductory
statements.
Candidates need be aware of the importance of the lettered sub-sections in questions. Sometimes the same
resource is being used for more than one part of the question. Usually the separate parts are examining the
same, or a closely related, topic. Most candidates appeared not to look ahead to the next part of the
question. If more had done this in 5(g), there would have been less repetition and fewer contradictory
answers between the three different parts. A significant number of candidates wrote in (g)(i) that few trees
were present in the area shown on the photograph, in order to explain the high risk of soil erosion, to be
followed by planting trees in (g)(ii) for what had already been done in this area to reduce the likelihood of
soil erosion. Tree planting reappeared in the next part, (g)(iii), which asked for what else the farmer might
do to prevent soil erosion. In Question 6, few candidates appeared to recognise that (b)(iii) followed on
naturally from both the resource used at the start of (b) for world oil supply and demand, and the calculations
asked for in (b)(i) and (ii).
Candidates again forfeited marks by not paying sufficient attention to command words. This need was clear
from some of the answers given to parts of Question 6. In 6(b)(i) and (ii), calculate the difference meant
that the final value, complete with sign if a minus value, was marked, and not any written information that
might have been inserted. Describe how, used in 6(c)(ii), required a different type of answer from
Describe used on its own in the previous part, 6(c)(i). Describe in (c)(i) required nothing more than
descriptive statements about tundra climate and vegetation; it was a simple test of knowledge. Describe
how in (c)(ii) required some use of the basic information, towards the main question theme of wilderness. A
good number of candidates found it difficult to switch from Describe to Explain when changing questions
from 6(f)(i) to (f)(ii). They carried on describing, which led to repetition of points without further elaboration.
Earlier, in Question 5(g)(i), a number of candidates made no attempt to describe anything from the
photograph. With this type of question, the answer was flawed. It is possible that they had looked at the
photograph before writing about soil erosion, but without written description which clearly showed this, no
marks could be awarded.

249

5014 Environmental Management November 2007

Comments
BALOL on individual questions
Section A
Question 1
A few candidates appeared to be unfamiliar with geothermal power, apart from knowing that it is was one of
the alternative sources of energy. This allowed them to gain marks in part (c) for renewable and clean / nonpolluting, but without many more from the other parts of the question. Steam or similar was the answer
required from the photograph in (a)(i), and not geothermal power station, which was essentially given to
candidates in the stem of the question. Wires or power lines were quite common incorrect answers in (a)(ii).
Instead of focusing on transfer to the power station in (a)(iii) and (a)(iv) as well, some candidates gave an
advantage of the valley for locating the power station itself in (iii) and disadvantages of the power station
in (iv). The underlying problem for many candidates was a failure to use the photographic evidence, which
showed far more about the pipeline transfer than power station details, such as taking up a large area on the
valley floor where the good land is located, resulting in visual pollution and a likely warming up the ground
surface around the hot water transfers. Depending on the level of candidate knowledge and understanding,
part (b) was either well answered or badly answered, with few answers in between.
Question 2
In (a)(i) the four points needed to be linked with a continuous clockwise sequence of arrows. Some
candidates failed to complete the cycle; a few others drew arrows from two directions in a pincer movement
towards intestines. R, P, and Q was the order required in (a)(ii); although this was the commonest answer, it
was by no means universal. In (a)(iii), most candidates could not go any further than very general
statements about being too expensive for countries with low GDPs, and for people who are poor. These
were rarely extended to include references to what could not be afforded, that would be useful for controlling
typhoid such as vaccination programmes and better health service provision. Answers to (b) were again
dominated by references to low cost, without exploring other advantages such as water availability in the
village, even during power cuts, and how it provides a clean water supply. Most candidates gave answers
that were either inaccurate or too narrow for the number of marks available in one or more parts of this
question.
Question 3
Part (a) proved to be deceptively awkward for those who failed to home in on environment, and concentrate
in particular on the final three precautions on the label. Some of the answers given to (b)(i) were more
appropriate to a question about the environmental effects of the over-use of fertilisers rather than pesticides,
since they were based on enrichment of water courses, the growth of algae and eutrophication. Only a
minority of candidates focused their whole answer upon pesticide over-use, such as deaths of other useful
species and interference with food chains. The full range of answer quality was evident in (b)(ii). The best
answers came from those who were able to refer to examples of natural predators, or to farming techniques
such as inter-cropping with plants repellent to the pest. Answers were often based upon references to the
development of genetically engineered crops with built-in pest resistance. Overall, there were many middle
of the range answers to this question and few that were good throughout.
Question 4
Some candidates calculated angles in (a)(i), which was not really necessary given the percentages marked
on the circle. Wise candidates realised that by plotting 50% first and 10% next in a clockwise direction, the
percentage markers allowed perfect accuracy. Not all completed their graphs with the key provided. The
short answer of greater use in the developing world was accepted in (a)(ii), although the best candidates
used the percentages in the table and stated that the total was only 13% in other more developed continents.
Social problems were largely ignored by candidates when they answered part (b). Few mentioned having to
walk further to collect wood and the adverse effects of this; most referred to the consequences of forest
removal for wildlife habitats, soils and the water cycle. Without a social reference, full marks could not be
awarded. Answers to part (c) were disappointing in general, because the typical answer included re-planting
and little else. Few references were seen to community forestry and sustainable management of existing
timber resources.

250

5014 Environmental Management November 2007

Section
BALOL B
Question 5
The majority of candidates contrived to get one of the three parts of (a) wrong. This was least likely to
happen in (a)(i); nevertheless, there were candidates who answered 1,000 without stating millions. Most
errors happened in (a)(iii), for which Asia was the commonest incorrect answer. Instead of taking note of
fastest rate of population growth in the question, candidates were over-influenced by Asias large total of
people.
In part (b)(i), most candidates chose a sensible scale and drew bars of equal width. It did not matter whether
the bars were arranged by countries or by dates, as long as this was clearly indicated. While small, careless
mistakes were occasionally made in the plotting, three mark responses were the norm. The only total
failures were from candidates who attempted to draw line graphs. Accurately drawn graphs showed that
Indias total population is expected to climb above that of China by 2050. This was the answer required for
the mark in (b)(ii), but too many candidates stopped short of stating this, and noted only increases in both
countries, or the greater increase in India. Part (b)(iii) was well answered by the many who included a good
range of relevant points beyond the simple answer of use (or not) of birth control measures. References to
examples, notably that of China under low population growth, enhanced answer quality. Relevant examples
are always credited, even when not specifically requested in the question (as here). A few candidates
successfully used the Demographic Transition Model as their framework for answering. Occasional totally
wrong answers were given, especially about reasons for economic growth in 1 and economic decline in 2.
Answers arranged as two lists of reasons rarely gained more than half marks, mainly because they suffered
from lack of or limited elaboration beyond the reason heading. Typical answers were of the type no birth
control in 1 and birth control in 2.
Marks in part (c) were given for both general comparative description and for the use of values. It was
impossible to gain full without using values as illustrations. Some candidates limited their answers to one
descriptive point from the line graph and one from the bar graphs. Often they filled all the lines without
mention of a value and most gained less than half the available marks. It became clear that some, mainly
weaker, candidates, did not understand what living on US$1 per day meant and assumed that there had
been increases in dollar income in sub-Saharan Africa and decreases in Asia. For the many who used the
correct technique of answering, namely description followed by use of equivalent values for sub-Saharan
Africa and Asia, this was one of most straightforward questions on the paper.
To most candidates the type of practical task in part (d)(i) posed no problems; there were many full mark
answers. Some began by calculating the number of degrees, but for plots of 25% and 1% this was not really
necessary. For the pie graphs to be fully complete, shading in the key was needed to separate out subSaharan Africa from the rest of the world, consistently. It was surprising to find several examples of
candidates using the type of shading given for rest of the world in the key in the one per cent segment on the
graph of world total of health workers. Others ignored the instruction to fill in the key. Part (d)(ii) was less
well answered than might have been expected. One unforeseen problem was the misinterpretation of health
workers for healthy workers, leading to frequent comments along the lines that only one per cent of workers
in sub-Saharan Africa were healthy enough to work. This made answering the question set much more
difficult. The best answers came from those candidates who recognised the link between hunger and lower
resistance to disease. Without adequate health services, young and old were at greatest risk.
Although weak candidates stayed too close to the information provided in the boxes in part (e), the majority
wrote enough to show that they understood that the diagram showed a repetitive, self-perpetuating cycle,
with poverty as its root cause. Greater candidate understanding was often demonstrated by references to
the need for aid, investment or help from outside in order to break the cycle. This question was intended to
lead candidates into part (f) about types of aid. Since the question referred to this poverty cycle,
Development Aid was regarded as the best choice in (f)(i) and Food Aid was considered to be the least
useful type of aid in (f)(ii). In both cases, Farm Aid was regarded as the middle choice, capable of being
used either positively or negatively for the question theme. Good choices made for easier justifications, and
many were awarded full marks. However, it was difficult for candidates to justify the reverse choices of Food
Aid in (i) and Development Aid in (ii), to produce answers worth more than one mark. A wide variety of
reasons were used by candidates, which were rewarded provided that they remained faithful to the poverty
cycle shown in part (e).

251

5014 Environmental Management November 2007

Part
(g) elicited a full and varied range of answers from candidates. In (g)(i) an essential pre-requisite for the
BALOL
award of marks was some reference to natural features which could be seen on the photograph, such as
steep slopes, bare surfaces and limited natural vegetation cover. Only then were candidates rewarded for
further elaboration about the contribution of these to a higher risk of soil erosion in this area. No description
from the photograph meant no marks. Fortunately only a minority of answers were general answers on soil
erosion, of this type. Some descriptions were better than others, for example, mountainous area suggested
that the candidate had looked at the photograph, but it could have been obtained solely from the caption,
and it was not as precise and useful a description as steep slopes. There were many answers based upon
only one observed natural feature. In (g)(ii) candidates who began from observation of the prominent
staircase of terraces fared best. Although some struggled to find the term terraces, alternative ways of
describing terrace were also accepted. Less successful were answers from candidates who suggested that
tree planting had taken place, especially if they had included comments about the lack of vegetation cover in
the previous part. Answers to (g)(iii) were inconsistent in quality and depth. Instead of referring to two of the
strategies named in the syllabus (tree planting, contour ploughing, dry land farming and windbreaks), quite a
number of candidates concentrated more on use of fertilisers and irrigation water. These agricultural
techniques are more significant for increasing farm output than preventing soil erosion. The more successful
answers tended to include references to tree planting, accompanied by one from contour ploughing or dry
land farming.
Question 5 covered familiar topics (especially population growth and soil erosion) and included questions
which required the use of practical skills. Together these probably explain why a majority of candidates (and
in certain Centres a very dominant majority) gained a higher mark than in Question 6.
Question 6
While the most popular answer, sedimentary, was also the correct answer in part (a)(i), plenty of candidates
were attracted to the other two rock types. In addition, some circled two of them, while others missed the
question out. Sedimentary rock was a common wrong answer to (a)(ii). Other answers frequently seen,
which were not credited, were fossil fuels and fossil by itself. Only a minority of candidates seemed aware
that oil was made from the decomposition of plants and small creatures. Part (a)(iii) was answered even
less well. Some left it unattempted while others showed themselves to be impossibly muddled between the
characteristics of permeable and impermeable rocks. A significant proportion of candidates believed that it
was the layer of sandstone rock which was trapping the oil and preventing it from moving. Under the
circumstances, it was a real pleasure to read the answers of candidates who had clear understanding of the
roles of sandstone in providing the storage spaces and clay in keeping the oil inside the sandstone layer.
Few mentioned the importance of rock structure for forming the trap; indeed more references to anticline
were noticed in the next part of the question, suggesting that candidates knew the name but could not
recognise it as one common type of oil trap. Answers to part (iv) were either totally incorrect or effective.
Few fell between these two extremes. Many candidates referred either to opencast or deep shaft mining, or
to both, as if oil were a solid instead of a liquid. Fortunately there were others, who understood that drilling
with pipes from the surface was the usual method. Oil is forced out to the surface by pressure or pumping.
Success in part (iv) meant that candidates were more likely to choose acceptable answers such as fire or
explosion in (v), rather than unacceptable answers such as breathing problems and tunnel collapses with
mining methods.
In response to the command word calculate in (b)(i) and (ii), it was the final answers that were marked
(12.4 and 19.2 respectively). A few candidates wrote about the differences without doing a calculation; a
small number of others made one careless error. For the final mark, the candidate needed to make clear
that one of the values was negative and the other positive. Quite a large number failed to claim this mark.
This question was intended to give candidates a start with their answers to part (iii). Unfortunately a lot of
candidates never looked at the pictograph again. If they had, they would have been able to offer answers
that were more precise, by referring to the size of oil surpluses in the Middle East, matched by equally large
oil deficits in the developed world. Depth and quality in part (iv) depended heavily on reference to an
example. The Exxon Valdez disaster in Alaska was the most popular (and generally successful) choice.
However, it was good to see candidates making use of local examples from Argentina and the Gulf. In part
(v) candidates frequently referred to factors which affected the likelihood or otherwise of an effective cleanup, but without bending the content towards the main theme of the question. Thus location (land or sea,
coastal or offshore), preparedness (developed or developing countries) and size of spill were regularly
mentioned, but not always used in a way that took the answer above half marks.

252

5014 Environmental Management November 2007

In
(c)(i) references to climate were more consistently accurate than were those to vegetation. Reference to
BALOL
coldness was the starting point for most candidates. Little precipitation, or most falling as snow, tended to
claim a second mark more often than use of temperature values. Perhaps as much as half the candidature
believed that tundra lands were covered by coniferous forests, in what was obviously confusion with the taiga
to the south. No trees is one of the defining characteristics of the tundra biome. Candidates were better at
selecting and stating relevant pieces of information unaltered than actually describing how they show that
the area is still a wilderness when answering (c)(ii).
Candidates had few problems in answering (d)(i). Plenty of help was available to them in the source
material provided. The economic needed to be separated out from the environmental, which not all
managed to do. However, some significant losses of marks were caused by candidates allowing their proenvironmental opinions to take over too early. The wording of this part of question (d) did not give the option
to talk about not-allowing more oil extraction in Alaska. Claiming all the marks in part (d)(ii) was more of a
challenge. In expressing an opinion, some candidates sat on the fence, which usually meant that they had
little to add to what they had already written in the previous part. The command word in part (ii) was explain
but too many continued to describe more than explain and restricted themselves to less than half marks.
This question gave more able candidates the opportunity for demonstrating understanding of the broader
issues associated with developing a highly prized world commodity in a beautiful and fragile environment. It
was not the candidates own opinion which mattered, but the quality of the explanatory comment.
Only those candidates with a firm knowledge of oil traps and oil extraction were able to maintain quality and
consistency of performance throughout Question 6. For other candidates, the number of marks increased in
each part of the question, being lowest in part (a) and highest in part (d).

253

BALOL
UNIVERSITY OF CAMBRIDGE INTERNATIONAL EXAMINATIONS
General Certificate of Education Ordinary Level

*7287040288*

5014/01

ENVIRONMENTAL MANAGEMENT
Paper 1

May/June 2008
2 hours 15 minutes

Candidates answer on the Question Paper.


Additional Materials:

Ruler

READ THESE INSTRUCTIONS FIRST


Write your Centre number, candidate number and name on all the work you hand in.
Write in dark blue or black pen.
You may use a soft pencil for any diagrams, graphs or rough working.
Do not use staples, paper clips, highlighters, glue or correction fluid.
DO NOT WRITE IN ANY BARCODES.
Answer all questions.
All questions in Section A carry 10 marks.
Both questions in Section B carry 40 marks.
At the end of the examination, fasten all your work securely together.
The number of marks is given in brackets [ ] at the end of each question or part question.

For Examiners Use


1
2
3
4
5
6
Total
This document consists of 23 printed pages and 1 blank page.
SPA (KN)
UCLES 2008

[Turn o

254

BALOL

Section A

For
Examiners
Use

(a) In September 2006 a newspaper report stated:

PROTESTS IN BANGLADESH OVER LARGE


OPENCAST MINE PLAN
The people protesting claim that the opencast mine will displace 40 000
villagers from their homes and land and ruin their livelihoods just for 1 000
new jobs. They believe that wells within 6 km of the mine will dry up and
rivers will be polluted. The mining company promises these advantages:
new jobs, foreign exchange, tax revenues for the government and new
infrastructure.
State one social disadvantage and two economic advantages that will result from the
opening of this mine. Explain each.
social disadvantage .........................................................................................................
explanation ......................................................................................................................
..........................................................................................................................................
..........................................................................................................................................
economic advantage 1 .....................................................................................................
explanation ......................................................................................................................
..........................................................................................................................................
..........................................................................................................................................
economic advantage 2 .....................................................................................................
explanation ......................................................................................................................
..........................................................................................................................................
......................................................................................................................................[6]
(b) After mining has finished in an area, how can the damaged environment be improved?
..........................................................................................................................................
..........................................................................................................................................
..........................................................................................................................................
..........................................................................................................................................
..........................................................................................................................................
......................................................................................................................................[4]
UCLES 2008

5014/01/M/J/08

255

For
Examiners
Use

3
2 BALOL
(a) The diagram shows a water cycle.

rain

soil
A
C
rock

(i)

B
soil

sea

Name the processes shown by the arrows.


A ...............................................................................................................................
B ...............................................................................................................................
C ...........................................................................................................................[3]

(ii)

Explain how water from the sea may return to the sea.
..................................................................................................................................
..................................................................................................................................
..................................................................................................................................
..................................................................................................................................
..................................................................................................................................
..............................................................................................................................[4]

(b) Suggest why some rural areas of the developing world are suffering from a shortage of
groundwater from wells and boreholes.
..........................................................................................................................................
..........................................................................................................................................
..........................................................................................................................................
..........................................................................................................................................
......................................................................................................................................[3]
UCLES 2008

5014/01/M/J/08

[Turn over

256

4
3 BALOL
(a) Look at the graph showing information about lead emissions in the UK.
0.6

For
Examiners
Use

Key

Annual average
(micrograms of lead/m3)

average lead emissions in urban areas


0.5

average lead emissions in rural areas

0.4
0.3
0.2
0.1
0.0
1980 1982 1984 1986 1988 1990 1992 1994 1996 1998 2000 2002 2003
Year

(i)

Describe the main trends in lead emissions in urban areas shown in the graph
between 1980 and 2003.
..................................................................................................................................
..................................................................................................................................
..................................................................................................................................
..............................................................................................................................[3]

(ii)

Suggest reasons for the trends shown in urban areas.


..................................................................................................................................
..................................................................................................................................
..................................................................................................................................
..................................................................................................................................
..............................................................................................................................[3]

(iii)

Suggest why the amount of lead emitted in rural areas is different from that in urban
areas.
..................................................................................................................................
..................................................................................................................................
..............................................................................................................................[2]

UCLES 2008

5014/01/M/J/08

257

For
Examiners
Use

BALOL
(b) Why is reducing lead emissions important?
..........................................................................................................................................
..........................................................................................................................................
......................................................................................................................................[2]

UCLES 2008

5014/01/M/J/08

[Turn over

258

6
4 BALOL
(a) The photograph shows a deforested area that is now used for grazing sheep. Some years
ago, a small part of it was enclosed by fences to investigate vegetational succession.

(i)

For
Examiners
Use

Look at the photograph. What evidence is there that vegetational succession is


taking place inside the fence?
..................................................................................................................................
..................................................................................................................................
..............................................................................................................................[2]

(ii)

What type of plant cover is likely if the succession continues to its climax?
..............................................................................................................................[1]

(iii)

Why does animal grazing stop vegetational succession?


..................................................................................................................................
..............................................................................................................................[1]

UCLES 2008

5014/01/M/J/08

259

For
Examiners
Use

BALOL
(b) How does a vegetational succession improve the soil over time?
..........................................................................................................................................
..........................................................................................................................................
..........................................................................................................................................
..........................................................................................................................................
......................................................................................................................................[3]
(c) The area shown in the photograph has high rainfall throughout the year. The present
ecosystem is poorer than that of the original forest. Why?
..........................................................................................................................................
..........................................................................................................................................
..........................................................................................................................................
..........................................................................................................................................
......................................................................................................................................[3]

UCLES 2008

5014/01/M/J/08

[Turn over

260

For
Examiners
Use

BALOL

Section B

(a) Look at the world map showing the main and other important ocean fishing areas.
World Ocean Fisheries

ATLANTIC
OCEAN

PACIFIC
OCEAN

Equator
PACIFIC
OCEAN

INDIAN
OCEAN

KEY
Main fishing areas
other important
fishing areas

(i)

Describe where the main fishing areas are located.


..................................................................................................................................
..................................................................................................................................
..................................................................................................................................
..................................................................................................................................
..............................................................................................................................[3]

(ii)

State one similarity and one difference between the locations of the main and other
important fishing areas.
Similarity ...................................................................................................................
..................................................................................................................................
Difference .................................................................................................................
..............................................................................................................................[2]

UCLES 2008

5014/01/M/J/08

261

For
Examiners
Use

BALOL
(b) Three factors to explain the locations of major ocean fisheries are listed below.

wide continental shelf

presence of ocean currents

densely populated coastal areas nearby

(i)

How many of these factors are natural (physical) factors?


..............................................................................................................................[1]

(ii)

Choose two of the factors. For each one, explain its importance for major ocean
fisheries.
Name of factor ..........................................................................................................
..................................................................................................................................
..................................................................................................................................
..................................................................................................................................
Name of factor ..........................................................................................................
..................................................................................................................................
..................................................................................................................................
..............................................................................................................................[4]

UCLES 2008

5014/01/M/J/08

[Turn over

262

10

BALOL
(c) Look at the graph showing total world ocean fish catches.

For
Examiners
Use

World Ocean Fish Catches


Million tonnes
90
80
70
60
50
40
30
20
10
0
1970
1980

(i)

1990

2000

State the amount of fish caught in 1970, 1987 and 2000.


..................................................................................................................................
..............................................................................................................................[2]

(ii)

Describe the evidence from the graph which suggests overfishing.


..................................................................................................................................
..................................................................................................................................
..................................................................................................................................
..................................................................................................................................
..............................................................................................................................[2]

UCLES 2008

5014/01/M/J/08

263

For
Examiners
Use

11

BALOL
(d) Some of the human causes of overfishing are shown in the spider diagram.
Larger size
fishing boats

High technology for


locating shoals of fish

Human causes
of overfishing

Bigger nets
in use
(i)

Refrigerated
factory ships

Briefly describe how each one contributes to overfishing.


..................................................................................................................................
..................................................................................................................................
..................................................................................................................................
..................................................................................................................................
..................................................................................................................................
..................................................................................................................................
..................................................................................................................................
..............................................................................................................................[4]

(ii)

In your view, which one is contributing most to overfishing? Explain why.


..................................................................................................................................
..................................................................................................................................
..................................................................................................................................
..............................................................................................................................[2]

UCLES 2008

5014/01/M/J/08

[Turn over

264

12

BALOL
(e) In some ocean fishing areas, fish catches are reduced by natural causes. This happens

For
Examiners
Use

off the coast of Peru in El Nino years.

Normal year

Andes
Dry
sinking
air
Desert
Peru

Cold sea surface


Cold Peru current

Plankton
Up-welling

Cold water

El Nino year

Rain

Andes

Peru

surface current

of warm water w oxygen and nutrients


Lo

Peru current
Cold water

Describe the differences between normal and El Nino years for


(i)

ocean currents
..................................................................................................................................
..................................................................................................................................

(ii)

warm and cold water


..................................................................................................................................
..............................................................................................................................[2]

UCLES 2008

5014/01/M/J/08

265

For
Examiners
Use

13

BALOL(iii) Explain why there are fewer fish off the coast of Peru in El Nino years.
..................................................................................................................................
..................................................................................................................................
..................................................................................................................................
..............................................................................................................................[2]

UCLES 2008

5014/01/M/J/08

[Turn over

266

14

BALOL
(f) Look at the graph for fish catches in Peru and the graph for El Nino years.

For
Examiners
Use

Total fish catches in Peru


11
10
9
8
7
million
tonnes

6
5
4
3
2
1
0

1980 1981 1982 1983 1984 1985 1986 1987 1988 1989 1990 1991 1992 1993 1994 1995 1996 1997 1998 1999 2000

The record of El Nino years in Peru

Strong

Weak
1980 1981 1982 1983 1984 1985 1986 1987 1988 1989 1990 1991 1992 1993 1994 1995 1996 1997 1998 1999 2000

(i)

State the three strong El Nino years.


..............................................................................................................................[1]

UCLES 2008

5014/01/M/J/08

267

15

BALOL(ii) Do El Nino years have an effect on the size of fish catches in Peru? Support your

For
Examiners
Use

answer with evidence from the graphs.


..................................................................................................................................
..................................................................................................................................
..................................................................................................................................
..................................................................................................................................
..................................................................................................................................
..................................................................................................................................
..............................................................................................................................[4]
(iii)

Is there any evidence, from the graph of fish catches, for overfishing in Peru during
the 1990s? Explain your answer.
..................................................................................................................................
..................................................................................................................................
..................................................................................................................................
..............................................................................................................................[2]

(g) (i)

Draw another spider diagram to show three strategies for the sustainable harvesting
of ocean fisheries and reducing overfishing.

[4]
UCLES 2008

5014/01/M/J/08

[Turn over

268

16

BALOL(ii) Are these strategies for the sustainable harvesting of ocean fisheries easy or

For
Examiners
Use

difficult to carry out? Explain your views on this.


..................................................................................................................................
..................................................................................................................................
..................................................................................................................................
..................................................................................................................................
..................................................................................................................................
..................................................................................................................................
..................................................................................................................................
..................................................................................................................................
..................................................................................................................................
..............................................................................................................................[5]
[Total: 40]

UCLES 2008

5014/01/M/J/08

269

For
Examiners
Use

17
6 BALOL
(a) Look at the photo of a weather station.

(i)

State why this is a good place to site a weather station.


..................................................................................................................................
..................................................................................................................................
..................................................................................................................................
..............................................................................................................................[2]

(ii)

Give one reason why a fence is usually built around weather stations.
..................................................................................................................................
..............................................................................................................................[1]

(iii)

Name a weather instrument that will be placed inside the white wooden box.
..............................................................................................................................[1]

UCLES 2008

5014/01/M/J/08

[Turn over

270

18

BALOL(iv) Draw a labelled diagram to show how wind speed is measured in a weather

For
Examiners
Use

station.

[3]
(b) Look at the table of climate data for two weather stations in Africa north of the Equator.
Station A
Jan

Feb

Mar

Apr

May

Jun

Jul

Aug

Sep

Oct

Nov

Dec

Temperature
(C)

16

19

23

28

31

34

34

34

33

30

24

19

Precipitation
(mm)

Jan

Feb

Mar

Apr

May

Jun

Jul

Aug

Sep

Oct

Nov

Dec

Temperature
(C)

22

24

28

31

29

29

27

25

26

27

25

21

Precipitation
(mm)

10

69

117

206

310

142

13

Station B

(i)

Complete the table below for Station A.


Station A

Station B

Highest monthly temperature (C)

31

Lowest monthly temperature (C)

21

Annual range of temperature (C)

10
wet and dry
season

Precipitation during the year

[3]

UCLES 2008

5014/01/M/J/08

271

For
Examiners
Use

19

BALOL(ii) When is the wet season at Station B?


..............................................................................................................................[1]
(iii)

Major climatic types in Africa


Equatorial

Savanna

Desert

Name the type of climate at Station A and at Station B.


Station A ...................................................................................................................
Station B ...............................................................................................................[1]
(iv)

State one reason for each of the choices made in part (iii).
Station A ...................................................................................................................
..................................................................................................................................
Station B ...................................................................................................................
..............................................................................................................................[2]

UCLES 2008

5014/01/M/J/08

[Turn over

272

20

BALOL
(c) Many parts of Africa are too dry for cultivation for part or all of the year. Four options to

For
Examiners
Use

allow farming in dry areas are listed below.


1
2
3
4

Practise extensive livestock farming


Plant new varieties of crops
Government builds large dams, reservoirs and irrigation canals
Use underground water supplies for trickle drip irrigation

(i)

For three of the options, state one advantage and one disadvantage of each for
farming in dry areas.
Option number ............
..................................................................................................................................
..................................................................................................................................
..................................................................................................................................
..................................................................................................................................
Option number ............
..................................................................................................................................
..................................................................................................................................
..................................................................................................................................
..................................................................................................................................
Option number ............
..................................................................................................................................
..................................................................................................................................
..................................................................................................................................
..............................................................................................................................[6]

(ii)

In your view, which one of the four options is the most sustainable for farming in dry
areas? Explain your choice.
..................................................................................................................................
..................................................................................................................................
..................................................................................................................................
..................................................................................................................................
..................................................................................................................................
..............................................................................................................................[3]

UCLES 2008

5014/01/M/J/08

273

For
Examiners
Use

21

BALOL
(d) (i) Name one area where desertification is a major problem.
..............................................................................................................................[1]
(ii)

There are both physical and human causes of desertification.


Explain this statement.
..................................................................................................................................
..................................................................................................................................
..................................................................................................................................
..................................................................................................................................
..................................................................................................................................
..................................................................................................................................
..............................................................................................................................[3]

(iii)

Which is more important as a cause of desertification human or physical factors?


Explain your view.
..................................................................................................................................
..................................................................................................................................
..................................................................................................................................
..............................................................................................................................[2]

(e) In big cities located in dry climates, such as Los Angeles, atmospheric pollution is a big
problem.
(i)

State two reasons why atmospheric pollution from traffic and industry is often
worse in places with a dry climate.
..................................................................................................................................
..................................................................................................................................
..................................................................................................................................
..............................................................................................................................[2]

UCLES 2008

5014/01/M/J/08

[Turn over

274

For
Examiners
Use

22

BALOL(ii)

Los Angeles

Number of days with atmospheric pollution worse than recommended health standards
Year

Number of
days

1975

210

1980

170

1985

160

1990

130

1995

95

2000

45

In the space below, draw a graph to show these values.

[4]

UCLES 2008

5014/01/M/J/08

275

For
Examiners
Use

23

BALOL(iii) Describe the trend from 1975 to 2000.


..............................................................................................................................[1]
(iv)

Where are strategies for reducing traffic emissions likely to be more effective in
developed world cities like Los Angeles, or in developing world cities like Cairo and
Beijing?
Explain your answer as fully as you can.
..................................................................................................................................
..................................................................................................................................
..................................................................................................................................
..................................................................................................................................
..................................................................................................................................
..................................................................................................................................
..................................................................................................................................
..................................................................................................................................
..............................................................................................................................[4]
[Total: 40]

UCLES 2008

5014/01/M/J/08

276

BALOL
UNIVERSITY OF CAMBRIDGE INTERNATIONAL EXAMINATIONS
GCE Ordinary Level

MARK SCHEME for the May/June 2008 question paper

5014 ENVIRONMENTAL MANAGEMENT


5014/01

Paper 1, maximum raw mark 120

This mark scheme is published as an aid to teachers and candidates, to indicate the requirements of
the examination. It shows the basis on which Examiners were instructed to award marks. It does not
indicate the details of the discussions that took place at an Examiners meeting before marking began.
All Examiners are instructed that alternative correct answers and unexpected approaches in
candidates scripts must be given marks that fairly reflect the relevant knowledge and skills
demonstrated.
Mark schemes must be read in conjunction with the question papers and the report on the
examination.

CIE will not enter into discussions or correspondence in connection with these mark schemes.

CIE is publishing the mark schemes for the May/June 2008 question papers for most IGCSE, GCE
Advanced Level and Advanced Subsidiary Level syllabuses and some Ordinary Level syllabuses.

277

Page 2

Mark Scheme
GCE O LEVEL May/June 2008

Syllabus
5014

Paper
01

BALOL
Notes on application of the mark scheme for section A
each line represents one mark.
ideas in brackets are not essential to the answer but anything underlined is
reward any equivalent way of expressing the ideas in the mark scheme
1

(a) Social disadvantage


* villagers will have to leave their homes/lose their livelihoods = 1d
disruption of communities/poverty if cannot replace their livelihoods
* illness/disease from drinking polluted water = 1 reason
Economic advantages
* better jobs/jobs for 1 000
because more skilled work (than agriculture)/more income for worker
* foreign exchange
because coal exported/will help pay for imports/other development
* tax revenues
for government as company pays taxes/ any specific economic benefit of it
* new infrastructure/ e.g. of
needed for access to/from mine/encourages more industry/economic development
social 1d + 1 associated reason = 2
economic 2* d points + 1 associated reason for each = 4

[6]

(b) restoration to what land was like before


removal of waste heaps
replacement of overburden
replacement of soil
replacement of vegetation/landscaping (allow the term once only)
original shape of land created/landscaping
hole filled in/land filling
removal of sediment from streams/clean up

[4]

(a) (i) A interception (allow transpiration/evaporation if given)


B run-off
C through flow/groundwater flow

[3]

(ii) * water evaporated from the sea = 1


condensation (produces cloud/water droplets) water droplets join to form raindrops
drops fall when heavy enough
run-off in rivers/over surface to the sea
gravity
infiltrates through soils/percolates through rocks to sea
through pore spaces/cracks = 3
max. 2 of the 3 if water is not stated to reach the sea by at least one of run-off or
infiltration
*point + 3
[4]
UCLES 2008

278

Page 3

Mark Scheme
GCE O LEVEL May/June 2008

Syllabus
5014

Paper
01

BALOL

(b) increasing population uses greater amounts of water


more taken out than can be replenished by rainfall
more droughts/less rain
global warming/higher temperatures cause more evaporation
therefore less water infiltrates to become groundwater
extraction for mines/industries/agriculture etc
avp

[3]

(a) (i) overall fall


erratic at first/until 1986
steep fall to 1986
gradual fall from 1986
only one significant fluctuation since 1986

[3]

(ii) legislation
reduction in the amount of lead in petrol (until none)
uptake of unleaded petrol
materials substituted for lead in industry
e.g. plastic/copper/tin etc
lead in paint much reduced
avp

[3]

(iii) fewer vehicles


fewer industries
avp

[2]

(b) lead is very toxic


lead does not break down
adversely affects human health/renal disease/sterility
reduces intelligence
avp

[2]

(a) (i) taller (than outside fencing)


bushes/shrubs
(small) trees/saplings
(greater) variety of vegetation (than outside fencing)

[2]

(ii) trees/forest/woodland

[1]

(iii) eat the new shoots/young plants and leaves/pull out roots

[1]

(b) adds humus/organic matter/plant material


increases nutrients/more plant foods/increased fertility
increases cohesion/stability/reduces soil erosion susceptibility
deepens the soil
increases water holding ability
increases infiltration/interception/water take up therefore reduces soil erosion
roots prevent soil erosion by holding the soil

[3]

UCLES 2008

279

Page 4

Mark Scheme
GCE O LEVEL May/June 2008

Syllabus
5014

Paper
01

BALOL
(c) decreased interception so more nutrient/soil loss in run-off
no/few roots to bind the soil so more soil erosion
more water into the soil so more leaching of nutrients
soil may become too shallow for trees to grow again
soil nutrients may reduce so that trees cannot grow
less biodiversity of vegetation
loss of habitats
therefore less biodiversity of animal/bird life
changed food chain (some development needed)
avp
5

[3]

(a) (i) Most are north of Equator/northern hemisphere,


only one south of the Equator,
in temperate latitudes,
both on western and eastern sides of the continents,
large areas enclose islands (e.g. Japan, British Isles, Iceland).
Three descriptive points such as these

[3]

(ii) Similarity
Both fringe the coastlines/in between and around islands
Difference
More of the other important areas south of the Equator/more common in the Indian
Ocean/different hemisphere/developed compared with developing
2 @ 1 mark

[2]

(b) (i) Two

[1]

(ii) Wide continental shelf;


shallow seas (under 200m deep) next to the land masses,
more light, minerals, plankton and other food supplies for fish,
further explanation of how these favour fish life,
easy access from populated land area,
significance of wide producing a large area for fish/fishing.
Presence of ocean currents;
bring fresh supplies of nutrients,
plankton rich (especially cold ocean currents),
where currents upwell to surface (e.g. Peru)/where currents meet (e.g. Newfoundland),
named examples of currents.
Nearby densely populated coastal areas;
fish are important food/protein source,
close to harbours for fishing boats to operate out of,
quick/easy to reach fishing grounds/take catch back to shore,
fish reach the market fresh (refrigerated ships not needed).
Likely 2 + 2, but allow 3 marks and 1 mark

[4]

UCLES 2008

280

Page 5

Mark Scheme
GCE O LEVEL May/June 2008

Syllabus
5014

Paper
01

BALOL
(c) (i) 1970 50; 1987 80; 2000 72 million tonnes
(allow +/ 1 for all values)
All three correct = 2 marks
Two correct = 1 mark

[2]

(ii) Steady rise in world catches until the peak in 1987,


noticeable dip downwards in early 1990s / around 1993,
catches at lower levels (closer to 70m tonnes) ever since,
comment about how overfishing is suggested
Two points along these lines which show understanding

[2]

(d) (i) Larger size fishing boats fish further from shore/in more difficult locations or weather
conditions, hold more before they need to return to shore, economies of scale favouring
use of more sophisticated equipment
High technology for locating shoals of fish mention of an example of the technology
such as radar and sonar, or technology in terms of operational capabilities of boats
Bigger nets in use trap larger shoals/numbers of fish, non-selective which means that
immature fish are caught as well
Refrigerated factory ships allow fishing much further from the shores/out in the oceans,
some processing at sea means that enormous numbers of fish need to be caught before
it returns to port
4 @ 1 mark

[4]

(ii) No mark for choice any choice is valid. The two marks are for explanation,
emphasising such as enormous numbers caught, finding and taking shoals of fish in the
oceans well beyond the traditional fishing grounds, non-selective taking young and
immature so that future stocks are affected. Contrast could be made with fishing
practices using traditional methods/small boats.
[2]
(e) (i) Ocean currents cold Peru current no longer up-wells to flow on the surface/cold Peru
current in an El Nino year flows below warm surface water.
Mention of the Peru current is essential for the mark here.
(ii) Warm and cold water all cold sea water both at the surface and in the ocean deeps is
replaced by a layer of warm water on the sea surface.
Sufficient reference to this might have been covered in the first part for the award of the
mark.
[2]
(iii) Plankton is a food supply for fish,
Peru current brings it to the surface where most fish live in normal years,
warm water in El Nino years is low in oxygen and nutrients.
Understood = 2 marks
Some understanding/selection of suitable content from diagrams without full or effective
use = 1 mark
[2]

UCLES 2008

281

Page 6

Mark Scheme
GCE O LEVEL May/June 2008

Syllabus
5014

Paper
01

BALOL
(f) (i) 1982, 1983 and 1998

[1]

(ii) Evidence to support Yes


Lowest catches of all in 1983 and 1994 which were both El Nino years,
in 1983 size of catch was half that of the previous and next years,
more noticeable drop in catches in 1994/only about 15% of years around it,
big drop in 1998 coinciding with quite a strong El Nino
Some weak evidence for No
1982/1990 El Nino event did not seem to have any effect on catches.
Entirely Yes answers are the most likely and easiest to justify.
Points along the lines of the ones listed; reward comment towards the focus of the
question. An answer is unlikely to be worth two or more marks without at least one
specific reference to values and dates.
[4]
(iii) The trend in fish catches is upwards throughout the 20 years,
in 2000 was the highest ever catch/between 34 times more than in 1980, variations in
catch sizes is greater in the 1990s than 1980s,
El Nino year of 1994 resulted in the lowest catch/greatest fall in catch size,
However, fish catches go up and down anyway/everywhere because of year to year
changes in natural conditions,
the record 1994 level was soon recovered in subsequent years,
fact that more fish are being caught could be an indication of the great size of fish stocks
here,
that the natural resource had been under-exploited in previous years,
would need significantly lower values in later (especially normal years) for conclusive
evidence for overfishing.
Credit points made along these lines.
Both marks can be gained from For or No evidence' answers, provided that they are
supported by specific information from the graph of fish catches.
[2]
(g) (i) Strategies for sustainable harvesting of ocean fisheries include;
quotas; closed seasons; conservation laws; conservation controls/monitoring; net types;
net sizes; territoriality
Three labels = 3 marks
Sustainable harvesting of fisheries' (or similar) written in centre of drawn spider diagram
= 1 mark
[4]
(ii) Problems for implementation of strategies include;
economic costs to boat owners, fishermen and economy of countries,
increasing demand for fish for animal feed (not just human food),
difficulty of monitoring/patrolling fishing grounds,
problems in international waters with boats from many countries,
difficulties of reaching international agreements and abiding by them,
some fishing stocks may have already fallen below safe biological limits e.g. cod in many
parts of the North Sea
Easy? general point that sustainable strategies are rarely easy to implement because
they involve costs for humans; only easy where fish stocks remain a plentiful natural
resource.

UCLES 2008

282

Page 7

Mark Scheme
GCE O LEVEL May/June 2008

Syllabus
5014

Paper
01

BALOL
Points made that are relevant to the question up to 4 marks.
Fifth mark reserved for an explanatory or concluding comment which answers the
question posed.
[5]
6

(a) (i) open place/in the countryside,


well away from the trees,
no sign of any buildings,
nothing to block sun, wind, rain,
soft surface
Two points made along these lines

[2]

(ii) Protect the instruments from animals/people,


ensure that they are not disturbed to increase chance of taking accurate readings

[1]

(iii) Thermometer or name of an individual type of thermometer

[1]

(iv) Title anemometer,


long pole,
three cups at the top,
cups facing in different directions,
meter below counting number of rotations
Three labels on the diagram 3 @ 1 mark
Maximum 1 mark for the name without a diagram or with a totally or largely incorrect
diagram
[3]
(b) (i) Highest temperature 34 and lowest 16 = 1 mark
Annual range 18 = 1 mark
Precipitation none, dry all year or desert = 1 mark

[3]

(ii) May/June to September or in summer

[1]

(iii) Station A Desert


Station B Savanna
Both needed for the mark

[1]

(iv) Station A No rain


Station B One wet season and one dry so that it is not Equatorial i.e. this given
information needs to be explained for the choice. An alternative (less likely) answer is
that temperature is highest before the wet season begins and not in mid-summer.
2 @ 1 mark

[2]

UCLES 2008

283

Page 8

Mark Scheme
GCE O LEVEL May/June 2008

Syllabus
5014

Paper
01

BALOL
(c) (i) Option 1 Extensive livestock farming
Advantage

long established/traditional method of farming,


depends on animals adapted to conditions such as camels

Disadvantage

dangers from overgrazing especially from goats and sheep,


low productivity/can only support a few people

Option 2 New varieties of crops


Advantage

drought resistant so that they survive the natural conditions,


seeds genetically developed to allow cultivation where not otherwise
possible

Disadvantage

may require greater inputs/higher level of technology for success,


expensive purchases for poor people/go into debt

Option 3 Large dams etc.


Advantage

enable more land to be cultivated/more than one crop a year, most


likely option to lead to high and reliable crop output

Disadvantage

water costly to buy/expensive to supply isolated places,


many more environmental problems from dam construction/use of
large amounts of irrigation water e.g. salinisation

Option 4 Underground supplies for trickle drip irrigation


Advantage

does not waste water/uses less than from dams etc.,


much less risk of environmental problems like salinisation

Disadvantage

underground supplies of water not available everywhere/may be


salty
only small areas likely to be cultivated

Essentially 2 marks per option, one for advantage and one for disadvantage
3 @ 2 marks
If maximum is not reached, allow one mark to be used as a floating mark to reward any
part that was particularly well answered.
[6]

UCLES 2008

284

Page 9

Mark Scheme
GCE O LEVEL May/June 2008

Syllabus
5014

Paper
01

BALOL
(ii) No marks for choice; a good choice makes for easier/more detailed explanation.
Award marks for positive comments for chosen option and negative comment about the
others.
Examples of potentially good answers
* Trickle drip irrigation; sustainability advantages over large scale irrigation
reduces salinisation risk, water use conserved by careful direction of water to plant
roots, smaller scale, many of problems of large schemes avoided such as removal of
communities for land to be flooded, not dependent on water supplies coming from other
areas or countries
* Extensive livestock farming; sustainability advantages compared with cultivation with
new varieties
not making as heavy a demand on the environment, much of it is nomadic in dry areas
so that grazing areas are changed, low level of technology leaving less of an imprint on
the land, using methods and a way of life which has already been sustained for
centuries
If full marks are not claimed by the likes of the positive and negative comments stated
above, one mark can be awarded for comment that is included which is well directed
towards the question theme of sustainable ways of farming.
Some understanding; perhaps not a clearly stated choice, or a good choice = 1 mark
More understanding for a good choice = 2 marks
As above and well related to question theme = 3 marks
[3]
(d) (i) Can be a large area like the Sahel; may be smaller such as the Indus Valley. However, it
cannot be just the name of a desert such as the Sahara or a region with little farming
such as just the Middle East
[1]
(ii) Physical cause is drier than usual conditions/drought as opposed to just a dry/desert
climate,
allows other natural factors such as wind and heavy rain to erode the soil/land with a
vegetation cover less than normal
Human causes are related to over-use of the land such as overcultivation and
overgrazing or clearance of vegetation for fuelwood
often the consequence of continued population increase/no surpluses accumulated any
more for use in dry years
Minimum 1 mark, maximum 2 marks for each of physical and human
[3]
(iii) No separate mark for view expressed. The generally accepted view is that a natural
process is being increased by human activities and actions. Reward all views from very
physical to very human.
View effectively justified = 2 marks
Some merit in support put forward = 1 mark

[2]

UCLES 2008

285

Page 10

Mark Scheme
GCE O LEVEL May/June 2008

Syllabus
5014

Paper
01

BALOL
(e) (i) Lack of rain to dissolve and wash away pollution particles in the atmosphere, desert
areas have many calm days/low wind speeds,
strong sunlight creates ideal conditions for formation of low level ozone,
inversion of temperature in lower layers which traps particles preventing them rising to
higher levels,
high pressure/sinking air keeps pollution near the surface.
Any two
[2]
(ii) Axes labelled for number of days and dates = 1 mark
suitable method chosen such as bars or a line graph = 1 mark
all plotted correctly = 2 marks
(at least two correct plots = 1 mark)

[4]

(iii) Decrease in number of days with pollution worse than recommended health standards
[1]
(iv) Evidence of a reduction here in Los Angeles so that it shows methods used are working
in this developed world city,
developing world cities are growing faster/traffic and industry are increasing more than in
the developed world,
pollution/traffic controls are less strong/less likely to be strictly enforced,
control measures are costly,
examples of measures used to reduce traffic and industrial emissions up to two marks
if used within the context of the answer
If the answer is based on more effective in developing cities, some progress can be
made with the line of argument that there is massive private car ownership and use in
developed world cities, so much more traffic that even with lower emissions pollution
levels are greater, public transport is more likely to run on newer cleaner fuels like
natural gas.
Mark according to the strength of the explanation
References to strategies with little comment, or some relevant comment without much
real support = 1 or 2 marks
Clear answer to the question supported by relevant explanation, including references to
strategies or other city/country examples = 3 or 4 marks
[4]

UCLES 2008

286

5014 Environmental Management June 2008

BALOL
ENVIRONMENTAL
MANAGEMENT

Paper 5014/01
Paper 1

General comments
There was little noticeable difference in final mark outcomes between totals of the four short questions in
Section A and each of the two long questions in Section B. Some outstanding scripts were seen from
candidates with a broad and thorough knowledge and understanding of the natural environment, human
activities and strategies for management, which allowed them to maintain the flow of full, high quality
answers throughout. A few candidates, fortunately fewer in numbers than in some previous years, penalised
their overall examination performance by not finishing all parts of Question 6. Their scripts displayed the
results of a mis-allocation of time. Overlong answers, well beyond mark availability for the short answer
questions in Section A, were later followed by empty spaces and unfinished sentences in some of the longer
questions towards the end of Section B. There was some evidence this year that more candidates than
previously were answering questions in order of preference rather than paper order. This showed good
examination technique for a paper in which candidates are expected to need all the time allocated to it.
Within Section A, a clear pattern of marks emerged highest marks from Question 1 (generously supplied
with useful source information in the stem of the question), high marks for Question 2 (examining a familiar
topic), significantly lower marks for Question 3 (about lead and its effects, which appeared to be less well
known) and equally low marks for Question 4 (answers not well directed towards the questions set). Within
Section B, the overall standard of answers from most candidates to Questions 5 and 6 was very similar. If
they were not, as many candidates scored better on Question 5 as did for Question 6. For some, part (a) in
Question 5 proved to be more difficult than parts (b)-(f). Overall, however, the vast majority of candidates
exhibited considerable familiarity with the topic of fishing. The final part, (g)(ii), was the least well answered
part of Question 5. Many candidates failed to focus on the question set, spending time describing how the
strategies named in (g)(i) would work, and seemed to assume that they were all easy to implement. The
most troublesome part of Question 6 appeared to be (c)(i). Only a minority of candidates showed much
knowledge and understanding of extensive pastoral farming. Likewise plant new varieties of crops often
failed to trigger references either to high-yielding varieties or GM crops. Consistently more successful were
the answers for large dams and trickle drip irrigation.
Huge variations were noticed in the speed with which candidates began to answer the actual question set.
Often the main mark-earning parts of answers were concentrated in the lower half, often within the last two
lines. Too many candidates began by repeating the question, instead of answering it. The use of a booklet
with spaces below each question for answering renders repetition unnecessary. Many candidates obviously
felt that they had fully answered a question once all the lines left for it had been filled. Wise candidates,
displaying good examination technique, looked at the number of marks available, and tried to include a wider
range of points and more detailed elaboration or exemplification in answers worth four or five marks. In both
Questions 5 and 6 the two longer final parts appeared to be less well answered than usual. The final part of
Question 6 suffered particularly badly from this. Many eight- and nine-line answers essentially referred to
nothing more than stating that developed countries were richer and people better educated, and that
developing countries were poorer and people were less well-educated. Education anyway was not
particularly relevant to this answer. When answering the four mark question in 5(f)(ii), many candidates
were satisfied after they had referred to just one of the El Nino years, having named the year and stated the
amount of catch. Instead they should have continued the search for other evidence from the graphs to
support their answer.

UCLES 2008

287

5014 Environmental Management June 2008

Comments
BALOL on individual questions
Section A
Question 1
The most frequent reasons for candidates achieving below half marks in part (a) were use of a social
advantage instead of disadvantage, or attempted use of a disadvantage which was more environmental than
social. Even when a social disadvantage was correctly identified, candidates used explanations which were
more economic than social. Choosing and explaining the two economic advantages were undertaken more
effectively and with far more consistency. Five- and six-mark answers were common. There was a much
greater range in the quality of answers to part (b). The weakest type of answer typically began with planting
trees, and included reclaiming land for farming, but contained few if any references to methods of achieving
them.
Question 2
- known of the three processes in (a)(i). A systematic approach to answering, beginning with evaporation of
sea-water and ending with surface or groundwater using the slope to flow back into the sea, yielded the most
sure reward in (a)(ii). Marks in this part seemed to be lost more by carelessness in completing the
explanation than by lack of understanding. Candidates found (b) the least easy part to answer. Many
candidates suggested and stuck with only one reason; even with good elaboration, it was not normally
enough for more than two marks, and one mark answers were frequent. Overuse by people, supported by
references to increasing population and greater frequency of droughts, were the reasons used in many of the
three-mark answers.
Question 3
To gain full marks in part (a)(i) candidates needed to recognise one of the times of sharp decline, the
fluctuations, and the later time of more gentle (but also more persistent) decline. Able candidates found this
easy, and made appropriate references to years and emissions. The weakest candidates typically filled all
the answer lines, yet used only one of these descriptive elements. For many less able candidates, this part
yielded their only marks for the whole question. Part (a)(ii) was the least well answered part. Few answers
were focused on trends, despite this being a direct continuation from the previous part. Even good
candidates spent too much time trying to give reasons for high emissions instead of seeking reasons for
emissions decline. Only a few candidates showed that they knew that road traffic was responsible for high
levels of emissions, before the now quite widespread adoption of unleaded fuels. Only a tiny minority
broadened their answers to include references to substitutes for lead in industry, such as plastic and copper.
References to fewer vehicles appeared more regularly in answers to (a)(iii). Answers to part (b) tended to
be either a success or a disaster. Many candidates appeared able only to conceive of pollution and dangers
to health as coming from the air; global warming, ozone depletion and human health problems from poor air
quality were referred to more often than was lead poisoning, and its adverse effects on human and animal
health.
Question 4
As in last years examination, some candidates did not state what could be seen on the photograph. Part
(a)(i) needed to be answered in terms of photograph-based evidence; apart from mention of trees, little direct
connection existed between photograph and answer from many candidates. Many candidatesdid not
mention trees, woodland or forest in (a)(ii). Grasses, bushes and small shrubs were suggested more often.
Part of the reason must have been limited understanding of vegetation succession. Something a little more
precise than what many candidates were able to offer was required in (a)(iii), such as eat the new shoots or
pull out young plants by their roots. Candidates who began with the role of mosses and lichens answered
part (b) really well. The majority gained one or more marks from references to increased humus, nutrients
and fertility. Other candidates, who either began with advantages of trees for soils or referred to farming
techniques for improving soils such as crop rotation, failed to demonstrate any understanding of succession.
Most answers to part (c) covered only a restricted range of reasons, which were inadequate for full marks.
Most frequently given were loss of habitats and biodiversity.

UCLES 2008

288

5014 Environmental Management June 2008

Section
BALOL B
Question 5
Answers to part 5(a)(i) were often no more than a list of places, for which some credit was given, provided
that it was based on continents or countries and not just names of oceans. Better answers came from
candidates who were able to detect features of the overall pattern, such as all but one of the main fishing
areas being located north of the Equator. Most candidates found answering (a)(ii) a little easier, most
frequently suggesting near to coasts, around islands and on continental shelves as similarities. Northern
versus southern hemisphere, even if not as briefly stated as this, was the most frequently suggested
difference.
In part (b) there were wide variations in candidate performance. Strong candidates packed their answers
with real content, particularly for ocean currents and continental shelves. The one disappointment was the
general absence of references to named ocean currents. It soon became clear to Examiners that some
weaker candidates added more after having answered part (e), but often the additions were of little worth,
because they concentrated on warm ocean currents. ;This only served to emphasise how limited was their
understanding. Most answers to (c)(i) were accurate, although many fewer marks would have been
awarded if Examiners had insisted on the inclusion of the unit (millions). Candidates should be reminded of
the need to state units when working from graphs. In part (c)(ii), some candidates viewed overfishing as
synonymous with a lot of fishing. Either they finished their answers having mentioned the rise to the peak in
1987, or having dealt only with the increase during the whole period between 1970 and 2000 (usually worth
no marks). Recognising the decrease from the late 1980s was an important requisite for full mark answers
on this section.
Part (d)(i) discriminated well between weak candidates, who added little to what was already given to them
in the labels on the spider diagram, and able candidates, who could include an example of high technology
for locating shoals of fish, and appreciated that larger sized fishing boats and refrigerated factory ships could
explore the oceans more widely, staying out at sea and fishing for much longer periods. All the causes were
equally capable of generating full mark answers in part (d)(ii). Choice was less important than candidate
knowledge and understanding.
With only a moderate amount of understanding, the answers needed for all three parts of (e) could be
obtained from the two diagrams. However, some candidates failed to realise that in order to establish clear
differences in parts (i) and (ii) their answer needed to be two-sided or comparative. Another common failing
was to refer to the differing opportunities for fishing, only to find that they needed to repeat their answers in
part (iii). Others interpreted well what the diagrams showed. In some answers it was easy to detect good
understanding of El Nino based upon knowledge.
In part (f)(i) a few candidates took 1982 and 1983 to be one year and included 1994 as a replacement for
one of them. By referring to the El Nino years of 1982 and 1990 in part (ii) it was possible for candidates to
argue that El Nino events had no or limited effects upon fish catches in Peru, but answers which only
followed this route could gain no more than half of the available marks. The evidence for reduced catches
was too overwhelming, particularly in the 1990s. The most convincing answers were those in which
candidates directly compared catch sizes in El Nino years with those in surrounding years. However, in part
(iii) the evidence for overfishing was not as definite. Some candidates successfully argued that continually
rising catches during the 1990s, except in El Nino years, at significantly higher levels than those in the
1980s, were a sign that overfishing had not occurred. Others noted the large drop in 1994 compared with
surrounding years, and argued equally well that this suggested overfishing had occurred. In this question,
the strength of supporting evidence mattered more than the opinion taken about overfishing. The least
satisfactory answers were those in which candidate opinion was not made clear.
The weakest answers to part (g)(i) came from candidates who merely reversed the causes of overfishing
from the spider diagram in part (d). These answers were largely worthless, since they were not usually
accompanied by any positive strategies for sustainable harvesting of ocean fisheries. Candidates had plenty
of strategies to choose from, there being six or seven of them named in the syllabus. Quotas, closed
seasons and restricted areas for fishing were the ones most frequently used. In order to gain the final mark,
candidates needed to write more than just strategies in the centre of their spider diagram. They had to
make clear what the strategies were for. Diagrams always need to be free-standing to be effective. Most of
the answers given to part (g)(ii) failed to reach the expected standard. Here it was difficult to justify easy to
implement as the answer, yet many candidates tried to do this. However, they tended to skirt around the
real question by spending most of the time describing how implementation of the strategies would reduce
overfishing. Those candidates who focused their responses upon the difficulties involved in carrying out the

UCLES 2008

289

5014 Environmental Management June 2008

strategies,
BALOL by reference to fishermens needs for income and food, international problems of effective policy
implementation and the difficulties of monitoring, seemed to have little trouble giving answers worth full or
almost full marks. Unfortunately, such answers were the exception.
Question 6
Successful answers to part (a)(i) often began with in an open area or similar. Least successful were those
that were totally negative, such as not in a city, or those which tried to describe the many types of weather
(mountain, coast, etc.) that could be recorded here. For protection alone was not a strong enough answer
in (a)(ii) unless animals or people were mentioned. Thermometers of any type gave an acceptable answer
to part (a)(iii). Some candidates answered temperature without naming the weather instrument. Others
gave the name of the white box. Barometer was quite a common answer as well. Although, inevitably, there
were candidates who confused anemometer with wind vane in (a)(iv), there were many well-drawn and
labelled diagrams. Unfortunately, a few candidates drew an excellent diagram of an anemometer, but
without name or labels, and could be given very limited reward for their efforts.
Occasionally candidates missed out part (b)(i) which was a careless error. Virtually every candidate who
attempted the question correctly picked out the highest and lowest temperatures of 34 C and 16 C.
Despite the guidance in Station B column, many did not attempt to work out the annual range for Station A,
(18 C), and left the box empty. Dry season was a poor answer to summarise precipitation during the year.
Something stronger, such as dry all year, was needed. Summer or May/June to September were
considered to be superior answers to March-September in (b)(ii). One of the most common answers was
August, but one month by itself does not make a season. While most candidates correctly selected Desert
climate for Station A in (b)(iii), as many, if not more, chose Equatorial climate for Station B. This confirmed
the problems that many candidates have with understanding climates. The choice of desert for Station A
was easy to explain in part (b)(iv) because of the absence of rain, but many of the attempts to explain
Savanna for Station B were unconvincing, relying only upon lifting wet and dry season from the table. Much
better were responses which either ruled out Equatorial because of the lack of rainfall all year, or used a
more complete picture of summer rains and winter dryness with not too much variation in the high
temperatures to justify their choice of Savanna climate.
Most candidates were only able to gain half the available marks for part (c)(i). It was clear that less was
known about extensive livestock farming than the other three. Few recognised this type of farming as a
traditional activity in places too dry for cultivation and little mention was made of the natural adaptation of
certain animals such as camels to dry environments. The better answers about new varieties of crops were
the ones which were adapted by the candidates to the question theme that the seeds, whether high
yielding or genetically modified, had been developed to grow in dry environments. Plenty of not very relevant
references to high yielding varieties of rice were seen. New varieties in the question triggered off many
barely relevant references to crop rotation, how it works and its advantages. Consistently the best answers
were seen for the third option of large dams, even if weaker candidates relied heavily upon expensive for
the disadvantage without further elaboration. There were some good answers for trickle drip irrigation,
although in some there was over-concentration on underground water supplies at the expense of the
irrigation method. Able candidates, with the support of knowledge and understanding, were the only ones to
state advantages and disadvantages with precision in this question. In part (c)(ii) option 3 was the most
popular choice. With relevant comments about amount of water and distance into the future that water
supplies were likely to last, many candidates claimed marks. More candidates using option 4 gained full
marks than for any of the other options, especially when they concentrated on explaining the advantages of
trickle drip over other types of irrigation, and not upon underground water supplies.
Naming a desert, such as the Sahara or a desert region such as the Middle East, was considered insufficient
in (d)(i). The most obvious answer was the Sahel or one of the countries within it. Naming appropriate
areas within the candidates home country was also rewarded. However, it was clear in this part, as well as
in the next two parts, that many candidates regarded a desert, and an area suffering desertification turning it
into a desert, as the same. Physical causes given in part (d)(ii) were sometimes no different from the
description of a desert climate. There was an absence of reference to drier periods that are longer and more
severe than normally expected (i.e. drought). While in general this part was high scoring, because most
candidates were clearly familiar with the human causes such as deforestation, overgrazing and overcultivation, some candidates used the whole answer to explain the difference between physical and human
factors without any mention of actual causes of desertification. Total human or total physical answers were
equally acceptable in (d)(iii) and explanation that matched the view expressed was what mattered. Those
who answered along the lines that human causes accelerate natural causes to extend desertification to new
areas gave many of the best answers, especially when it was linked to increased population pressure.

UCLES 2008

290

5014 Environmental Management June 2008

There
were wide variations in the quality of answer given to part (e)(i). Lack of rain and wind under high
BALOL
pressure conditions to remove the pollutants from the air earned full marks for many. However, from time to
time references to inversion of temperature and favourable conditions for the formation of photochemical
smog were successfully included in answers. Weakest of all were the totally irrelevant answers about ozone
hole formation and carbon dioxide concentrations leading to global warming. In (e)(ii) candidates who chose
a vertical scale of 10 squares for 50 days were the ones most certain to draw a fully accurate graph, because
all the plotted points fell on graph lines, whereas those who began with 0 at the bottom and 210 at the top
made one or more mistakes unless very careful, because only two of the plots were exactly on a graph line.
There was an almost equal candidate preference for bar and line graphs. There were a few total disasters,
caused by candidates using an unequal scale and simply writing in the values from the table to plot at every
5 or 10 grid squares. Over-simple answers, such as Decrease or It is declining, failed to gain the mark in
(e)(iii). Without reference to days or atmospheric pollution, the trend was not meaningfully described. The
result was the careless loss of a mark. The final part, (e)(iv), suffered from too many vague answers,
caused by candidate over-reliance upon weak generalisations lacking further support and development. Use
of more knowledge of strategies, more money for implementation, more law enforcement and better
education in developed than in developing countries generated low marks. Inclusion of actual strategies
such as catalytic converters, cleaner fuels and improved public transit systems began to strengthen answers
greatly.

UCLES 2008

291

BALOL
UNIVERSITY OF CAMBRIDGE INTERNATIONAL EXAMINATIONS
General Certificate of Education Ordinary Level

*3932422729*

5014/01

ENVIRONMENTAL MANAGEMENT
Paper 1

October/November 2008
2 hours 15 minutes

Candidates answer on the Question Paper.


Additional Materials:

Ruler
Protractor

READ THESE INSTRUCTIONS FIRST


Write your Centre number, candidate number and name on all the work you hand in.
Write in dark blue or black pen.
You may use a soft pencil for any diagrams, graphs or rough working.
Do not use staples, paper clips, highlighters, glue or correction fluid.
DO NOT WRITE IN ANY BARCODES.
Answer all questions.
All questions in Section A carry 10 marks.
Both questions in Section B carry 40 marks.
At the end of the examination, fasten all your work securely together.
The number of marks is given in brackets [ ] at the end of each question or part question.

For Examiners Use


1
2
3
4
5
6
Total
This document consists of 22 printed pages and 2 blank pages.
SP (KN)
UCLES 2008

[Turn o

292

For
Examiners
Use

BALOL

Section A

(a) The table shows information about how cultivable land and water resources are
distributed between North and South China.

(i)

North China

South China

Cultivable land

65%

35%

Water resources

19%

81%

What problem, for China, is suggested by the information in the table?


..................................................................................................................................
..............................................................................................................................[1]

(ii)

Look at the map of China.


N
0

500
km

Beijing

(Ye

C
W

g
Jia )
g
n
Cha gtse
n
(Ya
n

R.

Br
ah
map
utra Rive

llo
w

ua

ng
Ho
R iv
er)

Tianjin

Key
city

border of Chinese mainland

Xun Riv
er

major river and direction of flow


planned canal

Western canals to take water from the Tibetan Plateau


at altitudes above 4000 m in tunnels through high mountains

Central canal

Eastern canal across Taihang Mountains

[ ]
X

C and E will pass under the Huang Ho river in tunnels


The Three Gorges Dam

UCLES 2008

5014/01/O/N/08

293

BALOL

Describe how the Chinese government is planning to solve the problem you have
stated in (a)(i).

For
Examiners
Use

..................................................................................................................................
..................................................................................................................................
..............................................................................................................................[2]
(iii)

If these plans are carried out, describe the problems likely to be caused during and
after construction.
..................................................................................................................................
..................................................................................................................................
..................................................................................................................................
..................................................................................................................................
..................................................................................................................................
..............................................................................................................................[4]

(b) Many Chinese rivers are heavily polluted. Explain how this problem could be reduced.
..........................................................................................................................................
..........................................................................................................................................
..........................................................................................................................................
..........................................................................................................................................
......................................................................................................................................[3]

UCLES 2008

5014/01/O/N/08

[Turn over

294

For
Examiners
Use

4
2 BALOL
(a) The diagram shows an oil and gas trap.
(i)

Complete the key to the diagram using the type of shading shown on the diagram.
[3]
Key

ground surface

impermeable rock

water

water

oil
gas

(ii)

Name the type of oil trap shown in the diagram.


..............................................................................................................................[1]

(iii)

Explain how oil and gas below the surface of the ground are discovered.
..................................................................................................................................
..................................................................................................................................
..................................................................................................................................
..............................................................................................................................[2]

(b) Oil at 80C is transported by pipeline across the tundra in Alaska. Explain how
the construction of the pipeline and the transport of the oil may affect the tundra
ecosystem.
..........................................................................................................................................
..........................................................................................................................................
..........................................................................................................................................
..........................................................................................................................................
..........................................................................................................................................
......................................................................................................................................[4]

UCLES 2008

5014/01/O/N/08

295

5
3 BALOL
(a) The graph shows the effects, on crop yield, of using different amounts of chemical
fertiliser.

For
Examiners
Use

30
25
20
crop yield
(tonnes per
unit area)

15
10
5
0

20

40

60

80

100 120 140 160 180 200

amount of fertiliser applied (kg per hectare)

Describe the effects, on crop yield, of applying different amounts of chemical fertiliser.
..........................................................................................................................................
..........................................................................................................................................
..........................................................................................................................................
......................................................................................................................................[3]
(b) Explain how the over-use and misuse of chemical fertiliser is harmful to the
environment.
..........................................................................................................................................
..........................................................................................................................................
..........................................................................................................................................
..........................................................................................................................................
..........................................................................................................................................
......................................................................................................................................[4]
(c) How could a farmer increase crop yields using alternatives to chemical fertilisers?
..........................................................................................................................................
..........................................................................................................................................
..........................................................................................................................................
..........................................................................................................................................
......................................................................................................................................[3]
UCLES 2008

5014/01/O/N/08

[Turn over

296

6
4 BALOL
(a) Look at the photograph which was taken in a National Park in the taiga of Canada.

(i)

For
Examiners
Use

Describe the characteristics of the trees and the forest that can be seen in the
photograph.
..................................................................................................................................
..................................................................................................................................
..................................................................................................................................
..................................................................................................................................
..............................................................................................................................[3]

(ii)

Explain ways in which these trees are adapted to survive in the climate of the
taiga.
..................................................................................................................................
..................................................................................................................................
..................................................................................................................................
..................................................................................................................................
..............................................................................................................................[3]

UCLES 2008

5014/01/O/N/08

297

BALOL
(b) (i) The building on the photograph is one of the National Parks visitor centres. The

For
Examiners
Use

panels on the front provide it with energy. Name the type of energy.
..............................................................................................................................[1]
(ii)

Suggest reasons why there are both advantages and disadvantages of using this
type of energy in a National Park in this part of Canada.
..................................................................................................................................
..................................................................................................................................
..................................................................................................................................
..................................................................................................................................
..............................................................................................................................[3]

UCLES 2008

5014/01/O/N/08

[Turn over

298

For
Examiners
Use

BALOL

Section B

(a) The diagram shows ways by which precipitation reaches a river.

Precipitation

Soil
X

Y
r
Rive

Rock

(i)

Name the processes labelled X and Y.


X ...............................................................................................................................
Y ...........................................................................................................................[2]

(ii)

Explain how precipitation reaches the river by process X.


..................................................................................................................................
..................................................................................................................................
..............................................................................................................................[2]

(iii)

UCLES 2008

Add the letter I on the diagram where interception of precipitation takes place.

[1]

5014/01/O/N/08

299

BALOL(iv) In some areas on the diagram precipitation reaches the river quickly, while in other

For
Examiners
Use

areas it reaches the river more slowly. Explain why.


..................................................................................................................................
..................................................................................................................................
..................................................................................................................................
..................................................................................................................................
..................................................................................................................................
..................................................................................................................................
..................................................................................................................................
..............................................................................................................................[4]
(b) State three different reasons why people throughout the world live next to rivers.
1 .......................................................................................................................................
2 .......................................................................................................................................
3 ...................................................................................................................................[3]

UCLES 2008

5014/01/O/N/08

[Turn over

300

10

BALOL
(c) There can be disadvantages from living next to rivers. One is pollution. Look at the map

For
Examiners
Use

of North East China.

November 21 2005

City of Harbin (350 km downstream from Jilin) announces


it will shut off water to residents

Harbin is city of 9 million people; 80% of its water comes


from the Songhua River
the reason given by the authorities was pipe
maintenance
no one believed this; panic resulted, city shops sold out
of water in hours and all trains and planes out of Harbin
were packed with people
water supply was shut off for 5 days

December 8 2005

Toxic slick expected to reach Khabarovsk


in Russia, a city of half a million people
there was a delay of at least one
week after the explosion before
Russia was informed about toxins in
the Songhua River
after receiving the news, preparations
were made to switch off drinking
water supplies

November 18 2005

Date when information about the toxic slick in the river was
given to other authorities down river for the first time
80 km long slick was flowing down the Songhua River
officials remained silent hoping that the chemicals would
quickly dilute in the river water doing little harm
nitro-benzene is dangerous to people; it affects the
nervous system and can lead to cancers in the longer
term
chemicals could be absorbed by fish in the river and
enter the food chain
no one is sure how long it will take the bacteria in the
water and soil to decompose the pollutants; it depends
on levels of concentration of the pollutants and water
temperature (it happened in winter)
an expert living outside China warned that a heavy
concentration of chemicals absorbed by sediments on
the river bed would be released into the river water for
possibly 3 to 5 years

RUSSIA

Khabarovsk

Songhua
River

CHINA

Harbin
Jilin

November 13 2005

Chemical plant explodes at Jilin


series of explosions; 5 workers killed and 70 injured
giant orange cloud of smoke released; 40 000 nearby
residents evacuated
leak of highly toxic nitro-benzene into the Songhua River

(i)

Look at box 1 below the map. State two immediate effects of the chemical plant
explosion for people living in Jilin.
..................................................................................................................................
..................................................................................................................................
..............................................................................................................................[2]

UCLES 2008

5014/01/O/N/08

301

11

BALOL(ii) Although located 350 km away, effects from the toxic leak were still felt in Harbin.

For
Examiners
Use

Explain why.
..................................................................................................................................
..................................................................................................................................
..................................................................................................................................
..............................................................................................................................[2]
(iii)

Explain why this pollution event became an international issue.


..................................................................................................................................
..................................................................................................................................
..................................................................................................................................
..................................................................................................................................
..............................................................................................................................[3]

(iv)

When the water supply to Harbin was restored after 5 days, it was reported that
residents were told that

the chemicals have now passed the city


the water is now safe

Were these two statements accurate and reliable? Give your view and explain it.
..................................................................................................................................
..................................................................................................................................
..................................................................................................................................
..................................................................................................................................
..................................................................................................................................
..............................................................................................................................[4]

UCLES 2008

5014/01/O/N/08

[Turn over

302

12

BALOL
(d) Another disadvantage from living next to rivers is flooding. River floods are common in

For
Examiners
Use

tropical climates such as savanna and monsoon during the wet season.
Mumbai tropical monsoon climate
35

temperature (C)

30
25
20
15
10
5
0

700

precipitation (mm)

600
500
400
300
200
100
0

(i)

Average monthly temperatures for Mumbai

Month

Jan

Feb

Mar

Apr

May

Jun

Jul

Aug

Sep

Oct

Nov

Dec

24

24

26

28

30

29

27

27

27

27

28

26

In the space above precipitation, complete the climate graph for Mumbai by drawing
a line graph to show average monthly temperatures.
[3]

UCLES 2008

5014/01/O/N/08

303

For
Examiners
Use

13

BALOL(ii) When is the wet season in Mumbai?


..............................................................................................................................[1]
(iii)

River floods in surrounding farming areas are more widespread during September
and October than in June and July. Use the precipitation values to suggest why the
worst flooding happens in these two months.
..................................................................................................................................
..................................................................................................................................
..............................................................................................................................[2]

(iv)

Look at both temperature and precipitation. In which month is drought likely to be


the most serious for farmers? Explain your choice.
Month .......................................................................................................................
..................................................................................................................................
..................................................................................................................................
..............................................................................................................................[2]

(v)

Describe one method used by farmers to allow cultivation to continue during dry
periods of the year.
..................................................................................................................................
..................................................................................................................................
..................................................................................................................................
..................................................................................................................................
..............................................................................................................................[3]

UCLES 2008

5014/01/O/N/08

[Turn over

304

14

BALOL
(e) The benefits of high rainfall and river floods can be greater than the bad effects.
(i)

For
Examiners
Use

State two benefits of high rainfall and river floods for farmers.
..................................................................................................................................
..................................................................................................................................
..................................................................................................................................
..............................................................................................................................[2]

(ii)

Do you agree that the benefits of river floods can be greater than the bad effects for
people living close to large rivers? Explain your view.
..................................................................................................................................
..................................................................................................................................
..................................................................................................................................
..................................................................................................................................
..................................................................................................................................
..................................................................................................................................
..................................................................................................................................
..................................................................................................................................
..............................................................................................................................[4]
[Total: 40 marks]

UCLES 2008

5014/01/O/N/08

305

For
Examiners
Use

15
6 BALOL
(a) Look at the photograph of savanna vegetation in Africa.

Describe the features of the vegetation shown.


..........................................................................................................................................
..........................................................................................................................................
..........................................................................................................................................
..........................................................................................................................................
......................................................................................................................................[3]

UCLES 2008

5014/01/O/N/08

[Turn over

306

16

BALOL
(b) The plant and animal life shown in the photograph is only possible because of air, water

For
Examiners
Use

and minerals.
Suns energy

CO2 from air

rainwater

plant
soil

nutrients
(i)

Explain how green plants trap the suns energy to create living matter.
..................................................................................................................................
..................................................................................................................................
..................................................................................................................................
..................................................................................................................................
..................................................................................................................................
..................................................................................................................................

(ii)

Where do continued supplies of nutrients for plant growth come from?


..................................................................................................................................
..................................................................................................................................
..................................................................................................................................
..................................................................................................................................
..................................................................................................................................
..................................................................................................................................
[6]

UCLES 2008

5014/01/O/N/08

307

17

BALOL
(c) The diagram shows part of a food chain based upon what can be seen in the

For
Examiners
Use

photograph.

(i)

Explain how it shows part of a food chain.


..................................................................................................................................
..................................................................................................................................
..................................................................................................................................
..............................................................................................................................[2]

(ii)

Describe how further links might be added to this food chain.


..................................................................................................................................
..................................................................................................................................
..................................................................................................................................
..............................................................................................................................[2]

UCLES 2008

5014/01/O/N/08

[Turn over

308

18

BALOL
(d) Look at the diagram which shows the growth of the human population in the last 7000

For
Examiners
Use

years and its effects.

5000 BC
Solar Water Carbon
dioxide
energy

Earths
land area
rock

population
10 million

2000 AD
Solar Water Carbon
energy
dioxide

Earths
land area

Agricultural
land

rock
chemical fertilisers
and pesticides
new varieties of
seeds and animals
modern technology

UCLES 2008

population
6.1 billion

5014/01/O/N/08

309

For
Examiners
Use

19

BALOL Between 5000 BC and 2000 AD state what has


(i)

remained the same;


..................................................................................................................................
..................................................................................................................................

(ii)

decreased in size and numbers.


..................................................................................................................................
..................................................................................................................................
[2]

(iii)

What is the evidence from the diagram that there was increased pressure on the
Earths natural resources and ecosystems between the two dates?
..................................................................................................................................
..................................................................................................................................
..............................................................................................................................[2]

(e) (i)

In 5000 BC how did humans survive? State two ways.


..................................................................................................................................
..................................................................................................................................
..............................................................................................................................[2]

(ii)

State one advantage and one disadvantage of survival by these ways.


Advantage ................................................................................................................
..................................................................................................................................
Disadvantage ...........................................................................................................
..............................................................................................................................[2]

(iii)

Approximately what percentage of the Earths land area was agricultural land in
2000?
..............................................................................................................................[1]

UCLES 2008

5014/01/O/N/08

[Turn over

310

20

BALOL(iv) Three reasons are given on the diagram for the great increase in agricultural land

For
Examiners
Use

and food output by 2000. Choose two of them and describe in more detail how
each has increased food output.
1 ...............................................................................................................................
..................................................................................................................................
..................................................................................................................................
..................................................................................................................................
..................................................................................................................................
2 ...............................................................................................................................
..................................................................................................................................
..................................................................................................................................
..................................................................................................................................
..............................................................................................................................[6]
(f)

Great ecosystem losses result from increases in the human population and agricultural
output.
Ecosystem losses (up to 2000 and predicted for 2000 to 2050)

tundra
taiga (coniferous forests)
other temperate forests
hot desert
savanna
tropical rain forest
100

90

80

70

60

50

40

30

20

10

percentage
Losses

between 2000 up to 2000


and 2050

(i)

Which ecosystem suffered the greatest percentage loss up to 2000?


..............................................................................................................................[1]

UCLES 2008

5014/01/O/N/08

311

21

BALOL(ii) Losses up to 2000 varied from 1% to 75% between the different ecosystems.

For
Examiners
Use

Suggest reasons why percentage losses before 2000 varied so much between
ecosystems.
..................................................................................................................................
..................................................................................................................................
..................................................................................................................................
..................................................................................................................................
..................................................................................................................................
..............................................................................................................................[3]
(iii)

Name the ecosystem expected to have the greatest percentage loss between 2000
and 2050.
..............................................................................................................................[1]

(iv)
Sustainable forest management techniques
community forestry

sustainable forest
management techniques

sustainable harvesting
of hardwoods

agro-forestry

Choose one technique. Describe how it can be used for sustainable forest
management.
..................................................................................................................................
..................................................................................................................................
..................................................................................................................................
..................................................................................................................................
..................................................................................................................................
..............................................................................................................................[3]

UCLES 2008

5014/01/O/N/08

[Turn over

312

22

BALOL(v) Are these techniques easy or difficult to carry out? Explain your views on this.

For
Examiners
Use

..................................................................................................................................
..................................................................................................................................
..................................................................................................................................
..................................................................................................................................
..................................................................................................................................
..................................................................................................................................
..................................................................................................................................
..............................................................................................................................[4]
[Total: 40 marks]

UCLES 2008

5014/01/O/N/08

313

5014 Environmental Management November 2008

BALOL
ENVIRONMENTAL
MANAGEMENT

Paper 5014/01
Paper 1

General comments
There was a great range in quality of performance, from candidates who maintained the flow of perfect or
near perfect answers to all parts of all questions, to those who struggled over the majority of questions, and
managed only the occasional convincing answer. For most, the total mark out of 40 for Section A was on a
par with the individual marks for Questions 5 and 6 in Section B. For better or for worse, the standard
established early in the examination tended to be maintained throughout. Time issues with this paper are
now much reduced from what they were in its first two sessions. Examples of candidates genuinely
struggling to complete the examination in the time allowed were rare.
In Section A, Questions 1, 3 and 4 performed in similar ways; in each, one part proved to be significantly
more awkward for the majority of candidates, but the other parts were more regularly answered comfortably
and well. Question 2 was the exception; it caught out candidates with inadequate knowledge of the location
of oil deposits and their transport. In Section B, the general standard of answers to Question 5 in many
Centres was slightly better than that to Question 6. One of the main reasons for this was that the final two
parts of Question 6(f) were the least well answered questions on the whole paper. Despite being named in
the syllabus, only a minority of candidates seemed to know what was meant by community forestry, agroforestry or sustainable harvesting of hardwoods.
Some areas for improvement in examination technique remain. There has never been any need to repeat
the question; candidates should be advised always to begin to answer the question in the first sentence.
When most candidates equate filling all the lines with a full answer to the question, this advice has added
importance. The lines left in the booklet are for guidance only and cannot take full account of variations in
size of writing and precision of expression between candidates. It is only to be expected that there will be
occasions when candidates feel the need to write more. They can continue their answers in spaces below
the lines (where they exist), or in empty spaces at the bottom of the page or at the end of the booklet. More
attention could have been paid to the number of marks available for the answers. When describing from a
source provided, such as from the photographs in Questions 4(a)(i) and 6(a)(i), the number of marks
suggested the number of features that needed to be observed and described. These questions were both
worth three marks; common answers to 6(a)(i), along the lines of 'a lot of vegetation with many tress and
grasses', were never going to offer description that was sufficiently full to claim all three marks. In fact,
candidates should be advised to describe more fully than they believe is necessary for the number of marks,
to allow for the occasions when the points they make do not match up with those in the mark scheme.
Answers to questions worth four or more marks needed to contain some breadth or depth in order to be full
and effective. Answers to Questions 1(a)(iii), 2(b) and 5(a)(iv) suffered from an over-concentration on one
element only, or at best two. In 5(a)(iv), many candidates, after having explained validly enough how trees
slow down water movement towards the river, then filled up the lines using reverse information for areas
without tree cover. Instead they should have looked for other explanatory factors relevant to the area shown
in the diagram, such as variations in agricultural land uses, particularly the fields without crops and ploughed
up and down the slope. The four mark Question 5(e)(ii) offered an opportunity to provide depth through
reference to an example, such as the Nile Valley in Egypt supporting the benefits of living with floods, or
Bangladesh or a home country example to illustrate just how devastating effects of a big flood can be.

UCLES 2008

314

5014 Environmental Management November 2008

Comments
BALOL on individual questions
Section A
Question 1
In part (a)(i) quite a number of candidates skirted around the problem, unable to show that they clearly
recognised that water resources were concentrated in South China while there was twice as much cultivable
land in North China. Better progress was made with answers to part (a)(ii) for the south to north transfers
using canals. Two mark answers dominated in (a)(iii), because many candidates did not try to go beyond
great cost and direct effects on the environment. Great cost was rarely related to the need for so many
tunnels under the higher land between river valleys. Those candidates who continued the search for more
problems, in order to provide either the depth or breadth needed for a four mark question, were most likely to
refer to the difficulties of working in remote and inhospitable regions such as the Tibetan Plateau. Again in
part (b) two mark answers were more common than those worth three marks; typically the range of points or
their elaboration were insufficient for the needs of a three mark question.
Question 2
Candidates with good topic knowledge and understanding sailed through all parts of this question. Some of
the others stumbled in the first part and never recovered. When none of the three items in the key were
identified correctly in part (a)(i), it was an ominous sign for answers to the remaining parts. Folded rocks
was too vague an answer for upfold or anticline in (a)(ii). Many answers to part (a)(iii) were about obtaining
the oil, not its discovery. When drilling was mentioned in these answers, often it was not in the correct
context of trial drilling, the only certain way to prove the existence of oil underground. It was in their answers
to part (b) that weak candidates were most likely to claim one or two marks, usually for referring to melting of
the frozen ground and interference with ecosystems. Only more knowledgeable candidates seemed able to
assemble either the range or depth of relevant explanation required for a four mark answer.
Question 3
In part (a), as in other three mark questions on this paper, one and two mark responses were more common
than three mark answers. This reflected either general description without any reference to yields or fertiliser
amounts, or partial description frequently without any reference to the levelling off or slight decline in crop
yields after about 100 kg of fertiliser per hectare. Part (b) was the best answered part, with a much higher
chance than in other four mark answers of adequate content for all the marks. Precise explanation for what
happens when excess nitrates reach water courses was regularly given. Part (c) discriminated ruthlessly
well between able and weak candidates. The former referred to organic fertilisers such as manure and
compost, farming techniques such as mixed farming and crop rotation, and the use of nitrogen fixing crops
such as legumes. In contrast, sometimes after mentioning manure or compost, many weaker candidates
drifted away from the question theme of 'alternatives to chemical fertilisers'. High yielding varieties of seeds
were used in many answers, but without candidates considering what was needed for their high yields to be
maintained.
Question 4
Failed answers in part (a)(i) fell into one or two categories, either no direct description from the photograph,
or seeing an ecosystem which was not there to see on the photograph, such as tundra. On the other hand,
those candidates, who looked and concentrated on describing the characteristics of the trees and forest that
could actually be seen, often described more features than were needed for all three marks. Part (a)(ii) was
the least well answered part of this question, with many answers suffering from a continuation of the
description from the previous part, without any attempt to explain adaptations to the taiga climate. For some
candidates confusion with tundra also continued. Candidates with knowledge and understanding soon
claimed all the marks, most commonly with references to conical shape for the snow to slide off and/or for
flexibility in strong winds, evergreen nature to take advantage of the short growing season, and needle
leaves to reduce water loss. Virtually every candidate gave the correct answer of solar in (b)(i). Most found
it easy to claim two marks in (b)(ii), one for the general disadvantage of days without sunlight and one for the
general advantage of clean or non-polluting. However, the third mark proved to be more elusive for all but a
few, mainly the ones who considered the National Park or northerly location.

UCLES 2008

315

5014 Environmental Management November 2008

Section
BALOL B
Question 5
The best answers to part 5(a)(i) were infiltration and runoff. Most candidates knew these two water cycle
processes. Those candidates who also knew percolation and groundwater flow tended to answer part (a)(ii)
the best; although these names were not essential for claiming the marks, they were a sign of greater
candidate understanding than answers from those who merely tried to explain based upon what the diagram
showed. In part (a)(iii) some placed the letter I too far away from the trees, often within the precipitation
above the forested part. Part (iv) was less well answered. One common failing was to answer in general
terms about variations in surface run-off, instead of giving answers based on what this diagram showed. The
result was that the influences of factors such as relief, rock type and land use were explained in complete
isolation from the diagram. Another failing was to concentrate on just one factor, most often tree
interception, and fill most of the lines first by referring to areas with trees and then reversing the comments
for areas without trees, without adding anything new to the explanation. The superior quality of some
answers stood out, when forest was compared with farm land, and grass and crop growing areas were
compared with fields ploughed up and down the slope. Credit was given for clear references to different
areas on the diagram, where they were used to illustrate differences in speed for precipitation reaching the
river.
It was not difficult for candidates to find three sufficiently different reasons why people live next to rivers in
part (b). A few candidates, however, left the Examiner to do too much of the work by stating 'source of
water' or similar without identifying any specific uses.
Most answers to (c)(i) accurately included immediate effects, although in a few the immediate was ignored
as subsequent effects on people's health and water supplies dominated. In (c)(ii), one mark answers for the
river transporting toxic materials further downstream were more common than two mark answers, in which
candidates added explanation to the relevant stated information. In stronger answers candidates either
commented on the size of the slick or on the failure of officials to act more quickly to stop or disperse it.
Answers which failed to score any marks referred to transfers of the smoke cloud by winds, instead of
focusing on the toxic leak into the river. Some candidates carried the focus on air pollution and wind
transfers into part (c)(iii) with similarly poor results. Whilst there were few problems with candidate
understanding of the term 'international', some again failed to apply their answers to this example; many
failed to mention Russia in their answers. Without the naming of Russia, answers tended to be general and
unconvincing. In only a few answers were other issues referred to, such as the slowness of the Chinese
authorities to pass on information about the leak. One and two mark answers were more common than
those worth three marks. The quality of answers to part (c)(iv) directly reflected how well individual
candidates had read and studied the information in the boxes. Those who supported their negative answers
with a broad range of points gave the better answers worth three marks or more. However, the majority
were content to make only a couple of points, typically referring to chemicals in river bed sediments and in
fish (with consequences for the food chain), and worth two marks. Those who had made little study the
information were the ones most likely to try to defend positive answers about the accuracy and reliability of
both statements, with few opportunities to progress beyond one mark at best.
The temperature graph in part (d)(i) was accurately and neatly completed by the overwhelming majority of
candidates. The biggest disasters were from those who insisted on drawing a bar graph; they spent a lot of
time for little reward. 'Drawing a line graph' was the instruction in the question; even without this, candidates
could have been expected to know that temperature values (being continuous data) are always shown by a
line in climate graphs. June to September or October, or summer, were the acceptable answers in part
(d)(ii); a minority of candidates gave too restricted a range of months, for some as little as one month. Part
(d)(iii) was well understood in terms of water accumulation during the wet season after months of dry
weather. Supporting answers with values from the graph was the best way to ensure full marks. Some,
however, used values only to support the statement given in the question without any attempt to answer it.
Worse still were answers focused on explaining why the floods were greater in June and July! April and May
were considered to be equally good choices in part (iv); once chosen, as they were by most candidates,
explanation proved to be straightforward. The most obvious answer to part (v) was irrigation; drought
resistant varieties of seeds was another possibility. The main weakness of answers to this part was not
choice of method, but description that was adequate for all three marks, which was often lacking.
The best answers to part (e)(i) stated two benefits that were clearly different such as fertile silt deposits and
topping up water stores (both surface and underground) for use during dry months of the year. Another
common answer was a crop like padi that is grown in standing water. Vaguer answers about crops being
watered were limited to one mark at best. In part (ii), most candidates chose bad effects. Many answers

UCLES 2008

316

5014 Environmental Management November 2008

contained
BALOL the breadth necessary for three or four marks to be awarded to them. Although good effects was
the minority choice, some well explained answers were seen, which went well beyond the two benefits
already stated in answers to the previous part. Particularly strong were answers based on examples such as
the Ganges, Indus and Nile. However, merely repeating statements from the previous answer gained no
new credit until it was extended to include explanation. Although most candidates focused more on
explaining only one viewpoint, there were many sensibly balanced answers in which the need for good flood
control was stressed in order to obtain maximum benefits.
Question 5 examined familiar, previously visited, topic areas, for which the great majority of candidates were
well prepared. As always, the key to a high total mark was consistency of answering between the different
parts, which included answering well those parts worth four marks. Some candidates let too many marks slip
away by giving answers to (a)(iv), (c)(iv) and (e)(ii) which were too brief or too narrow in their coverage.
Question 6
Successful answers to part (a) had to be based on what could be observed from the photograph. Of course,
knowledge of savanna vegetation was helpful, but only provided that it did not take over and dominate the
answer. There were, for example, references to variations in vegetation cover between wet and dry seasons
and information about baobab trees that could not receive any credit. However, the trees in leaf indicating
that it was the wet season was credited. Candidates who concentrated on describing what could be seen,
such as the mixed cover of grasses, bushes and trees dotted around, soon reached full marks with only a
small amount of further supporting detail.
In part (b)(i), one way of claiming the first mark was by simply naming the process 'photosynthesis'. Up to
three more marks were available for more detailed explanation about how it works. Three of the possible
four marks for this part were regularly claimed by candidates. Likewise candidates could gain up to four
marks for their answers to (b)(ii). These were claimed much less frequently since the sources for continued
supplies of nutrients, such as from rock weathering, nutrient recycling or new surface deposits like river silt,
did not make up the main part of most answers. Instead the majority of candidates spent too much time
describing the role of plant roots in absorbing nutrients. Full and effective answers to both parts allowed
good candidates with real knowledge and understanding to be identified.
One mark answers to part (c)(i) were as common as two mark answers, because of candidate failure to
mention energy flows from the producer to the consumer shown. Yet virtually every candidate understood
how the diagram showed part of a food chain. Part (c)(ii) was well answered, with most not only mentioning
a possible consumer of the giraffe, but also taking the food chain to the tertiary consumer level with
references to humans or to the decomposers at the end of the food chain. This was probably the best
answered part of the second question.
The underlying message in the diagram used in part (d) was that the Earth's land area and basic natural
resources such as rock, solar energy and water had remained the same, while the human population had
massively increased, resulting in a decrease in natural flora and fauna and its replacement by a large area of
agricultural land. The increase shown in the amount of carbon dioxide was a reflection of forest clearances
and use of resources not shown, notably fossil fuels. The questions in parts (i) and (ii) fed into part (iii), and
gave candidates the opportunity to demonstrate how well they understood the main message. Accurate
answers to part (d)(i) were solar energy, water, the Earth's land area and rock. The worth of quite a few
answers was reduced by the inclusion of carbon dioxide. The best answers to part (d)(ii) included reference
to the decrease in size and extent of the Earth's natural ecosystems of plants and animals, however
expressed. The main answer spoiler here was 'Earth's land area', when stated without any qualification
about some of it having been taken into agricultural use. A few candidates attempted to give increases
instead of decreases. The effectiveness of many answers to part (d)(iii) was reduced by list-like statements
such as 'great increase in number of people' and 'increase in agricultural land' without any comment or
information use that would have demonstrated understanding. Answers to these three parts led to wide
mark variations between candidates, with all marks from zero to four in regular use.
Although there was considerable variation in answer quality in parts (e)(i) to (e)(iii), the mark distribution
between candidates was more regular. The shortest and clearest answers to (e)(i) were those in which the
candidates used the most appropriate terms 'hunting' and 'collecting' (or 'gathering'). Without the use of
these terms, it was sometimes difficult to work out exactly what the candidate meant; these made up many of
the one mark answers. Some answers obviously strayed into cultivation and pastoral farming, which made it
more difficult (but not impossible) for them to gain marks in (e)(ii). The most clearly acceptable answers in
this part were minimal environmental impact or sustainability (however expressed) for advantage and the
variability of food supplies or limited opportunities for advance (however expressed) for disadvantage.

UCLES 2008

317

5014 Environmental Management November 2008

Examiners
BALOL marked the many other answers suggestions on their merits. Despite a few answers of 75%,
most answers to (e)(iii) were correct, either 25% or one quarter (the preferred way for candidates in some
countries to express the answer). In the final part, (e)(iv), the full range of marks was in regular use. As in
the earlier six mark question in part (b), candidates were allowed a four to two ratio of marks between the
two parts, which enabled particularly strong answers to one part to be more adequately rewarded. Much
less candidate confusion between fertilisers and pesticides was observed than in previous examinations.
Some of the fullest answers were for the choice of 'new varieties of seeds and animals', especially if
references were made to both high yielding and genetically modified varieties. Conversely, many of the
weakest answers were for 'modern technology' with unsubstantiated references to faster and more efficient
food output quite widespread. The disasters came from candidates who did not use any of the three reasons
shown in the bullet points on the diagram.
In parts (f)(i) and (iii) not all candidates interpreted the graphs correctly to arrive at the answers 'other
temperate forests' and 'tropical rain forest', although the majority did. For many these were the only two
marks in part (f). Too many candidates ignored the all-important 'between ecosystems' part of the question
when answering part (ii). Instead they gave reasons for increasing percentage losses of forests in general.
Even among candidates attempting to answer the question set, there was a dearth of precise information
about any of the ecosystems in relation to opportunities for people and agriculture, either favourable or
unfavourable. Without real knowledge of any of the three techniques named in part (iv), the best that most
candidates could hope for was one mark for replanting trees. Some tried to work out or guess what
community forestry and agro-forestry meant, but rarely did they get any further than planting trees. The
choice of easy as the basis for answering part (v) often followed on from non-scoring answers to part (iv).
In contrast, answers based on knowledge in (iv), often about selective logging of hardwood trees, were
typically followed by strong answers about the difficulties of controlling logging companies and individuals
tempted by high profits and good incomes. Their answers stood out as beacons of light among the dark
mass of weak answers.
Most candidates finished Question 6 with a lower mark than the one obtained from answering Question 5
as a result of the poor finish in part (f). As usual, this type of summary applied less to able, high scoring
candidates, whose performances between the two questions were more balanced. Also candidates from a
few Centres bucked the general trend, especially when either community forestry and selective logging were
well known by their candidates.

UCLES 2008

318

BALOL
UNIVERSITY OF CAMBRIDGE INTERNATIONAL EXAMINATIONS
General Certificate of Education Ordinary Level

*3881632154*

5014/01

ENVIRONMENTAL MANAGEMENT
Paper 1

May/June 2009
2 hours 15 minutes

Candidates answer on the Question Paper.


Additional Materials:

Ruler

READ THESE INSTRUCTIONS FIRST


Write your Centre number, candidate number and name on all the work you hand in.
Write in dark blue or black pen.
You may use a soft pencil for any diagrams, graphs or rough working.
Do not use staples, paper clips, highlighters, glue or correction fluid.
DO NOT WRITE IN ANY BARCODES.
Answer all questions.
All questions in Section A carry 10 marks
Both questions in Section B carry 40 marks.
At the end of the examination, fasten all your work securely together.
The number of marks is given in brackets [ ] at the end of each question or part question.
For Examiners Use
1
2
3
4
5
6
Total

This document consists of 25 printed pages and 3 blank pages.


SPA (SHW 00245 4/08)
UCLES 2009

[Turn o

319

BALOL
1

Section A

(a) Look at the maps, which show countries with the largest known uranium ore deposits
and countries which used the largest amounts of uranium in 2007. Uranium is a nuclear
fuel.

For
Examiners
Use

Countries having the largest uranium deposits

Russia
Canada
Kazakhstan
USA

Niger
Equator

key
percentage of world
uranium deposits

Brazil
Namibia
Australia
South Africa

30
25
20
15
10
5
0

Countries having the largest uranium use

Russia
UK
France

Ukraine
Germany

USA

South
Korea

Japan

Equator

key
percentage of world
uranium use
30
25
20
15
10
5
0

UCLES 2009

5014/01/M/J/09

320

BALOL (i) Australia has 28% of the worlds known uranium ore deposits. Plot this information
on the map, using the key provided.
(ii)

[1]

For
Examiners
Use

Using the maps, which three countries were the largest importers of uranium in
2007?
............................................................................................................................ [1]

(iii)

What do the locations of the importing countries have in common?


............................................................................................................................ [1]

(b) How are countries, such as Namibia and Niger, likely to benefit from having large
uranium ore deposits?
..........................................................................................................................................
..........................................................................................................................................
..........................................................................................................................................
..........................................................................................................................................
.................................................................................................................................... [3]
(c) Explain why some countries are planning to expand nuclear power generation.
..........................................................................................................................................
..........................................................................................................................................
..........................................................................................................................................
..........................................................................................................................................
..........................................................................................................................................
.................................................................................................................................... [4]

UCLES 2009

5014/01/M/J/09

[Turn over

321

4
2 BALOL
(a) Look at the diagram, which gives information about the spread of bilharzia.

eggs in
faeces and urine

worms mate
inside human body
water surface
larvae hatch
and enter snail
worm
hatches
into water

(i)

For
Examiners
Use

worm bores into skin


of people in the water

Why are water snails known as hosts?


............................................................................................................................ [1]

(ii)

Why is bilharzia known as a water-based disease?


............................................................................................................................ [1]

(iii)

Why are irrigation canals ideal breeding grounds for snails?


............................................................................................................................ [1]

(b) How could the hazard of bilharzia be reduced?


..........................................................................................................................................
..........................................................................................................................................
..........................................................................................................................................
..........................................................................................................................................
.................................................................................................................................... [3]

UCLES 2009

5014/01/M/J/09

322

BALOL
(c) How do water-related diseases, such as bilharzia, affect humans and their activities?
..........................................................................................................................................

For
Examiners
Use

..........................................................................................................................................
..........................................................................................................................................
..........................................................................................................................................
..........................................................................................................................................
.................................................................................................................................... [4]

UCLES 2009

5014/01/M/J/09

[Turn over

323

6
3 BALOL
(a) Look at the climate graph.

temperature
(C)

10

10

10

10

15

15

50
40
precipitation 30
(mm)
20
10
0

(i)

For
Examiners
Use

temperature
(C)

50
40
30 precipitation
(mm)
20
10
0

Calculate the temperature difference between the warmest and coldest months
(annual temperature range).
..................................................... C [1]

(ii)

Which season has the most precipitation?


.......................................................... [1]

(iii)

Circle the term which best describes the temperature of the warmest month.
very hot

warm

cool

very cold
[1]

(iv)

Name the type of climate shown in the graph.


.......................................................... [1]

(v)

Circle the range of latitudes within which most areas with this climate are found.
30-45

45-60

60-75

75-90
[1]

UCLES 2009

5014/01/M/J/09

324

BALOL(vi) State two reasons why this climate is unsuitable for agriculture.
..................................................................................................................................

For
Examiners
Use

..................................................................................................................................
..................................................................................................................................
............................................................................................................................ [2]
(b) Explain why the ozone hole is still present in high latitudes, even though measures have
been taken to reduce it.
..........................................................................................................................................
..........................................................................................................................................
..........................................................................................................................................
..........................................................................................................................................
.................................................................................................................................... [3]

UCLES 2009

5014/01/M/J/09

[Turn over

325

8
4 BALOL
(a) Look at the photograph, which shows a method used by farmers in many parts of the
world.

(i)

For
Examiners
Use

Describe what has been done to allow farming to take place in this area.
..................................................................................................................................
..................................................................................................................................
..................................................................................................................................
..................................................................................................................................

(ii)

Why has it been done?


..................................................................................................................................
..................................................................................................................................
..................................................................................................................................
..................................................................................................................................
..................................................................................................................................
[5]

UCLES 2009

5014/01/M/J/09

326

BALOL
(b) The area in the photograph has a dry climate. Describe the methods farmers could use
to grow crops
(i)

For
Examiners
Use

if irrigation water is available,


..................................................................................................................................
..................................................................................................................................
..................................................................................................................................
..................................................................................................................................
..................................................................................................................................

(ii)

if irrigation water is not available.


..................................................................................................................................
..................................................................................................................................
..................................................................................................................................
..................................................................................................................................
..................................................................................................................................
[5]

UCLES 2009

5014/01/M/J/09

[Turn over

327

10

BALOL

Section B

For
Examiners
Use

(a) World population keeps on growing.


World population growth
9
8
7
6

billions 5
of
people 4
3
2
1
0 1804
1800

1922

1850

1900

1959

1950

1974

1987 1999

2013

2028

2000

year

2048

2050
expected

(i)

Draw in the line on the graph to show world population growth.

[1]

(ii)

How many years did it take for the world population to grow from one to two
billion?
............................................................................................................................ [1]

(iii)

What was the least number of years it took for world population to increase by one
billion?
............................................................................................................................ [1]

(iv)

Describe what the graph shows about expected future population growth compared
with past growth.
..................................................................................................................................
..................................................................................................................................
..................................................................................................................................
............................................................................................................................ [2]

UCLES 2009

5014/01/M/J/09

328

11

BALOL
(b) Some countries have population policies to reduce rates of population growth.
(i)

For
Examiners
Use

Name one country which has a population policy.


..................................................................................................................................

(ii)

Describe its main features.


..................................................................................................................................
..................................................................................................................................
..................................................................................................................................
..................................................................................................................................
..................................................................................................................................

(iii)

Comment on whether or not it has been successful.


..................................................................................................................................
..................................................................................................................................
..................................................................................................................................
[5]

(c) State two reasons why governments in some developing countries have not introduced
a population policy.
..........................................................................................................................................
..........................................................................................................................................
..........................................................................................................................................
.................................................................................................................................... [2]

UCLES 2009

5014/01/M/J/09

[Turn over

329

12

BALOL
(d) More food needs to be produced to feed the worlds growing population. Four agricultural
techniques used by farmers to increase food output are given on the spider diagram
below.
A
fertilise the soil

For
Examiners
Use

B
spray with pesticides

ways to
increase yields

D
increased use of
machinery

C
use irrigation water

Choose two of the techniques named. For each one,


(i)

briefly explain how it increases yields;

(ii)

state one disadvantage of its use.


Choice .......................................................
Explanation ..............................................................................................................
..................................................................................................................................
..................................................................................................................................
Disadvantage ...........................................................................................................
............................................................................................................................ [3]
Choice .......................................................
Explanation ..............................................................................................................
..................................................................................................................................
..................................................................................................................................
Disadvantage ...........................................................................................................
............................................................................................................................ [3]

UCLES 2009

5014/01/M/J/09

330

13

BALOL
(e) Another way of increasing farm yield is to use new seeds.

For
Examiners
Use

Sowing the seeds of change


In 1959 researchers in the Philippines cross-bred two rice plants: a semi-dwarf plant from China
with a strong, tall Indonesian plant. The result was a sturdy short plant called IR8. How it compared
with traditional varieties of rice plants is shown below.
Old plant

New plant

grows rapidly

shorter, stronger
plant

tall plant, can fall


over easily

can be planted
close together,
needs little space

needs to be
planted far apart

needs fertiliser
and pesticides

5 months
growing season

4 months
growing season

average yield 1.5


tonnes per ha

average yield 5.0


tonnes per ha

(i)

Why was this method given the name Green Revolution?


............................................................................................................................ [1]

(ii)

From the information above, give two different reasons why use of the new seeds
resulted in higher food output.
..................................................................................................................................
..................................................................................................................................
..................................................................................................................................
............................................................................................................................ [2]

(iii)

State one reason why the new plants were less at risk from bad weather during the
growing season than the old plants.
..................................................................................................................................
..................................................................................................................................
............................................................................................................................ [2]

UCLES 2009

5014/01/M/J/09

[Turn over

331

14

BALOL(iv) After new seeds were introduced, the wealth gap between rich large farms and poor
small farms became wider. Explain why the introduction of new seeds favoured rich
farms with large areas of land.

For
Examiners
Use

..................................................................................................................................
..................................................................................................................................
..................................................................................................................................
..................................................................................................................................
............................................................................................................................ [3]
(f)

About 30 years ago, research scientists started to develop GM (genetically modified)


crops. The bar graph shows world total area planted with GM crops for the 10 years
from 1996 to 2005.
Global GM plantings 1996-2005
(thousands of hectares)
100 000
90 000
80 000
70 000

60 000
GM crop
plants
50 000
(thousands
of hectares) 40 000
30 000
20 000
10 000
0

1996

1997

1998

1999

2000

2001

2002

2003

2004

2005

year
(i)

Describe what the graph shows about the growth in area planted with GM crops
before and after 1999.
..................................................................................................................................
..................................................................................................................................
..................................................................................................................................
............................................................................................................................ [2]

UCLES 2009

5014/01/M/J/09

332

15

BALOL(ii)

Global GM plantings by country in 2005

For
Examiners
Use

0
10

90

0
90

Percentages of global total area


USA
55
Argentina
20
Brazil
10
Canada
7
China
4
16 other countries 4

80

70

10

80

20

70

30

60
60

20

30

40
50

40

50
Complete the pie graph.
(iii)

[3]

The largest GM seed producing companies are American. They were full of hope
that their GM seeds would be used world-wide, in the same way as new seeds
were during the Green Revolution of the 1960s.
By 2005 the use of GM crops had not been as widespread and successful as the
companies had hoped. What evidence from the bar and pie graphs supports this
statement?
..................................................................................................................................
..................................................................................................................................
..................................................................................................................................
..................................................................................................................................
............................................................................................................................ [3]

UCLES 2009

5014/01/M/J/09

[Turn over

333

16

BALOL
(g)

Comments about GM crops

GM crops produce more food, more


reliably. We have created corn with a
natural pesticide; if an insect attacks the
plant, the insect will die, not the plant.
We have made a tomato that withstands
frost.

Supermarket boss in Europe

We are worried about the introduction of


unnatural plants into the environment.
The environment is a delicate system of
checks and balances, easy to upset.
Our fear is that GM crops will be harmful
to surrounding plant and insect life.

Politician in a poor African country

(i)

For
Examiners
Use

GM research scientist

Public opinion in Europe is hostile to


GM crops. It would be commercial
suicide for our company to start selling
foods made from GM crops. While there
is no evidence that eating GM foods is
harmful, many people are unwilling to
take the risk.

Environmental group

As usual, the big GM companies are


concentrating on rich countries.
They are unwilling to sell seeds in poor
countries, where their profits will be less.
We have not seen any of the benefits
so far and probably never will.

State different reasons why fewer GM crops are grown in Europe and Africa than in
the Americas (North and South).
..................................................................................................................................
..................................................................................................................................
..................................................................................................................................
............................................................................................................................ [2]

UCLES 2009

5014/01/M/J/09

334

17

BALOL(ii) Would you expect there to be a great increase in the global area of GM crop
plantings in the next 10 years?

For
Examiners
Use

..................................................................................................................................
..................................................................................................................................
..................................................................................................................................
(iii)

In your opinion, should there be a big increase in plantings of GM crops? State and
explain your views about this.
..................................................................................................................................
..................................................................................................................................
..................................................................................................................................
..................................................................................................................................
..................................................................................................................................
[4]
[Total: 40]

UCLES 2009

5014/01/M/J/09

[Turn over

335

18
6 BALOL

Information about the earthquake in Peru (August 2007)

On Wednesday 18th August 2007 at 18.41 local time, an undersea earthquake registering 7.9
on the Richter scale struck Perus coastal province of Ica. The epicentre was about 150 km south
east of Lima, the capital city.

For
Examiners
Use

key
city/town
earthquake
epicentre and
shockwaves
PERU

Lima

Pacific Ocean

land
Imperial

sea

Pisco
Ica

200

400 km

Reports from coastal cities within the first 24 hours


Pisco The first estimate was that 70% of the city was in ruins. Most houses had fallen so also
had churches and hotels. It was a city without lights, water and communications. Hundreds were
trapped in buildings.
Imperial The first estimate was that 80% of the adobe (mud) brick houses had fallen. Survivors
lit fires in their ruined homes during the cold winter night.
Ica People sifted through the rubble of the main church, which collapsed during a service when
packed with worshippers.
Lima People stood trembling in the streets as buildings shook around them.
The final death toll was estimated at between 450 and 500.

UCLES 2009

5014/01/M/J/09

336

19

BALOL
(a) State where the centre of the Peru earthquake was located in 2007.
..........................................................................................................................................

For
Examiners
Use

.................................................................................................................................... [1]
(b) (i)

How far away from the centre were the effects of the earthquake felt?
............................................................................................................................ [1]

(ii)

Describe how the impacts of the earthquake varied with distance from the centre.
..................................................................................................................................
..................................................................................................................................
..................................................................................................................................
..................................................................................................................................
............................................................................................................................ [3]

UCLES 2009

5014/01/M/J/09

[Turn over

337

20

BALOL
(c) The partly completed graph shows estimates for the percentage chance of damage to
buildings, according to types of building materials and earthquake strength.

For
Examiners
Use

Percentage estimates for damage to buildings in earthquakes


100
90
80
70
60
percentage
estimate of
damage

50
40
30
20
10
0
5

Richter scale
key
mud (adobe) and hand-made bricks
concrete and factory-made bricks
buildings with a steel frame
(i)

Percentage estimates for an earthquake Richter scale 9


mud (adobe) and hand-made bricks
concrete and factory-made bricks
buildings with a steel frame

100
33
20

Complete the bar graph by showing percentages for a Richter scale 9 earthquake.
[2]

UCLES 2009

5014/01/M/J/09

338

21

BALOL(ii) Describe what the graph shows about the percentage chance of damage to buildings
according to increasing earthquake strength and types of building materials used.

For
Examiners
Use

Increasing earthquake strength ................................................................................


..................................................................................................................................
..................................................................................................................................
Types of building materials .......................................................................................
..................................................................................................................................
............................................................................................................................ [3]
(iii)

Was the damage to buildings in the Peru earthquake similar to that expected for an
earthquake around 8.0 on the Richter scale? Explain as fully as you can from the
information given.
..................................................................................................................................
..................................................................................................................................
..................................................................................................................................
..................................................................................................................................
..................................................................................................................................
............................................................................................................................ [3]

UCLES 2009

5014/01/M/J/09

[Turn over

339

22

BALOL
(d) Look at the world map of major earthquake zones. All ten cities named have more than
7 million inhabitants.

For
Examiners
Use

Major earthquake zones

Tehran
Karachi

Los Angeles

Tokyo
Dhaka
Manila

Mexico City
Bogota
Jakarta

Lima

key
earthquake zone
large city

(i)

Describe the main features of the world distribution and pattern of major earthquake
zones shown on the map.
..................................................................................................................................
..................................................................................................................................
..................................................................................................................................
..................................................................................................................................
..................................................................................................................................
..................................................................................................................................
............................................................................................................................ [4]

(ii)

Explain why earthquakes occur less frequently, or not at all, outside the major
earthquake zones shown.
..................................................................................................................................
..................................................................................................................................
..................................................................................................................................
............................................................................................................................ [2]

UCLES 2009

5014/01/M/J/09

340

23

BALOL(iii) Choose one country in which the earthquake risk is high. Explain why strong
earthquakes occur regularly in that country.

For
Examiners
Use

Name of country .......................................................................................................


Explanation ..............................................................................................................
..................................................................................................................................
..................................................................................................................................
..................................................................................................................................
..................................................................................................................................
..................................................................................................................................
............................................................................................................................ [4]
(e) The large cities named on the map make a big contribution to national wealth, for example
Tehran contributes 40% of Irans national income each year. One big earthquake could
wreck the economy of Iran.
Tehran lies in such an earthquake-prone area that some have suggested moving the
whole city of 12 million people to a safer location.
(i)

Why is this unlikely to happen?


..................................................................................................................................
..................................................................................................................................
..................................................................................................................................
..................................................................................................................................
..................................................................................................................................
............................................................................................................................ [3]

(ii)

Describe three strategies used in cities in developed countries, such as Tokyo and
Los Angeles, to save lives when an earthquake strikes.
..................................................................................................................................
..................................................................................................................................
..................................................................................................................................
..................................................................................................................................
..................................................................................................................................
..................................................................................................................................
............................................................................................................................ [4]

UCLES 2009

5014/01/M/J/09

[Turn over

341

24

BALOL
(f)

Big cities and natural disasters


UN Report 2007
*
The number of natural disasters affecting urban populations has increased by four
times since 1975

For
Examiners
Use

Causes identified
*
World population growth
*
Growth of big cities and densely packed urban areas
*
Slums springing up in disaster-prone areas, such as on steep slopes, next to
swamps
*
Coastal locations of many big cities, with increasing risks from flooding and
tsunamis
Main conclusion
*
That the death toll from natural disasters affecting big cities does not have to keep
increasing
(i)

Big city growth is a much greater problem in developing than in developed countries.
Why?
..................................................................................................................................
..................................................................................................................................
..................................................................................................................................
..................................................................................................................................
............................................................................................................................ [3]

(ii)

Problems caused by natural disasters in coastal locations are similar in big cities in
both developing and developed countries. Explain why.
..................................................................................................................................
..................................................................................................................................
..................................................................................................................................
..................................................................................................................................
............................................................................................................................ [3]

UCLES 2009

5014/01/M/J/09

342

25

BALOL(iii) How likely is it that the death toll from natural disasters affecting large cities will
stop increasing, as the UN says it should? State and explain your views about this.

For
Examiners
Use

..................................................................................................................................
..................................................................................................................................
..................................................................................................................................
..................................................................................................................................
..................................................................................................................................
..................................................................................................................................
..................................................................................................................................
............................................................................................................................ [4]
[Total: 40]

UCLES 2009

5014/01/M/J/09

343

BALOL
UNIVERSITY OF CAMBRIDGE INTERNATIONAL EXAMINATIONS
GCE Ordinary Level

MARK SCHEME for the May/June 2009 question paper


for the guidance of teachers

5014 ENVIRONMENTAL MANAGEMENT


5014/01

Paper 1, maximum raw mark 120

This mark scheme is published as an aid to teachers and candidates, to indicate the requirements of
the examination. It shows the basis on which Examiners were instructed to award marks. It does not
indicate the details of the discussions that took place at an Examiners meeting before marking began,
which would have considered the acceptability of alternative answers.
Mark schemes must be read in conjunction with the question papers and the report on the
examination.

CIE will not enter into discussions or correspondence in connection with these mark schemes.

CIE is publishing the mark schemes for the May/June 2009 question papers for most IGCSE, GCE
Advanced Level and Advanced Subsidiary Level syllabuses and some Ordinary Level syllabuses.

344

Page 2

Mark Scheme: Teachers version


GCE O LEVEL May/June 2009

Syllabus
5014

Paper
01

BALOL
1

(a) (i) plot for Australia (28%) with correct shading, bar width and length and appropriately
positioned
[1]
(ii) USA, France and Japan

[1]

(iii) Northern Hemisphere / Developed world

[1]

(b) export revenues


can be used to develop the country / industry / infrastructure etc.
potential to develop nuclear power (with sufficient finance)
employment
revenues would raise living standards
avp

[3]

(c) exhaustion of fossil fuels


cleaner than fossil fuels / concern over air pollution from fossil fuels
does not produce greenhouse gases / fossil fuels produce greenhouse gases
concern about global warming
does not produce acid rain / fossil fuels cause acid rain
wish to diversify sources of energy / not rely on potentially volatile areas of supply of gas / oil
rapid industrial growth in some countries
only need to import small quantities of uranium
advantages compared with other alternative energy sources
avp
[4]
2

(a) (i) larvae hatch into worms inside them

[1]

(ii) the carrier lives in water

[1]

(iii) still / shallow water / water present all year

[1]

(b) install toilets / sanitation / treat human waste before it enters the water
do not enter the water
wear impermeable cover on feet and legs
control / clear snails
drain water bodies containing snails
introduce fish predators to eat snails
avp

[3]

(c) weakness
anaemia
kidney / bladder infections
death
reduced resistance to other diseases
* reduced ability to work
* reduced agricultural output / food supply
avp
at least one * point for max.

[4]

UCLES 2009

345

Page 3

Mark Scheme: Teachers version


GCE O LEVEL May/June 2009

Syllabus
5014

Paper
01

BALOL
3

(a) (i) 22

[1]

(ii) summer

[1]

(iii) cool

[1]

(iv) Tundra

[1]

(v) 6075

[1]

(vi) too cold / short summer


short / two / three months growing season
soil / ground often frozen / frost / snow covered for long periods
strong winds
low precipitation
avp

[2]

(b) still using CFCs


in (old) refrigerators, air conditioning systems
still using halons
in fire extinguishers
especially in countries which lack alternatives
gases remain in atmosphere for long periods
elaborated upon
avp
4

[3]

(a) (i) terracing / land cut into steps


flat areas
across slope / parallel to contours
narrow(er) on steep(er) slopes / wide(r) on gentle(r) slopes
bunds / low walls at the edges
(ii) steep slope
(terrace) encourages / increases infiltration
decreases run-off
conserves soil / soil not washed down the slope (terrace or bund)
contour ploughing so water sinks into furrows
plough ridges prevent water from flowing downhill
easier farming practices
minimum 1 mark for each part

[5]

(b) (i) method of storage of water described


channels / pipes bring water from storage
sprinkler / other suitable method on terrace (to max 2)
care needed not to over water / not to induce run off
wells dug to tap into underground water supplies
avp

UCLES 2009

346

Page 4

Mark Scheme: Teachers version


GCE O LEVEL May/June 2009

Syllabus
5014

Paper
01

BALOL
(ii) dry land farming
widely spaced plants, no competition for small amount of water available
mulch / straw etc. on surface, protects from evaporation
protects from wind erosion
grass strips between crops, reduce soil erosion explained
GM drought resistant crops
Plant seeds / crops after the rain
Avp
Allow well developed points to 2 as above. Max. 3 each part.
5

(a) (i) Line drawn linking the dots on the graph

[5]
[1]

(ii) 118 years

[1]

(iii) 12 years

[1]

(iv) growth expected to slow down,


especially after 2028,
expected to take 20 years to increase by one billion (from 14 and 15 years previously),
growth still a lot faster than past growth up to 1959.
Basic point = 1 mark Elaboration based on graph use = 2nd mark

[2]

(b) (i) No mark for the country name, but an acceptable name will be essential for full marks
from the other two parts of the question.
(ii) Reference to methods for achieving aims general features of population policies
include education and family planning advice, ready availability of contraceptives,
permissable use of sterilisation and abortion, statutory limits on numbers of children with
penalties, financial incentives, and promoting social change such as later marriages.
Policies can be everything from strong and enforced, as in China, to weak and voluntary,
as in the many African and Middle Eastern countries.
Up to three marks for general answers not applied well, or at all, to a named country
Up to four marks for answers which demonstrably refer to the named country
(iii) Basic comment on success = 1 mark. Reserve one mark for this part, which can only be
gained in the context of an accepted named country.
Up to three marks available for this part for reference to advantages and/or
disadvantages of the population policy and comments, which are clearly relevant to the
named country.
[5]

UCLES 2009

347

Page 5

Mark Scheme: Teachers version


GCE O LEVEL May/June 2009

Syllabus
5014

Paper
01

BALOL
(c) Possible reasons:
poverty / lack of funds,
other more pressing needs e.g. recovering from natural / human disasters,
against social traditions / religious beliefs,
political corruption and inefficiency,
oil rich / wealthy so no need to worry about population growth,
population growth considered necessary for international status / against rivals
Two reasons @1 mark

[2]

(d) Fertiliser;
improves soil fertility,
replaces nutrients taken out of soil by previous crops,
organic fertilisers maintain soil texture / structure
... or similar points
disadvantage possible choices include cost, over-use of chemical fertilisers leading to
leaks into surface water courses / underground stores, unfavourable environmental
consequences.
Pesticides;
destroys insects etc. which eat / damage crops,
kills weeds which would compete for nutrients / water with the crops
... or similar
disadvantage chemical pesticides also kill other useful insects, destroy habitats for birds
and wildlife, has knock-on effects in natural food chains
Irrigation water;
water allows plant growth when rainfall is too little or too unreliable,
allows use of otherwise favourable conditions for crops (such as fertile soils, sunshine),
increases size of crops / fruit and the amount produced
... or similar
disadvantage overuse leading to salinisation, costs (of all types) of building infrastructure
for supply (especially large dams), competing use for a scarce natural resource
Machinery;
sowing/spraying etc. done more evenly / in a more controlled way, more can be done more
quickly while weather conditions are favourable, large scale / more efficient operations
possible,
larger areas brought into cultivation by new technology
... or similar
disadvantage heavy machinery compresses soil / damages soil structure, encourages loss
of wildlife habitats through clearance of vegetation for easy use of machinery, high costs to
small farmers in developing countries
In general, it will be two marks for explanation and one for disadvantage, because only one
disadvantage is required; however, good / full elaboration about the one disadvantage can
receive two marks when the explanation is not worth two marks. Keep one mark for each
part.
2 @3 marks = 6 marks

[3][3]
UCLES 2009

348

Page 6

Mark Scheme: Teachers version


GCE O LEVEL May/June 2009

Syllabus
5014

Paper
01

BALOL
(e) (i) Revolution = change / big difference
Green = referring to plants / output from farming
(as opposed to earlier Industrial Revolution)
Decent understanding shown for the mark

[1]

(ii) new seeds


more plants per hectare because they can be planted closer together,
yield 3.5 tonnes per hectare more / more than three times greater,
shorter growing season allows more chance of growing a second crop
Two reasons such as these, clearly stated

[2]

(iii) Old seeds are tall plants, more prone to falling over in poor weather,
suggested weather conditions named such as strong winds or heavy rain
Growing season for new seeds is one month shorter,
less time for adverse weather to affect them I harvested one month earlier before
weather changes
Reason clearly stated = 1 mark
Elaborated upon including references to weather = 1 mark

[2]

(iv) More intensive farming so more inputs are needed / more capital is needed,
for buying fertilisers and pesticides in order to obtain promised high yields,
other costs may be for irrigation water and machinery,
seeds are bought instead of using own seeds from previous year
Once farmers achieve higher output they have the income to keep on investing,
small farmers are more likely to get into debt if not successful,
larger farmers buy more land for increased output and profits
Points made along these lines, with something from both parts for full marks
(f)

[3]

(i) More rapid rate of growth before 1999 than after,


elaborated upon by evidence / use of data e.g. size of increase in three years up to 1999
took five years to achieve after 1999,
some speeding up of annual increases from 2003 without ever matching the big increase
from 1997 to 1998
General point made for one mark
Further detail for the second mark.

[2]

(ii) Correct plot of pie graph percentages = 2 marks


One obvious error or just minor errors = 1 mark
Countries for sectors clearly identified = 1 mark

[3]

(iii) Bar graph showed annual increases in plantings but without a big change I take-off,
pie graph shows that they were only planted in 21 countries in 2005,
more than half was in the USA which is the home of the GM companies,
four countries account for over 90% of plantings (or similar use of the %s)
Three points made along these lines.

[3]

UCLES 2009

349

Page 7

Mark Scheme: Teachers version


GCE O LEVEL May/June 2009

Syllabus
5014

Paper
01

BALOL
(g) (i) Lack of GM crops in Europe is due to public resistance to them (logical or otherwise).
There may also be an environmental element; although this is not explicitly tied to
Europe in the comments, but it is reasonable to infer that it will apply more in developed
countries in Europe.
Lack of GM crops in Africa is stated as being for economic reasons - not worth selling in
poor countries where prices have to be low for use.
Different reasons clearly stated = 2 marks
One sided, or differences stated more weakly = 1 mark

[2]

(ii) Expect?
No no evidence from the 10 years to 2005 that a big increase will happen,
reasons for not using them such as people's views and poverty do not change
quickly
Yes have great advantages for increased food output, even from areas where
conditions are difficult at present,
world will need more food for its growing population,
crops are being used more for bio-fuels as well,
so that resistance to use might diminish or be overcome
(iii) Should there be? This is personal candidate opinion.
If no, it is most likely to reflect environmental concerns.
If yes, the needs to increase output for feeding people and economic development for
developing countries are likely reasons.
Mark as one as the two answers overlap (even though they are kept separate).
Statement of views with limited or generalised explanation = 1 or 2 marks
Clear statement of views, with meaningful explanatory support = 3 or 4 marks
6

[4]

(a) From map, such as undersea or offshore in province of Ica / Pisco,


or from the introduction, 150 km south east of Lima

[1]

(b) (i) From introduction, up to 150 km away in Lima


From map, allow any distance between 150 and 190 km

[1]

(ii) 70% 80% damage to buildings in places close to epicentre such as Pisco and Imperial,
also all the other signs of severe earthquake shocks like people trapped and
infrastructure failure are present,
whereas 150 km away in Lima there is mention of buildings shaking, but without any
reference to them falling down
Points made along these lines which indicate decline in severity away from the
epicentre.
[3]

UCLES 2009

350

Page 8

Mark Scheme: Teachers version


GCE O LEVEL May/June 2009

Syllabus
5014

Paper
01

BALOL
(c) (i) Plots accurate = 1 mark
Attempted use of the same shading types = 1 mark

[2]

(ii) use of percentages from graph to show examples of increased damage with earthquake
strength e.g. mud 5% to 50% to 100%,
same for the other two types of buildings materials although percentages increases are
less dramatic,
they only reach 33% and 20% for a scale 9 earthquake compared with 100%,
comment about the great importance of building materials / inability of mud bricks to
survive a strong earthquake shock.
Three points made along these lines, with one mark reserved for each of earthquake
strength and building materials.
[3]
(iii) 80% of the adobe brick houses in Imperial had fallen, which is between the percentages
given for Richter scale 7 and 9 where the strength of the Peru earthquake lies,
Pisco is located closer to the epicentre where damage at 70% appears to be less,
but churches and hotels in Pisco, less likely to be made of mud bricks, fell down due to
proximity to the epicentre,
earthquakes of 7 and above cause significant damage as the collapse of the church in
Ica shows,
overall summary that damage reasonably close to what can be expected from an
earthquake of that strength
Three points made along these lines.

[3]

(d) (i) Earthquake zones are found in long belts / relatively narrow zones,
Follow the lines of the plate boundaries,
one runs north south up the west of North and South America,
another goes east-west across Asia and into Europe / Mediterranean,
a third follows the western side of the Pacific Ocean linking island chains
along coasts / islands / borders between continents and seas,
Most of Africa / Australia outside major earthquake zones.
Accept both comments about the world pattern and more detail about alignment / course
followed by the earthquake zones, including named references to places
4 points @ 1 mark

[4]

(ii) Earthquake zones follow the plate boundaries (especially destructive / converging
margins),
earth movements are much less frequent and strong away from plate boundaries,
where earthquakes depend more on the presence of faults
Clear answer showing understanding = 2 marks
Some idea about the importance of plate boundaries = 1 mark

[2]

UCLES 2009

351

Page 9

Mark Scheme: Teachers version


GCE O LEVEL May/June 2009

Syllabus
5014

Paper
01

BALOL
(iii) Maximum 3 marks for general answers about plate movement, causing friction which
leads to shaking of the ground and earthquakes. Likewise for answers which name a
country (e.g. Peru) without including supporting detail specific to that country.
For more than three marks, specific information about the country needs to be included,
most easily done by identifying the type of plate boundary, typically destructive, but
conservative if USA / California is chosen and by naming the plates. Likely country
choices are Pakistan (convergence of Indian and Eurasian plates) or Peru (convergence
of Nazca and South American plates).
4 @ 1 mark

[4]

(e) (i) Reasons include the cost of doing this,


people and companies used to living, working and doing business there,
they have made big investments there,
all the transport infrastructure focuses on Tehran,
anyway, people living in earthquake zones take the view that it will never happen to
them,
logistics of moving so many people,
another approach is that damage from any future earthquake can be limited by good
building techniques and emergency planning.
Point like these, made in a convincing way for the question for three marks.

[3]

(ii) Constructing buildings that are designed to be earthquake-proof; details of how this is
done
Planning for an emergency by having trained emergency teams; equipment ready for
use / trained rescuers and sniffer dogs
Educating people about what to do and what not to do in an earthquake
Land use zoning - keeping industrial zones with oil refineries etc. separate from housing
zones
One mark for each of three clearly different strategies = 3 marks
Final mark available for rewarding quality of statements, or any elaboration = 1 mark [4]
(f)

(i) World population growth is greater in developing than developed countries due to high
birth rates / rates of natural increase,
a lower percentage of the population are urban than in developed countries,
rural to urban migration is widespread and will continue for many more years,
cities in developing countries are full of young people of child-bearing ages,
problems such as housing are not under control.
Three points made along these lines.

[3]

UCLES 2009

352

Page 10

Mark Scheme: Teachers version


GCE O LEVEL May/June 2009

Syllabus
5014

Paper
01

BALOL
(ii) Many of the world's big cities are coastal to benefit from port access, flat land, wet
climate etc.,
cities on the coast are equally at risk from stormy weather / tsunamis,
and from rising sea levels associated with climatic change,
some evidence that severe storms and coastal flooding are increasing,
although more defences / precautions are likely in developed world cities, economic
costs of a disaster are much larger
Three points made along these lines.

[3]

(iii) Likely to stop increasing?


Precautions and preparations exist for all natural disasters flood defences, warning
systems, trained emergency teams, well constructed buildings,
in theory, there is no reason why the death toll should increase, which is the main
message from the UN,
in practice, authorities in big cities in developing countries are struggling to cope with
existing problems,
constant growth means no breathing space to plan ahead to stop worse future problems,
adverse effects of natural disasters increase as the proportion of self-help housing
increases and moves onto less suitable building sites,
overall conclusion may not be the optimistic one hoped for by the UN.
View supported by one or two points / ideas, but limited overall = 1 or 2 marks
Better understanding shown by making a range of points = 3 or 4 marks

[4]

UCLES 2009

353

5014 Environmental Management June 2009

BALOL
ENVIRONMENTAL
MANAGEMENT

Paper 5014/01
Paper 1

General comments
Although there was little noticeable difference in final mark outcomes between the total of the four short
questions in Section A and the average mark total of the two long questions in Section B, there was a
pronounced difference in distribution, since the average mark out of 40 for Question 5 was somewhat higher
than for the total of Section A and for Question 6. Being unable to finish the paper in the time allowed did
not seem to be a major issue, since in only a few scripts were there unanswered parts of questions within
Question 6. However, there were signs of tailing-off in answer quality in the three parts of 6(f) for some
candidates. There remained a general tendency to give over-long and over-elaborate answers to the short
questions in Section A at the expense of full answers to questions in the second half of Question 6.
Once again some outstanding scripts were received from candidates well immersed in the knowledge and
understanding of the physical properties of the natural environment, related human activities and strategies
for management. These candidates produced answers of real quality throughout the examination. At the
lower end of the ability scale, there were some candidates who appeared comfortable only when answering
questions heavily dependent on the source materials provided. They struggled to give effective answers to
questions which placed a higher premium on knowledge and understanding of syllabus content. For all
candidates, consistently the most challenging questions were those which required higher levels of
application, such as 6(a)(iii) based on use of graph information and 6(d)(iii) based on knowledge.
Within Section A, a clear pattern of marks emerged highest total mark from Question 1 (examining a
generally familiar topic), variable marks from high to low for Questions 2 and 3, and significantly lower
marks for Question 4. Many answers to Question 4 suffered from limited description from the photograph
and frequent repetition of answers between parts (b)(i) and (ii), despite the fact that they were totally
different questions.
Within Section B, the overall standard of answers from most candidates to Questions 5 was noticeably
higher than for Question 6. Some of the questions with higher marks attached to them within Question 5
were consistently well answered, notably 5(b) and 5(d); most candidates found the more open questions in
5(g)(ii) and (iii) to be quite accessible (compared with the equivalent question (f)(iii) in Question 6).
Questions which discriminated well between more and less able candidates included (e)(iv), (f)(i) and (f)(iii).
Within Question 6 questions which regularly yielded good marks were more restricted; they tended to be (a),
(b), (c)(i) and (ii) and (e). However, many of the three- and four-mark questions proved to be more
challenging for some, including (c)(iii), (d)(i) and (iii) and all three parts of (f). Overall, the typical mark for
Question 6 was three or four marks lower than for Question 5, even though a majority of candidates
showed plentiful familiarity with the topic of earthquakes, which made up most of the question.
Based on candidate performance in this paper, the following points emerged which might be useful in guiding
future candidates towards improved examination technique.

Begin to answer the actual question set straight away; do not begin by writing out all or parts of the
question itself. Often main mark-earning points were concentrated in the bottom half of the spaces left
for answering, frequently within the last two lines.

Do not just repeat and re-use what is in the question without elaborating or giving examples. Many
candidates failed to give examples of 'bad weather' in 5(e)(iii) and 'natural disasters' in 6(f)(ii).

Look at the number of marks for the question. These are an important guide to the number and range of
points needed for full marks. Questions worth 3, 4 or 5 marks need a broader range of points, or more
elaboration and use of examples, than those worth 1 or 2 marks.

UCLES 2009

354

5014 Environmental Management June 2009

When describing from graphs, candidates should always state or use values from the graph to support
BALOL
their answers. This makes answers less vague and more precise, and speeds up the award of marks.

When asked to give an opinion or view, candidates should state this clearly and support it as strongly
and precisely as possible. Justifying and explaining the viewpoint is more important than the view itself
for earning marks.

Comments on individual questions


Section A
Question 1
Very weak answers were rare. More mistakes were made in answering part (a)(i) than in any other part; a
few candidates drew the bar on the map of uranium use, while others made the bar much wider than those
already drawn. Some missed this part out. A majority, however, tried to use the scale and remained faithful
to the bar width and shading used on the map. Most worked out the correct answers of USA, France and
Japan in (a)(ii). However, those who merely named countries with the tallest bars on the top map in this part
found it almost impossible to find valid answers to (a)(iii). Most answers to part (b) began with export of
uranium for foreign exchange earnings, followed by what this then allowed in terms of development and
improved standards of living. The widest variations in answer quality were to part (c). Some candidates
understood really well the merits of nuclear power compared with fossil fuels and other alternative energy
sources; many answers were seen that would have been worth more than four marks had they been
available. Other candidates over-repeated one or two points by stating one explanation for fossil fuels and
the direct opposite for nuclear energy.
Question 2
Precision in answering was a requirement in part (a), although thanks to the help provided by the diagram,
few candidates failed to gain at least one of the first two marks. In the best answers to part (b) candidates
looked for a range of ways; stating three different ways was the safest course to all three marks. One and
two mark answers reflected more limited responses and sometimes included quite long references to clean
drinking water supplies, which were not really relevant to this question. In part (c) candidates were not
required to stick with bilharzia, although the majority did. Some displayed excellent knowledge of its effects
on people. Equally impressive were some of the answers about effects on peoples' activities especially
those which examined further consequences related to not being able to work and how this contributed to
poverty cycles. The worth of some potentially good answers was reduced by failure to refer to effects for
both humans and their activities. In weak answers three or four lines were filled while making just one
scoring point.
Question 3
Only more able candidates gained all five marks in the first five parts of (a). Consistently the best answered
part was the calculation of temperature range in (a)(i), despite the need to allow for the negative. June or
July to September was as common an answer as summer in (a)(ii). 'Late summer' was a sensible answer
from those who would have liked autumn as well as summer for their answers, while recognising that the
question wanted only one season to be named. It was from part (a)(iii) onwards that a surprisingly large
number of candidates started to go wrong. Whether it was their answer of 22C in (a)(i) which lured them
along the wrong course was hard to determine, but many circled very hot or warm in (a)(iii) followed by
savanna or monsoon instead of tundra in (a)(iv) and 30-45 in (a)(v). Some of these candidates continued
along this wrong route in (a)(vi) by referring to it being too hot and too wet, apparently forgetting that the
climate graph was showing negative temperature values for the majority of months. The typical answer to
(a)(vi) was imprecisely expressed with vague references to low temperatures and low precipitation instead of
cool summer or short growing season. In part (b), (as usual for answers referring to the hole in the ozone
layer), there was a certain amount of candidate confusion with carbon emissions and the greenhouse effect.
Only when candidates mentioned CFCs and/or halons was it certain that they were focused on the ozone
hole. Among those candidates who were, good understanding was shown about why an immediate shortterm fix of the problem is not possible.

UCLES 2009

355

5014 Environmental Management June 2009

Question
BALOL 4
As commented on in previous examination reports, again some candidates did not describe what could be
seen in the photograph. To be fair, the majority did recognise and often began with terraces when answering
part (a)(i). What was lacking was any fuller description, such as creation of flat areas across the slope,
narrower on steeper upper slopes and wider on more gentle lower slopes, and held back by bunds or low
walls. Too many candidates wanted to keep referring to vegetation, but little is to be seen in this area; there
were many references to windbreaks. Part (a)(ii) was much better answered, often in terms of decreased
run-off and likelihood of soil erosion. Broader responses included advantages for farming as well as for good
farming practices. While most candidates concentrated on describing methods of irrigation in part (b)(i) just
as required, a significant number repeated references to water saving methods of irrigation such as trickle
drip and clay pot irrigation in part (b)(ii), or simply extended their answers to include other sources of
irrigation water such as from underground. They did not appear to regard these as methods of irrigation
when answering the second part of the question. As a result there was a dearth of mark-earning references
to dry land farming and drought resistant crops in (b)(ii). The maximum mark for part (b)(i) was three; thus
two or three were the most common marks awarded for answers to part (b), almost always entirely gained
from (b)(i).
Section B
Question 5
While a majority of candidates in (a)(i) added a line on the graph to link all nine dots, some drew only a part
line (usually from 1922 to 2048) and others left the question unattempted. The correct answers of 118 years
in (a)(ii) and 12 years in (a)(iii) were widespread; the most frequent incorrect answers were 122 and 4 years
for candidates who misinterpreted the graph and used 1800. In (a)(iv), among candidates in the lower half
of the ability range, there was a general failure to notice or to focus on the 'expected' growth, which was
marked for them on the horizontal axis of the graph between 2000 and 2050. This led to many one-mark
answers, in which early growth up to 1922 or 1959 was too narrowly compared with later growth (both actual
and expected). In contrast, answers that were well homed in on comparing growth before and after 2000
were more likely to identify expected slowing down compared with the furious growth rate between 1974 and
2000. Many candidates claimed the second mark when they supported this by comparing number of years
for the population to grow by one billion, such as 20 years from 2028 to 2048 compared with the already
calculated 12 years from 1987 to 1999.
Part (b) was one of the best answered questions on the whole paper, even if there was a wide variation in
standard of answer between those candidates with and without knowledge of a national population policy.
China was the almost universal choice, but equally good answers were occasionally seen from candidates
who chose either Thailand or India. Often the description was stronger than the comment, but not always,
and some well structured and argued total answers were presented. Candidates who entered the name of
developed countries such as Switzerland and the USA tended to betray from the start a lack of real
population policy knowledge; at best their answers contained general comments able to be credited with one
or two marks. In part (c) poverty and religious/cultural tradition were the two most commonly cited reasons.
Although there were many two-mark answers, some candidates filled four or more lines referring to or
repeating only one reason, for which no more than one mark was available no matter how good the
elaboration and detail.
Part (d) was another generally well answered question. Each of the four techniques was regularly chosen,
and none of them seemed to lead to full marks more easily than the others. Disadvantages in part (ii) were
particularly well known, even for the likes of heavy machinery damaging soil structure. A good mixture of
environmental and human disadvantages were referred to. One widespread misunderstanding by
candidates emerged; although chemical pesticides harm wildlife and plants when they leak into waterways,
they do not cause eutrophication in streams in the same way that nitrates do when released from fertilisers.
In weaker responses in part (i), what was expected to be explanation was often more description than
explanation; examples included naming different types of machinery and their uses, and methods for
obtaining and distributing irrigation water. There was a good sprinkling of six-mark answers; overall four- and
five-mark answers were common. Answers as low as one and two marks were rare.
In part (e)(i) candidates could not get a satisfactory answer by direct use of the information provided.
Although worth only one mark, this question was a good discriminator. Candidates needed to convey the
idea of major change leading to substantial increases in crop output. Part (e)(ii) was a much easier
question, intended to be accessible to all, since candidates were directed towards direct use of the
information given. Few candidates failed to gain both marks. One mark answers were typical in part (e)(iii),

UCLES 2009

356

5014 Environmental Management June 2009

for
identifying the reason such as shorter and stronger plants or shorter growing season. The failure of many
BALOL
candidates to claim the second mark was due to lack of further explanation. Too many relied upon repeating
'bad weather' from the question without any real application to plant characteristics. The full range of marks
were used in (e)(iv). In non-scoring answers, candidates either tended merely to repeat the previous
information about the advantages of the new seeds without reference to the theme of the question, or simply
argued that large farmers have a larger area of land and therefore can grow more crops. In full mark
answers, candidates referred to large farmers being more likely to be able to afford to buy the new seeds
and fertilisers etc. needed for successful growth, and then rounded off with a comment to explain the
widening gap, such as poor farmers getting into debt after having borrowed money, or large farmers being
able to buy up more and more land from small farmers.
Careful study of the graph in part (f)(i) shows that growth was proportionately greater between 1996 and
1999 than after this date. This was lost on those candidates who merely looked at the increase in the total
amount between 1996 and 2005, which led to the erroneous conclusion of faster growth since 1999. Many
of these candidates just ignored the date of 1999 in the question. It was interesting that some candidates
who failed to score any marks in this part then displayed a much better understanding of what the graph
showed in their answers to part (f)(iii). Unfortunately for them, marks are never transferred between different
parts of separate questions. All correct three-mark answers were the norm in part (ii). A few made a mess
of drawing the graph by identifying the 55% sector for the USA and then attempting to fit all the other sectors
within it, beginning each time from 0 and leaving 45% of the circle not used. In the weakest answers to part
(iii) candidates merely repeated percentages without any comment, for no marks. Most, however, made
sensible use of the percentages to emphasise the dominance of the USA and other countries in the
Americas for GM production. It was not a requirement for all three marks that bar graph evidence was used
as well, but in the majority of three-mark answers candidates supported their answers with its use to show
that no take-off in growth had occurred.
Part (g)(i) was the easy starter for two marks; only the careless, often those who referred to Europe and
Africa as one, lost marks. Parts (g)(i) and (ii) shared a common mark in order to allow maximum flexibility in
marking, to the benefit of individual candidates. A candidate could give totally different answers to each part
and still be fully rewarded, provided that the justification and explanation were strong enough. Many did so;
candidates regularly stated in (ii) that they expected an increase once world food shortages became more
severe than they are at present and then argued against their further introduction in (iii) on the basis of
possible damage to ecosystems ... or the other way around. As always when marking this type of question
in which candidates are asked to express an opinion or justify a view, it was the quality of the support that
controlled the mark level reached. Answers worth one or two marks were often over-reliant upon the
information already given about GM crops earlier in the question and narrow in coverage. Answers worth
three or more marks were more substantial and precise, and usually broader in their scope. Able, articulate
candidates are favoured by this type of question, which is why they are often placed (as here) in the final
part.
Question 6
The three questions that made up parts (a) and (b) were straightforward questions based on use of the map
and information below it. The only significant mark losses were in (b)(ii). Some candidates filled all five lines
just to make the general statement that impacts of the earthquake decreased with distance from the
epicentre. No attempt was made to describe how the amount and type of damage changed with distance
away from the centre. Indeed often there was no mention of any damage, only the repeat of the word
'impacts' from the question itself.
Few candidates failed to complete the bar graph accurately in part (c)(i); if there was a mistake, it was most
likely to be inaccurate plotting of 33% for concrete and factory-made bricks. Two-mark answers were more
common than full three-mark answers to (c)(ii), as many candidates failed to use supporting values from the
graph in at least one of their answers. This was a requirement for full marks for a question with the
instruction to 'Describe what the graph shows'. Part (c)(iii) was another question which separated strong
from weak candidates. The most successful answers came from candidates who used the graph to interpret
likely percentages for damage estimates for a Richter scale 8 earthquake, then stated these and compared
them with damage described for places near the epicentre such as Pisco, Ica and Imperial. The weakest
answers came from candidates who spent most of the time making the narrow, irrelevant point that 7.9 was
only 0.1 different from 8.0 on the Richter scale and therefore damage can be expected to be similar. They
did no more than state that the damage in Peru was about the same. Some candidates mistakenly tried to
explain less damage in Lima on the basis of earthquake proof strengthened buildings, without taking the
city's location over 150 km away from the earthquake epicentre into account.

UCLES 2009

357

5014 Environmental Management June 2009

The
majority of answers to part (d)(i) were disappointing. Instead of focusing on the shading showing the
BALOL
locations of major earthquake zones, as requested in the question, significant numbers of candidates had
eyes only for the cities. Their sole focus was on the relationship between the locations of the cities and
earthquake zones. Only in better than average answers to this question were attempts made to describe the
linear pattern of the earthquake zones, including references to such as from north to south up the west
coasts of the Americas, from east to west from southern Europe through South and Central Asia, and from
north to south following the island chains on the western side of the Pacific. The locational elements of the
world distribution, both positive (as above) and negative (such as absence of zones in most of Africa and
Australia), were left un-noticed by the majority. To be successful, answers to parts (d)(ii) and (iii) needed to
be focused on plate boundaries and what happens there. In too many there were references only to plate
movement, or lack of it in the case of part (ii), without making any telling explanation. To gain all four marks
in (iii) something precise for the country named had to be included, such as San Andreas fault for west coast
USA answers, Nazca and South American plates for Peru or Chile, and Eurasian and one or more from
Pacific, Philippine and Indo-Australian plates for Asian countries. Accurate general explanation for how
earthquakes are caused at destructive and/or conservative boundaries could gain the other three marks, and
did so from more knowledgeable candidates.
Answers to the two parts of (e) were much better. In answering part (i) candidates needed to take number of
marks into account. Some candidates made and re-made only one valid point; while the question did not
specify three reasons, broader answers were more successful than those restricted to just one reason. Most
frequently referred to were the horrendous costs, the impossible logistics and the lack of non-earthquake
locations in Iran, all expressed of course in many different ways. The question in part (ii) specified three
strategies. The ones most commonly and successfully used were education, preparation of emergency
services and earthquake-proof buildings. The fourth mark was awarded on the basis of fuller description,
included for one or more of them.
The reasons for poor candidate performance were probably many and varied, but the questions which made
up part (f) (both individual parts and overall) were the worst answered questions on the paper, by some
margin. Many answers to part (f)(i) suffered from candidates continuing with the previous earthquake theme.
Instead of answering the question set about big city growth, they explained why earthquakes were a much
greater problem in developing than in developed countries. Only the minority who immediately switched to
reasons such as rural to urban migration and high rates of natural population increase gave direct and
relevant answers. Many answers to (f)(ii) went little further than repeating the theme of the question. Some
again were entirely earthquake answers. Only those candidates who showed an awareness of natural
disasters most likely to affect coastal areas, such as tsunamis, cyclones and flooding, were able to develop
answers which matched the theme of the question. These were the ones that were worth two or three
marks. Again many answers to (f)(iii) were entirely earthquake orientated; this time much of the information
already stated in part (e)(ii) was repeated, with only a minimum of comment towards the theme of the
question, usually along the lines that either preparations are more likely in urban than in rural areas or in
developed rather than developing world cities. Only those candidates who attempted to take a broad look at
natural disasters were able to make the range of points expected in a four-mark answer. Answers given by a
few very able candidates stood out for the clarity and variety with which their views were stated and
explained. Included were some perceptive comments about likely continued city growth in the developing
world and the apparent growing frequency and intensity of some natural disasters, particularly climate related
ones.

UCLES 2009

358

BALOL
UNIVERSITY OF CAMBRIDGE INTERNATIONAL EXAMINATIONS
General Certificate of Education Ordinary Level

*0185367153*

5014/01

ENVIRONMENTAL MANAGEMENT
Paper 1

October/November 2009
2 hour 15 minutes

Candidates answer on the Question Paper.


Additional Materials:

Ruler
Protractor

READ THESE INSTRUCTIONS FIRST


Write your Centre number, candidate number and name on all the work you hand in.
Write in dark blue or black pen.
You may use a soft pencil for any diagrams, graphs or rough working.
Do not use staples, paper clips, highlighters, glue or correction fluid.
DO NOT WRITE IN ANY BARCODES.
Answer all questions.
All questions in Section A carry 10 marks.
Both questions in Section B carry 40 marks.
At the end of the examination, fasten all your work securely together.
The number of marks is given in brackets [ ] at the end of each question or part
question.

For Examiners Use


1
2
3
4
5
6
Total

This document consists of 22 printed pages and 2 blank pages.


UCLES 2009

[Turn o

359

BALOL

Section A

For
Examiners
Use

(a) Look at the soil diagrams, P and Q, showing soils from different parts of the world.
P
surface

Q
surface
pH 4
organic matter

pH 8 thin salty crust


no organic matter
Key

sandy
B
water level

main direction
of soil water
movement
solid rock

(i)

Which soil is the most acidic?

......................................

(ii)

On profile Q shade the part of the soil which has no air content.

(iii)

Which soil is in an area with a desert climate? Explain your answer.

[1]
[1]

..................................................................................................................................
............................................................................................................................ [1]
(b) Explain how human activity causes salination (salinization) of soils.
..........................................................................................................................................
..........................................................................................................................................
..........................................................................................................................................
..........................................................................................................................................
..........................................................................................................................................
.................................................................................................................................... [4]

UCLES 2009

5014/01/O/N/09

360

BALOL
(c) Why is soil P likely to cause problems for crop farmers?
..........................................................................................................................................

For
Examiners
Use

..........................................................................................................................................
..........................................................................................................................................
..........................................................................................................................................
.................................................................................................................................... [3]

(a) Look at the maps showing the size of the Aral Sea, an area of freshwater in Asia, in
1973, 1986 and 2004.

1973

1986

2004

Key
0

100
km

deep water
shallow water
land

(i)

Describe the changes which have taken place since 1973.


..................................................................................................................................
..................................................................................................................................
..................................................................................................................................
..................................................................................................................................
............................................................................................................................ [3]

UCLES 2009

5014/01/O/N/09

[Turn over

361

BALOL(ii) Suggest possible reasons for the changes.


..................................................................................................................................

For
Examiners
Use

..................................................................................................................................
..................................................................................................................................
..................................................................................................................................
............................................................................................................................ [3]
(b) (i)

What is El Nino?
..................................................................................................................................
..................................................................................................................................
..................................................................................................................................
..................................................................................................................................

(ii)

Why does it alter rainfall amounts in Peru and other areas of the world?
..................................................................................................................................
..................................................................................................................................
..................................................................................................................................
............................................................................................................................ [4]

(a) (i)

Name an instrument used to measure atmospheric pressure.


............................................................................................................................ [1]

(ii)

Read the description of part of an instrument which measures atmospheric


pressure:
It contains a collapsible metal box which is partly evacuated of air. When pressure
rises, the top of the box bends in.
Explain why a change in air pressure causes this change.
............................................................................................................................ [1]

UCLES 2009

5014/01/O/N/09

362

BALOL
(b) (i) The diagram shows isobar lines in a pressure system known as a cyclone.

For
Examiners
Use

Key
pressure in
millibars

960
968
980
992

On the diagram, shade the area with the lowest pressure.

[1]

(ii)

Complete the diagram by drawing in the isobar lines between 980 and 992 mb. [1]

(iii)

Does the diagram show that the winds will be weak or strong? Explain your
answer.
..................................................................................................................................
............................................................................................................................ [1]

(iv)

What features of a cyclone cause damage?


............................................................................................................................ [2]

(v)

Explain how improved weather forecasting can reduce the impact of a cyclone.
..................................................................................................................................
..................................................................................................................................
..................................................................................................................................
..................................................................................................................................
............................................................................................................................ [3]

UCLES 2009

5014/01/O/N/09

[Turn over

363

6
4 BALOL
(a) (i)

Look at the photograph, which shows an oil refinery in an area of natural beauty.

For
Examiners
Use

State two features of the oil refinery which people could consider to be ugly.
..................................................................................................................................
............................................................................................................................ [2]
(ii)

For what other reasons could people object to the construction of an oil refinery at
this location?
..................................................................................................................................
..................................................................................................................................
..................................................................................................................................
..................................................................................................................................
............................................................................................................................ [3]

(b) Many tourists visit this area. Suggest one reason why they come.
..........................................................................................................................................
.................................................................................................................................... [1]

UCLES 2009

5014/01/O/N/09

364

BALOL
(c) Suggest why the Government and Local Authorities allowed an oil refinery to be built in
this coastal area, even though it is important for tourism.

For
Examiners
Use

..........................................................................................................................................
..........................................................................................................................................
..........................................................................................................................................
..........................................................................................................................................
..........................................................................................................................................
.................................................................................................................................... [4]

UCLES 2009

5014/01/O/N/09

[Turn over

365

BALOL

Section B

For
Examiners
Use

(a) The graph shows percentages of people without access to water supply and sanitation
in three continents.
100
90
Key

% of people without access to


water supply and sanitation

80

water supply
70

sanitation

60
50
40
30
20
10
0
Africa

(i)

Asia

Europe

World

Average percentages for the World


without water supply
18
without sanitation
40
Add the percentages for the World to the graph.

(ii)

[2]

Compare the percentages for the World with those for each continent.
Africa
..................................................................................................................................
..................................................................................................................................
Asia
..................................................................................................................................
..................................................................................................................................
Europe
..................................................................................................................................
..................................................................................................................................
[3]

UCLES 2009

5014/01/O/N/09

366

BALOL(iii) State one reason why more people in the world have access to water supply than
to sanitation.

For
Examiners
Use

..................................................................................................................................
..................................................................................................................................
............................................................................................................................ [2]
(b) (i)

If people do not have access to a piped water supply, from where do they obtain
their drinking water? Name one source.
............................................................................................................................ [1]

(ii) How safe, for drinking, is the water from this source? Explain your answer.
..................................................................................................................................
..................................................................................................................................
............................................................................................................................ [2]
(c) One method for a country to increase supplies of clean water is desalination (extracting
fresh water from sea water).
Why is desalination widely used in some Middle Eastern countries, especially Saudi
Arabia, UAE and Kuwait, but rarely used elsewhere?
..........................................................................................................................................
..........................................................................................................................................
..........................................................................................................................................
..........................................................................................................................................
.................................................................................................................................... [3]

UCLES 2009

5014/01/O/N/09

[Turn over

367

10

BALOL
(d) Over three million people in the world die each year from water-related diseases. Malaria
is the largest killer.
(i)

For
Examiners
Use

The graph shows rainfall for Maputo, the capital of Mozambique. It is located 25
south of the Equator in Africa.

% of workforce
with malaria

40

30

20

10

rainfall
(mm)

140
120
100
80
60
40
20
0

month

UCLES 2009

5014/01/O/N/09

368

11

BALOL

The percentage of the workforce with malaria at a factory in Maputo varied during
the year.

J
6

F
17

M
19

Percentage of workforce with malaria


A
M
J
J
A
S
O
22 25 13 11
8
11
8

N
5

For
Examiners
Use

D
4

Draw a line graph above the rainfall graph to show these percentages for Maputo.
[3]
(ii)

When is the wet season in Maputo?


............................................................................................................................ [1]

(iii)

At what time of year are there most cases of malaria among the workforce?
............................................................................................................................ [1]

(iv)

How and why does the pattern of rainfall during the year affect the percentage of
workers with malaria? Explain as fully as you can.
..................................................................................................................................
..................................................................................................................................
..................................................................................................................................
............................................................................................................................ [3]

(e) Information about malaria


Deaths per 100,000 people with malaria
1900 1925 1950 1975 2000
In the world
200 170
50
20
25
In Africa
220 210 180 120 160
* 500 million cases a year world-wide
* 23 million people die from it every year
* At least 1 million deaths are in Africa
* Over 2 billion people (1 in 3 in the world) are at risk
* Most sufferers contract malaria two or three times a year
(i)

Describe how the information shows that malaria is a more serious problem in
Africa than in the rest of the world.
..................................................................................................................................
..................................................................................................................................
..................................................................................................................................
..................................................................................................................................
............................................................................................................................ [3]

UCLES 2009

5014/01/O/N/09

[Turn over

369

12

BALOL(ii) Malaria is an important factor keeping people and countries in Africa poor. State
two ways it does this.

For
Examiners
Use

..................................................................................................................................
..................................................................................................................................
..................................................................................................................................
............................................................................................................................ [2]
(f)

The diagram shows how the female mosquito spreads malaria.


1
2
6

larva
pupates
adult
emerges
and feeds

female
lays eggs

it rests
for 2 or 3
days

adult mates
in a swarm

female mosquito
takes a blood
meal
(i)

Why is malaria a water-bred disease?


..................................................................................................................................
............................................................................................................................ [1]

UCLES 2009

5014/01/O/N/09

370

13

BALOL(ii) Details about the two early methods of controlling malaria are given below.
Method 1 Use of cheap medicines, especially chloroquine
Problem: so widely used to treat tropical fevers that mosquitoes have developed
resistance to it.

For
Examiners
Use

Method 2 Spraying the pesticide DDT on the breeding grounds; only small
amounts needed to be used, yet it was highly effective in killing mosquitoes
Problem: its careless use killed many fish as well as beneficial insects and birds. It
has such a bad name that many African governments are now afraid to use it.
For each method, name the stage in the diagram which it was trying to control.
Method 1 ..................................................................................................................
Method 2 ............................................................................................................ [2]
(iii)

For as long as they were effective, both methods were suitable for use in poor
African countries. Explain why.
..................................................................................................................................
..................................................................................................................................
..................................................................................................................................
............................................................................................................................ [2]

UCLES 2009

5014/01/O/N/09

[Turn over

371

14

BALOL(iv) Methods for improved malaria control in Africa are available.


A

Spraying the inside and


outside of huts with DDT.

Sleeping under mosquito


nets treated with insecticide;
these are re-treated with
insecticide every six months.

Use the drug artemisia,


made from a Chinese herb
it cures 90% of patients
within 3 days at a cost of
up to US$10 per person;
acting quickly reduces the
chance of drug resistance
developing. It is in short
supply.

For
Examiners
Use

Describe how these methods are improvements upon the two early methods
already named.
..................................................................................................................................
..................................................................................................................................
..................................................................................................................................
..................................................................................................................................
..................................................................................................................................
............................................................................................................................ [3]

UCLES 2009

5014/01/O/N/09

372

15

BALOL
(g) Read the newspaper reports below.

For
Examiners
Use

A From South Africa

DDT can work


In 1996 the South African government, under pressure from environmental groups both at home
and abroad, abandoned the use of DDT. The number of cases of malaria jumped from around
10,000 per year to more than 60,000 by 2000.
When the government re-introduced DDT spraying in the middle of 2000, the results were dramatic.
By the middle of 2001, the number of cases of malaria was cut in half, and deaths from it fell from
432 to 146.
B From Kenya

Project to distribute free ITNs hailed as a great success


In 2003 Kenyas Ministry of Health began the distribution of 13.5m ITNs (insecticide treated nets).
The cost was kept low by government subsidies. Even so, 3.4m had to be given away free to the
poorest, unable to afford even subsidised nets.
Partly funded by the WHO (World Heath Organisation), this campaign was better supported by
education than previous ones. In the past, nets that were given away free were re-sold, or used for
fishing, or thrown away instead of being re-treated when the insecticide wore off.
Four years later, early results show that childhood deaths from malaria have fallen by almost 50%.
Three hospitals along the malaria-prone coast reported a 57% drop in malaria admissions by 2006.
(i)

Describe the evidence which shows that the improved malaria control methods can
work.
..................................................................................................................................
..................................................................................................................................
..................................................................................................................................
............................................................................................................................ [2]

(ii)

Suggest reasons why many African countries are slow to use these new methods.
..................................................................................................................................
..................................................................................................................................
..................................................................................................................................
..................................................................................................................................
..................................................................................................................................
..................................................................................................................................
..................................................................................................................................
............................................................................................................................ [4]

UCLES 2009

5014/01/O/N/09

[Total: 40 marks]
[Turn over

373

16
6 BALOL
Some parts of the Earths surface are mineral-rich.

For
Examiners
Use

Some mineral-rich regions


B
A
C
G

E
F

Key
mineral-rich
(a) (i)

Minerals from these regions include oil, copper, tin, diamonds, gold, iron-ore, nickel
and uranium.
Name two of the regions shown on the map. From the list, name one mineral mined
in the region.
Letter .......... Name of region ..................................... Mineral ..........................
Letter .......... Name of region ..................................... Mineral .......................... [4]

(ii)

Most of the minerals in the list have more than one use; sometimes their uses are
very different.
Elaborate on this statement by choosing one mineral and naming some of its uses.
..................................................................................................................................
..................................................................................................................................
..................................................................................................................................
............................................................................................................................ [2]

UCLES 2009

5014/01/O/N/09

374

17

BALOL(iii) Explain why some parts of the Earths surface are mineral-rich while others are
mineral-poor.

For
Examiners
Use

..................................................................................................................................
..................................................................................................................................
..................................................................................................................................
..................................................................................................................................
..................................................................................................................................
............................................................................................................................ [3]
(b) The diagram shows how long the known reserves of eight minerals are expected to
last.
Minerals World Reserves in 2005
iron ore
uranium

200
150

coal

100
50

oil

copper

Key
gold

tin

50

years

nickel
(i)

How many years are reserves of nickel expected to last?


............................................................................................................................ [1]

(ii)

Explain why all these minerals will eventually run out.


..................................................................................................................................
..................................................................................................................................
..................................................................................................................................
..................................................................................................................................
.............................................................................................................................. [3]

UCLES 2009

5014/01/O/N/09

[Turn over

375

18

BALOL(c) Two ways to extend the length of time before natural resources run out are shown
in the flow diagrams below.
A

B
bottle
bank

consumer

consumer

glass
bottles
and jars

refillable
bottles

filler

For
Examiners
Use

retailer

filler

recycling
plant

retailer
glass
Factory
(i)

Describe how each one increases the number of years before natural resources
run out.
..................................................................................................................................
..................................................................................................................................
..................................................................................................................................
..................................................................................................................................

(ii)

Which one is better for the environment? Explain your choice.


..................................................................................................................................
..................................................................................................................................
............................................................................................................................ [4]

UCLES 2009

5014/01/O/N/09

376

19

BALOL
(d) Another way of extending the life of natural resources is to develop and use alternatives.
Hydro-electric power (HEP) is an example of an alternative energy source.

For
Examiners
Use

Hydro-electric power station in the Alps

water pipe

HEP
station

(i)

Why is this a good location for generating hydro-electric power?


..................................................................................................................................
..................................................................................................................................
..................................................................................................................................
............................................................................................................................ [2]

(ii)

Explain what people have done to enable energy to be generated here.


..................................................................................................................................
..................................................................................................................................
..................................................................................................................................
..................................................................................................................................
............................................................................................................................ [3]

(iii)

In a location like this, the cost of making electricity from HEP is almost the same as
that from oil. State two advantages of HEP over oil for generating electricity.
..................................................................................................................................
..................................................................................................................................
..................................................................................................................................

UCLES 2009

............................................................................................................................ [2]
5014/01/O/N/09
[Turn over

377

20

BALOL(iv) Oil provides 40% of world energy, compared with only 6% from HEP. Why is the
difference so great?

For
Examiners
Use

..................................................................................................................................
..................................................................................................................................
..................................................................................................................................
..................................................................................................................................
..................................................................................................................................
............................................................................................................................ [3]
(e) Production of biofuels from crops such as sugar cane, palm oil and corn (maize) is
increasing. The fuel from them can be used as alternatives to petrol and diesel in cars
and trucks.
Biofuel production in 2005
(percentage of world total)
Brazil
41
USA
39
EU (European Union)
16
China
3
India
1
Complete a divided bar graph to show biofuel production in 2005.

0%

20%

40%

60%

80%

100%
[3]

UCLES 2009

5014/01/O/N/09

378

21

BALOL
(f) Some information about biofuel production in Brazil and the USA is given below.
Brazil
(from sugar cane)

USA
(from corn)

0.4

0.7

Fossil fuel input (transport and processing)


needed for making ethanol (%)

11

70

Carbon dioxide reduction compared with


oil (%)

90

20

Ethanol output from one hectare of land


(equivalent litres of petrol)

3,000 6,000

1,500 3,000

Average cost of production of ethanol,


compared with petrol (petrol = 1.0)

(i)

Which crop makes cheaper ethanol?


..........................................................

(ii)

For
Examiners
Use

[1]

Give one reason why it is cheaper.


..................................................................................................................................
............................................................................................................................ [1]

(iii)

In which one of the two countries is the environmental impact from ethanol lower?
Explain your choice.
..................................................................................................................................
..................................................................................................................................
..................................................................................................................................
..................................................................................................................................
............................................................................................................................ [3]

UCLES 2009

5014/01/O/N/09

[Turn over

379

22

BALOL
(g)

UN (United Nations) report on


biofuels
* They will cause severe environmental
and social damage instead of saving
the planet
* They will increase demand for land
and water resources at a time when
demand for food is rising
* Demand for palm oil has already led
to rainforest clearances in South East
Asia
* However, the report admits that
biofuels will reduce greenhouse gas
emissions

Comment from the Government of


Brazil
* We have produced ethanol for more
than 30 years; we developed the
technology which makes us the worlds
most efficient producer
* 45% of our energy is from renewables,
one of the highest percentages of any
country in the world
* We are making a bigger contribution
than most countries to reducing
greenhouse gases
* Only 4% of our crop area is planted
with crops for making ethanol
* It is on existing farm lands, not new
land from rainforest clearances

For
Examiners
Use

Targets for future use of biofuels


* USA seven times increase by 2022
* EU to meet 10% of energy needs by 2020
Is it right for the USA, EU, Brazil and other countries to go ahead and increase the
output of biofuels? Answer as fully as you can.
..........................................................................................................................................
..........................................................................................................................................
..........................................................................................................................................
..........................................................................................................................................
..........................................................................................................................................
..........................................................................................................................................
..........................................................................................................................................
..........................................................................................................................................
..........................................................................................................................................
.................................................................................................................................... [5]
[Total: 40 marks]

UCLES 2009

5014/01/O/N/09

380

BALOL
UNIVERSITY OF CAMBRIDGE INTERNATIONAL EXAMINATIONS
GCE Ordinary Level

MARK SCHEME for the October/November 2009 question paper


for the guidance of teachers

5014 ENVIRONMENTAL MANAGEMENT


5014/01

Paper 1, maximum raw mark 120

This mark scheme is published as an aid to teachers and candidates, to indicate the requirements of
the examination. It shows the basis on which Examiners were instructed to award marks. It does not
indicate the details of the discussions that took place at an Examiners meeting before marking began,
which would have considered the acceptability of alternative answers.
Mark schemes must be read in conjunction with the question papers and the report on the
examination.

CIE will not enter into discussions or correspondence in connection with these mark schemes.

CIE is publishing the mark schemes for the October/November 2009 question papers for most IGCSE,
GCE Advanced Level and Advanced Subsidiary Level syllabuses and some Ordinary Level
syllabuses.

381

Page 2

Mark Scheme: Teachers version


GCE O LEVEL October/November 2009

Syllabus
5014

Paper
01

BALOL
1

(a)

(i) P

[1]

(ii) shading of layer below water level

[1]

(iii) Q because water is drawn upwards / alkaline / salty crust shows evaporation / no organic
matter suggests few plants grow
[1]

(b) over irrigation = 1


water drains down and dissolves salts / minerals water is drawn to the surface
evaporation
salts / minerals deposited on surface
repeated causes build up of salts = 3

[4]

(c) pH 4 too acidic for (most) crops


fertiliser / manure needed
sand very porous
fertilisers / nutrients wash down quickly out of reach of crop roots
soil too dry in periods of no rain
expense of improving soil / fertiliser / irrigation

[3]

(a)

(b)

(i) become smaller


larger islands / more islands
islands became a peninsula / joined together (almost) split into two / three
northern part completely detached
becoming shallower
greater change 19862004 than 197386
avp

[3]

(ii) reduced rainfall / drought


global warming
increased evaporation
increased extraction of water for irrigation / other purpose
diversion of rivers feeding into the sea
increased silt / sand deposition
avp

[3]

(i) reversal of ocean currents


warm water flows eastwards across the Pacific / from Indonesia / to Peru
at end of year
every 38 years
(ii) pressure systems alter
trade winds weaken / reverse
cause droughts when winds offshore instead of onshore
cause increased rainfall when winds onshore instead of offshore
low pressure leading to more evaporation and rainfall
avp
at least one from each part for max.

[4]

UCLES 2009

382

Page 3

Mark Scheme: Teachers version


GCE O LEVEL October/November 2009

Syllabus
5014

Paper
01

BALOL
3

(a)

(b)

(a)

(i) barometer / barograph / aneroid barometer

[1]

(ii) weight of air

[1]

(i) shading of area with lowest pressure

[1]

(ii) drawing of isobars at 984 and 988

[1]

(iii) strong because isobars close / steep pressure gradient

[1]

(iv) strong winds


heavy rains / floods
winds causing storm surges in coastal areas

[2]

(v) time to put shutters over windows / board up properties


time to take down potentially dangerous metal signs / other objects
time to evacuate people
time to warn the population
time to mobilise emergency services, move to shelters
stock up on supplies of food and water

[3]

(i) chimneys
storage tanks / description of
avp

[2]

(ii) danger of spillage marine pollution, effect on wildlife / food chain


air pollution, carbon dioxide / sulphur dioxide / nitrous oxide etc.
takes away farmland, loss of livelihood / food
avp
allow each point up to two if well developed

[3]

(b) fishing / boating / attractive scenery / avp

[1]

(c) industrial country


demand for power
demand for refined products from petroleum
cheaper to locate by sea because of cheaper transport costs
industrial lobby more influential than conservationists
provide employment
stimulate local economy
avp

[4]

UCLES 2009

383

Page 4

Mark Scheme: Teachers version


GCE O LEVEL October/November 2009

Syllabus
5014

Paper
01

BALOL
5

(a)

(i) both correctly plotted = 1 mark


attempt to use the same two types of shading = 1 mark

[2]

(ii) Africa both worse than the world average, especially people without water supply
Asia without water supply is the same, but without sanitation is 14% worse
Europe much better than the world average for both, especially for sanitation for which
it is 34% better
these are just some examples of answers; percentage differences other than the ones
quoted here are equally valid
general statements only, without necessarily focusing on the significant or indicating size
of differences = 1 or 2 marks according to completeness
complete statements using key comparisons; highly likely that percentage differences
will be used = 3 marks
[3]
(iii) basic answer is that water supply is easier and cheaper to provide than sanitation
more expensive infrastructure needed to lay pipes to take dirty water away, and to build
and operate treatment works
governments have traditionally given higher priority to water supply for public health, by
community work it is often more feasible to pipe clean water short distances than
undertake sanitation works / access to a stated local water supply
water supply is seen as a more important basic human need
valid reason established = 1 mark
some worthwhile development / elaboration = 1 mark
(b)

(i) possible sources


from the atmosphere rainfall catchment
from the surface rivers, ponds/lakes, irrigation canals
from underground wells, springs

[2]

[1]

(ii) answer will depend on source chosen in (i), since these sources can vary from very
unsafe (mainly surface sources) to quite safe (underground sources)
however, there are exceptions to both, such as ice-melt rivers and contaminated wells
(either naturally by arsenic in parts of Bangladesh, or by human activities such as
spraying pesticides on farmland)
mark according to validity in association with answer to part (i)

[2]

(c) sea water is the most expensive source of all for fresh water
countries named in the Middle East are oil-rich
also they are desperate for water because of their desert locations
water need has risen well above water availability from natural underground stores
other desert countries can have fresh water from outside their borders (e.g. Nile in Egypt)
points made along these lines 3 @ 1 mark
maximum 1 mark for a correct plot using an incorrect method

[3]

UCLES 2009

384

Page 5

Mark Scheme: Teachers version


GCE O LEVEL October/November 2009

Syllabus
5014

Paper
01

BALOL
(d)

(i) plots all correct = 2 marks


no more than three mistakes = 1 mark
plots linked by a line (whether or not all plots are correct) = 1 mark

[3]

(ii) summer, or November to March (accept January to March as the minimum and October
to April as the maximum)
[1]
(iii) end of summer or April to May as the minimum (allow as wide as February to July)

[1]

(iv) malaria cases are highest at the end of wet season / lowest at end of dry season
after the three driest months (J/J/A) cases fall to their lowest in N/D/J
related to amount of standing water / breeding sites for the mosquitoes
these take time to form during the wet season / slow to dry up in dry season
therefore there is a relationship between rainfall and cases of malaria but with a time lag
points made along these lines; mark the answer as a whole according to amount of
understanding shown
[3]
(e)

(i) death rate from people ill with malaria has always been higher in Africa than in the rest of
the world
rate has reduced since 1900 more quickly in the rest of the world
90% reduction 19001975 compared with under 50% in Africa
more significant rise again in Africa from 1975 to 2000
at least a third of the people in the world who die from malaria are Africans
three descriptive points such as these

[3]

(ii) contracting malaria two or three times a year reduces working capacity
costs of medicines / preventative measures
frightens off investment / tourists from outside (non-malarial) countries
high mortality rates among babies and young children encourages high birth rates
keeps people in the poverty trap
two different ways stated = 2 marks
(f)

[2]

(i) female lays eggs in standing water where larva pupates


.... or similar

[1]

(ii) method 1 Stage 4


method 2 Stages 1, 6

[2]

(iii) cheap methods cost is critical to poor African countries


cost-effective; only small amounts of DDT needed for effective results

[2]

(iv) A targets places where people are most likely to be bitten / to contract malaria
less environmental damage will be caused here than in breeding grounds
B insecticide in the nets kills the mosquitoes (instead of merely keeping the insect off
the sleeping person's body)
no general use of insecticide which damages beneficial insects / wildlife as well
C new drug to which there is no mosquito resistance yet; fact that it acts quickly over
three days reduces the chance of mosquitoes developing resistance
a full answer for one method can gain two marks, allowing the maximum to be achieved
from comment about any two of the three 'improved' methods
[3]
UCLES 2009

385

Page 6

Mark Scheme: Teachers version


GCE O LEVEL October/November 2009

Syllabus
5014

Paper
01

BALOL
(g)

(i) in South Africa, without DDT, cases of malaria increased 6 fold in 4 years; within a year
of re-using DDT, cases cut by half, deaths by almost two thirds
in Kenya, reduction by 50% and more in childhood deaths and hospital admissions
values quoted and stated without a context = 1 mark
values quoted and evidence described in question context = 2 marks

[2]

(ii) poverty is one factor even cheap items like mosquito nets needed to be subsidised or
given away free to the poorest in Kenya
US$10 for drug treatment is a lot of money in countries where many people earn less
than US$1 per day
food for survival is more important
ignorance is another as shown by the way mosquito nets were either wasted or used
for other things in Kenya
typical developing country problems affecting distribution to rural areas where they are
most needed, including inadequate transport, poor organisation, corrupt officials, lack of
instruction and education
belief among many that malaria in Africa can never be stopped because of the great
number of breeding sites for mosquitoes during the wet season
the above are just some of the reasons that can be used
reasons stated in a general / non-precise way; may be over-reliant upon content in
newspaper reports without much adaptation to the question = 1 or 2 marks
either a good range of reasons, or one or two reasons well supported by detail = 3 or
4 marks
[4]
[Total: 40]
6

(a)

(i) A
B
C
D
E
F
G
H

Northern Canada gold, copper, nickel, iron-ore, tin


Baltic / Sweden / Scandinavia iron-ore, nickel
Rocky Mountains copper, gold, iron-ore
Andes Peru / Bolivia / Chile copper, tin, iron-ore
Brazilian Plateau gold diamonds, iron-ore, nickel
Southern Africa gold, diamonds, copper, nickel, uranium
Middle East oil
Western Australia gold, nickel, iron-ore, uranium

2 marks for names


2 marks for minerals
should the area not be precise enough for the name mark, the mineral mark can still be
awarded, provided the letter for area is given
no letter and no name = no marks
[4]

UCLES 2009

386

Page 7

Mark Scheme: Teachers version


GCE O LEVEL October/November 2009

Syllabus
5014

Paper
01

BALOL
(ii) good choices include:
oil not only petrol, diesel etc., but also plastics, polythene, paints etc.
lead cables, batteries, roofing
potash fertilisers, explosives, glass, soap, medicines
diamonds jewellery, industrial uses for cutting
two uses for one mineral = 1 mark
three or more different uses, or even two with relevant comment after stating clearly
different uses = 2 marks
[2]
(iii) certain geological conditions are needed for their formation in deposits large enough to
be worth mining
areas of old hard igneous rocks / old shield area (e.g. Canada, Baltic, Southern Africa) is
one example
another is in young fold mountain ranges with recent and great earth movements
fossil fuels relate to presence of tropical forests / shallow swamps 150300 million years
ago
basic ideas; credit elaboration or exemplification
(b)

(i) 75 (allow 7080)

[3]
[1]

(ii) formed at a slow rate / takes millions of years for new deposits to form
being used up at a faster rate by humans than they can ever be formed
reference to an example of how long it takes for vegetation to rot to become coal and oil
minerals will still exist but in amounts too small to mine for economic use
three points made along these lines credit use of or reference to mineral examples [3]
(c)

(i) A shows re-use same bottle used more than once


B shows recycling the glass is made into another bottle
both save on the natural resources used for making glass
(ii) A only needs cleaning / washing out before it can be used again
B needs more transport (bottle bank to recycling and glass plants)
B uses/needs more energy in the glass factory to make it into another bottle
with fewer stages and less energy use, re-use must be better for the environment than
recycling
mark both parts together
some relevant points made = 1 or 2 marks
good understanding and well argued = 3 or 4 marks

(d)

[4]

(i) two essential requirements fulfilled


supply of water from dam / lake / from stated natural sources
head of water to drive the turbines from difference in height / steep relief / plant on
valley floor below the steep mountain side
mention of both needed for 2 marks

[2]

UCLES 2009

387

Page 8

Mark Scheme: Teachers version


GCE O LEVEL October/November 2009

Syllabus
5014

Paper
01

BALOL
(ii) built a dam to increase water supply / make a lake or reservoir
built a pipe to link the reservoir to the power station
fed the water pipe through the mountain at a high level / horizontally
placed the HEP works on the valley floor below the steep water drop
made an outflow lake below the power station
three points made along these lines.

[3]

(iii) HEP is renewable will never run out


HEP is greener no release of greenhouse gases
HEP sites are available in more countries around the world than are oil deposits
any two of these (or any others that are valid)

[2]

(iv) oil is easier to use / more flexible in use


HEP is only electricity whereas oil is a direct fuel and can also be used for making
electricity
oil is easier to transport to where it is needed being a liquid / does not have the transfer
losses of electricity through wires
HEP requires certain physical conditions before a power station can be set up
world is geared to use of oil
for a long time it was a very cheap fuel
three points made along these lines; allow good elaboration of one point up to a
maximum of 2 marks
minimum of 1 mark for a definite point about oil
[3]
(e) correct plot = 2 marks
correct except for one major or two minor mistakes = 1 mark
countries identified for sectors shown (irrespective of method used) = 1 mark
(f)

(i) sugar cane

[3]
[1]

(ii) output from one hectare of land is greater


lower costs for fossil fuel and transport to make it
one of these

[1]

(iii) the carbon dioxide reduction from sugar cane in Brazil is much greater / by 70%
this means that greenhouse gas emissions are much lower
also less land is needed to make more ethanol saving on farm inputs
some of which, like sprays, can damage the environment
less need to make new vegetation, clearances destroying habitats, releasing CO2
points made along these lines
it is only feasible to answer in terms of sugar cane in Brazil

[3]

UCLES 2009

388

Page 9

Mark Scheme: Teachers version


GCE O LEVEL October/November 2009

Syllabus
5014

Paper
01

BALOL
(g) for more use of biofuels
already shown to have cost and environmental advantages compared with the use of fossil
fuels applies to corn from the USA even though the advantages are not as great as for
sugar cane from Brazil
avoids the main problem with fossil fuels greenhouse gas emissions
even the generally hostile UN report had to admit to the environmental advantages of
biofuels
increased output can be achieved on existing cropland; new land clearances are not always
necessary
improvements in technology are increasing efficiency of ethanol production
particularly attractive in countries without any or enough fossil fuel deposits of their own
against more use of biofuels
crop growing competes for scarce natural resources such as water and land
some think that the priority in crop growing should be for food crops for people, not industrial
crops, especially since world population is still growing so quickly
palm oil and sugar cane are tropical crops which grow in areas formerly covered by
rainforests, thereby contributing to further world losses in biodiversity; risks to forests will
increase because ethanol production from them is cheaper than from temperate crops like
maize
only an outline response, restricted to one or two pertinent points, which may keep being
repeated = 1 or 2 marks
more substantial response, with a clear view expressed after consideration of arguments for
and against further biofuel use = 3 or 4 marks
as above but with a higher level of argument = 5 marks

[5]
[Total: 40]

UCLES 2009

389

General Certificate of Education Ordinary Level


5014 Environmental Management November 2009
Principal Examiner Report for Teachers

BALOL
ENVIRONMENTAL
MANAGEMENT

Paper 5014/01
Paper 1

General comments
Despite the great range in overall performance between candidates, it was possible to detect a general
pattern of candidate performance within the paper. Of the three parts, each worth 40 marks (Section A,
Question 5 and Question 6), typically the highest mark was for Question 5, followed almost equally either
by Question 6 or Section A. Question 5 covered topic areas that appeared to be familiar to almost all
candidates, whereas Question 6 included parts which placed a higher demand on more specialised
knowledge, such as about the formation and distribution of minerals.
In Section A, there were fewer high scoring (9 and 10 mark) answers than in previous examinations, but
also fewer very low scoring answers (worth only 1 or 2 marks). In other words, there was usually one part
that was known to each candidate, but equally there was one part which challenged the most able in its
demands. There was evidence of answer quality tailing off in parts (f) and (g) of Question 6 among weaker
candidates. It was clear that many of the able candidates were becoming stretched for time on reaching
these two final parts: however, clear cut examples of candidates, who genuinely did not have sufficient time
to complete the examination within the time allowed, were few and far between. Looking back through their
earlier answers, most had begun by writing excessively long answers to questions worth two or three marks
in the first half of the paper. Future candidates need to be made aware that they must keep an eye on the
clock and allocate their time accordingly for this examination.
If there was a pattern to the marks in Section A, Question 3 yielded the highest mark and Question 4 the
lowest, but there was considerable variation from Centre to Centre. Quality of performance in Question 1
seemed to be heavily dependent on degree of candidate familiarity with salination. The equivalent in
Question 2 was knowledge and understanding of El Nino. Within Question 3 the first and final parts, (a)(i)
and (b)(v) were the main mark scorers. The weakest answers in Question 4 were most frequently given by
candidates who made little use of the photograph and believed that tourists came to see the refinery, which
negated one of the main themes of the question. In Section B, the knowledge gaps exposed most often
were for desalination in 5(c) and mineral formation in 6(a)(iii).
Three main areas for improvement in examination technique, useful for alerting future candidates, were
highlighted in this examination. One was the need to read ahead to the next part of the question before
beginning to answer the current question, to reduce the chances of repeating part or all of the answer in the
following question (where it really belonged in the first place). Many answers to Questions 2(a)(i) and (ii)
were illustrations of this. After one descriptive statement like 'decrease in size of the Aral Sea' in (a)(i) the
candidate continued 'because people are using more water', and then carried on from there, only to find that
they needed to repeat the same answer when they turned over the page and read the question for (a)(ii).
Most simply repeated the same answer, without any thought of going back and amending the answers
already written in (a)(i).
Another was the need to refer first in comparison questions to the item that came first in the question. For
example in 6(d)(iii) the focus needed to be on HEP rather than oil, whereas in the next part the focus needed
to be switched more towards oil. The third was giving list-like answers to longer four and five mark
questions, such as 5(g)(ii) and 6(g) in this paper. Listed answers such as 'too poor', 'lack of education',
'developing countries' and 'against their traditions' in 5(g)(ii) failed on most occasions to deliver answers that
were worth more than one of the four marks.
In addition on this paper, given its length and the large number of separate short structured questions,
candidates can never afford the luxury of repeating the question before beginning the answer. The message
highlighted in last year's report remains valid that the lines left for answering are for guidance only, and
cannot take into account the many variations in size of handwriting and precision of expression between
individual candidates. Remember that candidates tend to equate filling all the lines with giving a full answer

UCLES 2009

390

General Certificate of Education Ordinary Level


5014 Environmental Management November 2009
Principal Examiner Report for Teachers
to
the question, irrespective of any relationship between number of points made and number of marks
BALOL
available.

Comments on individual questions


Section A
Question 1
Most gave the correct answer, soil P, in part (a)(i), but this did not stop quite a number of candidates offering
instead one of the letters A-C for the soil horizons. Many fewer correctly shaded in the zone below the water
level in profile Q in part (a)(ii), while a clear majority gave the wrong answer of soil P in (a)(iii), attracted by
the label 'sandy' on the profile. Answers to part (b) were either high scoring from candidates with knowledge
and understanding of salination, or worthless from candidates who tried to apply causes other than
excessive use of irrigation water in hot climates. References to deforestation and soil erosion were common
in answers of the latter type. Part (c) was better answered. Although a few candidates limited the worth of
their answers by using only one reason, often related to soil acidity, most referred to several reasons, most
commonly based upon them being 'hungry soils', short of water, from which useful nutrients were soon
washed downwards out of the reach of plant roots.
Question 2
Most answers to part (a)(i) lacked finer descriptive detail beyond the most obvious descriptive points of
becoming smaller with more shallow water and an increase in areas of land in between. Thus one and two
mark answers were much more common than three mark answers. Others spent more time suggesting
reasons than describing, and then found that they needed to repeat most of the answer in (a)(ii). Many good
suggestions were made in answers to (a)(ii), from able candidates, which covered a range of possibilities
including greater direct human use, diversion of water from river sources into the Aral Sea, increased
deposition of sediments and climate change. In part (b), El Nino was described as everything from a
Protocol to the result of global warming by the minority of candidates who had no idea what it was. Others
with imperfect knowledge and understanding began to make incorrect statements about El Nino at some
point in their answers. Many of these were the opposites of the truth, such as El Nino causing droughts in
Peru. A few with generally good knowledge became stuck at three marks because they described more than
they explained when answering (b)(ii). Despite these criticisms, it was pleasing to find that candidate
understanding of El Nino continued to increase, especially useful now that it enjoys a higher profile than
previously in the wider news media.
Question 3
Although a few candidates confused barometer with anemometer in part (a)(i), this was the most commonly
claimed mark in the question. Answers to (a)(ii) suffered from candidates attempting only to reword the
description instead of simply stating that the change was caused by the weight of air. Candidate
performance in parts (b)(i)(iii) varied greatly between Centres; often there was no clear pattern from
candidates within a Centre which suggested great variations in levels of individual candidate understanding.
Shading sometimes spilled out beyond the area in the Centre of the diagram below 952 mbs; some shaded
in below 956 but above 952 mbs to give a result looking like a 'do-nut' in (b)(i). In (b)(ii), either one line or
three lines between 980 and 992 mbs were more common incorrect answers than no lines drawn in at all. All
candidates struggled to explain strong winds in (b)(iii) unless they referred to the closeness of the isobars or
to the steep (or large) difference in pressure between Centre and edges. The two main reasons for failure to
claim both marks in part (b)(iv) were just stating 'because of the winds and rain' without any adjectives to
describe them, or only referring to one from the list of choices including 'strong winds', 'heavy rains' and
'coastal storm surges'. Warnings, precautions and evacuations formed the basis for most answers to (b)(v).
Answers most likely to claim all three marks were those which rung true for cyclone such as moving inland
and retreating to already built shelters. Least successful were answers which were more about preparations
taken well in advance, or about the methods of improved weather forecasting themselves.

391

General Certificate of Education Ordinary Level


5014 Environmental Management November 2009
Principal Examiner Report for Teachers
Question
BALOL 4
Most weak answers to this question were rooted in inadequate observation of the photograph. The least well
answered part was (a)(i); in the majority of answers nothing was stated that was based on direct observation.
Mention of the clearly visible chimneys was often almost incidental in the context of air pollution, the theme
which dominated many an answer. Some answers to part (a)(ii) suffered from being too list-like with an
overuse of the word 'pollution'. Much better were answers in which candidates attempted longer explanation
and stated more clearly reasons why people might have objected. Two of the most common answers to (b)
were 'natural beauty' (merely repeating what was stated in the first question) and 'to see the oil refinery'.
Whereas the photograph evidence, if used by the candidate, pointed towards boating and fishing. There was
a wide range of acceptable answers for part (c). If answers remained at one or two marks, the most likely
reason was over-reliance upon one line of explanation. Other candidates, who targeted a range of points
and made sensible comment about the advantages of a coastal location, were the ones most likely to claim
all four marks. The minority who returned to the theme of the oil refinery as a tourist attraction struggled to
gain any marks when this dominated their entire answer.
Section B
Question 5
To claim both marks in (a)(i) the two bars needed to be accurately plotted, and the candidate had to make an
obvious attempt to follow the shading pattern already used. Only a few candidates failed to do both of these.
Those candidates who compared but without stating any percentages in (a)(ii) were limited to two marks;
those who just stated percentages without any comment were also limited to two marks provided that
comparative percentages were stated, and to one mark if non-comparative percentages were listed.
Candidates who answered along the lines that 'water supply and sanitation were low in Europe' were given
no marks. Two and three mark answers were the most common. The basic answer in part (a)(iii) was that
water supply is easier and cheaper to supply than is sanitation. Once this was clearly stated or hinted at in
answers, it only needed a minimum of elaboration to reach two marks. Marks, whether zero, one or two,
tended to go in line with candidate ability.
The easy mark was in (b)(i). A few candidates, however, made life difficult for themselves by answering
(b)(ii) with poor choices sometimes lists of more than one that were totally different (such as well and
river), sometimes desalination (most likely taken from the next question), or even worse oceans. Again the
quality of explanation given closely reflected ability. The wording of the question allowed candidates to
explain everything from totally safe to totally unsafe, as well as all levels of safety in between. The weakest
answers to (c) came from candidates who believed that all that was needed was to allow the sea water to
evaporate in the hot sun. A lot of candidates considered the coastal locations of these countries to be the
major factor. However, those with a real understanding of desalination gave the effective answers based
upon the expense of this process and how it is only carried out where the need is great (as in these desert
countries) and the financial resources exist (due to oil revenues).
For most candidates part (d)(i) offered an easy chance to claim three marks; inaccurate plots of the
percentages were rare. A few insisted on drawing bars, perhaps as the only graphical technique they know.
More surprisingly this was the part of Question 5 most likely to be unanswered by candidates. The reasons
for this are difficult to identify, although there are always a few candidates in each examination session who
seem to be more uncomfortable with practical graph questions than with the written answers. Many answers
to the next two parts, (d)(ii)and (d)(iii), were inadequate because candidates merely named one month for
'season' (normally January) and for 'time of year' (almost always May). Ranges of months as narrow as two
to three months or as wide as six or seven months were accepted, as also were summer in (d)(ii) and end of
summer in (d)(iii). In other words, the mark scheme was quite flexible provided that candidates looked
beyond one month. While almost all candidates showed themselves to be aware of the relationship between
the wet season and high incidence of malaria, a good number struggled to relate and explain the increase in
cases towards the end of, and immediately after, the wet season in part (iv). Only more able candidates
were able to apply what they knew to this particular example in a sufficiently precise way to claim all three
marks.
In part (e)(i) some candidates ignored the command word 'Describe' and filled all the lines trying to explain
why malaria is a more serious problem in Africa than in the rest of the world. From the information given,
some items were more useful to candidates for answering this question than others. Most useful were the
comparative figures for deaths per 100,000 people between Africa and the rest of the world. Candidates,
who made full use of the significant differences in numbers per 100,000 and in the changing trends shown
between 1900 and 2000, produced most of the two mark and all of the three answers. Weaker candidates

392

General Certificate of Education Ordinary Level


5014 Environmental Management November 2009
Principal Examiner Report for Teachers
tended
BALOLto use only the comparative values for 1900 and filled the lines by merely repeating what was given,
with a minimum of comment. For them, one mark was the typical outcome. The question seemed to be a
missed opportunity for some. Full mark answers were much more common to part (ii), with references to
reducing the capacity to work and the cost of medicines or preventative measures, dominating in the many
two mark answers.
Answers like 'the female anopheles mosquito breeds in water' were never going to be worth a mark with
'water-bred' in the question in (f)(i). A surprising number of candidates were shown to be under the false
impression that the disease was spread by contaminated water supplies. However, most did claim the mark.
Likewise full two mark answers dominated in part (ii), after candidates identified stage 4 for Method 1 and
either stage 6 or 1 for Method 2. Since none of the other stage numbers were persistent distractors, the
minority of incorrect answers were more likely to have been based on a total lack of individual candidate
understanding. Again the vast majority of candidates could identify 'low cost' and 'only needed in small
amounts' from the details supplied in part (iii). To score marks in (iv) candidates needed to concentrate on
describing how the new methods were improvements. Some failed to do this, instead merely stating their
good points. The importance of having the mosquito nets treated with insecticide was the improvement that
candidates missed most often.
In (g)(i), most candidates did stick to description this time instead of trying to give reasons. The worth of
some answers to (g)(ii) was greatly reduced by the use of lists. Many of the reasons suggested were valid,
but they were stated in such a short, general way as to be of little value. It was no surprise that poverty
dominated the answers; well developed, this could take the answer up to three marks. The most effective
answers came from candidates who explained using other reasons, especially ignorance (as opposed to just
'uneducated') and problems of distribution to reach the rural areas, where the majority of Africans live.
Question 5 examined familiar, previously visited, topic areas, for which the great majority of candidates were
well prepared. Despite highlighting weaknesses in this report, this question was well answered by the
majority of candidates and was quite high scoring. The key to a high total mark, as always, was consistency
of performance between the different parts, which favoured candidates without gaps in their knowledge and
who best obeyed question commands.
Question 6
Four mark answers to part (a)(i) were most common when the regions lettered F (Southern Africa) and G
(Middle East or the Gulf) were chosen, irrespective of the part of the world where the candidate lived.
Location knowledge for other regions was shown to be patchy, with perhaps Alaska for region A being the
most regular mistake. To answer part (ii) well some mineral choices were better than others; oil was
perhaps the best choice for stating a wide variety of uses. Uranium was a less good choice unless
candidates were able to elaborate upon its two major uses, in the way that many more showed they could do
after having chosen diamonds. A lot of answers to part (iii) suffered from inadequate knowledge of mineral
formation. Some candidates approached the answer by referring to the past conditions needed for the
formation of fossil fuels, but it was impossible to write a full mark answer without reference to geological
conditions. Some candidates confused minerals as used here with minerals as nutrients in soils. Certainly
there were many vague, general answers about how climate or soils or the movement of the plates affected
mineral formation.
The answer to part (b)(i) was intended to be 75 years; this was far and away the most common answer,
although a range between 70 and 80 years was allowed since candidates had at least shown that they
understood what needed to be done. Most answers to (b)(ii) were too narrow; typically candidates filled all
the lines making the one point that they are non-renewable resources, often without more telling points such
as the millions of years for new deposits to form, and present human use occurring at a faster rate than the
resources can ever be formed. Few candidates showed awareness of the widespread availability of minerals
in the rocks of the Earth's surface compared with their limited occurrence in deposits of sufficient size to be
economic to mine.
The two questions that made up part (c) were higher scoring. The majority of candidates understood the
main difference shown between the two diagrams and chose to explain A in part (ii) on the grounds of fewer
stages in the operation and less energy use for transport and in factory processes. The most common
answers which failed to progress beyond two marks were those in which the candidate tried to justify the
choice of B as being better for the environment, which was a big challenge in this particular example.

393

General Certificate of Education Ordinary Level


5014 Environmental Management November 2009
Principal Examiner Report for Teachers
For
a two mark answer to part (d)(i) a candidate needed to state how it fulfilled the two essential
BALOL
requirements for HEP generation, namely a supply of water, and a head of water to provide the force to drive
the turbines. Both were needed. Candidates could use either direct evidence from the sketch (such as
presence of the dam and reservoir, difference in height with the HEP station sited on the valley floor) or what
was likely in high mountain areas such as the Alps (water from melting ice, high annual rainfall and great
variations in height between mountain tops and valley bottoms). Non-scoring answers were rare. The best
answers to part (ii) came from candidates who approached the answer in a logical way starting with
building the dam high in the mountains, and then leading the water by a pipe under the mountains so that it
could drop with great force into the HEP station on the valley floor. Unfortunately more common were
answers in which the human additions were treated as separate items, or ones in which candidates imagined
the sketch showed what they knew about other HEP schemes, such as water being discharged out of the
dam directly into the HEP station. The result was that one and two mark answers were much more common
than those worth three marks. Some candidates made hard work of reaching the two most obvious answers
of renewable and less polluting in part (iii), sometimes prolonged by beginning with oil and only later
switching to how HEP was different. Then in (d)(iv) many answers were over-focused on HEP sometimes to
the exclusion of direct statements about oil. Many candidates, who in the end did reach three marks, needed
to fill all the lines, and sometimes more, to do this. The mark scheme included many simple advantages of
oil, which must have been known by many more candidates than actually used them, such as ease of use,
variety of uses (as a fuel), ease of transport, a long history of use and its relative cheapness (at least until
recently).
Divided bar graphs are a less widely used alternative to pie graphs. They are easier to draw than pie
graphs, especially when a graph paper background is provided. There were many accurate constructions in
part (e) from candidates who knew what to do. The most common mishap was to show Brazil as 42%
instead of 41. A minority of candidates were determined to try to draw a bar graph within the frame, despite
the percentage scale being provided; others attempted to show all the percentages beginning at 0% and not
going beyond 41%. These candidates could still claim the mark available for shading in or designating the
countries, provided this was done clearly.
The three questions in part (f) proved to be more challenging than expected. Even in what had been
imagined to be the very easy part (i) there were many answers of Brazil instead of the named crop (sugar
cane). More worryingly was the frequency of corn and USA answers. These suggested either that the
information in the table was not fully understood, or that candidates were running short of time and could not
study the table as carefully as was needed. Then in part (ii) some lost the mark by merely re-stating that it
was cheaper by quoting the average costs of production (0.4 and 0.7), instead of explaining either by
reference to lower fossil fuels inputs or greater output per hectare. Something similar happened in part (iii).
Many answers were dominated by descriptively repeating the values in the table, which for this answer
needed to be used in a more explanatory manner. In the best answers the lower fossil fuel use in Brazil was
linked to the greater reduction of carbon dioxide, followed by mention of the likely environmental benefits of
this.
The typical answer to part (g) was worth two or three marks, based on selective use of the information
supplied to support the candidate's expressed view. Marks higher than this were reserved for candidates
who introduced a broader perspective or an overview. This was most frequently done by reference to the
other alternative energy sources and how their possibilities for further use compared with biofuels. Only
more able candidates showed themselves able to do this, which is why the question can be said to have
discriminated well between candidates. However, there were some candidates who appeared still not to
understand what biofuels were. Perhaps they missed the explanation given at the start of (e); they saw
biofuels as emitting all the same polluting gases as from burning oil and diesel in motor vehicles.
For many candidates the total mark for Question 6 was below that for Question 5, to a large measure
because of a decline in performance from part (f) onwards. The more able the candidate, the less that this
general summary of performance applied.

394

BALOL
UNIVERSITY OF CAMBRIDGE INTERNATIONAL EXAMINATIONS
General Certificate of Education Ordinary Level

*8934866174*

5014/11

ENVIRONMENTAL MANAGEMENT
Paper 1

May/June 2010
2 hours 15 minutes

Candidates answer on the Question Paper.


Additional Materials:

Ruler
Protractor

READ THESE INSTRUCTIONS FIRST


Write your Centre number, candidate number and name on all the work you hand in.
Write in dark blue or black pen.
You may use a soft pencil for any diagrams, graphs or rough working.
Do not use staples, paper clips, highlighters, glue or correction fluid.
DO NOT WRITE IN ANY BARCODES.
Answer all questions.
All questions in Section A carry 10 marks.
Both questions in Section B carry 40 marks.
At the end of the examination, fasten all your work securely together.
The number of marks is given in brackets [ ] at the end of each question or part
question.

For Examiners Use


1
2
3
4
5
6
Total

This document consists of 20 printed pages.

UCLES 2010

[Turn o

395

BALOL
1

Section A

The diagram shows events which affected a village situated by Lake Turkana in Kenya,
Africa.
reduced
rainfall

water in lake
falls each year

increase of fluoride
concentrations
in the water

teeth rot

donkeys
carry good
quality water
from 8 km
away

TV crew raise
money to buy
35 donkeys for
the villagers

TV crew from
Belgium make a
programme about
the problems

For
Examiners
Use

villagers drink
the water and
eat the fish

bones
become
spongy

cancer

increase in
ill health

(a) (i)

What chemical caused the increase in ill health of the villagers?


..............................................

[1]

(ii)

Circle a box which shows how raising awareness in the developed world can help
with problems in developing countries, such as Kenya.
[1]

(iii)

Write one benefit for people, into box Y, based on the information in the diagram.
[1]

(b) List four problems that could be written in box X.


..........................................................................................................................................
..........................................................................................................................................
..........................................................................................................................................
..........................................................................................................................................
..........................................................................................................................................
......................................................................................................................................[4]

UCLES 2010

5014/11/M/J/10

396

BALOL
(c) Do you think it was appropriate to give the villagers donkeys, rather than a truck?

For
Examiners
Use

Give reasons for your answer.


..........................................................................................................................................
..........................................................................................................................................
..........................................................................................................................................
..........................................................................................................................................
......................................................................................................................................[3]

Look at the bar graphs of countries with the largest coal reserves, shown as percentages of
the worlds total reserves.

Russia
Poland
Ukraine

Kazakhstan

USA
China
India
Colombia

Equator

key
percentage of world
coal reserves
20
country
15
boundary
10
5
0
(a) (i)

Australia
South Africa

Name the country with the largest percentage of coal reserves. State its
percentage.
country ....................................... percentage .......................................%

(ii)

Which two continents have the smallest percentages of coal reserves?


continents ....................................... and .......................................

(iii)

[1]

Circle the hemisphere that has the smallest percentage of the worlds coal
reserves.
northern

UCLES 2010

[1]

southern

5014/11/M/J/10

[1]

[Turn over

397

BALOL
(b) Suggest why a country may not use all its coal reserves in the future.
..........................................................................................................................................

For
Examiners
Use

..........................................................................................................................................
..........................................................................................................................................
..........................................................................................................................................
......................................................................................................................................[3]
(c) Coal and water are both sources of power. What are the advantages of hydro-electric
power?
..........................................................................................................................................
..........................................................................................................................................
..........................................................................................................................................
..........................................................................................................................................
..........................................................................................................................................
......................................................................................................................................[4]

UCLES 2010

5014/11/M/J/10

398

5
3 BALOL
(a) Look at the photograph of a type of farming.

(i)

Circle two of the words below to describe the type of farming shown in the
photograph.
extensive
commercial

(ii)

For
Examiners
Use

intensive
subsistence

[2]

Why is the area shown in the photograph suitable for using machinery?
..............................................................................................................................[1]

(iii)

The structure, shown in the photograph, is used to irrigate the land. Describe this
method of irrigation.
..................................................................................................................................
..................................................................................................................................
..................................................................................................................................
..................................................................................................................................
..................................................................................................................................
..............................................................................................................................[3]

(b) Is the method of irrigation you have described in (a) (iii) suitable for all places? Explain
your answer.
..........................................................................................................................................
..........................................................................................................................................
..........................................................................................................................................
..........................................................................................................................................
......................................................................................................................................[4]
UCLES 2010

5014/11/M/J/10

[Turn over

399

6
4 BALOL
(a) Look at the pie graphs showing the percentages of the worlds species, in certain groups,
which are in danger of extinction.

birds

mammals

amphibians
key

plants

percentage of worlds
species in danger
of extinction

(i)

The value for birds is 12%. Complete the pie graph.

(ii)

Which group contains the greatest percentage of species in danger of extinction?


..............................................

(iii)

[1]

[1]

What percentage of the worlds species of amphibians is at risk of extinction?


.........................%

(iv)

For
Examiners
Use

[1]

Explain why the extinction of one species has an effect on others.


..................................................................................................................................
..................................................................................................................................
..................................................................................................................................
..................................................................................................................................
..................................................................................................................................
..............................................................................................................................[4]

UCLES 2010

5014/11/M/J/10

400

BALOL
(b) Suggest three different reasons why it is difficult to prevent the extinction of plant
species.

For
Examiners
Use

..........................................................................................................................................
..........................................................................................................................................
..........................................................................................................................................
..........................................................................................................................................
..........................................................................................................................................
......................................................................................................................................[3]

UCLES 2010

5014/11/M/J/10

[Turn over

401

BALOL
5

Section B

(a) Look at the world map showing the distribution of hot deserts.

Tropic of Cancer

For
Examiners
Use

Equator

Tropic of Capricorn

(i)

C Cairo

hot desert

R Riyadh

cold ocean currents

Name the continent with the largest area of hot desert.


..............................................................................................................................[1]

(ii)

Use the map to suggest why the hot desert area is larger in this continent than in
the others.
..............................................................................................................................[1]

(iii)

All the ocean currents shown on the map are cold. How can you work this out from
the map?
..................................................................................................................................
..............................................................................................................................[1]

(iv)

In El Nino years, how and why does the desert climate change along the coast of
Peru?
..................................................................................................................................
..................................................................................................................................
..............................................................................................................................[2]

(v)

Apart from the ocean currents, state two similarities of the location of hot deserts in
the different continents.
..................................................................................................................................
..................................................................................................................................
..................................................................................................................................
..............................................................................................................................[2]

UCLES 2010

5014/11/M/J/10

402

BALOL
(b) Climate data for Cairo and Riyadh is given in the table below. (Their locations are shown
on the world map.)

For
Examiners
Use

Climate data temperature and precipitation


J

temperature ( C)

13

15

18

21

25

28

29

29

26

24

20

15

precipitation (mm)

temperature ( C)

15

16

21

25

30

34

34

33

31

25

21

15

precipitation (mm)

20

23

25

10

Cairo, Egypt

Riyadh, Saudi Arabia

(i)

Describe the evidence from the climate data that both places have a hot desert
climate.
..................................................................................................................................
..................................................................................................................................
..............................................................................................................................[2]

(ii)

State the main differences in temperature and precipitation between the two
places.
..................................................................................................................................
..................................................................................................................................
..................................................................................................................................
..............................................................................................................................[3]

(iii)

Plants do not cover all the ground surface in desert areas due to the climate.
Explain how both temperature and precipitation limit vegetation growth.
..................................................................................................................................
..................................................................................................................................
..................................................................................................................................
..................................................................................................................................
..................................................................................................................................
..............................................................................................................................[3]

UCLES 2010

5014/11/M/J/10

[Turn over

403

10

BALOL
(c) Look at the diagram which shows three plants with adaptations to allow them to survive
the hot desert climate.

For
Examiners
Use

metres
10

date
palm

9
8
7
6
5
cactus

4
thorny
shrub

3
2
1

desert

surface

0
1
2
3

water table

4
5
6

(i)

On the diagram, add labels to show the different ways plants are adapted to living
in the difficult desert climate.
[4]

(ii)

Why is the root system of the cactus different from those of the other plants?
..................................................................................................................................
..................................................................................................................................
..............................................................................................................................[2]

UCLES 2010

5014/11/M/J/10

404

11

BALOL
(d) Read the passage below about the traditional way of life of the Bedouin people in
Arabia.

For
Examiners
Use

The Bedouin wander the desert with herds of camels, sheep and goats. They are forced
to move in search of new pasture for their animals. They move along routes across the
desert which they have followed for centuries; these are controlled by the availability of
water.
Of all the animals, the camel is considered the most valuable. The Bedouin make use
of its milk, hide, hair and meat. Most importantly the camel is a beast of burden, the
transport of the desert.
The camel is well adapted to desert conditions. Its hump made up of fat is a store of
food. Its height keeps its head up above the normal height of wind-blown sand. Its broad
feet allow it to walk better than any humans through sand.
The Bedouin exchange their animal products for food, such as wheat and dates, grown
by oasis dwellers. They are traders as well as animal herders. In the southern part of
Arabia a great network of trade routes grew up for the spice trade.
Today the Bedouins traditional way of life is under threat, as Saudi Arabia and its
neighbours have grown rich with oil extraction. Oil pipelines now cut across traditional
lines of Bedouin movement. Urban settlements are expanding into the desert as are new
areas of irrigation for food supply for the cities. Other types of work are now available at
the oil wells and in the refineries. Some Bedouin are being attracted towards the cities,
especially the young. Bedouin, who try to maintain traditional ways of living, no longer
have the large empty areas through which to roam. Everything now favours settled
agriculture rather than nomadic animal farming.
(i)

The traditional type of Bedouin agriculture is known as pastoral nomadism. State


the evidence from the passage for this type of agriculture.
..................................................................................................................................
..................................................................................................................................
..................................................................................................................................
..............................................................................................................................[2]

(ii)

Give two different reasons why the Bedouin rely more on their camels than any of
their other animals.
..................................................................................................................................
..................................................................................................................................
..................................................................................................................................
..............................................................................................................................[2]

UCLES 2010

5014/11/M/J/10

[Turn over

405

12

BALOL(iii) The traditional Bedouin way of life was an example of sustainable living. Explain
how and why.

For
Examiners
Use

..................................................................................................................................
..................................................................................................................................
..................................................................................................................................
..................................................................................................................................
..............................................................................................................................[3]
(iv)

Changes are now taking place in Arabia resulting from the discovery and extraction
of oil. Describe some of the economic and likely social effects on the Bedouin
people.
..................................................................................................................................
..................................................................................................................................
..................................................................................................................................
..................................................................................................................................
..................................................................................................................................
..................................................................................................................................
..................................................................................................................................
..............................................................................................................................[4]

UCLES 2010

5014/11/M/J/10

406

13

BALOL
(e) Desertification is happening around the edges of many hot desert areas in Africa and
Asia. Look at the flow diagram of causes of desertification below.

For
Examiners
Use

population growth

higher demand for food

.......................................

increased soil erosion

desertification
(i)

Complete the diagram by filling in the space between higher demand for food and
increased soil erosion.
[1]

(ii)

Which factor in the diagram is the most important in causing desertification?


Explain your answer.
..................................................................................................................................
..................................................................................................................................
..............................................................................................................................[2]

(iii)

Explain why only slow progress is being made to reduce rates of population growth
in many developing countries in Africa and elsewhere.
..................................................................................................................................
..................................................................................................................................
..................................................................................................................................
..................................................................................................................................
..................................................................................................................................
..................................................................................................................................
..................................................................................................................................
..................................................................................................................................
..................................................................................................................................
..............................................................................................................................[5]

UCLES 2010

5014/11/M/J/10

[Total: 40]
[Turn over

407

14
6 BALOL
(a) The coal that is mined today was formed millions of years ago.
300 million years ago

For
Examiners
Use

today

mine

recent rocks

coal seam
rocks 300 million
years old

UCLES 2010

5014/11/M/J/10

408

15

BALOL (i) Explain how coal is formed.


..................................................................................................................................

For
Examiners
Use

..................................................................................................................................
..................................................................................................................................
..................................................................................................................................
..............................................................................................................................[3]
(ii)

State two reasons why coal is called a fossil fuel.


..................................................................................................................................
..................................................................................................................................
..................................................................................................................................
..............................................................................................................................[2]

(iii)

Why is carbon dioxide released into the atmosphere when coal is burnt?
..................................................................................................................................
..................................................................................................................................
..................................................................................................................................
..............................................................................................................................[2]

UCLES 2010

5014/11/M/J/10

[Turn over

409

16

BALOL
(b) The bar graph shows total world energy consumption in 1987, 1997 and 2007.

For
Examiners
Use

1987

years

1997

2007

10

11

( billion tonnes of oil equivalent )

key

(i)

oil

coal

natural gas

HEP

nuclear

World energy consumption in 2007


(billion tonnes of oil equivalent)
Oil
4.0
Coal
3.1
Natural gas
2.6
HEP
0.7
Nuclear
0.6
Divide up the bar for 2007 to show the values of these five different energy
sources.
Complete the key to show the shading or colours used.
[4]

(ii)

Describe how the bar graph shows total world energy consumption has changed
since 1987.
..................................................................................................................................
..................................................................................................................................
..............................................................................................................................[2]

UCLES 2010

5014/11/M/J/10

410

17

BALOL(iii) How do the values for 2007 show the great importance of fossil fuels in world
energy supply?

For
Examiners
Use

..................................................................................................................................
..................................................................................................................................
..................................................................................................................................
..................................................................................................................................
..............................................................................................................................[3]
(c) Describe the advantages of oil over coal for
(i)

extraction from the ground,


..................................................................................................................................
..................................................................................................................................
..................................................................................................................................
..................................................................................................................................

(ii)

transporting to place of use,


..................................................................................................................................
..................................................................................................................................
..................................................................................................................................
..................................................................................................................................

(iii)

ease of use.
..................................................................................................................................
..................................................................................................................................
..................................................................................................................................
..................................................................................................................................
[6]

UCLES 2010

5014/11/M/J/10

[Turn over

411

18

BALOL
(d)

Two energy sources in the UK


coal burning
power station

For
Examiners
Use

wind turbines

full output of an average station


32,000 megawatts of energy a day

full output of one standard sized wind turbine


32 megawatts of energy a day

average output achieved


30,000 megawatts a day

average output achieved


8 megawatts a day

percentage of full output achieved

percentage of full output achieved

on average 94%

on average ....................................

(i)

In the space, fill in the average percentage of the full output achieved by a standard
wind turbine in the UK.
[1]

(ii)

Explain the advantages of using coal fired power stations instead of wind turbines
for generating electricity in the UK.
..................................................................................................................................
..................................................................................................................................
..................................................................................................................................
..................................................................................................................................
..................................................................................................................................
..................................................................................................................................
..................................................................................................................................
..............................................................................................................................[4]

UCLES 2010

5014/11/M/J/10

412

19

BALOL(iii) State the environmental damage caused by coal fired power stations.
..................................................................................................................................

For
Examiners
Use

..............................................................................................................................[2]
(iv)

Explain why the air pollution that results is both a local and an international
problem.
..................................................................................................................................
..................................................................................................................................
..................................................................................................................................
..................................................................................................................................
..................................................................................................................................
..............................................................................................................................[3]

(e)

World top 10 producers of nuclear power (2007)


(million tonnes of oil equivalent for energy produced)
Rank
1
2
3
4
5
6
7
8
9
10

Country
USA
France
Japan
Russia
South Korea
Germany
Canada
Ukraine
Sweden
China

Amount
192
100
63
36
32
31
21
20
15
14

Continent
North America
Europe
Asia
Europe / Asia
Asia
Europe
North America
Europe
Europe
Asia

Source BP Statistical Review of World Energy June 2008

(i)

Describe what the table shows about the world distribution of nuclear power
production. (You should refer to continents without nuclear power production as
well as those with production).
..................................................................................................................................
..................................................................................................................................
..................................................................................................................................
..................................................................................................................................
..................................................................................................................................
..............................................................................................................................[3]

UCLES 2010

5014/11/M/J/10

[Turn over

413

20

BALOL(ii) Which are the stronger arguments for more use of nuclear power, or arguments
against more nuclear power use?

For
Examiners
Use

State the arguments for and against more nuclear power use, and explain your
view.
..................................................................................................................................
..................................................................................................................................
..................................................................................................................................
..................................................................................................................................
..................................................................................................................................
..................................................................................................................................
..................................................................................................................................
..................................................................................................................................
..................................................................................................................................
..................................................................................................................................
..................................................................................................................................
..............................................................................................................................[5]
[Total: 40]

414

BALOL
UNIVERSITY OF CAMBRIDGE INTERNATIONAL EXAMINATIONS
GCE Ordinary Level

MARK SCHEME for the May/June 2010 question paper


for the guidance of teachers

5014 ENVIRONMENTAL MANAGEMENT


5014/11

Paper 11, maximum raw mark 120

This mark scheme is published as an aid to teachers and candidates, to indicate the requirements of
the examination. It shows the basis on which Examiners were instructed to award marks. It does not
indicate the details of the discussions that took place at an Examiners meeting before marking began,
which would have considered the acceptability of alternative answers.
Mark schemes must be read in conjunction with the question papers and the report on the
examination.

CIE will not enter into discussions or correspondence in connection with these mark schemes.

CIE is publishing the mark schemes for the May/June 2010 question papers for most IGCSE, GCE
Advanced Level and Advanced Subsidiary Level syllabuses and some Ordinary Level syllabuses.

415

Page 2

Mark Scheme: Teachers version


GCE O LEVEL May/June 2010

Syllabus
5014

Paper
11

BALOL
Section A
1

(a) (i) Fluoride.

[1]

(ii) Circle round either box referring to TV crew.

[1]

(iii) Better health / longer life expectancy / childrens teeth improve / less incidence of cancer
/ stronger childrens bones, etc.
[1]
(b) Credit problems, such as:
lower life expectancy,
increased death rate,
need for more medical services / clinics / doctors / nurses,
increased demands on the family of the ill,
reduced productivity / village economy falls,
reduced income (of ill individuals).
Any four points, 1 mark for each.

[4]

(c) Credit reasons supporting decision, such as:


yes donkeys (more) appropriate because
cheap(er) to use,
cheap(er) to maintain / expense of spare parts for a truck,
easier to maintain / difficulty of obtaining spare parts for a truck,
dont pollute the air (as much as a truck),
difficulty of obtaining fuel for a truck,
donkeys fertilise the soil.
If answer is no, truck would be better credit appropriate reasons, such as:
can carry more water,
faster,
can be used for other heavy / large loads that a donkey cannot carry,
need to provide feed / water may be difficult in the dry season.
Any three points, 1 mark for each.
2

[3]

(a) (i) USA,


30 (and any between), both needed for the mark.

[1]

(ii) South America and Europe

[1]

(iii) Southern

[1]

(b) Credit reasons, such as:


type of coal may not be very useful,
too low grade to be economic to mine,
fall in price,
cost of recovering the coal / labour costs etc.,
alternative energy sources developed by the country,
fall in demand due to changing technology,
waiting for new technology to become profitable / gasification,
government decision to try to combat global warming.
Any three points, 1 mark for each.

[3]
UCLES 2010

416

Page 3

Mark Scheme: Teachers version


GCE O LEVEL May/June 2010

Syllabus
5014

Paper
11

BALOL
(c) Credit advantages, such as:
cheap once set up,
clean / does not pollute the atmosphere / contribute to global warming / no carbon emissions,
renewable / usually available,
water not polluted so can be used downstream,
can provide power for relatively inaccessible mountain areas,
dam can be used for water control / to prevent floods,
reservoir can be used for recreation / to promote tourism,
quickly switched on / off for changing demand.
Any four points, 1 mark for each.
3

[4]

(a) (i) Intensive, commercial, circled or indicated in some other way.


1 mark for each.

[2]

(ii) Level / gently sloping.

[1]

(iii) Water stored in reservoir / from rivers / lakes,


pumps,
gravity flow,
piped to field,
sprinklers / sprays,
rotate,
cover a large area,
sprinkler moved to a different part of the field.
Any three points, 1 mark for each.

[3]

(b) Credit any two unsuitable types of area and any relevant reason linked to it e.g.
unsuitable in areas with high temperatures / where rapid evaporation,
leads to salts in the soil,
leads to large water loss,
unsuitable in areas with unreliable rainfall / without rivers / lakes,
source of water may not be adequate / method uses large quantities of water,
unsuitable on steep slopes,
difficulty of moving the big machinery / expense of pumping the water.
If a different method of irrigation has been described in (a)(iii), credit points relevant to the
method described.
Any four points, 1 mark for each.

[4]

UCLES 2010

417

Page 4

Mark Scheme: Teachers version


GCE O LEVEL May/June 2010

Syllabus
5014

Paper
11

BALOL
4

(a) (i) Segment drawn correctly and shaded.

[1]

(ii) Plants.

[1]

(iii) 32 / 33 / 34.

[1]

(iv) Credit links, such as:


plants are food for other species,
plants are habitat for other species,
animals are food for other species,
plants rely on birds / insects for pollination,
plants rely on birds / animals for dispersal.
Allow credit to 2 marks each if developed by examples or detail.
Any four points, 1 mark for each.

[4]

(b) Credit different reasons, such as:


increasing populations need for firewood / farmland / other specific purpose,
effect of genetic modification on wild species,
spread of plant diseases,
spread of insects / other pests,
cost of prevention may be too high for many countries,
spread of fires difficult to control.
Any three points, 1 mark for each.

[3]

UCLES 2010

418

Page 5

Mark Scheme: Teachers version


GCE O LEVEL May/June 2010

Syllabus
5014

Paper
11

BALOL
Section B
5

(a)

(i) Africa

[1]

(ii) Continent is wider in tropical latitudes where the desert climate occurs, or desert goes
right across the continent from west to east coasts.
[1]
(iii) From the direction of flow all from high to low latitudes, from cold towards warm ocean
waters, and therefore carry cold water.
[1]
(iv) Becomes wetter,
warm Pacific current replaces the cold Peruvian current off the coast,
winds blow from sea to land/wet and warm winds from the sea.
How = 1 mark
Why = 1 mark

[2]

(v) The two similarities are:


western sides of the continents,
across the tropics/sub-tropics/2030 North and South of Equator.
1 mark for each similarity.
(b)

[2]

(i) Most conclusive evidence is low total annual precipitation/less than 250mm
= 1 mark
e.g. only 29mm in Cairo/81mm in Riyadh,
also very hot summer temperatures (especially in Riyadh),
30 C plus is higher than at the Equator.
Any one of these for the 2nd mark.

[2]

(ii) Riyadh is hotter in summer (5 months with 30C+), and has a larger annual range of
temperature (19 compared with14). Cairo is drier (29mm total against 81mm in Riyadh),
although Riyadh has more months with zero precipitation (7 against 5). Riyadh has a
more distinct wet period in late winter/spring (Feb-Apr).
Two marks for a two-sided difference with supporting values/months.
One mark for a heavily one-sided difference or a comparative without support.
Reserve 1 mark each for temperature and precipitation; otherwise any route to three
marks.
[3]
(iii) High temperatures lead to high rates of transpiration/evaporation/great water loss, fierce
sun from cloudless skies bakes the land and burns off green vegetation, total rainfall is
low and unreliable, limited effectiveness because of dry ground and high temperatures.
Points made along these lines for one mark each.
A well developed point may be worth two marks.
Any route to three marks.
[Note no credit here for plant adaptations against these climatic problems.]

[3]

UCLES 2010

419

Page 6

Mark Scheme: Teachers version


GCE O LEVEL May/June 2010

Syllabus
5014

Paper
11

BALOL
(c)

(i) Possible labels:


tap root of date palm,
large branching root system of the scrub,
both have deep roots reaching water table,
shallow surface rooting system of cactus,
succulent stem of cactus,
tough/thick bark of the date palm,
thorny nature of cactus and scrub plant,
only a small part of the scrub plant above the surface.
Four labels correctly placed = 4 marks

[4]

(ii) Cactus only has surface roots to trap water after occasional desert rains, its main
adaptations are above the surface storing moisture in its fleshy stem, whereas the other
plants rely more upon greatest possible use of underground water, either down to the
water table or extensive coverage.
Two points made along these lines.
(d)

[2]

(i) Wandering the desert for new pastures with their herds = nomadism
Reliance on animals for all their needs = pastoral
Stating only from the passage = 1 mark
Explaining how this makes it pastoral nomadism = 2nd mark

[2]

(ii) Main reason is that the camel is the beast of burden to transport their belongings as they
move around the desert searching for fresh pastures.
Other acceptable reasons are:
has the greatest number of uses food and drink plus others,
best adapted of the animals to living in desert conditions,
useful for other aspects of their lives such as trading.
Two reasons sufficiently distinct.

[2]

(iii) Still carried out after having been followed for centuries, changing pastures means that
grazing takes place only where vegetation exists, group moves on before overgrazing
occurs, have an extra income from trading, way of life is self sustaining/does not depend
on large inputs from outside.
Some understanding = 1 mark
Understanding with some support = 2 marks
Well understood and supported by relevant references to ways of life = 3 marks

[3]

UCLES 2010

420

Page 7

Mark Scheme: Teachers version


GCE O LEVEL May/June 2010

Syllabus
5014

Paper
11

BALOL
(iv) Effects can be positive/beneficial, or negative/disadvantages.
Positive effects mainly economic; they include work on the oil wells and in the
refineries, likely to be much better paid; can live in one place instead of the nomadic
desert existence. Urban living with all the modern conveniences and less exposed to
natural conditions and changes.
Negative effects some economic because land traditionally used and crossed over by
them is being taken away for irrigated farming and oil. Migration with animals made
more difficult by pipelines crossing the desert. Social disruption caused by migrations
of young folk, likely in future to be short of people to carry on the traditions, and do the
work as their parents get older. Once disrupted, their way of life is in danger of being
lost for ever.
Effects mentioned, but not really explained; mainly one effect, or all negative or positive
leading to a narrow answer.
[12 marks]
Both social and economic effects covered, including both good and bad.
Reasonable breadth of coverage.
(e)

[34 marks]

(i) Overgrazing or over cultivation; allow deforestation/very intensive farming.

[1]

(ii) The underlying/basic cause = 1 mark


Elaborated upon/emphasised = 2nd mark

[2]

(iii) Background about why rates are high i.e. high birth rates compared with low and
declining death rates, resulting in high rates of natural increase.
The explanation can be for both of these, but it is most likely that answers will be about
why birth rates remain high with references to social and economic factors such as
family planning availability, social customs, role of certain religions, lack of female
education, limited career opportunities for women.
The slow progress element can bring in the role of governments and the existence or
otherwise of population policies. Opportunities for case study use e.g. 1 child policy in
China showing what could be done, but what has not been done elsewhere in
developing countries.
National economic factors also play a part the poverty of many African countries which
can't afford population policies even if the will exists.
General points without development, or over-concentration on only one.

[12 marks]

Useful range of relevant points made, with supporting detail.

[34 marks]

As before, but with a reference to the African and other developing countries part of the
question.
[5 marks]
[Total: 40]

UCLES 2010

421

Page 8

Mark Scheme: Teachers version


GCE O LEVEL May/June 2010

Syllabus
5014

Paper
11

BALOL
6

(a)

(i) Formed where there used to be dense forests (e.g. TRF), these die, decay and
decompose, covered over by new surface deposits, compressed over millions of years
into layers of coal.
Understood and compete = 3 marks
Some or partial explanation = 12 marks

[3]

(ii) Fossil fuel living matter from millions of years ago preserved in rocks.
Fossil fuel in the sense of time taken to form up to 300 million years.
1 mark for each reason.

[2]

(iii) Plants absorb carbon dioxide from the air as part of photosynthesis, some of it remains
stored and is released when burnt, carbon in plants when burnt combines with oxygen in
the air.
Understood = 2 marks
Some understanding = 1 mark
(b)

[2]

(i) All plots accurate = 3 marks


Key completed in agreement with graph = 1 mark
If not, 1 or 2 correct plots = 1 mark
3 or 4 correct plots = 2 marks

[4]

(ii) The gap between ten year periods has increased = 1 mark
supported by values read off graph 1.2 for 198797 and 2.0 for 19972007.
2 marks

[2]

(iii) Oil, coal and natural gas identified as being the fossil fuels,
total of 9.7 billion out of 11 billion,
estimate or calculation of the percentage for emphasis (88%),
comment on the low contribution from alternatives.
Three statements along these lines.
(c)

[3]

(i) Oil can be brought to surface using drilling machinery, whereas underground seams of
coal need men and mines, oil can be obtained both offshore and onshore.
(ii) Oil is transported by pipelines and tankers over long distances, since it is a liquid rather
than a solid it flows and is more easily transferred between different forms of transport,
coal is bulkier to load and unload.
(iii) Oil, being a liquid, is easier to control in use amount can be measured easily and it can
be switched on and off, coal, being a solid, involves more mechanical and human work,
heating up and reducing heat are slower/less finely tuned processes with coal.
Entirely or heavily one sided statements = 1 mark max. for each
Two sides comparative statements = up to 2 marks each
Reserve a minimum of 1 mark for each part.
Unclaimed marks can be topped up by any second comparative statements, within any
of the headings.
Any mixture of 1 and 2 mark statements; maximum three marks per part.
[6]
UCLES 2010

422

Page 9

Mark Scheme: Teachers version


GCE O LEVEL May/June 2010

Syllabus
5014

Paper
11

BALOL
(d)

(i) 25%

[1]

(ii) Two aspects to this; the basic statements are:


a lot of wind turbines are needed to match the output from one coal fired power station
(1000 at best, in reality 4000 of them)
reliability because the coal fired station can deliver a consistently high output close to
full, whereas the wind turbine is only 25% efficient.
Fuller development of these statements can include:
great areas of land would need to be covered to give the same output as one compact
power station,
wind farms must have greater costs of construction and operation, there are days
without wind (why the wind turbine is only 25% efficient), on these days another source
like a coal fired station is needed.
Part answer 1 or 2 marks
Fuller answer showing more understanding 3 or 4 marks

[4]

(iii) Sulphur dioxide and nitrogen oxides are released, these cause acid rain which destroys
forests/kills trees, lakes and water courses become too acid for plant/water life.
Two points made along these lines.

[2]

(iv) Local problem:


greatest concentration of pollution emissions in area around chimney,
washed down to surface when raining (wet acid rain),
damage around power stations includes rotting stonework and dead trees.
International problem:
winds are capable of carrying pollutants in the atmosphere long distances to other
countries,
example of where this is a problem such as UK to Scandinavia.
Points made along these lines minimum 1 mark for each of local and international.
Otherwise 3 marks for 3 for explanatory points.
[3]
(e)

(i) Producers are concentrated in three continents (North America, Europe and Asia),
all are in the northern hemisphere,
mostly developed world countries,
none of them are from southern hemisphere continents dominated by developing
countries,
nuclear power is rich world energy source/beyond the means and technology of poor
countries.
Three points made along these lines, including some reference to both continents with
and without nuclear power stations.
[3]

UCLES 2010

423

Page 10

Mark Scheme: Teachers version


GCE O LEVEL May/June 2010

Syllabus
5014

Paper
11

BALOL
(ii) Arguments for nuclear power include clean in the sense of giving no carbon emissions,
not contributing to global warming, source uses relatively little raw material (uranium)
compared with the amount of energy produced, known technology which has been made
more efficient.
Arguments against nuclear power include generates wastes that are radioactive without
satisfactory means of storage, remain dangerously radioactive for many years,
radioactive leaks are highly dangerous to living things, causing cancers, explosion of
Chernobyl contaminated a wide area, known world reserves of uranium are running
down.
Some arguments for or against, but simple/shallow/unsupported statements. May be
very unbalanced e.g. dominated by arguments against.
[12 marks]
More substantial arguments, with something for both sides, even if not in perfect
balance. More understanding shown. View may be explained no better than in an
earlier statement.
[34 marks]
Arguments made for both sides. The view expressed is an overview and is supported by
the explanation.
[5 marks]
[Total: 40]

UCLES 2010

424

General Certificate of Education Ordinary Level


5014 Environmental Management June 2010
Principal Examiner Report for Teachers

BALOL
ENVIRONMENTAL
MANAGEMENT

Paper 5014/21
Paper 21

General comments
This paper invited candidates to consider environmental issues and methods of gathering and interpreting
data in the context of one country, Costa Rica. Many candidates understood and made good use of the
source material and their written responses were sufficiently clearly expressed that the Examiners could be
confident that marks awarded were deserved. The mathematical and graphical questions did pose some
difficulties for a minority of candidates.
Candidates had no problems completing the paper in the time available.
Overall the pattern of this paper is very similar to past papers and Centres should work through past papers
to help candidates see how to make the best use of the information given for each question.

Comments on specific questions


Question 1
(a)

Many candidates identified at least one reason why exports were not occurring, the idea that all the
products were needed in the country was most often given credit.

(b)

The vast majority of candidates gave at least one creditworthy point. However, vague statements
about making more money or profit needed to be qualified as shown in the mark scheme to gain
credit.

(c)

Most candidates completed the three rows as requested on the planting plan. However, care was
needed to keep the spacing even and the correct number of plants. Some candidates planted at
the same density as the original or half the original density. In part ii the graphs were usually well
plotted but sometimes labels were missing from axes. In part iii nearly all candidates correctly
read a figure from the graph they had plotted. In part iv candidates usually identified that there was
no increase in yield, surprisingly this was not always supported by a second reason for not planting
at 80 thousand plants per hectare.

(d)

The trends in soil erosion were usually well described and the second marking point was often
made. In part ii the question was a little more demanding as candidates needed to consider their
graph.
Planting at 70 thousand plants per hectare was not the best answer though many were drawn to
state this. Answers between 50-60 thousand plants per hectare usually gained both marks for
supporting reasons. In part iii soil erosion was sometimes described as wearing rock away rather
than removal of topsoil. In part iv There were some very good descriptions of the process of soil
erosion that gained maximum marks. Unfortunately some candidates went on to describe soil
erosion due to mining which did not gain credit.

(e)

Candidates realised that Plan One was inadequate but sometimes struggled to give clear answers
that could be given credit. In part ii neat tables were often drawn; to gain the third mark there
needed to be a clear indication that 25 items of data could be recorded. In part iii it was clear that
candidates realised this was a better plan but sometimes their answers were too vague to gain
credit. To just say more data is gathered was not regarded as a clear answer.

425

General Certificate of Education Ordinary Level


5014 Environmental Management June 2010
Principal Examiner Report for Teachers
Question
BALOL 2
(a)

Most candidates gave the correct answer of 4000 dollars. In part ii weaker candidates just said
because they need to be relocated. However many realised that the government would still gain
revenue in the longer term and it would help to prevent objections to the scheme.

(b)

That the larger lake could generate more electricity was clearly stated by many candidates though
few seemed to appreciate that power generation could be continuous or that the water for HEP
would not run out.

(c)

Candidates nearly all described the need to prevent overfishing and a reasonable number went on
to suggest that the number of tourists could be controlled.

(d)

There were many clearly expressed answers gaining maximum marks. When figures were used
from the data table they successfully supported the answer given. In part ii a range of figures from
the table was suggested and some suggested the average figures. However sample one nitrate
was furthest from the general pattern of the data.
In part iii there were many correct references to reliability or the need to calculate an average.

(e)

There were many candidates who gave clear concise answers and gained maximum marks. The
Examiners were pleased to see that the details of eutrophication had been well understood.

(f)

This question was the hardest on the paper. Candidates needed to look carefully at the pattern of
movement of the pesticide and then describe the movement. Some data from the source helped
some answers but most candidates used too many references to the letters rather than considering
the rate of movement. However, in part ii candidates could gain a mark even if the previous
section had proved to be demanding. In part iii most candidates realised that the pesticide was
going to enter the lake although few appreciated that the consequence of this was unknown but the
risk was not worth taking.

Question 3
(a)

Most candidates understood the three parts to this question and gave appropriate answers in their
own words having selected an appropriate statement.

(b)

Many candidates gave answers that did not quite make the point that the process was using
renewable energy or that it was non-polluting or that oxygen is not a greenhouse gas. Some
candidates stated that carbon dioxide would be released although the source made no reference to
this gas. There was an assumption by some candidates that any industrial process must be
damaging the environment. In part ii most candidates gave thoughtful answers clearly outlining
some arguments in favour and against the development. Most candidates gained three or four
marks here and it was very rare to see answers that were not trying to balance the arguments.

426

BALOL
UNIVERSITY OF CAMBRIDGE INTERNATIONAL EXAMINATIONS
General Certificate of Education Ordinary Level

*7726080987*

5014/12

ENVIRONMENTAL MANAGEMENT
Paper 1

May/June 2010
2 hours 15 minutes

Candidates answer on the Question Paper.


Additional Materials:

Ruler

READ THESE INSTRUCTIONS FIRST


Write your Centre number, candidate number and name on all the work you hand in.
Write in dark blue or black pen.
You may use a soft pencil for any diagrams, graphs or rough working.
Do not use staples, paper clips, highlighters, glue or correction fluid.
DO NOT WRITE IN ANY BARCODES.
Answer all questions.
All questions in Section A carry 10 marks.
Both questions in Section B carry 40 marks.
At the end of the examination, fasten all your work securely together.
The number of marks is given in brackets [ ] at the end of each question or part question.

For Examiners Use


1
2
3
4
5
6
Total
This document consists of 22 printed pages and 2 blank pages.
UCLES 2010

[Turn o

427

For
Examiners
Use

BALOL
1

Section A

(a) Look at the diagram which shows methods of producing energy from household waste.

heat

water

steam

turbine

electricity

incinerator
suitable
non-organic waste
household
waste
collected
and
sorted

unsuitable
waste

gas

organic
waste

ane

th
me

diesel

digester
containing
bacteria
Circle the word listed below which describes the type of energy source shown in the
lower part of the diagram.
biomass

geothermal

hydro

solar

wind

[1]

(b) Why is the organic waste separated from the other waste?
..................................................................................................................................... [1]
(c) Describe what happens in the digester.
..........................................................................................................................................
..................................................................................................................................... [2]
(d) Suggest why some household waste is unsuitable for use in the incinerator.
..........................................................................................................................................
..........................................................................................................................................
..........................................................................................................................................
..........................................................................................................................................
..........................................................................................................................................
..................................................................................................................................... [3]

UCLES 2010

5014/12/M/J/10

428

For
Examiners
Use

BALOL
(e) What are the advantages of using household waste in these ways?
..........................................................................................................................................
..........................................................................................................................................
..........................................................................................................................................
..........................................................................................................................................
..........................................................................................................................................
..................................................................................................................................... [3]
[Total: 10]

UCLES 2010

5014/12/M/J/10

[Turn over

429

4
2 BALOL
(a) Look at the diagram of ocean zones and the pie chart showing the potential fish harvest
from three zones.
continental
shelf

For
Examiners
Use

open ocean in which areas of upwelling


are less than 1%

10%

90%
sea level
200 m

continental
shelf

continental
slope
abyssal
plain

abyssal
plain

4 km

ocean
deep
12 km
open
oceans

areas of
upwelling
continental
shelf and
coastal
waters

potential fish production

UCLES 2010

5014/12/M/J/10

430

BALOL (i) What percentage of the potential fish production from the oceans is expected to

For
Examiners
Use

come from the continental shelf and coastal waters?


.............................................. % [1]
(ii)

Use both diagrams to comment on the potential fish production from the open
ocean.
..................................................................................................................................
..................................................................................................................................
............................................................................................................................. [2]

(b) (i)

Why are fish important in the human diet in some parts of the world?
............................................................................................................................. [1]

(ii)

Describe other valuable resources which oceans can provide.


..................................................................................................................................
..................................................................................................................................
..................................................................................................................................
..................................................................................................................................
..................................................................................................................................
............................................................................................................................. [3]

(c) Suggest why a full exploitation of the resources of the open oceans is unlikely.
..........................................................................................................................................
..........................................................................................................................................
..........................................................................................................................................
..........................................................................................................................................
..........................................................................................................................................
..................................................................................................................................... [3]
[Total: 10]

UCLES 2010

5014/12/M/J/10

[Turn over

431

6
3 BALOL
(a) Look at the map, which gives information for a recent year about the worlds main
exporting and importing countries of fresh pineapple, together with the main trade routes
of fresh pineapple.

For
Examiners
Use

SOME
EU COUNTRIES

CANADA

JAPAN

USA
China

Tropic of
Cancer
Philippines

Thailand
Costa
Rica

Equator

Kenya

Ivory
Coast

Indonesia

Tropic of Capricorn

Key
The main importing countries of the
European Union are Belgium, Luxembourg,
the Netherlands, UK, France, Italy, Germany,
Portugal and Spain.

fresh pineapple exports


tonnes
200,000
100,000
0
Exporting country names in lower case

main fresh pineapple trade routes


20,000 to 100,000 tonnes per year

Countries which together


import 86% of the world
imports of fresh pineapple

100,000 to 400,000 tonnes per year


over 400,000 tonnes per year

Importing country names in capital letters

(i)

Name the country which exports the most fresh pineapple and state the amount it
exports.
country ................

(ii)

amount ................ tonnes [1]

Name one continent with large imports of fresh pineapples.


............................................................................................................................. [1]

UCLES 2010

5014/12/M/J/10

432

For
Examiners
Use

BALOL(iii) State one similarity in the direction of the sea trade routes shown.
............................................................................................................................. [1]
(iv)

Suggest why some countries export more fresh pineapples than others.
...................................................................................................................................
..................................................................................................................................
..................................................................................................................................
..................................................................................................................................
..................................................................................................................................
............................................................................................................................. [3]

(b) Pineapples are often grown on plantations. Describe how commercial farming, such as
plantation farming, is organised to produce large amounts of a cash crop for export.
..........................................................................................................................................
..........................................................................................................................................
..........................................................................................................................................
..........................................................................................................................................
..........................................................................................................................................
..........................................................................................................................................
..................................................................................................................................... [4]
[Total: 10]

UCLES 2010

5014/12/M/J/10

[Turn over

433

8
4 BALOL
(a) Look at the photograph showing vegetation growing on the edge of a clearing in a
tropical rainforest.

(i)

For
Examiners
Use

Describe the features of the vegetation in the photograph.


..................................................................................................................................
..................................................................................................................................
..................................................................................................................................
..................................................................................................................................
..............................................................................................................................[3]

UCLES 2010

5014/12/M/J/10

434

BALOL(ii) Explain how the leaves and stems are adapted to the climate in which tropical

For
Examiners
Use

rainforest grows.
..................................................................................................................................
..................................................................................................................................
..................................................................................................................................
..................................................................................................................................
..................................................................................................................................
..................................................................................................................................
............................................................................................................................. [4]

(b) Some areas of tropical rainforest have been made into National Parks. Is this method of
conservation likely to have any benefits for local people? Explain your views.
..........................................................................................................................................
..........................................................................................................................................
..........................................................................................................................................
..........................................................................................................................................
..........................................................................................................................................
..................................................................................................................................... [3]
[Total: 10]

UCLES 2010

5014/12/M/J/10

[Turn over

435

For
Examiners
Use

10

BALOL
5

Section B

(a) Look at the pie graphs showing the distribution of water on the Earths surface.
land and sea

fresh water and


salt water
fresh
water

land
salt
water

ocean

Describe the evidence from the pie graphs for each of statements A and B.
A There is a lot more water than land on the Earths surface.
..........................................................................................................................................
..........................................................................................................................................
B The amount of water available for human use is very small.
..........................................................................................................................................
......................................................................................................................................[2]
(b) The diagram gives information about fresh water stores on the Earths surface.
lakes
0.3%

rivers
less than
0.1%

groundwater
(above 1000 m)
10.6%
deep groundwater
(below 1000 m)
14%

ocean

UCLES 2010

ice & snow


75%

5014/12/M/J/10

436

11

BALOL (i) Show the percentages for ice and snow, total groundwater, and lakes and rivers on

For
Examiners
Use

a pie graph.
fresh water stores on the Earths surface

[3]
(ii)

Explain more fully why the amount of water available for human use is so small.
..................................................................................................................................
..................................................................................................................................
..................................................................................................................................
..................................................................................................................................
..................................................................................................................................
..............................................................................................................................[3]

(iii)

How can people obtain their water supply from groundwater stores? Describe one
method.
..................................................................................................................................
..................................................................................................................................
..................................................................................................................................
..............................................................................................................................[2]

(iv)

State two advantages of using groundwater stores for water supply.


..................................................................................................................................
..................................................................................................................................
..............................................................................................................................[2]

UCLES 2010

5014/12/M/J/10

[Turn over

437

12

BALOL
(c) In many countries large dams have been built to increase natural stores in rivers and

For
Examiners
Use

lakes.
(i)

Name or state the location of an example of a large dam.


..............................................................................................................................[1]

(ii)

Describe its advantages and explain why it was built.


..................................................................................................................................
..................................................................................................................................
..................................................................................................................................
..................................................................................................................................
..................................................................................................................................
..............................................................................................................................[4]

(iii)

Why are decisions to build large dams often controversial? Describe some of the
economic, social and environmental factors which help to explain why some people
object to the building of large dams.
..................................................................................................................................
..................................................................................................................................
..................................................................................................................................
..................................................................................................................................
..................................................................................................................................
..................................................................................................................................
..................................................................................................................................
..................................................................................................................................
..................................................................................................................................
..............................................................................................................................[5]

UCLES 2010

5014/12/M/J/10

438

For
Examiners
Use

13

BALOL
(d) Look at the spider diagram showing some uses of rivers.
domestic for
cooking and washing
irrigation water
for crops

waste disposal
uses of
rivers

navigation and
shipping

recreation, swimming
and canoeing
cooling water
for power stations
(i)

Which two uses are most likely to lead to pollution of river water? Explain why.
Use 1 .........................................................
..................................................................................................................................
..................................................................................................................................
..................................................................................................................................
Use 2 .........................................................
..................................................................................................................................
..................................................................................................................................
..............................................................................................................................[4]

(ii)

Conflicts of interest can arise between people using rivers for different purposes.
Giving examples, explain some of these conflicts of interest.
..................................................................................................................................
..................................................................................................................................
..................................................................................................................................
..................................................................................................................................
..................................................................................................................................
..................................................................................................................................
..................................................................................................................................
..............................................................................................................................[3]

UCLES 2010

5014/12/M/J/10

[Turn over

439

For
Examiners
Use

14

BALOL
(e) Look at the graphs showing access to water supply and sanitation.
water supply

sanitation

90

90

80

80

70

70

60

60

% access

100%

% access

100%

50

50

40

40

30

30

20

20

10

10

world

urban

rural

world

urban

rural

Use values from the graphs to describe how they show that access to sanitation is
(i)

less widely available than for water supply;


..................................................................................................................................
..................................................................................................................................
..............................................................................................................................[2]

(ii)

particularly poor in rural areas.


..................................................................................................................................
..................................................................................................................................
..............................................................................................................................[2]

UCLES 2010

5014/12/M/J/10

440

15

BALOL(iii) Give reasons why access to water supply and sanitation is much poorer in rural

For
Examiners
Use

than in urban areas.


..................................................................................................................................
..................................................................................................................................
..................................................................................................................................
..................................................................................................................................
..................................................................................................................................
..............................................................................................................................[3]
(f)

Costs from lack of safe water supply and sanitation


1.6 million children a year die
from diarrhoea

economic loss of the time taken


in fetching water every day

Explain why
(i)

children are most at risk from water-related diseases;


..................................................................................................................................
..................................................................................................................................
..................................................................................................................................
..................................................................................................................................

(ii)

economic costs result from people having to walk long distances to fetch water.
..................................................................................................................................
..................................................................................................................................
..................................................................................................................................
..............................................................................................................................[4]
[Total: 40]

UCLES 2010

5014/12/M/J/10

[Turn over

441

16
6 BALOL
(a) (i)

Complete the table of birth and death rates and population change for selected
countries from Europe and Asia.
Country

Birth rate
per 1000

Death rate
per 1000

Population change
per 1000

11.0

10.2

+0.8

8.7

10.7

China

14.5

7.1

India

23.8

8.3

Saudi Arabia

31.5

3.7

UK
Germany

For
Examiners
Use

+27.8
[3]

(ii)

How is Germany different from the other countries in the table?


..............................................................................................................................[1]

(b)

World distribution of birth rates

Key
Live births per
1000 population
Very high
above 40
High
2540
Moderate
1524.9
Low
below 15
(i)

Equator

Name the continent with the largest number of countries with very high birth rates
above 40 per 1000.
..............................................................................................................................[1]

(ii)

In which continent are there the most countries with birth rates below 15 per
1000?
..............................................................................................................................[1]

UCLES 2010

5014/12/M/J/10

442

17

BALOL(iii) Describe the distribution of countries with high and very high birth rates above 25

For
Examiners
Use

per 1000.
..................................................................................................................................
..................................................................................................................................
..................................................................................................................................
..................................................................................................................................
..............................................................................................................................[3]
(iv)

Give reasons why some countries have very low birth rates, below 15 per 1000.
..................................................................................................................................
..................................................................................................................................
..................................................................................................................................
..................................................................................................................................
..................................................................................................................................
..................................................................................................................................
..................................................................................................................................
..............................................................................................................................[4]

UCLES 2010

5014/12/M/J/10

[Turn over

443

For
Examiners
Use

18

BALOL
(c) Look at the pyramids showing population structure for Ethiopia and the UK.
Ethiopia

UK

75+
70-74
65-69
60-64
55-59
50-54
45-49
40-44
35-39
30-34
25-29
20-24
15-19
10-14
5-9
0-4

90+
85-89
80-84
75-79
70-74
65-69
60-64
55-59
50-54
45-49
40-44
35-39
30-34
25-29
20-24
15-19
10-14
5-9
0-4

10 9 8 7 6 5 4 3 2 1 0
% males

6 5 4 3 2 1 0
% males

0 1 2 3 4 5 6 7 8 9 10
% females

0 1 2 3 4 5 6
% females

(i)

On each pyramid, shade in the age group with the highest total percentage of
population.
[1]

(ii)

What is the approximate total percentage of population in Ethiopia below the age
of 15?
..............................................................................................................................[1]

(iii)

What is the approximate total percentage of population aged 65 and above in the
UK? Circle one answer.
6.5

(iv)

9.5

12

16

[1]

State two differences in shape between the population pyramids for Ethiopia and
the UK.
..................................................................................................................................
..................................................................................................................................
..............................................................................................................................[2]

(v)

The UK has an ageing population. State the evidence for this from its population
pyramid.
..................................................................................................................................
..................................................................................................................................
..............................................................................................................................[2]

UCLES 2010

5014/12/M/J/10

444

19

BALOL(vi) Some countries like Ethiopia have a young population; others like the UK have an

For
Examiners
Use

ageing population. Their population problems are different.


Explain how and why their problems are different.
..................................................................................................................................
..................................................................................................................................
..................................................................................................................................
..................................................................................................................................
..................................................................................................................................
..................................................................................................................................
..................................................................................................................................
..............................................................................................................................[4]

UCLES 2010

5014/12/M/J/10

[Turn over

445

20

BALOL
(d) Look at the world map which shows the course of the dividing line between the rich

For
Examiners
Use

North and poor South.


N
North
America
26,900
T. of Cancer

th
Nor
The South
The

Europe
12,100

Africa
690
Latin
America
3,100

Equator

T. of Capricorn

Asia
2,200
Th
Th e N
e or
So th
ut
h

Oceania
13,900
Key
Average income per
head in US $

(i)

Rank the six continents by income per head from highest to lowest. State whether
each continent is mainly developed or developing.
Continent

Developed or Developing

1 ............................................................. ...............................................................
2 ............................................................. ...............................................................
3 ............................................................. ...............................................................
4 ............................................................. ...............................................................
5 ............................................................. ...............................................................
6 ............................................................. ........................................................... [2]
(ii)

Why does the North-South dividing line not continue running west to east all the
way across the world?
..................................................................................................................................
..............................................................................................................................[1]

UCLES 2010

5014/12/M/J/10

446

21

BALOL(iii) How well does the line split the world between a rich North and a poor South?

For
Examiners
Use

Answer as fully as you can describing where the fit is good and not so good.
..................................................................................................................................
..................................................................................................................................
..................................................................................................................................
..................................................................................................................................
..................................................................................................................................
..................................................................................................................................
..............................................................................................................................[3]
(iv)

Look back to the map showing the world distribution of birth rates in part (b).
How well would the North-South dividing line separate countries with high and low
birth rates?
..................................................................................................................................
..................................................................................................................................
..................................................................................................................................
..................................................................................................................................
..............................................................................................................................[2]

UCLES 2010

5014/12/M/J/10

[Turn over

447

For
Examiners
Use

22

BALOL
(e) Two strategies for reducing the wealth gap between rich and poor countries:
Improved trade such as Fair Trade
Non-governmental aid organisations (NGOs) from rich countries.
(i)

How is Fair Trade different from other trade?


..................................................................................................................................
..................................................................................................................................
..................................................................................................................................
..............................................................................................................................[2]

(ii)

Explain some of the advantages and disadvantages of aid for helping poor countries
and reducing the wealth gap between them and rich countries.
..................................................................................................................................
..................................................................................................................................
..................................................................................................................................
..................................................................................................................................
..................................................................................................................................
..................................................................................................................................
..................................................................................................................................
..............................................................................................................................[4]

(iii)

In your view, which of the two strategies is better for reducing the wealth gap
between rich and poor countries in the long term? Explain your choice.
..................................................................................................................................
..................................................................................................................................
..................................................................................................................................
..................................................................................................................................
..............................................................................................................................[2]
[Total: 40]

UCLES 2010

5014/12/M/J/10

448

BALOL
UNIVERSITY OF CAMBRIDGE INTERNATIONAL EXAMINATIONS
GCE Ordinary Level

MARK SCHEME for the May/June 2010 question paper


for the guidance of teachers

5014 ENVIRONMENTAL MANAGEMENT


5014/12

Paper 12, maximum raw mark 120

This mark scheme is published as an aid to teachers and candidates, to indicate the requirements of
the examination. It shows the basis on which Examiners were instructed to award marks. It does not
indicate the details of the discussions that took place at an Examiners meeting before marking began,
which would have considered the acceptability of alternative answers.
Mark schemes must be read in conjunction with the question papers and the report on the
examination.

CIE will not enter into discussions or correspondence in connection with these mark schemes.

CIE is publishing the mark schemes for the May/June 2010 question papers for most IGCSE, GCE
Advanced Level and Advanced Subsidiary Level syllabuses and some Ordinary Level syllabuses.

449

Page 2

Mark Scheme: Teachers version


GCE O LEVEL May/June 2010

Syllabus
5014

Paper
12

BALOL
Section A
1

(a) Biomass circled (or otherwise clearly indicated).

[1]

(b) Non-organic waste cannot be used in digester / to use organic waste in the digester.

[1]

(c) Basic points:


Bacteria break down organic matter,
Methane gas is produced.

[2]

(d) Some are toxic materials,


examples of items in household waste which include metals such as batteries,
details of toxic content such as lead, cadmium etc.,
chlorine based plastics,
incombustible materials,
glass.
Credit explanatory elaboration.
Three suggestions along these lines, 1 mark for each.

[3]

(e) Ideas such as:


reduces need for landfill / to find landfill sites,
advantages such as lessens leakage of pollutants into ground water,
and other advantages such as less smell / visual pollution,
constant source of waste for energy production,
advantages for reduction in fossil fuel use,
waste turned into something useful / source of income,
combined heat and power to local housing.
Three points made along these lines, 1 mark for each.

[3]
[Total: 10]

UCLES 2010

450

Page 3

Mark Scheme: Teachers version


GCE O LEVEL May/June 2010

Syllabus
5014

Paper
12

BALOL
2

(a) (i) 6268% / accept

[1]

(ii) Realisation that production is very small,


from a very large area of open ocean / 90% of the ocean area,
valid comment on the upwelling.
Two points, 1 mark for each.
(b) (i) Most likely answer that fish provide protein,
also allow build muscle / fish oils if given.

[2]

[1]

(ii) Useful natural resources such as oil / gas,


also sand and gravel from shorelines,
energy sources such as wave / tidal power,
useful sites for offshore wind farms,
water supply from desalination,
locations for recreation / tourism,
world's major shipping routes,
possible sources for medicines,
salt source.
Credit located examples of any of these for a second mark.
Three points, one mark for each or two points with an example.

[3]

(c) Basic point difficulties of exploration and exploitation,


elaboration of these with references to:
water depth / pressure,
distance from shore for supplies / high transport costs,
weather problems such as cold / storms / strong winds,
at or beyond the limits of known technology,
overall expense compared with obtaining resources on land.
Three points made along these lines, 1 mark for each.

[3]
[Total: 10]

UCLES 2010

451

Page 4

Mark Scheme: Teachers version


GCE O LEVEL May/June 2010

Syllabus
5014

Paper
12

BALOL
3

(a) (i) Thailand amount 1,000,000 to 1,300,000 tonnes.

[1]

(ii) North America / Europe

[1]

(iii) All go northwards / towards developed world countries.

[1]

(iv) Possible reasons:


shorter distances to main importing countries,
which means that they arrive fresher at market,
also cheaper transport costs,
country produces the cheapest / cheaper pineapples than others,
trade agreements in place with exporting countries,
higher domestic demand for pineapples in producing country,
differences in amount produced / scale of the farming operations.
Three reasons along these lines, 1 mark for each.

[3]

(b) Very large farms,


high capital inputs,
some owned by companies rather than individuals,
large / skilled / specialised workforce,
modern / scientific methods of farming such as mechanisation,
further examples such as use of pesticides, fertilisers, irrigation,
use of high quality seeds / plants / hybrids,
monoculture / one crop only.
Points such as these; further elaboration might be given for the points stated.
Four points, 1 mark for each.

[4]
[Total: 10]

UCLES 2010

452

Page 5

Mark Scheme: Teachers version


GCE O LEVEL May/June 2010

Syllabus
5014

Paper
12

BALOL
4

(a) (i) Observations about the stems:


straight / vertical / thick stems.
Observations about the leaves:
large / broad leaves,
drip tips,
depression along central vein,
leaflets from central point / some long and narrow,
some lighter at edges.
General observation about high density of vegetation.
Three points, 1 mark for each.

[3]

(ii) Leaves large to catch sunlight, maximise photosynthesis, great competition for light in
the forests.
Drip tips to shed excess rainwater, depression along central vein gives a channel for
shedding water during the frequent heavy downpours.
Leathery leaves because of great heat and intense sunlight, keeps the leaf rigid for
photosynthesis / transpiration.
Vertical / straight stems tall to compete and reach for sunlight, trees can grow tall in
the constantly hot and wet climate, strong stems as support in heavy rain.
Minimum 1 mark for each of leaves and stems.
Otherwise up to two marks for each reasoned adaptation.
Four points, 1 mark for each.

[4]

(b) Has benefits for local people suggestions include:


direct employment (e.g. guides, park rangers, drivers),
indirect employment (e.g. in markets, shops, making crafts),
make use of public services / facilities established for tourists,
forests preserved so can continue to collect fruits etc. from it.
Little benefit to local people suggestions include:
jobs go to outsiders not locals (especially better paid work),
tourism brings air / noise / litter pollution with it,
traditional culture and ways of life destroyed without any rewards,
deprived of traditional land / long established activities,
diseases brought in to which they have little resistance,
damage to crops from forest animals.
Points can be made from one or both viewpoints.
Three points, 1 mark for each.

[3]
[Total: 10]

UCLES 2010

453

Page 6

Mark Scheme: Teachers version


GCE O LEVEL May/June 2010

Syllabus
5014

Paper
12

BALOL
Section B
5

(a) A almost three quarters ocean (plot = 71%), or vice versa, or similar
B fresh water makes up a tiny percentage of water on Earth (plot = 3%), or similar
(b)

[2]

(i) Reasonably accurate plot of ice and snow 75%, groundwater 25%, with thinnest of
sectors less than 1% for lakes and rivers = 2 marks
Part accuracy = 1 mark
Labels or key for sectors = 1 mark
[3]
(ii) [No further credit for tiny percentage of fresh water]
75% of fresh water locked up in ice and snow only available when these melt,
most located in cold places where few people live anyway,
easy to reach surface sources like rivers and lakes make up only 0.4 %,
more groundwater available but more difficult/costly to obtain than surface stores,
much of the groundwater is deep and out of the reach of humans,
comment about the value of rivers as water supply for people, yet they are only 0.1%,
high costs of desalination.
Three explanatory points like these.

[3]

(iii) May be all human power, animal power such as sakeer/Persian wheel, mechanical such
as tube wells; can be traditional or modern.
In some places it is just a matter of collecting water from springs or other natural surface
outpourings.
Name with basic description = 1 mark
Further descriptive detail = 2nd mark

[2]

(iv) Possible advantages of groundwater stores include reliability of supply, all-year/


all-season availability, not subject to short term weather variations, free from
evaporation, clean/not polluted.
Two advantages given, 1 mark for each.
(c)

(i) Name of large dam or clear location (more than just a river name).

[2]
[1]

(ii) References to advantages such as its many uses (electricity, irrigation water, water
supply for people and industry), flood control, navigation, tourism.
Further detail such as increase in crop areas and yields, reclamation of desert land for
crops, etc.
The explain why it was built part of the question allows reference to broader economic
factors, plus physical factors.
General advantages of large dams/nothing specific to named example, or no named
example in part (i) maximum of 2 marks.
General advantages of large dams that could apply to the named example, but lacks
specificity maximum of 3 marks.
Valid example in part (i) and includes specific information related to it up to four marks.
[4]

UCLES 2010

454

Page 7

Mark Scheme: Teachers version


GCE O LEVEL May/June 2010

Syllabus
5014

Paper
12

BALOL
(iii) Examples of objections that might be used:
Economic high costs, places the country in debt/need to borrow from rich countries,
diverts spending away from other areas/projects.
Social people often displaced from best land on valley floors, family disruption/loss of
communities and traditions, not always as well compensated as promised, moved on to
inferior land.
Environmental clearances of forests/vegetation, loss of habitats, disruption of river flow
and ecosystems below the dam.
Narrow answers with one type of factor only referred to, or broader answers but lacking
in supporting detail worth 1 or 2 marks.
Broader answers with at least two factors covered and some substance to the comments
worth 3 or 4 marks.
All three factors covered with support; particularly if comment is included about their
controversial nature worth 5 marks.
[5]
(d)

(i) Two uses most likely to lead to water pollution are waste disposal and ships and
navigation = 1 mark for these choices.
Explanation such as untreated waste, litter and toxic materials disposed of, oil/diesel
from ships' engines or cleaning out of tanks, or disposal of waste over the sides.
Other uses can be credited for explanation provided that candidates show how they lead
to water pollution e.g. washing out nitrates/pesticides related to irrigation water for crops,
and untreated sewage for domestic uses.
Either 1 mark for choice + 3 marks for explanation (1 + 3).
Or no marks for choice and 4 marks for explanation (0 + 4).

[4]

(ii) Possible conflicts that candidates might refer to:


waste disposal upstream then use of water for drinking/recreation downstream
shipping is often densest in most populated river sections
irrigation water is taken out of the river so that not enough is left downstream for all the
other users
mention of other conflicts such as fishermen and river life destroyed by pollution.
Mention of at least two examples, and explanation for at least one for all three marks. [3]
(e)

(i) Access to sanitation is lower than for water supply for all three (world, urban, rural),
differences are about 24% world, 15% urban and 34% rural.
Strong general statement = 1 mark.
Use of values 1 or 2 marks.

[2]

(ii) Access to sanitation in rural areas is the lowest of all six values, under half the access of
urban areas/41% difference between them.
Strong general statement = 1 mark.
Use of values 1 or 2 marks.

[2]

UCLES 2010

455

Page 8

Mark Scheme: Teachers version


GCE O LEVEL May/June 2010

Syllabus
5014

Paper
12

BALOL
(iii) Possible reasons:
poverty lack of money for providing the necessary infrastructure
low level of development lack of industries/businesses to stimulate improvement
traditional farming societies, in some places nomadic
population more spread out making it more expensive/difficult to provide services,
remote from political decision making in the city.
Credit clear statement of reasons such as the above, and any elaboration.
Three reasons for 3 marks or two reasons and an elaboration (2 + 1 marks).
(f)

[3]

(i) Less time to build up immunity,


many infants are under-nourished so that their resistance is low,
reference to diarrhoea or another water related disease in question context,
children more likely to play in water/less likely to know the risks.
(ii) Long journey to collect water consumes time that could be used for productive work, in
many African countries women are both water carriers and the main crop growers,
other ways to make money such as by craft occupations/helping husbands, frees up time
to help with/engage in community projects.
Minimum 1 mark, maximum 3 marks for each part.

(a)

(b)

[4]

(i) Germany ( 2.0), China (+ 7.4), India (+ 15.5), 1 mark each.


Maximum of 2 marks without + signs.

[3]

(ii) Natural decrease instead of natural increase, however expressed.

[1]

(i) Africa

[1]

(ii) Europe

[1]

(iii) Every country in Africa above 25 (many in Africa not enough),


attempt to describe distribution of groups of very high countries in East/West Africa,
block of high birth rates from the Middle East into Pakistan,
examples of three or more countries in this block,
elsewhere in Asia only one other (Mongolia)/North Korea,
only one in all of South America (Bolivia),
a few in Central America and the Caribbean (Mexico, etc.)
Three descriptive statements such as the above.

[3]

(iv) Have readily available family planning for all,


often more economically developed countries that can afford this,
also attitudes of people are different with women more career orientated,
education available to all both male and female to high levels,
socially acceptable/normal to have small families,
government pensions and social services to look after elderly,
costs money to bring up children/children no longer seen as work assets.
Four points made along these lines.

[4]

UCLES 2010

456

Page 9

Mark Scheme: Teachers version


GCE O LEVEL May/June 2010

Syllabus
5014

Paper
12

BALOL
(c)

(i) Ethiopia 04 shaded in; UK 3539 shaded in.

[1]

(ii) 46 (allow 4547)

[1]

(iii) 16 circled (or otherwise clearly indicated as the answer)

[1]

(iv) Ethiopia more triangular/pyramidal; UK is more upright/straight up and down,


Ethiopia widest at the base; UK narrows towards the base,
UK is taller.
Any two differences like these related to shape, 1 mark for each.

[2]

(v) Ageing populations have an increasingly high proportion of elderly people, for the UK
this is shown by age groups above 65 being well represented (16%), bulge in middle
aged groups to swell soon the elderly age groups.
Evidence stated and understood = 2 marks.
Some understanding but answer incomplete = 1 mark.

[2]

(vi) With young populations:


high costs for education (also for certain health services, agriculture for food),
unemployment is often a major issue with problems for finding work for all,
population likely to continue growing for many years as they reach marrying age,
continued pressure on resources.
With ageing populations:
high and increasing costs for care and pensions,
at time when working population paying taxes is decreasing,
funding gap which is going to increase with time,
solutions such as immigration for more workers are unpopular with the public.
Separate answers/answer for young or ageing much stronger = 1 or 2 marks.
Differences established and explained = 3 or 4 marks.
(d)

(i) 1 North America 2 Oceania 3 Europe all Developed.


4 Latin America 5 Asia 6 Africa all Developing.
All correct = 2 marks.
No more than one or two mistakes = 1 mark.

[4]

[2]

(ii) The direct evidence is the high average income in Oceania (Australia and New Zealand);
as the continent with the second highest average it cannot be left in the same group as
Africa and South America.
From knowledge candidates may also be able to explain in relation to Japan, one of the
world's most developed industrial countries; the line takes a sharp turn to keep Japan in
the north (= less likely answer).
Understanding shown = 1 mark.

[1]

UCLES 2010

457

Page 10

Mark Scheme: Teachers version


GCE O LEVEL May/June 2010

Syllabus
5014

Paper
12

BALOL
(iii) Examples of where the line indicates a big divide are between North America and Latin
America (along the Mexico-USA border), and between Europe and Africa (through the
Mediterranean Sea) = good fits. Also between Oceania and Asia.
Lack of evidence for route across Asia; reason for position of line cannot be worked from
an average value for Asia only, but the average is a lot below those for Europe and
North America.
Comment favourable to the divide as a good indicator is easier to justify by reference to
the size of the differences in average income between Northern and Southern
continents, especially the US$ 23,800 difference between North and Latin America. One
warning is that average income in individual countries can vary greatly from the
continental averages.
Enough to show good understanding = 3 marks.
Some valid points supporting the assessment of fit = 2 marks.
One or two valid points without an assessment, or unsupported assessment = 1 mark.
[3]
(iv) All low birth rate values below 15 are on the 'North' side of the line,
all high values above 25 are on 'South' side of line,
conclusion that the divide is well supported
This is one example of a 2 mark answer.
Could focus more on less evidence in Asia, or use areas of moderate birth rates, but this
seems more likely to lead to 1 mark rather than 2 mark answers.
[2]

(e)

(i) Fair trade considers the producers/suppliers rather than trading using world market
prices which is what happens in normal trade. It often includes some guarantees for the
producer such as keeping prices above production levels even when world prices
tumble. Often help with community projects is built in (or similar).
Differences understood and clearly stated = 2 marks.
Some attempt to state difference, or understanding but weakly expressed = 1 mark.

[2]

(ii) Advantages of aid emergency relief aid can help desperate people in desperate
situations after human or natural disasters. Development aid can support projects with
longer lasting benefits to local communities such as clean water supplies, wells, rain
water storage and small dams to increase farm output in dry times, clinics etc.
Disadvantages of aid can be given for the wrong things big prestigious/political
projects which give poor value for money spent. In some African countries like Ethiopia
there are fears that many years of aid is leading to a culture of dependency.
Some balance between advantages and disadvantages, and especially if comment
towards reducing the wealth gap is incorporated = 3 or 4 marks.
Stronger on either advantages or disadvantages, or shallow on both = 1 or 2 marks.

[4]

UCLES 2010

458

Page 11

Mark Scheme: Teachers version


GCE O LEVEL May/June 2010

Syllabus
5014

Paper
12

BALOL
(iii) No mark for choice all marks for justification.
Many of the positives and negatives can be applied to both; everything depends on
candidate use.
Possible lines of explanation positives such as these:
Fair Trade income/aid both transfer wealth from rich to poor countries.
Both can have knock-on benefits for communities in general as well as people
specifically targeted.
Both can establish long lasting arrangements with give poor people some security for the
future.
Negatives such as these:
Both come with strings attached to them; people/companies/organisations and
governments in rich countries can walk away from them.
Long term tie-ups lead to dependence/over-reliance upon one source with no back-up.
Remote communities can be at mercy of economic downturns affecting developed world.
Choice well explained = 2 marks.
Some support for choice = 1 mark.

[2]
[Total: 40]

UCLES 2010

459

General Certificate of Education Ordinary Level


5014 Environmental Management June 2010
Principal Examiner Report for Teachers

BALOL
ENVIRONMENTAL
MANAGEMENT

Paper 5014/12
Paper 12

General comments
Despite the many and varied patterns of performance between both individual candidates and Centres, the
general pattern on this year's paper was for total mark out of 40 for questions 1-4 not to be too different from
the average mark out of 40 for questions 5 and 6. On the typical script the mark for question 5 was a few
marks higher than that for question 6. The content of question 5 covered well known topics within the
Hydrosphere unit, while question 6 was more wide ranging in its coverage of the Biosphere unit. Not many
gaps in knowledge were exposed on this paper, but where they were, they tended to be Centre specific and
most often for Fair Trade and aid in 6(e).
There is always pressure on time for candidates to complete this paper. It is important that candidates begin
to answer questions straightaway and do not spend too much time on the short questions in Section A for
which breadth of response rather than depth is expected. There was evidence of answer quality tailing off
from part (d)(ii) onwards in Question 6, particularly among candidates in the lower half of the ability range,
but occasionally from very able candidates who had not kept a sufficiently close eye on the clock. While
unanswered questions were rare throughout the paper, the last page for answering 6(e) was the one most
likely to be left blank.
The main weakness exposed in this year's paper was the study and use of world maps. Some candidates
did not seem to know where to begin when they were asked to describe the distribution of high and very high
birth rates in 6(b)(iii). The question was attempting to help them by making them concentrate on only two of
the four types of shading on the map, and high birth rates were only present in limited areas of the world. Yet
a significant number of candidates were not able to go further than the answer they had already given about
Africa in (b)(i). They were not expected to know the names of lots of different countries, but there was
nothing to stop them spending a minute looking carefully at the map and noticing that every African country
had a high birth rate above 25, and that birth rates were particularly high in certain parts of Africa that could
be described. Average incomes per head marked on the world map in part 6(d) were little used in answers
to parts (ii) and (iii), even though candidates had needed to use them answering part (i). Before answering
part 6(d)(iv) some sensible, thinking candidates drew in the course of the North-South dividing line on the
map of birth rates, which really did help them to answer part (iv) successfully. Second only to this was
candidate failure to answer photograph questions which asked for description based on observation rather
than knowledge, as in 4(a)(i). Certainly knowledge of tropical rainforest was helpful in guiding the candidate
to know what to look for and to do using the appropriate language, but there were no signs of tall trees and
buttress roots on the photograph as some candidates imagined.
It is worth alerting future candidates to the good practice of beginning to answer the question straightaway
without repeating the question. It remains an important issue on this paper for two reasons time and the
fact that a majority of candidates equate filling all the lines with giving a full answer to the question. Most
stop answering once all the lines have been filled. Emphasise to future candidates that the lines left for
answering are for guidance only, and cannot take into account the many variations in size of handwriting and
precision of expression between individual candidates. What is essential is that candidates tailor the number
of points made and the amount of development to the number of marks available, even if it means extending
the answer into the spaces below or on to a supplementary answer sheet. When they do this, advise them
to ensure that they clearly mark up any extra answers with the question number, since answers might be
written some distance away from the main question.

460

General Certificate of Education Ordinary Level


5014 Environmental Management June 2010
Principal Examiner Report for Teachers
Comments
BALOL on individual questions
Section A
Question 1
Biomass was the most popular choice of answer in (a)(i); geothermal was the main distraction for
candidates. Relatively few candidates made it sufficiently clear in (a)(ii) that non-organic waste was
unsuitable for use in the digester. Weak candidates merely repeated without much rewording what was
labelled on the diagram in their answers to (a)(iii), although most recognised the role of bacteria in breaking
down organic matter and releasing methane gas. While it was rare for any of the lines left for answering part
(d) to remain unfilled, there was too much on the unsuitability of organic materials for the incinerator and not
enough about other more unsuitable household waste. Two from toxic materials, metals, glass and batteries
made up many two mark answers. Part (e) was consistently better answered than (d). Most candidates did
look to give three advantages in line with the number of marks for the question, often a mixture of positive
uses and what incineration stopped from happening otherwise.
Question 2
The answer range accepted in (a)(i) was between 62 and 68% using the pie graph, but a few candidates
gave nonsense answers trying to use stated percentages from the diagram of ocean zones. Instead of
obeying the instruction to use both diagrams in (a)(ii), many candidates only used one of them, thereby
restricting themselves to one of the two marks. Important as protein was the most widely known answer in
(b)(i). Part (b)(ii) was often the best answered part. Most candidates included in their answers mineral
resources such as gas and oil along with different types of resources attractive to tourists, but sources of
medicine, use for shipping or for power, and desalination were regularly referred to as well. Less acceptable
were references to rocks and other minerals and metals which are not extracted from the sea. Few answers
to part (c) progressed beyond one or two marks. There was an over-reliance on references to depth or size
of the oceans without explaining how and why they made human exploitation so difficult. Some
straightforward points such as expense compared with obtaining resources on land, or the effects of bad
weather, were mentioned by only a few candidates.
Question 3
Stating the amount rather than naming the country of export was the main challenge in part (a)(i). Values of
between 1 million and 1.3 million tonnes for Thailand were accepted. Some candidates used Costa Rica
instead of Thailand. In (a)(ii) candidates had a choice of continent, either North America or Europe, although
quite a number named the country, the USA, instead. Some candidates stated the direction in (a)(iii) in
terms of 'from developing to developed', which was accepted along with 'in a northerly direction'. Part (a)(iv)
was one of the least well answered questions on the paper. Most candidates had the idea that good climatic
and other natural conditions made the best answer to the question; frequently they repeated this idea without
looking more broadly for other possible and better reasons. More important factors such as close proximity,
existence of trade agreements and low costs of production were ignored by a majority of candidates. In
contrast, part (b) was one of the best answered questions on the paper. Here the typical candidate did look
for a range of ways for achieving high outputs. Large size, high inputs, specialised production methods and
others were regularly referred to in the many three and four mark answers.
Question 4
As in previous years with the photograph question, some candidates in (a)(i) were guilty of making little or no
use of what the photograph showed. The mere mention of tropical rainforest in the stem of the question was
enough justification for some to include characteristics of tall trees in their answers. There were plenty of
one mark answers solely for comment on the density of the vegetation, whereas points in the mark scheme
were focused on characteristics of the stems and leaves that were clearly visible in the photograph. The
clarity and organisation of the answer for part (a)(ii) had a strong influence on the number of marks obtained.
For explanatory points about leaves and stems to be made fully relevant, candidates needed to match a
vegetation characteristic with a related feature of the tropical climate. Some candidates were more proficient
at doing this than others. Some of the best explanations were seen for broad (or big) leaves, drip tips and
leathery leaves; vertical or straight stems for competing to reach sunlight were referred to less regularly even
in stronger answers. Answers to part (b) were of mixed standard. Explanation could refer to either one or
both viewpoints; using the latter approach opened up the range of possible points and made it more likely for
a candidate to claim all three marks. The most frequent references were to tourism for both positive and

461

General Certificate of Education Ordinary Level


5014 Environmental Management June 2010
Principal Examiner Report for Teachers
negative
BALOL views. Candidates in general failed to explore a range of points which is why one and two mark
answers were more commonly found than three mark ones.
Section B
Question 5
The quickest way to the two starter marks in (a) was for a candidate to support the answers stating that there
is a large amount of ocean and a tiny amount of fresh water with fractions or percentages, such as three
quarters and 3%. Many did. The main construction problem for candidates in completing the pie graph in
(b)(i) was showing the tiny total percentage for rivers and lakes. The most common reason for a candidate
losing one of the marks was making their sector too wide; a reasonable tolerance was allowed, but some of
the sectors drawn in were closer to ten rather than to one percent. In the weakest answers to (b)(ii)
candidates merely repeated percentages without any comment and left too much work for Examiners to do
on their behalf. Only a little explanation was needed about why large percentages of fresh water on Earth
were out of human reach; comment about the tiny amount available in more accessible rivers and lakes was
also rewarded, as well as comment about their lack of cleanliness. Most candidates described how a well
works sufficiently fully for both marks in (b)(iii). The most common advantage stated in (b)(iv) was clean /
safe / non-polluted water, although 'fresh' by itself was not credited because it was part of the question.
Stating the second advantage, often reliability of supply or its equivalent, was a good discriminator between
able and weak candidates. Most of the available marks in part (b) were claimed with some regularity.
The most popular large dam choices in (c)(i) were the Aswan, Three Gorges and Hoover dams. Also it was
good to see many candidates from Centres in South Asia using examples of dams in their own countries.
Full marks for part (c)(ii) were only awarded if an acceptable dam had been named and if some information
about advantages specific to it was included. Many full and good answers to part (iii) were seen. To their
credit, the vast majority of candidates attempted to include at least one advantage for each of three factors
specified in the question. Those writing answers worth four or five marks were the ones who elaborated
further. Candidates who referred to an example, or who commented on the questions 'controversial' theme,
were the ones most certain to claim all five marks.
One mark was separated out for making what was considered to be the best choice of two uses in part (d)(i),
namely waste disposal and navigation and shipping. However, failure to select these two uses did not bar
the candidate from gaining all four marks provided that their explanations were sufficiently strong. For
example, there were many strong answers relating to irrigation water for crops in relation to nitrate leaks
leading to eutrophication. Only a shallow explanation or really poor choices such as recreation, stopped
answers reaching at least half marks in this question. There was a sharp divide in answer quality in (d)(ii)
between candidates who merely continued answers from the previous part about how the different river uses
led to pollution and those who identified conflicts between different groups of river users. There was plenty
of the latter, and many explained two or more conflicts to ensure that they claimed all three marks.
Candidates approached answering parts (e)(i) and (ii) in many different ways. In both parts, however, the
one mark answers came mainly from those who compared two bars without further comment. The two mark
answers came from those who either compared values more widely, often stating differences in size, or who
supported their use of values with a strong general comment. An example of a strong general comment for
part (ii) was 'access to rural areas is the lowest of all six values', a statement which was made by some more
able candidates. The poorest answers to part (ii) came from those who merely compared the size of the two
rural bars, which did not answer the question. Part (iii) was well answered by many; they were most likely to
refer to poverty, less concentrated populations and lower levels of influence over politicians, although in fact
these points were most often made the other way around in relation to urban areas. Poverty was the most
common one mark answer.
Very few candidates failed to gain at least one mark from part (f). Low levels of immunity was often the
starting point for answers in (f)(i). Economic consequences of spending time fetching water such as not
undertaking productive work was the most frequently claimed mark in part (ii). To be worth more than two
marks, candidates needed to do more, and to bear in mind that the question was worth four marks. Looking
for more explanation came up with points such as many infants under-nourished and children often play in
and around water in the first part, and specifying types of work such as crop growing and craft occupations in
the second part.

462

General Certificate of Education Ordinary Level


5014 Environmental Management June 2010
Principal Examiner Report for Teachers
Question
BALOL 5 was well answered throughout. Quite unusually there were no parts within the question in which
candidates regularly under-performed compared with what had been expected. The higher the mark, the
greater the consistency of performance across the questions, and the greater willingness of the candidate to
give the amount needed for all the marks available.
Question 6
Somewhat unaccountably some candidates made no attempt to complete the table in (a)(i). Of those who
did, few made any numerical errors, although a few were careless in not including the plus signs. In (ii) it
was not enough for the candidate to write words to the effect that Germany's death rate is higher than its
birth rate; to be awarded the mark, they needed to go to the next stage and recognise the natural decrease
in population.
While Africa was the almost universal answer to part (b)(i) (only the occasional answer of Asia broke the
trend), Europe was quite regularly replaced by North America or Oceania in (b)(ii). This probably reflects
candidate failure to read the question carefully enough. Answers to (b)(iii) varied greatly in quality. Some
merely repeated the mention of Africa without any more careful description from the map, such as the fact
that every country in Africa was shown to have a birth rate above 25, or that very high birth rates above 40
were concentrated in countries in West and East Africa. One or two mark answers often included mention of
the Middle East and parts of Asia. Three mark answers typically included some reference to the Americas
reflecting a broader perspective on the world distribution. India and Indonesia were the two countries most
likely to be wrongly identified as having high birth rates. The reasons for low birth rates in developed
countries and in countries with a strict population policy such as China were well known in (b)(iv). The total
mark most closely reflected the breadth of points made and how well stated they were.
While some candidates shaded in more than one age group on each pyramid, and others shaded in age
group 0-4 on the pyramid for Ethiopia only in part (c)(i), the vast majority of candidates picked up the easy
mark. They had to work harder to gain the mark in part (ii); answers within the range 45 to 47 were the only
ones accepted. There were many answers of 23%, worked out for either males or females, but not for the
total population. The correct answer of 16% was far from being the most commonly circled answer in part
(iii). Some of those who did circle 16% had sufficient savvy to make use of this percentage when answering
part (v). Candidates experienced few problems answering part (iv) provided that they homed in on pyramid
'shape'. Many did, and often reached two marks despite problems trying to write about the shape of the UK's
pyramid. Many did not, however, write about pyramid shape and these candidates frequently used a lot of
percentages for different age groups to no avail. The weakest answers to part (v) were from those who
stated that the largest age group in the UK was 35-39 and then said no more. The bulge in middle-aged
groups was mark worthy only when it was placed in the context that it would soon lead to a swelling of the
elderly age groups. One mark answers, showing a little understanding when they referred to the height of
the UK pyramid going up to 90+, were common. Absolutely the full range of quality of responses was seen
in part (vi). Zero mark answers were normally those that did not answer the question, either by writing about
reasons why Ethiopia has a young population and the UK does not, or by concentrating on general problems
arising from great population growth in a developing country such as Ethiopia. Middle of the way two mark
answers typically touched on one problem for the young (such as costs of education) and one problem for
the old (such as costs of health care or pensions). In superior answers, the correct context was established
and the explanation for both was fuller.
A few responses to part (d)(i) showed that even for the easiest of questions some candidates can do things
wrong. Occasionally continents were not ranked by income; sometimes Latin America was marked as
developed and Oceania as developing. The main weakness in answers to both parts (ii) and (iii) was that
candidates failed to use, or obviously show that they had used, the income information on the world map.
Without this, the case for extending the North-South dividing line southwards to encompass Oceania as part
of the North was not made clear in (ii) and answers to (iii) relied on over-general comments. In fact, in part
(iii), many dealt better with the not so good element of the fit particularly when they referred to regions that
they knew had high average incomes per head like the Middle East. There was not always a lot of evidence
in the answers given to part (d)(iv) that candidates had looked back to the map of birth rates. In many
answers there was nothing beyond a general statement about high birth rates in developing and low birth
rates in developed countries. Answers written after another look at the world map of birth rates stood out as
being of a different quality. Such candidates were more likely to notice that there were both low and
moderate birth rates, not just low, in countries north of the line. A below 15 per 1000 population birth rate
country like China stood out as an exception to the dominant high and very high birth rates of countries south
of the line. Marks were not awarded unless there was definite evidence that the birth rate map had been restudied in the light of this question.

463

General Certificate of Education Ordinary Level


5014 Environmental Management June 2010
Principal Examiner Report for Teachers
From
those candidates who were not familiar with Fair Trade there was frequent confusion with Free Trade in
BALOL
(e)(i). Whilst most candidates, who did know what Fair Trade was, referred to a guaranteed minimum price
to overcome the problem of widely fluctuating world market prices in primary goods, a few were also aware
of its role in supporting the wider community. Many answers for advantages to (e)(ii) suffered from
vagueness; there was much about helping with education, medical care, birth control, development and
generally improving living standards, but little in the way of specific references to types of aid and the nature
of help given. Over-dependency on aid was the disadvantage that was mentioned most. One common
misconception was that aid led to people getting in debt and having to pay back all the money given.
Answers to this question were probably more disappointing than for any other question on the paper. Three
and four mark answers were the exception rather than the rule. To an extent there was a relationship
between answer quality in part (e)(iii) and the strength of answers that had already been given to parts (i)
and (ii). However, by concentrating on the strategy with which they were more familiar, some candidates
managed to claim both marks. Even so, most answers were narrower than this and worth only one mark.
Choice mattered little here; knowledge and understanding mattered more.
For nearly all candidates the total mark for Question 6 was below that for Question 5. Many candidates
began to lose momentum from part (d)(ii) onwards. This general summary applied least to more able
candidates, who could cope better with the study of the world maps and knew more about Fair Trade and
aid.

464

BALOL
UNIVERSITY OF CAMBRIDGE INTERNATIONAL EXAMINATIONS
General Certificate of Education Ordinary Level

*1379653749*

5014/13

ENVIRONMENTAL MANAGEMENT
Paper 1

May/June 2010
2 hours 15 minutes

Candidates answer on the Question Paper.


Additional Materials:

Ruler

READ THESE INSTRUCTIONS FIRST


Write your Centre number, candidate number and name on all the work you hand in.
Write in dark blue or black pen.
You may use a soft pencil for any diagrams, graphs or rough working.
Do not use staples, paper clips, highlighters, glue or correction fluid.
DO NOT WRITE IN ANY BARCODES.
Answer all questions.
All questions in Section A carry 10 marks.
Both questions in Section B carry 40 marks.
At the end of the examination, fasten all your work securely together.
The number of marks is given in brackets [ ] at the end of each question or part question.

For Examiners Use


1
2
3
4
5
6
Total
This document consists of 22 printed pages and 2 blank pages.
DC (A
UCLES 2010

[Turn o

465

For
Examiners
Use

BALOL
1

Section A

(a) Look at the diagram which shows methods of producing energy from household waste.

heat

water

steam

turbine

electricity

incinerator
suitable
non-organic waste
household
waste
collected
and
sorted

unsuitable
waste

gas

organic
waste

ane

th
me

diesel

digester
containing
bacteria
Circle the word listed below which describes the type of energy source shown in the
lower part of the diagram.
biomass

geothermal

hydro

solar

wind

[1]

(b) Why is the organic waste separated from the other waste?
..................................................................................................................................... [1]
(c) Describe what happens in the digester.
..........................................................................................................................................
..................................................................................................................................... [2]
(d) Suggest why some household waste is unsuitable for use in the incinerator.
..........................................................................................................................................
..........................................................................................................................................
..........................................................................................................................................
..........................................................................................................................................
..........................................................................................................................................
..................................................................................................................................... [3]

UCLES 2010

5014/13/M/J/10

466

For
Examiners
Use

BALOL
(e) What are the advantages of using household waste in these ways?
..........................................................................................................................................
..........................................................................................................................................
..........................................................................................................................................
..........................................................................................................................................
..........................................................................................................................................
..................................................................................................................................... [3]
[Total: 10]

UCLES 2010

5014/13/M/J/10

[Turn over

467

4
2 BALOL
(a) Look at the diagram of ocean zones and the pie chart showing the potential fish harvest
from three zones.
continental
shelf

For
Examiners
Use

open ocean in which areas of upwelling


are less than 1%

10%

90%
sea level
200 m

continental
shelf

continental
slope
abyssal
plain

abyssal
plain

4 km

ocean
deep
12 km
open
oceans

areas of
upwelling
continental
shelf and
coastal
waters

potential fish production

UCLES 2010

5014/13/M/J/10

468

BALOL (i) What percentage of the potential fish production from the oceans is expected to

For
Examiners
Use

come from the continental shelf and coastal waters?


.............................................. % [1]
(ii)

Use both diagrams to comment on the potential fish production from the open
ocean.
..................................................................................................................................
..................................................................................................................................
............................................................................................................................. [2]

(b) (i)

Why are fish important in the human diet in some parts of the world?
............................................................................................................................. [1]

(ii)

Describe other valuable resources which oceans can provide.


..................................................................................................................................
..................................................................................................................................
..................................................................................................................................
..................................................................................................................................
..................................................................................................................................
............................................................................................................................. [3]

(c) Suggest why a full exploitation of the resources of the open oceans is unlikely.
..........................................................................................................................................
..........................................................................................................................................
..........................................................................................................................................
..........................................................................................................................................
..........................................................................................................................................
..................................................................................................................................... [3]
[Total: 10]

UCLES 2010

5014/13/M/J/10

[Turn over

469

6
3 BALOL
(a) Look at the map, which gives information for a recent year about the worlds main
exporting and importing countries of fresh pineapple, together with the main trade routes
of fresh pineapple.

For
Examiners
Use

SOME
EU COUNTRIES

CANADA

JAPAN

USA
China

Tropic of
Cancer
Philippines

Thailand
Costa
Rica

Equator

Kenya

Ivory
Coast

Indonesia

Tropic of Capricorn

Key
The main importing countries of the
European Union are Belgium, Luxembourg,
the Netherlands, UK, France, Italy, Germany,
Portugal and Spain.

fresh pineapple exports


tonnes
200,000
100,000
0
Exporting country names in lower case

main fresh pineapple trade routes


20,000 to 100,000 tonnes per year

Countries which together


import 86% of the world
imports of fresh pineapple

100,000 to 400,000 tonnes per year


over 400,000 tonnes per year

Importing country names in capital letters

(i)

Name the country which exports the most fresh pineapple and state the amount it
exports.
country ................

(ii)

amount ................ tonnes [1]

Name one continent with large imports of fresh pineapples.


............................................................................................................................. [1]

UCLES 2010

5014/13/M/J/10

470

For
Examiners
Use

BALOL(iii) State one similarity in the direction of the sea trade routes shown.
............................................................................................................................. [1]
(iv)

Suggest why some countries export more fresh pineapples than others.
...................................................................................................................................
..................................................................................................................................
..................................................................................................................................
..................................................................................................................................
..................................................................................................................................
............................................................................................................................. [3]

(b) Pineapples are often grown on plantations. Describe how commercial farming, such as
plantation farming, is organised to produce large amounts of a cash crop for export.
..........................................................................................................................................
..........................................................................................................................................
..........................................................................................................................................
..........................................................................................................................................
..........................................................................................................................................
..........................................................................................................................................
..................................................................................................................................... [4]
[Total: 10]

UCLES 2010

5014/13/M/J/10

[Turn over

471

8
4 BALOL
(a) Look at the photograph showing vegetation growing on the edge of a clearing in a
tropical rainforest.

(i)

For
Examiners
Use

Describe the features of the vegetation in the photograph.


..................................................................................................................................
..................................................................................................................................
..................................................................................................................................
..................................................................................................................................
..............................................................................................................................[3]

UCLES 2010

5014/13/M/J/10

472

BALOL(ii) Explain how the leaves and stems are adapted to the climate in which tropical

For
Examiners
Use

rainforest grows.
..................................................................................................................................
..................................................................................................................................
..................................................................................................................................
..................................................................................................................................
..................................................................................................................................
..................................................................................................................................
............................................................................................................................. [4]

(b) Some areas of tropical rainforest have been made into National Parks. Is this method of
conservation likely to have any benefits for local people? Explain your views.
..........................................................................................................................................
..........................................................................................................................................
..........................................................................................................................................
..........................................................................................................................................
..........................................................................................................................................
..................................................................................................................................... [3]
[Total: 10]

UCLES 2010

5014/13/M/J/10

[Turn over

473

For
Examiners
Use

10

BALOL
5

Section B

(a) Look at the pie graphs showing the distribution of water on the Earths surface.
land and sea

fresh water and


salt water
fresh
water

land
salt
water

ocean

Describe the evidence from the pie graphs for each of statements A and B.
A There is a lot more water than land on the Earths surface.
..........................................................................................................................................
..........................................................................................................................................
B The amount of water available for human use is very small.
..........................................................................................................................................
......................................................................................................................................[2]
(b) The diagram gives information about fresh water stores on the Earths surface.
lakes
0.3%

rivers
less than
0.1%

groundwater
(above 1000 m)
10.6%
deep groundwater
(below 1000 m)
14%

ocean

UCLES 2010

ice & snow


75%

5014/13/M/J/10

474

11

BALOL (i) Show the percentages for ice and snow, total groundwater, and lakes and rivers on

For
Examiners
Use

a pie graph.
fresh water stores on the Earths surface

[3]
(ii)

Explain more fully why the amount of water available for human use is so small.
..................................................................................................................................
..................................................................................................................................
..................................................................................................................................
..................................................................................................................................
..................................................................................................................................
..............................................................................................................................[3]

(iii)

How can people obtain their water supply from groundwater stores? Describe one
method.
..................................................................................................................................
..................................................................................................................................
..................................................................................................................................
..............................................................................................................................[2]

(iv)

State two advantages of using groundwater stores for water supply.


..................................................................................................................................
..................................................................................................................................
..............................................................................................................................[2]

UCLES 2010

5014/13/M/J/10

[Turn over

475

12

BALOL
(c) In many countries large dams have been built to increase natural stores in rivers and

For
Examiners
Use

lakes.
(i)

Name or state the location of an example of a large dam.


..............................................................................................................................[1]

(ii)

Describe its advantages and explain why it was built.


..................................................................................................................................
..................................................................................................................................
..................................................................................................................................
..................................................................................................................................
..................................................................................................................................
..............................................................................................................................[4]

(iii)

Why are decisions to build large dams often controversial? Describe some of the
economic, social and environmental factors which help to explain why some people
object to the building of large dams.
..................................................................................................................................
..................................................................................................................................
..................................................................................................................................
..................................................................................................................................
..................................................................................................................................
..................................................................................................................................
..................................................................................................................................
..................................................................................................................................
..................................................................................................................................
..............................................................................................................................[5]

UCLES 2010

5014/13/M/J/10

476

For
Examiners
Use

13

BALOL
(d) Look at the spider diagram showing some uses of rivers.
domestic for
cooking and washing
irrigation water
for crops

waste disposal
uses of
rivers

navigation and
shipping

recreation, swimming
and canoeing
cooling water
for power stations
(i)

Which two uses are most likely to lead to pollution of river water? Explain why.
Use 1 .........................................................
..................................................................................................................................
..................................................................................................................................
..................................................................................................................................
Use 2 .........................................................
..................................................................................................................................
..................................................................................................................................
..............................................................................................................................[4]

(ii)

Conflicts of interest can arise between people using rivers for different purposes.
Giving examples, explain some of these conflicts of interest.
..................................................................................................................................
..................................................................................................................................
..................................................................................................................................
..................................................................................................................................
..................................................................................................................................
..................................................................................................................................
..................................................................................................................................
..............................................................................................................................[3]

UCLES 2010

5014/13/M/J/10

[Turn over

477

For
Examiners
Use

14

BALOL
(e) Look at the graphs showing access to water supply and sanitation.
water supply

sanitation

90

90

80

80

70

70

60

60

% access

100%

% access

100%

50

50

40

40

30

30

20

20

10

10

world

urban

rural

world

urban

rural

Use values from the graphs to describe how they show that access to sanitation is
(i)

less widely available than for water supply;


..................................................................................................................................
..................................................................................................................................
..............................................................................................................................[2]

(ii)

particularly poor in rural areas.


..................................................................................................................................
..................................................................................................................................
..............................................................................................................................[2]

UCLES 2010

5014/13/M/J/10

478

15

BALOL(iii) Give reasons why access to water supply and sanitation is much poorer in rural

For
Examiners
Use

than in urban areas.


..................................................................................................................................
..................................................................................................................................
..................................................................................................................................
..................................................................................................................................
..................................................................................................................................
..............................................................................................................................[3]
(f)

Costs from lack of safe water supply and sanitation


1.6 million children a year die
from diarrhoea

economic loss of the time taken


in fetching water every day

Explain why
(i)

children are most at risk from water-related diseases;


..................................................................................................................................
..................................................................................................................................
..................................................................................................................................
..................................................................................................................................

(ii)

economic costs result from people having to walk long distances to fetch water.
..................................................................................................................................
..................................................................................................................................
..................................................................................................................................
..............................................................................................................................[4]
[Total: 40]

UCLES 2010

5014/13/M/J/10

[Turn over

479

16
6 BALOL
(a) (i)

Complete the table of birth and death rates and population change for selected
countries from Europe and Asia.
Country

Birth rate
per 1000

Death rate
per 1000

Population change
per 1000

11.0

10.2

+0.8

8.7

10.7

China

14.5

7.1

India

23.8

8.3

Saudi Arabia

31.5

3.7

UK
Germany

For
Examiners
Use

+27.8
[3]

(ii)

How is Germany different from the other countries in the table?


..............................................................................................................................[1]

(b)

World distribution of birth rates

Key
Live births per
1000 population
Very high
above 40
High
2540
Moderate
1524.9
Low
below 15
(i)

Equator

Name the continent with the largest number of countries with very high birth rates
above 40 per 1000.
..............................................................................................................................[1]

(ii)

In which continent are there the most countries with birth rates below 15 per
1000?
..............................................................................................................................[1]

UCLES 2010

5014/13/M/J/10

480

17

BALOL(iii) Describe the distribution of countries with high and very high birth rates above 25

For
Examiners
Use

per 1000.
..................................................................................................................................
..................................................................................................................................
..................................................................................................................................
..................................................................................................................................
..............................................................................................................................[3]
(iv)

Give reasons why some countries have very low birth rates, below 15 per 1000.
..................................................................................................................................
..................................................................................................................................
..................................................................................................................................
..................................................................................................................................
..................................................................................................................................
..................................................................................................................................
..................................................................................................................................
..............................................................................................................................[4]

UCLES 2010

5014/13/M/J/10

[Turn over

481

For
Examiners
Use

18

BALOL
(c) Look at the pyramids showing population structure for Ethiopia and the UK.
Ethiopia

UK

75+
70-74
65-69
60-64
55-59
50-54
45-49
40-44
35-39
30-34
25-29
20-24
15-19
10-14
5-9
0-4

90+
85-89
80-84
75-79
70-74
65-69
60-64
55-59
50-54
45-49
40-44
35-39
30-34
25-29
20-24
15-19
10-14
5-9
0-4

10 9 8 7 6 5 4 3 2 1 0
% males

6 5 4 3 2 1 0
% males

0 1 2 3 4 5 6 7 8 9 10
% females

0 1 2 3 4 5 6
% females

(i)

On each pyramid, shade in the age group with the highest total percentage of
population.
[1]

(ii)

What is the approximate total percentage of population in Ethiopia below the age
of 15?
..............................................................................................................................[1]

(iii)

What is the approximate total percentage of population aged 65 and above in the
UK? Circle one answer.
6.5

(iv)

9.5

12

16

[1]

State two differences in shape between the population pyramids for Ethiopia and
the UK.
..................................................................................................................................
..................................................................................................................................
..............................................................................................................................[2]

(v)

The UK has an ageing population. State the evidence for this from its population
pyramid.
..................................................................................................................................
..................................................................................................................................
..............................................................................................................................[2]

UCLES 2010

5014/13/M/J/10

482

19

BALOL(vi) Some countries like Ethiopia have a young population; others like the UK have an

For
Examiners
Use

ageing population. Their population problems are different.


Explain how and why their problems are different.
..................................................................................................................................
..................................................................................................................................
..................................................................................................................................
..................................................................................................................................
..................................................................................................................................
..................................................................................................................................
..................................................................................................................................
..............................................................................................................................[4]

UCLES 2010

5014/13/M/J/10

[Turn over

483

20

BALOL
(d) Look at the world map which shows the course of the dividing line between the rich

For
Examiners
Use

North and poor South.


N
North
America
26,900
T. of Cancer

th
Nor
The South
The

Europe
12,100

Africa
690
Latin
America
3,100

Equator

T. of Capricorn

Asia
2,200
Th
Th e N
e or
So th
ut
h

Oceania
13,900
Key
Average income per
head in US $

(i)

Rank the six continents by income per head from highest to lowest. State whether
each continent is mainly developed or developing.
Continent

Developed or Developing

1 ............................................................. ...............................................................
2 ............................................................. ...............................................................
3 ............................................................. ...............................................................
4 ............................................................. ...............................................................
5 ............................................................. ...............................................................
6 ............................................................. ........................................................... [2]
(ii)

Why does the North-South dividing line not continue running west to east all the
way across the world?
..................................................................................................................................
..............................................................................................................................[1]

UCLES 2010

5014/13/M/J/10

484

21

BALOL(iii) How well does the line split the world between a rich North and a poor South?

For
Examiners
Use

Answer as fully as you can describing where the fit is good and not so good.
..................................................................................................................................
..................................................................................................................................
..................................................................................................................................
..................................................................................................................................
..................................................................................................................................
..................................................................................................................................
..............................................................................................................................[3]
(iv)

Look back to the map showing the world distribution of birth rates in part (b).
How well would the North-South dividing line separate countries with high and low
birth rates?
..................................................................................................................................
..................................................................................................................................
..................................................................................................................................
..................................................................................................................................
..............................................................................................................................[2]

UCLES 2010

5014/13/M/J/10

[Turn over

485

For
Examiners
Use

22

BALOL
(e) Two strategies for reducing the wealth gap between rich and poor countries:
Improved trade such as Fair Trade
Non-governmental aid organisations (NGOs) from rich countries.
(i)

How is Fair Trade different from other trade?


..................................................................................................................................
..................................................................................................................................
..................................................................................................................................
..............................................................................................................................[2]

(ii)

Explain some of the advantages and disadvantages of aid for helping poor countries
and reducing the wealth gap between them and rich countries.
..................................................................................................................................
..................................................................................................................................
..................................................................................................................................
..................................................................................................................................
..................................................................................................................................
..................................................................................................................................
..................................................................................................................................
..............................................................................................................................[4]

(iii)

In your view, which of the two strategies is better for reducing the wealth gap
between rich and poor countries in the long term? Explain your choice.
..................................................................................................................................
..................................................................................................................................
..................................................................................................................................
..................................................................................................................................
..............................................................................................................................[2]
[Total: 40]

UCLES 2010

5014/13/M/J/10

486

23

BALOL

BLANK PAGE

UCLES 2010

5014/13/M/J/10

487

24

BALOL

BLANK PAGE

488

BALOL
UNIVERSITY OF CAMBRIDGE INTERNATIONAL EXAMINATIONS
General Certificate of Education Ordinary Level

*8322734003*

5014/11

ENVIRONMENTAL MANAGEMENT
Paper 1

October/November 2010
2 hours 15 minutes

Candidates answer on the Question Paper.


Additional Materials:

Ruler
Protractor

READ THESE INSTRUCTIONS FIRST


Write your Centre number, candidate number and name on all the work you hand in.
Write in dark blue or black pen.
You may use a soft pencil for any diagrams, graphs or rough working.
Do not use staples, paper clips, highlighters, glue or correction fluid.
DO NOT WRITE IN ANY BARCODES.
Answer all questions.
All questions in Section A carry 10 marks.
Both questions in Section B carry 40 marks.
At the end of the examination, fasten all your work securely together.
The number of marks is given in brackets [ ] at the end of each question or part
question.

For Examiners Use


1
2
3
4
5
6
Total

This document consists of 24 printed pages.

UCLES 2010

[Turn o

489

BALOL
1

Section A

(a) Look at the photograph, which shows part of the largest copper mine in the world,
located in the Rocky Mountains, USA.

(i)

For
Examiners
Use

Use evidence from the photograph to describe the mining method shown.
..................................................................................................................................
..................................................................................................................................
..................................................................................................................................
..................................................................................................................................
..................................................................................................................................
............................................................................................................................ [4]

UCLES 2010

5014/11/O/N/10

490

BALOL(ii) The ore from the mine contains only 1% copper. What problems will result from
this?

For
Examiners
Use

..................................................................................................................................
..................................................................................................................................
..................................................................................................................................
..................................................................................................................................
............................................................................................................................ [3]
(b) What do you think should be done with such a large hole after mining has finished?
Give reasons for your answer.
..........................................................................................................................................
..........................................................................................................................................
..........................................................................................................................................
..........................................................................................................................................
.................................................................................................................................... [3]

UCLES 2010

5014/11/O/N/10

[Turn over

491

4
2 BALOL
(a) Look at the graph showing changes in the number of dead zones in the oceans. A
dead zone is found in very shallow, coastal waters. These areas used to have healthy
ecosystems, but not much life can now survive in them.
number of
dead zones
in oceans

(i)

number of
dead zones
in oceans

500

500

400

400

300

300

200

200

100

100

0
1965

1975

1985

1995

0
2005

How many dead zones were known in 1965 and 2005?


1965 .................................. 2005 .....................................

(ii)

[2]

Which ten-year period had the largest increase in dead zones?


...................................................................

UCLES 2010

For
Examiners
Use

[1]

5014/11/O/N/10

492

BALOL
(b) (i) Name two heavy metals.
............................................................................................................................ [1]
(ii)

For
Examiners
Use

Explain how heavy metals and plastics can cause the death of marine animals.
..................................................................................................................................
..................................................................................................................................
..................................................................................................................................
..................................................................................................................................
............................................................................................................................ [3]

(c) Why is it difficult to prevent the formation of dead zones?


..........................................................................................................................................
..........................................................................................................................................
..........................................................................................................................................
..........................................................................................................................................
.................................................................................................................................... [3]

UCLES 2010

5014/11/O/N/10

[Turn over

493

6
3 BALOL
Look at the maps showing the extent of ice cover north of the tundra zone in September
1979 and September 2007.

For
Examiners
Use

1979
Greenland

North America

Europe
Arctic Ocean

key
ice
sea water
land not
covered
by ice

Asia

2007
Greenland

North America

Europe
Arctic Ocean

Asia

UCLES 2010

5014/11/O/N/10

494

BALOL
(a) (i) Describe one main similarity and one main difference in the extent of the ice cover
in 2007 compared with 1979.

For
Examiners
Use

similarity ...................................................................................................................
difference ............................................................................................................ [2]
(ii)

On the 2007 map write T to show a land area with a tundra climate.

[1]

(b) Describe and explain:


(i)

the effect of snow and ice on the incoming rays from the Sun,
..................................................................................................................................
..................................................................................................................................
............................................................................................................................ [2]

(ii)

why insolation is low in high latitudes.


..................................................................................................................................
..................................................................................................................................
............................................................................................................................ [2]

(c) Can the changes in ice cover shown on the maps be linked to climate change?
You should give reasons for agreeing or disagreeing, or both.
..........................................................................................................................................
..........................................................................................................................................
..........................................................................................................................................
.................................................................................................................................... [3]

UCLES 2010

5014/11/O/N/10

[Turn over

495

8
4 BALOL
(a) Look at the diagram, which shows part of the food web in Yellowstone National Park,
USA.

black
bear

wolf

grizzly
bear

coyote

cougar
(mountain
lion)

elk

moose

For
Examiners
Use

plants

Many wolves once lived in Yellowstone National Park but there were none living there by
the 1970s. Wolves from Canada have recently been released in the area.
(i)

What is likely to happen to the wolf population over time?


............................................................................................................................ [1]

(ii)

Which animal can help to control the population of wolves?


...................................................................

(iii)

How many feeding (trophic) levels are shown?


...................................................................

(iv)

[1]

[1]

Use an example from the food web diagram to explain the meaning of the following
terms:
predator ....................................................................................................................
..................................................................................................................................
competition ...............................................................................................................
..................................................................................................................................
producer ...................................................................................................................
..................................................................................................................................
food chain .................................................................................................................
............................................................................................................................ [4]

UCLES 2010

5014/11/O/N/10

496

BALOL
(b) Fires which start naturally in Yellowstone National Park are allowed to burn.

For
Examiners
Use

Explain the likely effects of this policy on animals in any ecosystem.


..........................................................................................................................................
..........................................................................................................................................
..........................................................................................................................................
..........................................................................................................................................
.................................................................................................................................... [3]

UCLES 2010

5014/11/O/N/10

[Turn over

497

10

BALOL
5

Section B

(a) Look at the map which shows major plate boundaries in Europe, Africa and Asia.

For
Examiners
Use

Iceland

Eurasian
Plate
North
American
Plate

Q
Pacific
Plate

African
Plate
Q
P
Indo-Australian
Plate
South
American
Plate

Key:
Plate boundaries
Direction of plate movement
Volcano Vesuvius
(i)

State the direction of plate movement along the boundary marked P.


............................................................................................................................ [1]

(ii)

Name the type of plate boundary at P.


............................................................................................................................ [1]

(iii)

Volcanic eruptions are frequent in Iceland. Explain why.


..................................................................................................................................
..................................................................................................................................
..................................................................................................................................
............................................................................................................................ [2]

UCLES 2010

5014/11/O/N/10

498

11

BALOL(iv) A high percentage of Icelands energy needs come from renewable alternative
sources. Describe how volcanic activity can be harnessed for electric power and
heating.

For
Examiners
Use

..................................................................................................................................
..................................................................................................................................
..................................................................................................................................
..................................................................................................................................
..................................................................................................................................
............................................................................................................................ [3]
(v)

The earthquake risk is high in countries located along plate boundary Q. Explain
why earthquakes often occur along plate boundaries of this type.
..................................................................................................................................
..................................................................................................................................
..................................................................................................................................
..................................................................................................................................
..................................................................................................................................
..................................................................................................................................
............................................................................................................................ [4]

UCLES 2010

5014/11/O/N/10

[Turn over

499

12

BALOL
(b) Governments of countries with a high earthquake risk can prepare for future earthquakes.
Describe strategies they can use that will help to
(i)

For
Examiners
Use

stop buildings from collapsing in an earthquake,


..................................................................................................................................
..................................................................................................................................
..................................................................................................................................
..................................................................................................................................
..................................................................................................................................

(ii)

reduce the number of deaths after the earthquake.


..................................................................................................................................
..................................................................................................................................
..................................................................................................................................
..................................................................................................................................
..................................................................................................................................
[5]

UCLES 2010

5014/11/O/N/10

500

13

BALOL
(c) Look at the table which shows strong earthquakes along plate boundary Q during the
nine years between 2000 and 2008.

For
Examiners
Use

Strong earthquakes above 6.0 on the Richter Scale along plate boundary Q (20002008)
Date

Richter scale

Location

Estimated deaths

2000
2001 January

7.9

Gujarat, India

20,000

7.0

Algeria

December

6.5

Bam, Iran

2004 December

8.9

Off the coast of Indonesia

2005 February

6.4

Kerman, Iran

March

8.7

Off the coast of Indonesia

October

7.6

Northern Pakistan and Kashmir

2006 April

6.0

Western Iran

May

6.2

Yogyakarta, Indonesia

5,000

6.5

Baluchistan, Pakistan

300

2002
2003 May

2,000
30,000
250,000
300
1,300
79,000
70

2007
2008 October
(i)

Estimated deaths is the heading for the last column in the table. Why is this used
instead of just Deaths?
..................................................................................................................................
..................................................................................................................................
............................................................................................................................ [2]

(ii)

Name the two countries with the largest number of strong earthquakes during
these nine years.
............................................................................................................................ [1]

(iii)

Describe the pattern of strong earthquakes during the nine years shown in the
table.
..................................................................................................................................
..................................................................................................................................
..................................................................................................................................
............................................................................................................................ [2]

UCLES 2010

5014/11/O/N/10

[Turn over

501

14

BALOL(iv) What does the information in the table suggest about the chances of people
predicting when and where earthquakes will occur along plate boundary Q?

For
Examiners
Use

..................................................................................................................................
..................................................................................................................................
..................................................................................................................................
............................................................................................................................ [2]
(v)

The stronger the earthquake, the greater the number of deaths. Describe evidence
in the table for and against this statement.
..................................................................................................................................
..................................................................................................................................
..................................................................................................................................
..................................................................................................................................
..................................................................................................................................
..................................................................................................................................
..................................................................................................................................
............................................................................................................................ [4]

(vi)

In your view, how strong is the evidence for this statement? Explain your answer.
..................................................................................................................................
..................................................................................................................................
..................................................................................................................................
............................................................................................................................ [2]

UCLES 2010

5014/11/O/N/10

502

15

BALOL
(d) Look at the timeline showing dates of major eruptions of the volcano Vesuvius in

1850

1800

1900

For
Examiners
Use

19
44

19
06

18
5
18 0
5
18 5
61
18
72

18
22
18
35

southern Italy.

1950

2000

eruption and date


(i)

Describe differences in the pattern of eruptions between the nineteenth (18001900)


and twentieth centuries (19002000).
..................................................................................................................................
..................................................................................................................................
..................................................................................................................................
............................................................................................................................ [2]

(ii)

More than half a million people live in the area around Vesuvius. It is the most
densely populated part of southern Italy.
State the most likely reason why so many people live near this volcano.
............................................................................................................................ [1]

UCLES 2010

5014/11/O/N/10

[Turn over

503

16

BALOL(iii) Scientists keep a close watch on Vesuvius in the hope of predicting the next
eruption and warning people before it happens. Four ways scientists watch and
study volcanoes are given in the spider diagram below.
thermometers to measure
temperatures in the crater

For
Examiners
Use

seismographs to
record small earthquake
shocks

watching and studying


volcanoes

observations of
emissions of gas
and steam

tilt meters to
record small changes in
ground shape

Describe how these ways are useful for predicting the next volcanic eruption.
..................................................................................................................................
..................................................................................................................................
..................................................................................................................................
..................................................................................................................................
..................................................................................................................................
............................................................................................................................ [3]
(iv)

Suggest why the scientists might not always make correct predictions about
volcanic eruptions.
..................................................................................................................................
..................................................................................................................................
..................................................................................................................................
............................................................................................................................ [2]

UCLES 2010

5014/11/O/N/10

504

17

BALOL(v) When scientists predict that Vesuvius is about to erupt again, more than half a
million people will need to be evacuated from their homes. How easy or difficult will
it be to put this strategy into action? Answer as fully as you can.

For
Examiners
Use

..................................................................................................................................
..................................................................................................................................
..................................................................................................................................
..................................................................................................................................
..................................................................................................................................
............................................................................................................................ [3]
[Total: 40 marks]

UCLES 2010

5014/11/O/N/10

[Turn over

505

18
6 BALOL
(a) The table below shows the composition of the lower atmosphere.
gas

percentage

nitrogen

78%

oxygen

21%

other gases
(i)

For
Examiners
Use

1%

In the frame below, draw a divided bar graph to show these percentages and
complete the key.
key:
nitrogen
oxygen

100

percentage

other gases

[3]
(ii)

Among the other gases are water vapour, carbon dioxide and ozone.
Explain the importance of each of these gases for life on Earth.
Water vapour ............................................................................................................
..................................................................................................................................
..................................................................................................................................
..................................................................................................................................
Carbon dioxide .........................................................................................................
..................................................................................................................................
..................................................................................................................................
..................................................................................................................................
Ozone .......................................................................................................................
............................................................................................................................ [6]

(iii)

How is the natural balance of the gases, nitrogen, oxygen and carbon dioxide,
maintained in the atmosphere?
..................................................................................................................................
..................................................................................................................................
..................................................................................................................................
..................................................................................................................................
..................................................................................................................................
............................................................................................................................ [3]

UCLES 2010

5014/11/O/N/10

506

19

BALOL
(b) Some human actions are disrupting this natural balance.

For
Examiners
Use

The graph below shows total world carbon emissions since 1860.
400

billions of tonnes of carbon

350
300
250
200
150
100
50
0 1860 1880 1900 1920 1940 1960 1980 2000
year
(i)

On the graph, draw a summary line to show the change between 1860 and 2000.
[1]

(ii)

Describe the trends shown before and after 1960 and quote values to support your
answer.
..................................................................................................................................
..................................................................................................................................
..................................................................................................................................
..................................................................................................................................
............................................................................................................................ [3]

(iii)

Explain why many people believe that increased emissions of carbon dioxide are
causing global warming.
..................................................................................................................................
..................................................................................................................................
..................................................................................................................................
..................................................................................................................................
..................................................................................................................................
............................................................................................................................ [3]

UCLES 2010

5014/11/O/N/10

[Turn over

507

20

BALOL
(c) Information about the two countries in the world with the greatest carbon dioxide
emissions in 2007 is given below.

For
Examiners
Use

USA
China

percentage of total world


carbon dioxide emissions
in 2007

percentage of total world


carbon dioxide emissions
in 2007
Key
% share of world
CO2 emissions
emissions per head,
1tonne per head

carbon dioxide emissions


per head (2007)

(i)

carbon dioxide emissions


per head (2007)

Describe what the information above shows about the share of total world emissions
from these two countries.
..................................................................................................................................
..................................................................................................................................
..................................................................................................................................
..................................................................................................................................
............................................................................................................................ [3]

UCLES 2010

5014/11/O/N/10

508

21

BALOL(ii) The cartoon below is trying to explain why carbon dioxide emissions are increasing
so fast in China.

For
Examiners
Use

an Companies invest in Ch
Americ
in a

Factory
clothes,
trainers
,
toys

Explain what the cartoon shows about the reasons for carbon dioxide emissions
increasing in China.
..................................................................................................................................
..................................................................................................................................
..................................................................................................................................
..................................................................................................................................
............................................................................................................................ [3]

UCLES 2010

5014/11/O/N/10

[Turn over

509

22

BALOL
(d) When complete, the flow diagram below will show the worldwide effects of global
warming.

For
Examiners
Use

worldwide effects of global warming

ice sheets melting

economic losses for farmers, house


owners, companies and governments

Fill in the boxes by choosing the best comment for each box from the list below.
* sea defences breached

* previously populated areas abandoned

* rising sea levels

* flooding of low lying coastal areas such as deltas


[3]

UCLES 2010

5014/11/O/N/10

510

23

BALOL
(e) In November 2008 the new President in the Maldives said that he intended to set up a
wealth fund from tourist income. This fund would be used to buy land in another country
where his people could move, should global warming be worse than expected.
Global sea levels are expected to rise 2558 cm by 2100.

For
Examiners
Use

Read the information about the Maldives.


The Maldives
Population

380,000
capital city Mal 100,000

Geography

coral islands in the middle of the Indian Ocean


about 1200 islands, of which about 250 are populated
highest point 2.4 m above sea level; average 1.5 m

Economy

main income from tourism with about 500,000 visitors a year


average income about US$ 4,600 per head

(i)

Explain why the President of the Maldives is more worried than leaders in most
other countries about possible threats from global warming.
..................................................................................................................................
..................................................................................................................................
..................................................................................................................................
............................................................................................................................ [2]

(ii)

The President of the Maldives said We can do nothing to stop climate change on
our own.
How true is this statement? Explain your answer.
..................................................................................................................................
..................................................................................................................................
..................................................................................................................................
............................................................................................................................ [2]

(iii)

The President plans to buy land in another country for his people to go to if sea
levels rise. Describe one likely problem with this plan.
..................................................................................................................................
..................................................................................................................................
..................................................................................................................................
............................................................................................................................ [2]

UCLES 2010

5014/11/O/N/10

[Turn over

511

24

BALOL
(f) Read the information below.

For
Examiners
Use

2008 a bad year for climatic hazards


millions face drought in Ethiopia
11 million affected by monsoon floods in India
128,000 killed by a cyclone that struck Myanmar (Burma)

A I blame climate
change for all
these disasters

(i)

B these are
just normal climate
events

C large numbers
are affected because
of poverty

Explain the viewpoints of persons B and C.


..................................................................................................................................
..................................................................................................................................
..................................................................................................................................
..................................................................................................................................
..................................................................................................................................
..................................................................................................................................
..................................................................................................................................
............................................................................................................................ [4]

(ii)

How far do you agree with person A? Explain your answer.


..................................................................................................................................
..................................................................................................................................
..................................................................................................................................
............................................................................................................................ [2]
[Total: 40 marks]

512

BALOL
UNIVERSITY OF CAMBRIDGE INTERNATIONAL EXAMINATIONS
GCE Ordinary Level

MARK SCHEME for the October/November 2010 question paper


for the guidance of teachers

5014 ENVIRONMENTAL MANAGEMENT


5014/11

Paper 1, maximum raw mark 120

This mark scheme is published as an aid to teachers and candidates, to indicate the requirements of
the examination. It shows the basis on which Examiners were instructed to award marks. It does not
indicate the details of the discussions that took place at an Examiners meeting before marking began,
which would have considered the acceptability of alternative answers.
Mark schemes must be read in conjunction with the question papers and the report on the
examination.

CIE will not enter into discussions or correspondence in connection with these mark schemes.

CIE is publishing the mark schemes for the October/November 2010 question papers for most IGCSE,
GCE Advanced Level and Advanced Subsidiary Level syllabuses and some Ordinary Level
syllabuses.

513

Page 2

Mark Scheme: Teachers version


GCE O LEVEL October/November 2010

Syllabus
5014

Paper
11

BALOL
Section A
1

(a)

(i) Surface / opencast


Removed in layers
Roadways between levels
Explosives / blasting
Ore removed by dumper trucks / (large) lorries

[4]

(ii) Large quantities of waste


Disposal will use a large surface area / difficult to find a suitable area for the waste
Ore needs to be concentrated before transporting / high cost of transporting low grade
ore
Cost of concentrating the ore
Increases in size/depth of mine
[3]
(b) Credit any sensible use @ 1, with reasons @ 1 each, e.g.
Fill the hole with the waste
This would free up land which stored the waste for other use
Put the soil back / landscape it
Use it as a park / for farming / other appropriate land use
Use it to store rain
Lake for recreation
Use it for landfill
Would store large amounts of urban waste
Cheap / easy way to dispose of it
Use methane from decomposition for energy
Use it as a repository for toxic waste, etc

[3]
[Total: 10]

(a)

(i) 1965 46 52 (or between)

2005 402 408 (or between)

(ii) 1985 to 1995

[2]
[1]

(b) (i) Arsenic, copper, lead, cadmium, mercury, manganese, chromium, nickel etc.
2 metals = 1

[1]

(ii) Heavy metals toxic


Plastics
choking / entangling / strangling etc.
eaten because looks like jelly fish and fills stomach (causing death)
At least 1 for each substance. Allow 2 for a well developed point.

[3]

UCLES 2010

514

Page 3

Mark Scheme: Teachers version


GCE O LEVEL October/November 2010

Syllabus
5014

Paper
11

BALOL
(c) (Rivers may bring) pollutants from more than one country
Difficult to obtain total international agreement
Need for economic growth / industries
Cost of measures to reduce pollution
Material moved along coast by waves
Oceans linked
Ocean currents move materials between them

[3]
[Total: 10]

(a)

(i) Ideas, such as:


Similarity same extent off Europe / in Greenland
Difference smaller in 2007
decrease off Canada / off North America
more fragmented / more islands of ice (off Eastern Canada)
One similarity and one difference

[2]

(ii) T marked in a tundra area

[1]

(b) (i) High reflectivity / high albedo


Because of white surface

[2]

(ii) Low angle suns rays


Pass through more atmosphere
More energy absorbed / reflected / scattered by atmosphere
Energy spread over a large surface area

[2]

(iii) Agree because global warming


will cause ice to melt
thinning ice will melt faster
less snow falls to make ice
warmer oceans will melt sea ice
Disagree because 1979 may have been a year when insolation was reduced
because of reduced sunspot activity
because of absence of ozone hole
because of more particulates in the air than usual to reflect / scatter
the suns rays
e.g. from large volcanic eruptions of ash
Credit any reasoned argument for both. May take the converse approach for 2007

[3]

[Total: 10]

UCLES 2010

515

Page 4

Mark Scheme: Teachers version


GCE O LEVEL October/November 2010

Syllabus
5014

Paper
11

BALOL
4

(a)

(i) It will increase

[1]

(ii) Grizzly bear

[1]

(iii) 4

[1]

(iv) Predator
Kills / eats / preys on other animals / carnivore e.g. wolf / grizzly bear / cougar /
mountain lion / coyote
Competition
Organisms which need the same resource e.g. coyotes and wolves for food from elk
Producer
Plants use solar energy to make food by photosynthesis
Food chain
Nutrients are passed from one organism to another e.g. such as plant to moose to
coyote to wolf
[4]
(b) Credit reasoned effects, such as:
Animals flee if can move quickly enough
(So) become concentrated in a smaller area
Slow ones / those hibernating may die
Animals underground may survive
Loss of food for herbivores
Habitats destroyed
Will affect breeding success if fire in the breeding season

[3]
[Total: 10]

UCLES 2010

516

Page 5

Mark Scheme: Teachers version


GCE O LEVEL October/November 2010

Syllabus
5014

Paper
11

BALOL
Section B
5

(a) (i) Moving apart / away from each other towards west and east;
(ii) Constructive / diverging;

[1]
[1]

(iii) At this plate boundary / as plates diverge magma reaches the surface.
New material from mantle / interior of the Earth.
Eruptions of lava are frequent / regular.
Usually non-violent outpourings of runny lava / basalt.
Two points made along these lines = 2 marks

[2]

(iv) Geothermal power named.


Description about how it is obtained up to all 3 marks.
Surface sources of hot / boiling water such as hot springs taken in pipes and
used to heat buildings and homes.
Any route to three marks

[3]

(v) Plates are moving together which makes it a destructive boundary.


Eurasian plate is crashing up against Indo-Australian / African plates.
One plate (oceanic plate) is destroyed in the subduction zone.
Great friction / pressure as rock is forced to move against rock.
Creates shock waves underground felt at the surface as earthquakes.
Possible to use a sketch to show some of these points, equally valid.
Four points made along these lines.

[4]

(b) (i) To stop buildings from collapsing:


Steel frames, damping and bracing systems, foundation piles of alternate layers of steel
and rubber, good quality building materials, checks made that builders are sticking to
standards set, low rise buildings.
(ii) To reduce number of deaths after the quake:
Train specialist emergency and rescue teams ready for immediate action, sniffer dogs,
stockpiles of emergency supplies such as tents, blankets, drugs and drinking water, drills
and people educated in advance about what to do.
Other points can be considered if relevant and precise.
Credit both separate and developed points.
Min 2, max 3 for each part, but in exceptional circumstances allow 4:1.

[5]

UCLES 2010

517

Page 6

Mark Scheme: Teachers version


GCE O LEVEL October/November 2010

Syllabus
5014

Paper
11

BALOL
(c)

(i) Usually impossible to know exact numbers of dead.


Variety of reasons for this such as buried bodies not recoverable, chaos/confusion
and remoteness of the location.
Larger the earthquake / casualties then the less the chance of an exact number.
Points made along these lines 2 @ 1 mark
(ii) Indonesia and Iran.

[2]
[1]

(iii) Not regular / uneven pattern / haphazard.


Some years with none (e.g. 2000, 2002, 2007),
One year with three (2005).
Gaps between them within a year variable as well / sometimes consecutive months.
Concentration in the years 200306.
Two descriptive points such as these

[2]

(iv) No reliable chance of people predicting when and where, but it shows that some places
along the boundary are more at risk of activity, although anywhere along the boundary
remains at risk.
General conclusion stated = 1 mark
Elaborated upon with table evidence = 1 mark

[2]

(v) Evidence for this includes:


The strongest (Indonesia 2004) had greatest loss of life.
Two other strong ones (7.9 and 7.6) had big losses of life.
Least strong (Iran 2006) had smallest loss of life.
Evidence against this includes:
Only 1300 dead in the second strongest.
Third largest loss of life (Iran 2003) was among least strong.
It and another at 6.5 varied greatly with loss of life (30,000 and 300).
Wide variations in deaths for earthquakes between 6.0 and 6.5.
Points made along these lines.
General point = 1 mark Supported by comment to appropriate references = 2 marks.
Min. 1 and max. 3 for each of 'for' and 'against'.
[4]
(vi) Stronger for the earthquakes of greater strength above 7.0 than those for below.
There is definitely an element of truth in it but with wide variations, mention in passing of
factors that might matter more than strength such as location, preparedness etc.
View with some support = 1 mark
Well supported view = 2 marks

[2]

UCLES 2010

518

Page 7

Mark Scheme: Teachers version


GCE O LEVEL October/November 2010

Syllabus
5014

Paper
11

BALOL
(d) (i) Nineteenth.
Many of them, with six in a 50 year period in the middle of the century.
Twentieth.
Only two of them, with long gaps between eruptions (34 years, 38 years and 56 years).
Weakly stated difference / unsupported / very one sided = 1 mark.
Well supported difference stated = 2 marks.

[2]

(ii) Fertile soils or its equivalent such as minerals in an agricultural context.

[1]

(iii) Realisation that they are all ways of monitoring changes to give as early a warning as
possible of a full eruption. 1 mark.
Further descriptive comment such as:
Seismograph and tiltmeters record physical movements / changes.
The other two are more to do with heating up and signs of increased activity.
Comment about how either of these are useful indicators.
Earthquakes sometimes trigger off volcanic eruptions.
2 points made = 2 marks
Overall 1 mark + 2 marks

[3]

(iv) Increase in activity may not lead to a full eruption / all the activity may be confined to the
crater.
Volcanoes (especially those on destructive margins) are unpredictable.
Pressure building up inside the volcano may have no external signs with an eruption that
is sudden and violent.
One worthwhile suggestion = 1 mark.
Fuller elaboration, or wider range of suggestions = 2 marks.

[2]

(v) Easy?
Italy is one of the rich developed countries; developed countries have the money to
make advance plans and train people to put it into effect.
There is a plentiful transport system of roads, railways, private cars.
Difficult?
Massive logistical problems in trying to move those numbers of people in only a short
period of time; and to where?
Problems of persuading people to leave before an actual eruption begins.
Reasons for people's reluctance to leave homes and possessions behind, such as fears
from looting etc.
Many people will believe that the volcano will not affect them.
Comments made along these lines, for any answers from all easy to all difficult. No need
to address both sides for full marks.
Some comment but not developed points mentioned rather than explained = 12 marks.
Fuller comment showing wider appreciation = 3 marks.
[3]
[Total: 40]

UCLES 2010

519

Page 8

Mark Scheme: Teachers version


GCE O LEVEL October/November 2010

Syllabus
5014

Paper
11

BALOL
6

(a) (i) Correct plot = 2 marks.


One segment correct = 1 mark.
Completing the key (provided that the graph is attempted) = 1 mark.

[3]

(ii) Water vapour


This is the source for cloud formation and precipitation in the atmosphere (water cycle).
Water is needed for plants (the producers) to grow and to keep both animals and
humans alive. Water is lost by transpiration and perspiration.
Carbon dioxide
Soaked up by plants for photosynthesis; plants are the producers upon which
consumers depend, herbivores directly and carnivores indirectly, and they release
oxygen also it absorbs heat radiated from the surface and stops the Earth cooling
down massively on a night. Without it, the Earth would be much colder and lifeless.
Ozone
Located high in the atmosphere, it absorbs ultraviolet rays which would otherwise harm
plants, animals and humans (e.g. skin cancer).
Minimum 1 mark, maximum 3 marks for each of these.
Otherwise any combination / route to the 6 mark total.

[6]

(iii) Through various cycles and energy flows,


e.g. carbon cycle: from intake of carbon dioxide by green plants to decay and respiration
by plants and animals, e.g. nitrogen cycle from use by plants after the work of bacteria in
root nodules to death, decay and excretion from plants and animals, return to
atmosphere by dentrifying bacteria.
Basic idea or mention of cycle(s) without use = 1 mark.
Elaboration or exemplification = 2 or 3 marks.
(b) (i) Line drawn linking tops of bars to show trend.

[3]
[1]

(ii) Before 1960.


Steady (but persistent) increase.
Increase of 140 billion tonnes in 100 years.
After 1960.
Noticeably faster increase.
Larger increase of 200 billion tonnes in just 40 years.
Trends without fully establishing the difference = 1 mark.
Trends with difference emphasised = 2 marks.
Emphasised trends supported by use of values = 3 marks.

[3]

(iii) 'Thicker layer' / greater concentration of carbon dioxide in the upper atmosphere.
More radiated heat trapped while incoming sunlight stays the same.
The greenhouse effect, accelerating a natural process to increase average world
temperatures by about 0.8C between 1900 & 2000.
Carbon dioxide is just one of the greenhouse gases along with methane, CFCs and
nitrogen oxides.
Three points made along these lines.

[3]

UCLES 2010

520

Page 9

Mark Scheme: Teachers version


GCE O LEVEL October/November 2010

Syllabus
5014

Paper
11

BALOL
(c) (i) Together they account for about half of total world emissions.
China responsible for slightly more carbon dioxide emissions than USA, but much lower
amount per head than in USA.
Per head consumption in USA is 4 times greater than in China.
Three of these.

[3]

(ii) Many industries owned by American companies have moved to China.


They have taken their carbon emissions with them.
Goods made in Chinese factories are exported and sold in the USA.
Moving emissions to China has left the USA a cleaner place.
Any further comment about the reasons for this movement.
Three points made along these lines.

[3]

(d) Order of infilling for the boxes:


Rising sea levels.
Sea defences breached.
Flooding of low lying coastal areas such as deltas.
Previously populated areas abandoned.
All placed in the correct order = 3 marks.
2 or 3 correct = 2 marks.
1 correct = 1 mark.

[3]

(e) (i) Nowhere on the Maldives is above 2.4 metres above sea level; unlike other countries,
there is no place to retreat away to avoid rising sea levels, and its income heavily
dependent on tourism will just disappear.
Some understanding = 1 mark.
Well understood including comparative mention with other countries = 2 marks.

[2]

(ii) Only answer easy to justify is totally true if climate change is due to the enhanced
greenhouse effect. This is because industries and transport in developed countries have
mainly been responsible for the emissions, the USA in particular, many of which have
been reluctant to adopt measures for a reduction. Also relevant are references to recent
and likely future increases in China and India.
OR
A supported view that climate change is natural is a way to try to explain a partly true or
a not true answer.
View and some supporting explanation = 1 mark
Precisely explained / supported view = 2 marks

[2]

(iii) Possible problems:


Finding a country willing to sell land.
Land and environments elsewhere are likely to be very different from home.
What will the economic opportunities be?
Getting everyone to leave especially as flooding is likely to be phased.
Likely problem identified with little further description = 1 mark.
Identified and further elaborated upon = 2 marks.

[2]

UCLES 2010

521

Page 10

Mark Scheme: Teachers version


GCE O LEVEL October/November 2010

Syllabus
5014

Paper
11

BALOL
(f)

(i) Person B
Some of these climatic disasters are regular events in the areas named.
e.g. annual monsoon in India, cyclone season each year in Bangladesh.
Ethiopia is on the edge of the Sahara desert and in the Sahel. They are to do with the
world pattern of pressure and winds i.e. normal climatic events, such as the hurricane
season in the Gulf of Mexico.
Person C
Poor people in developing countries are more badly affected by natural hazards than are
rich people in developed countries. The answer lies in preparation before the event and
provision of emergency services after the event, both of which are money related (for
both people and the economy).
Reference also to big populations and rapidly increasing populations in Asia and Africa
could also be made.
Views explained 2 marks for each view 2 @ 2 marks.
(Exceptionally only allow 3:1 or 1:3).

[4]

(ii) Some people like A are now of the opinion that climate change is causing climate
hazards that already exist to be worse / more intense / more frequent e.g. stronger
cyclone seasons, more protracted droughts.
However, it is possible that because of better communications more people learn about
them faster; it could be that there are simply more people to be affected in areas
regularly hit by them.
Explaining strength of view A, or why the views of B and C may be stronger.
Mark the explanation rather than the view expressed.
Statements made that support view taken = 1 mark.
'How far' part of the question addressed as well = 2 marks.

[2]
[Total: 40]

UCLES 2010

522

General Certificate of Education Ordinary Level


5014 Environmental Management November 2010
Principal Examiner Report for Teachers

BALOL
ENVIRONMENTAL
MANAGEMENT

Paper 5014/11
Paper 11

General comments
Despite the many variations in performance between candidates, the general pattern on this year's paper
was for the total mark for Questions 1- 4 to be slightly higher than the average mark for Questions 5 and 6.
Often the total for Section A was similar to the better of the two marks from Section B. There seemed to be
less pressure on time to complete this paper than in some previous years; however, it was noticeable that
the quality of answers from weaker than average candidates tailed off from part (e) of Question 6 onwards.
Also, fewer answers were continued beyond the lines left for answering than in previous examinations. The
questions which were generally well answered included 4(a)(ii), (iii) and (iv) about food webs, 5(a)(iv) about
geothermal power, 5(b)(i) and (ii) about strategies for preparing against earthquake risks, 6(a)(ii) about the
importance of various gases in the atmosphere for life on Earth, and the flow diagram in 6(d). Conversely
the ones which proved to be more difficult were 1(a)(ii), 2(c), 3(b)(i) and (ii), 5(c)(iii) and (iv), and 6(a)(iii).
Weaknesses seen in this session's examination answers suggested two ways in which candidate
performance in future examinations could be improved. The first is to read more carefully the question set
before starting to write the answer. If necessary, underline key words in the question to try to reduce the
chances of drifting away from the actual question set during answering. It soon became clear to Examiners
that many candidates were missing 'largest number' in the question for 5(c)(ii) and were not homing in on
the pattern of strong earthquakes during the years in 5(c)(iii). Instead, in both questions many answers
were based on earthquake strength, making use only of values from the Richter scale in the table. Likewise
in Question 6(f)(i) candidates were merely required to explain the two viewpoints of persons B and C, and to
reserve their comments on them and their own opinions on global warming for the next part (f)(ii). It did not
appear to be common practice for candidates to look ahead to the next question before answering earlier
questions on the same topic.
The second is to make better use of the resource information provided in those questions. Despite the
strong question wording 'Use evidence from the photograph' in 1(a)(i), most candidates made little further
use of the photograph. This meant that the majority of answers fell well short of the amount of description
needed for a full four mark answer. The focus of Question 6(b)(ii) was the different trends before and after
1960. A majority of candidates who obeyed the question and quoted values stated only total values for 1960
and 2000. While these showed that the amount of carbon emissions had greatly increased, they did not
address the question theme of trends before and after 1960. To do this, candidates needed to use the
values better, for example by comparing the increase of 200 billion tonnes of carbon in just 40 years from
1960 to 2000 with the smaller increase of 140 billion tonnes in the longer period of 100 years between 1860
and 1960.
In Section A, Question 4 was consistently the highest scoring of the four questions, followed by Questions
2, 1 and 3 in that order of average performance. Candidate performances between the different parts of
Question 4 were more even than in the other three questions, which reflected good candidate knowledge
and understanding of food webs. Least well known and understood were the albedo effect and insolation,
examined in Question 3(b), which caused it to rank lowly for marks earned. The most difficult part of
Question 1 proved to be part (a)(ii), with few candidates able to identify problems such as large quantities of
waste and its disposal, or the costs of concentrating the ore to reduce the otherwise prohibitively high costs
of transportation. In Question 2, least well known were the names of two heavy metals; however, since both
were needed for one mark only, more serious mark losses resulted from the limited scope and depth of most
answers to parts (b)(ii) and (c), which carried a majority of the total marks.
In Section B, unattempted questions were few and far between. Candidate knowledge of plate tectonics
was shown to be strong throughout, which meant that parts (a) and (b) of Question 5 were consistently well
answered. Less well answered were questions based on the table of strong earthquakes along plate
boundary Q in 5(c), with the exception of parts (i) and (v). Candidates frequently gave answers to parts (ii),
(iii), (iv) and (vi), which did not match what the question asked for or which drew wrong conclusions from the

UCLES 2010

523

General Certificate of Education Ordinary Level


5014 Environmental Management November 2010
Principal Examiner Report for Teachers
table
evidence. Most of the answers to 5(d) were better focused on the questions set, even if candidates did
BALOL
not always give the amount of detail needed to claim full marks. Candidates picked up marks regularly
enough as they worked their way through the different parts of Question 6, but without always achieving the
same consistency as in Question 5. Some did not know how to draw a divided bar in (a)(i); others found it
straightforward. Some did not recognise that (a)(iii) was about the carbon and nitrogen cycles. Some
explained how the greenhouse effect operates in (b)(iii) while others gave irrelevant answers about the
effects of global warming or made mistaken comments about the hole in the ozone layer. Answers to the
various parts of 6(c), (d) and (e) were typically worth some marks; usually they were in line with question
needs. What was often lacking was sufficient breadth of question coverage to claim all the marks. Question
(f)(i) was the part that was most likely not to be answered in the manner intended.

Comments on individual questions


Section A
Question 1
While opencast mining was almost universally recognised and named in (a)(i), most candidates did not
describe the method in sufficient detail for a four mark answer. Most appeared to rely upon their own
knowledge of opencast mining instead of using the good evidence from the photograph, as the question
directed them to do. The closest many candidates came to earning their first mark in (a)(ii) was for
references to increasing the size of the open pit, although rarely was there any further mention of problems
caused by the need to do this. In part (b) the most common use stated for the large hole was to fill it in with
waste (presumably mining waste), but reasons for this were often lacking. Somewhat surprisingly only the
occasional candidate seemed to search for other uses such as landfill, water storage and recreation.
Question 2
For many candidates, reading values from the graph in (a)(i) was a straightforward task, but even here
occasional candidates misread 1965 for 1975 or gave the answer 400 for 2005 when the value on the graph
was clearly centred between 400 and 410. Most also claimed the mark for the ten-year period 1985 to 1995
in (a)(ii). After this, less able candidates in particular found marks more difficult to obtain. Having to name
two heavy metals in (b)(i) proved to be a challenge too far for most candidates. After correctly naming one,
most commonly copper or lead, the second one named was often a refined product, such as steel or
aluminium. Therefore it was not surprising that more marks were earned for plastics than heavy metals in
answers to part (b)(ii). There were many one mark answers to part (c). Most typically the mark was for
people dumping waste in water. What was lacking was further development such as reasons for this or the
role of water movement. Typically the remaining two marks were left unclaimed.
Question 3
Most candidates recognised the great shrinkage in the extent of sea ice between 1979 and 2007 that was
shown on the maps. This meant that few encountered problems answering part (a)(i). The letter T needed
to be placed on land, and not in Greenland, and only about half the candidates marked T in an acceptable
location in (a)(ii). Part (b) proved to be the most difficult part in Section A. The high reflectivity of white
snow and ice surfaces on light rays from the Sun was little appreciated by candidates when answering (b)(i).
The key to answering (b)(ii) was the low angle of the Sun's rays, leading to more absorption as they pass
through more atmosphere, and spreading the effects over a wider surface area than in the tropics. Among
this cohort of examination candidates, this was little known. Part (c) led candidates into a more familiar
topic, global warming. Most chose to agree with the statement; some were able to relate this to warmer
oceans and ice melting. A few, unfortunately, deviated badly into filling the remaining lines with references to
the hole in the ozone layer and its alleged consequences for global warming.
Question 4
Only part (a)(i) was poorly answered; decline and extinction were more common answers than increase, the
answer which the question had tried to lead them towards. The correct answer of grizzly bear was the most
frequent response in (a)(ii). The correct answer of four was also the most common answer in (a)(iii). The
four terms related to food webs that were listed in the question for (a)(iv) were well known and understood;
they were explained in many different individual ways, but some of the most successful answers used
examples for predator and competition from the diagram. Explaining food chains seemed to be more difficult
for candidates; not all managed to include the idea of nutrients passing from one organism to another.

524

General Certificate of Education Ordinary Level


5014 Environmental Management November 2010
Principal Examiner Report for Teachers
Answers
BALOL to part (b) tended to have breadth rather than depth. Stating three different effects, such as habitat
loss, loss of sources of food and animals migrating to other areas, was the normal style of answering. Some
answers would have benefited from greater precision of expression, such as loss of food for herbivores, or
more mobile animals being the ones to migrate.
Overall in Section A, marks for Questions 1, 2 and 3 were below the mark obtained for Question 4. Only
stronger candidates seemed able to perform more consistently across the questions. Scripts from weaker
candidates almost inevitably included one or two parts in each of Questions 1, 2 and 3 which failed to score
any marks, often a reflection of inadequate knowledge and understanding.
Section B
Question 5
Most candidates claimed the starter marks for parts (a)(i) and (ii). Only a few who either gave just one
compass direction in (a)(i), such as towards the west, or believed that P was a destructive boundary in (a)(ii),
forfeited marks. The two basic points claimed by a majority of candidates in part (a)(iii) were magma from
the Earth's interior, reaching the surface where the plates split apart. Part (a)(iv) was consistently well
answered; most of the references were to geothermal power, but some candidates also referred to hot water
being carried away in pipes for use in heating systems within buildings. Although there were very few zero
mark answers to (a)(v), equally there were few answers where the explanation was sufficiently full to raise
answers from the typical mark of two or three into four. Sometimes this was because candidates, having
accurately explained what was happening at destructive margins, drifted into fold mountain formation or
volcanic activity rather than continuing with the earthquake theme of the question. A few included a diagram;
when labelled, these enhanced the worth of candidates' answers.
Part (b) covered a topic that was familiar to most candidates. The two questions discriminated well
according to candidate ability. More able candidates gave answers to part (i) which included more precise
details about building construction than simply saying 'using good building materials'. This meant that they
achieved a better balance in marks earned between the two parts. Part (ii) was the one which less able
candidates found the easier part; they were able to make mark earning points such as previous education
and practice drills, and making medical and other preparations.
Candidates had few problems understanding question needs to part (c)(i). Those who were most certain to
claim the second mark were the ones who suggested definite reasons for the estimate, such as numbers
buried under rubble, or chaos and confusion following a big disaster. Indonesia and Iran were the countries
needed in part (ii). The most common incorrect answer was Indonesia and India; candidates seemed to be
looking for the two countries with the strongest earthquakes, instead of the 'number' of strong earthquakes
as asked for in the question. Part (c)(iii) was one of the least well answered questions on the paper.
Instead of answering for 'the pattern of strong earthquakes during the nine years shown', many candidates
looked instead for the pattern according to earthquake strength, for example between those above and
below 7.0 on the Richter scale. Quite simple answers were expected, such as the non-regular pattern, with
some years with none (2000, 2002 and 2007) and one year with three (2005). The wording of part (iv)
meant that it was intended to be a more challenging question, because candidates were required to make a
summary judgement. The most common answer from those who found this difficult was that 'Most
earthquakes occur between October and May', which had no significance. Some perceptive answers were
seen, often along the following lines. While there is no reliable way of knowing when and where an
earthquake will actually occur, the table showed places and countries along plate boundary Q where the risk
seemed to be greater, although it existed everywhere. Part (v) was much better answered, particularly by
those who used a systematic approach to answering. They dealt first with 'evidence for' and giving
examples, and then with 'evidence against' followed by more examples. Two and three mark answers
tended to come from candidates who cut corners on giving examples as supporting evidence from the table.
There were wide variations in answer quality to part (vi). A candidate who came to the view that evidence
for 'the stronger the earthquake, the greater the number of deaths' was weak, found it more difficult to claim
both marks than the one who recognised that those earthquakes above 7.5 did, on most occasions, lead to
greater loss of life than those below 6.5. What candidates of both views did recognise was the importance of
other factors, often human ones. Good understanding led to some very effective answers. The least
successful answers to part (vi) came from candidates who merely repeated the style of answering already
used in part (v) and listed examples for or against their stated view.
The better answers to part (d)(i) made mention of the differences in both number and frequency of major
eruptions. Fertile soils was the answer expected in (d)(ii), or an answer in relation to farming which came
down to meaning the same. This was the most popular answer. While other economic activities such as

525

General Certificate of Education Ordinary Level


5014 Environmental Management November 2010
Principal Examiner Report for Teachers
tourism
BALOLand obtaining minerals can be locally important, these cannot explain the great numbers of people
who live around active volcanoes in countries like Italy and Indonesia. These were unacceptable as 'the
most likely reason'. Although there were some weak answers to part (d)(iii), containing little more than the
information given in the spider diagram, most candidates seemed to understand well how the ways listed
could indicate an imminent eruption. Most candidates were able to make one or more sensible suggestions
about why perfect prediction of a volcanic eruption is impossible in (d)(iv). Many of the good answers to
(d)(v) concentrated on the logistical problems of moving and relocating so many people. It was the sign of a
very good answer when the candidate additionally tried to take account of the fact that Italy is a rich
developed country. This means that the authorities have more money and human resources to make
advance plans, and to train people to put the evacuation plans into effect.
Question 5 was well answered throughout. Most candidates had a sound knowledge of tectonic activity,
which they were able to use and to apply to the questions set. For many, (c) was the least well answered
part. There were many clues in the table of information about earthquakes along plate boundary Q which
might have been better used by candidates in the later parts of (c), such as (ii), (iii) and (iv). Using the word
'pattern' in the question seemed to cause more problems for candidates answering (c)(iii) than it did for
(d)(i).
Question 6
Completion of the divided bar graph in part (a)(i) was either a comfortable three marks or a challenge
(perhaps for about one third of candidates). While a few candidates, among those who knew what was
expected of them, made mistakes while plotting the 1% or 21% or both, the rest drew accurate graphs. Of
the third, most tried to draw three separate bar graphs within the graph paper background; all they could
claim was the mark for completing the key, provided that the shading matched the types used on their graph.
Part (ii) was well answered. Candidates showed a widespread appreciation of the importance of all three
gases, and many took the opportunity to fill the extra lines left for answers for water vapour and carbon
dioxide. The dual role of carbon dioxide, for photosynthesis and for preventing the Earth from becoming too
cold for life, was regularly referred to. When answering (iii), some less able candidates could do no more
than state percentages from the table at the top of the page. Some of the others, who did realise that the
question related to natural cycles, such as carbon and nitrogen, provided few further details. Having seen
what the question wanted, this was a waste. Although some of the full mark answers were entirely about the
carbon cycle, more commonly some information about the nitrogen cycle was included as well in these
answers.
The best place for drawing the summary line in (b)(i) was through the centre of the top of each bar; however,
the mark was awarded provided that the line touched any part of each bar. The solution used most was
drawing an increasingly steep curved line through the top left corner of each bar. Curved and straight lines
away from the bars were incorrect. The easy mark in (b)(ii) was for the noticeably faster increase after 1960;
further marks mainly depended on the candidate's selection and use of values to illustrate this trend. There
was an over-concentration on quoting totals for 2000 compared with 1960 instead of, for example, the larger
increase of 200 billion tonnes in only 40 years between 1960 and 2000, which better illustrated the theme of
the question. A greenhouse effect answer, and how it works, was what was expected in (b)(iii). If this was
the focus of the candidate's answer, the three marks were soon claimed. Answers from some candidates
drifted more into the effects of global warming, while the worst answers came from those candidates who
continue to believe that global warming is caused by the hole in the ozone layer. Every time a global
warming question is set, there seems to be a cohort of candidates with this belief.
The information given in part (c)(i) shows three main features about the share of world emissions between
the USA and China: that together the two countries account for about half of the world's emissions; that total
emission from China are slightly higher than from the USA; and that emissions per head in the USA are
much higher than in China. In typical answers candidates identified two of the three. They could obtain the
third mark in another way by using the values to make a statement such as emissions per head are four
times greater in the USA than in China. The quality of the responses to part (c)(ii) closely reflected levels of
candidate understanding. One mark answers tended to be narrow in scope and to rely upon simple narrow
statements about what information the cartoon showed, with little added; whereas three mark answers were
strong on comment about American companies exporting both their manufacturing (due to cheaper costs of
production) and their carbon emissions to China.
Some candidates seemed to start to answer part (d) too quickly, judging by the amount of crossing out. After
one or two attempts, the great majority did find the best order. A tiny number of answers showed no hint of
understanding. In those answers with one mistake, the flow sequence was most likely to be lost after 'rising
sea levels' as 'flooding of low lying coastal areas' was placed ahead of 'sea defences breached'.

526

General Certificate of Education Ordinary Level


5014 Environmental Management November 2010
Principal Examiner Report for Teachers

BALOL

One mark answers were the norm in (e)(i), mainly because candidates paid too little attention to the need to
explain why the threat in the Maldives was greater than in other countries. Most other countries, even delta
countries like Bangladesh and the Netherlands, do have areas of higher land within their borders. Even
without knowing or stating this, candidates could have made stronger use of the information supplied about
the physical make-up of the Maldives as well as their economic dependence on tourism. Again one mark
answers were more common than two mark answers in (e)(ii). Here most candidates showed a basic
appreciation of question need when they referred to all the world's countries having a shared responsibility.
Less often was this stated in the context of the Maldives with its tiny total population. Part (e)(iii) provoked a
wide variety of suggestions. The most convincing ones were related to the availability of habitable land now
that total world population is so large, and to the likely economic possibilities and problems. This question
definitely favoured more able candidates; answers from less able candidates were typically about economic
costs of the move and nothing else.
The worth of many potentially good answers to (f)(i) was reduced by candidates expressing their own
opinions, rather than concentrating on explaining the viewpoints of persons B and C, as required by the
question. This was most likely to happen from candidates strongly of the opinion that humans are
responsible for climate change. The good answers came from candidates who recognised that natural
climatic events do vary greatly in frequency and intensity from year to year. Throughout recorded time the
Indian monsoon, summer rains in savanna lands and the annual tropical cyclone season have varied from
year to year, with significant consequences for people living in the regions affected. Any adverse effects hit
poor countries and poverty-stricken people hardest, because they do not have the resources to reduce or
offset these effects. While some candidates did little more than repeat the comments made by person A
when answering (f)(ii), stronger candidates showed that they were aware that the frequency and intensity of
natural hazards seems to be increasing in some parts of the world. Examiners marked the strength of the
candidate's explanation, not the opinion held.
It is difficult to find an explanation why the total mark for Question 6 was usually, but not always, one or two
marks below that for Question 5, since the topics examined seemed to be familiar enough to most
candidates, and Examiners saw good answers to every one of the individual questions. Perhaps it was more
a problem of varied candidate performance between questions, which led to slightly lower mark
accumulations overall without a consistent pattern as to where different candidates gained and lost marks.

527

BALOL
UNIVERSITY OF CAMBRIDGE INTERNATIONAL EXAMINATIONS
General Certificate of Education Ordinary Level

* 3 9 6 7 5 7 8 7 5 9 *

5014/12

ENVIRONMENTAL MANAGEMENT
Paper 1

October/November 2010
2 hours 15 minutes

Candidates answer on the Question Paper.


Additional Materials:

Ruler
Protractor

READ THESE INSTRUCTIONS FIRST


Write your Centre number, candidate number and name on all the work you hand in.
Write in dark blue or black pen.
You may use a soft pencil for any diagrams, graphs or rough working.
Do not use staples, paper clips, highlighters, glue or correction fluid.
DO NOT WRITE IN ANY BARCODES.
Answer all questions.
All questions in Section A carry 10 marks.
Both questions in Section B carry 40 marks.
At the end of the examination, fasten all your work securely together.
The number of marks is given in brackets [ ] at the end of each question or part question.

For Examiners Use


1
2
3
4
5
6
Total

This document consists of 25 printed pages and 3 blank pages.


UCLES 2010

[Turn o

528

BALOL
1

Section A

For
Examiners
Use

(a) Look at the diagram of part of the Earths crust.

C
Key
E
A

different rock types

r of
laye

altere

d ro
ck

(i)

UCLES 2010

Use letters A to F from the diagram to identify the following:


two areas of igneous rock,

areas ..

one area of sedimentary rock,

area ....

one area of metamorphic rock,

area ....

one area of folded rock,

area ....

one area of faulted rock.

area ....

and ....

[3]

5014/12/O/N/10

529

BALOL(ii) Draw simple labelled diagrams to show two ways in which geological factors can
make the mining of minerals difficult.

For
Examiners
Use

[4]
(b) Why are minerals sometimes mined, even when geological factors make the mining
difficult?
..........................................................................................................................................
..........................................................................................................................................
..........................................................................................................................................
..........................................................................................................................................
..........................................................................................................................................
...................................................................................................................................... [3]

UCLES 2010

5014/12/O/N/10

[Turn over

530

4
2 BALOL
(a) Look at the map of the North Pacific Ocean.

For
Examiners
Use

r ile
Ku N. Pacific Drift Ca
lifo
o
rn
w
ia
Si
n
Cu
Japan
Ku
ro

rre

Hawaii

nt

Northern Equatorial Current


Equator
Key

(i)

main ocean current

minor ocean current

ocean

area where plastic


waste is trapped

Name one cold ocean current marked on the map.


.............................................................................................................................. [1]

(ii)

Use the map to explain why a piece of plastic from a beach in Japan could later be
found on a beach in Hawaii.
..................................................................................................................................
..................................................................................................................................
.............................................................................................................................. [2]

(iii)

In the darker shaded area, on the map of the North Pacific ocean, large amounts of
plastic waste are trapped. Why?
..................................................................................................................................
.............................................................................................................................. [1]

(b) What are the characteristics of plastic waste which cause it to be a very serious problem
in the oceans?
..........................................................................................................................................
..........................................................................................................................................
..........................................................................................................................................
..........................................................................................................................................
...................................................................................................................................... [3]
UCLES 2010

5014/12/O/N/10

531

BALOL
(c) Suggest ways of reducing the amount of plastic waste entering the oceans.
..........................................................................................................................................

For
Examiners
Use

..........................................................................................................................................
..........................................................................................................................................
..........................................................................................................................................
...................................................................................................................................... [3]

UCLES 2010

5014/12/O/N/10

[Turn over

532

6
3 BALOL
(a) (i)

Describe an instrument used to record sunshine hours and explain how it records
them.

For
Examiners
Use

..................................................................................................................................
..................................................................................................................................
..................................................................................................................................
..................................................................................................................................
............................................................................................................................. [3]
(ii)

Why do sunshine hours vary from day to day in a place?


.............................................................................................................................. [1]

11

11

10

10

Jan Feb Mar Apr May Jun Jul


month
Key
Calgary

(i)

Aug Sep Oct Nov Dec

average daily hours of sunshine

average daily hours of sunshine

(b) The graph shows average daily sunshine hours for each month at two places with a
temperate climate. Calgary is in the interior and Vancouver is on the west coast of
Canada in the northern hemisphere.

Vancouver

In which month were the average sunshine hours for Calgary and Vancouver most
similar and by how much did they differ?
month ........................................... difference ........................................... hours [1]

UCLES 2010

5014/12/O/N/10

533

BALOL(ii) Which season has the largest differences in average sunshine hours between
Calgary and Vancouver?

For
Examiners
Use

.............................................................................................................................. [1]
(iii)

Describe the pattern of average daily sunshine hours over the year in Calgary.
..................................................................................................................................
.............................................................................................................................. [1]

(iv)

Giving reasons, compare how suitable solar power would be for Calgary and
Vancouver.
..................................................................................................................................
..................................................................................................................................
..................................................................................................................................
..................................................................................................................................
.............................................................................................................................. [3]

UCLES 2010

5014/12/O/N/10

[Turn over

534

8
4 BALOL
(a) Look at the photograph of a hot desert area.

(i)

For
Examiners
Use

Describe the nature and distribution of the vegetation shown.


..................................................................................................................................
..................................................................................................................................
..................................................................................................................................
..................................................................................................................................
.............................................................................................................................. [3]

(ii) Explain how plants are adapted to survive in a hot desert climate.
..................................................................................................................................
..................................................................................................................................
..................................................................................................................................
..................................................................................................................................
..................................................................................................................................
............................................................................................................................. [4]

UCLES 2010

5014/12/O/N/10

535

BALOL
(b) What are the problems for pastoral farming in hot desert areas like this?
..........................................................................................................................................

For
Examiners
Use

..........................................................................................................................................
..........................................................................................................................................
..........................................................................................................................................
..................................................................................................................................... [3]

UCLES 2010

5014/12/O/N/10

[Turn over

536

10

BALOL
5

Section B

(a) Look at the diagram showing some of the features of trees and forests that are useful to
life on Earth.

For
Examiners
Use

Useful features of trees and forests


Leaves of the trees

Forest canopy

trap light energy from the sun

transpire moisture into atmosphere

fall to surface for new nutrients

see question (b)(i)

Tree roots

see question (b)(ii)


Explain how leaves
(i)

use energy from the sun to support animal life on Earth,


..................................................................................................................................
..................................................................................................................................
..................................................................................................................................
..................................................................................................................................
............................................................................................................................. [3]

UCLES 2010

5014/12/O/N/10

537

11

BALOL(ii) support nutrient cycling,


..................................................................................................................................

For
Examiners
Use

..................................................................................................................................
............................................................................................................................. [2]
(iii)

contribute to the world water cycle.


..................................................................................................................................
..................................................................................................................................
............................................................................................................................. [2]

(b) Fill in the bullet points below with features of the forest canopy and tree roots that are
useful to life on Earth, as was done in the diagram for leaves of trees.
(i)

Forest canopy useful features

.................................................................................................................................
........................................................................................................................... [2]
(ii)

Tree roots useful features

.................................................................................................................................
........................................................................................................................... [2]

UCLES 2010

5014/12/O/N/10

[Turn over

538

12

BALOL
(c) One of the worlds largest surviving areas of natural forest is in the Amazon Basin,
mostly in Brazil.

For
Examiners
Use

Location of tropical rainforest in Brazil

mouth of River
Amazon

ATLANTIC
OCEAN
Manaus
Santarem

BRAZIL

Brasilia
Cuiaba
Key:
Border of Brazil
Sao Paulo

Cities

Rio de
Janeiro

Early clearances

ecomonic core
of Brazil

Recent clearances
(post 1970)
Southern Limit
of natural rainforest

UCLES 2010

5014/12/O/N/10

539

13

BALOL (i) Describe the differences in location between areas of early and recent rainforest
clearances.

For
Examiners
Use

..................................................................................................................................
..................................................................................................................................
..................................................................................................................................
..................................................................................................................................
............................................................................................................................. [3]
(ii)

Suggest reasons which might explain these differences.


..................................................................................................................................
..................................................................................................................................
..................................................................................................................................
..................................................................................................................................
..................................................................................................................................
.............................................................................................................................. [3]

UCLES 2010

5014/12/O/N/10

[Turn over

540

14

BALOL
(d) The plan to pave the BR163 road between the towns of Cuiaba and Santarem (towns
located on the map) has caused a lot of controversy.

For
Examiners
Use

Should the rest of the BR163 be paved?


Otherwise known as the soyabean highway, the BR163 is the 1770 km long road linking
Cuiaba in the middle of Brazil to the deep water port of Santarem on the Amazon. It was
begun in the 1970s. Distances along it are huge.
Roadside sign near Santarem
Cuiaba

1767 km

Brasilia

2910 km

Rio de Janeiro 4114 km


3922 km

Sao Paulo

At the moment, half of it is unpaved dirt track, making travel difficult and slow. During the
wet season it becomes a sea of red mud; trucks can be stuck for days, weeks, or even months
after bridges are washed away. Under pressure from the strong farming business lobby,
the government is considering paving the rest of the road with a hard surface. The paved
section north of Cuiaba passes through already important areas of soyabean and beef cattle
production, both major exports of Brazil.
Although the government owns the 100 km wide stretch on each side of the road, trees on
the unpaved section have already been cleared as far as the eye can see. Cattle graze among
the tree stumps. The only lorries on the road are carrying timber, either tree trunks or sawn
planks. Illegal logging is what dominates here, not the rule of law.

Everyone has their own view on paving the BR163.

Trees, not gold,


provide the wealth in
the Amazon.

We all dream
of the day it will be
completely paved.

Logging company manager

There is a land
rush here, just with the
prospect of the road
being paved.

Land agent
Truck driver

All our problems for


exporting will vanish. The world
price of soyabean doubled
between 2006 and 2008.

We are worried. If
this goes ahead, half
the rainforest will be gone
by 2030.

Environmentalist
Soyabean farmer
UCLES 2010

5014/12/O/N/10

541

15

BALOL (i) State the economic advantages of paving the remainder of the BR163 road.
..................................................................................................................................

For
Examiners
Use

..................................................................................................................................
..................................................................................................................................
..................................................................................................................................
..................................................................................................................................
............................................................................................................................. [3]
(ii)

How strong are the economic reasons for paving the road? Explain what you think.
..................................................................................................................................
..................................................................................................................................
..................................................................................................................................
............................................................................................................................. [2]

(iii)

How far do you agree with the environmentalist that half the Amazon rainforest
will be gone by 2030? Answer as fully as you can with the help of the information
given.
..................................................................................................................................
..................................................................................................................................
..................................................................................................................................
..................................................................................................................................
..................................................................................................................................
..................................................................................................................................
..................................................................................................................................
............................................................................................................................. [4]

UCLES 2010

5014/12/O/N/10

[Turn over

542

16

BALOL
(e) Describe what makes tropical rainforests unique (different from all the other forests in
the world).

For
Examiners
Use

..........................................................................................................................................
..........................................................................................................................................
..........................................................................................................................................
..........................................................................................................................................
..........................................................................................................................................
..................................................................................................................................... [3]
(f)

Look at the spider diagram showing examples of sustainable ways to use tropical
rainforests.
B
tribes living by hunting,
A
fishing and collecting
small-scale shifting
cultivation

C
rubber collecting
from wild trees

sustainable uses
of tropical rainforests
D
sustainable
harvesting of
hardwoods

(i)

E
creating National
Parks

F
building jungle lodges
for ecotourism

Describe how the types of activities listed in AC are sustainable.


..................................................................................................................................
..................................................................................................................................
..................................................................................................................................
..................................................................................................................................
..................................................................................................................................
............................................................................................................................. [3]

(ii)

Why are all of these in decline?


..................................................................................................................................
..................................................................................................................................
..................................................................................................................................
............................................................................................................................. [2]

UCLES 2010

5014/12/O/N/10

543

17

BALOL(iii) State two ways in which sustainable logging of hardwoods is different from the
logging taking place along the sides of the BR163 in Brazil.

For
Examiners
Use

..................................................................................................................................
..................................................................................................................................
..................................................................................................................................
............................................................................................................................. [2]
(iv)

What is ecotourism, and can it save the rainforest and its peoples?
..................................................................................................................................
..................................................................................................................................
..................................................................................................................................
..................................................................................................................................
..................................................................................................................................
..................................................................................................................................
..................................................................................................................................
............................................................................................................................. [4]
[Total: 40 marks]

UCLES 2010

5014/12/O/N/10

[Turn over

544

18
6 BALOL
(a) The diagram shows a rain gauge.

For
Examiners
Use

Rain gauge

grass

(i)

On the diagram, name the main parts of the rain gauge.

(ii)

Why is it partly buried in the ground?

[3]

..................................................................................................................................
............................................................................................................................. [1]
(iii)

Explain how an accurate measurement is made of the amount of rain water


collected.
..................................................................................................................................
..................................................................................................................................
..................................................................................................................................
............................................................................................................................. [2]

UCLES 2010

5014/12/O/N/10

545

19

BALOL(iv) Four possible sites for locating a school rain gauge are marked AD on the plan of
the school and its surroundings.

school
buildings

path

For
Examiners
Use

pa

th

drive
D
hard surface

open area
(grass)

grass area
trees

Which one of these sites is best for obtaining accurate rainfall measurements?
Explain why.
..................................................................................................................................
..................................................................................................................................
..................................................................................................................................
............................................................................................................................. [2]
(v)

Choose two of the other sites and explain why they are less good.
..................................................................................................................................
..................................................................................................................................
..................................................................................................................................
............................................................................................................................. [2]

UCLES 2010

5014/12/O/N/10

[Turn over

546

20

BALOL
(b) (i)

Deaths from climatic hazards


(percentages of the world total)

For
Examiners
Use

Show these percentages on a pie graph.

climatic hazard

% of deaths

tropical cyclones

63

floods

33

drought

4
[3]

(ii)

World map showing the distribution of tropical cyclones

Tropic of
Cancer
Equator
Tropic of
Capricorn

Key:
tracks of tropical cyclones
main areas of formation
areas affected

UCLES 2010

5014/12/O/N/10

547

21

BALOL

Look at the world map showing the distribution of tropical cyclones.


State what they have in common for places where they form, direction of movement
and areas affected.

For
Examiners
Use

formation ...................................................................................................................
..................................................................................................................................
..................................................................................................................................
movement..................................................................................................................
..................................................................................................................................
areas affected............................................................................................................
..................................................................................................................................
.............................................................................................................................. [4]
(iii)

Explain what makes tropical cyclones so dangerous for people, sometimes leading
to great loss of life.
..................................................................................................................................
..................................................................................................................................
..................................................................................................................................
..................................................................................................................................
..................................................................................................................................
..................................................................................................................................
..................................................................................................................................
.............................................................................................................................. [4]

UCLES 2010

5014/12/O/N/10

[Turn over

548

22

BALOL
(c) One country which regularly suffers from drought is Niger in West Africa.
Location of Niger

For
Examiners
Use

Tropic of Cancer

Niger

Equator

(i)

Use the map to describe the geographical location of Niger.


..................................................................................................................................
..................................................................................................................................
..................................................................................................................................

(ii)

Why does its location make it more difficult for it to receive aid in an emergency?
..................................................................................................................................
..................................................................................................................................
............................................................................................................................. [3]

UCLES 2010

5014/12/O/N/10

549

23

BALOL(iii)

precipitation (mm)

temperature (C)

Climate graph for Niamey in Niger

For
Examiners
Use

35

35

30

30

25

25

20

20

15

15

10

10

300

300

250

250

200

200

150

150

100

100

50

50

J
J
month

Describe the main characteristics of this climate.


..................................................................................................................................
..................................................................................................................................
..................................................................................................................................
..................................................................................................................................
..................................................................................................................................
............................................................................................................................. [4]
(iv)

Name the climatic type in Niger.


.............................................................................................................................. [1]

UCLES 2010

5014/12/O/N/10

[Turn over

550

24

BALOL(v) Average annual rainfall is 554 mm. Looking at the climate graph, explain why crop
and livestock farmers in Niger depend greatly upon this amount of rain falling every
year.

For
Examiners
Use

..................................................................................................................................
..................................................................................................................................
..................................................................................................................................
............................................................................................................................. [2]
(vi)

Average annual rainfall in Niger is described as unreliable. What is meant by this


and how can it lead to drought?
..................................................................................................................................
..................................................................................................................................
..................................................................................................................................
............................................................................................................................. [2]

(d)

Effects of two droughts


Niger 2005

Europe 2006

Over 3 million of its 13 million people affected Record low output for many crops up to
by food shortages
50% lower than average
Niger is a debt-ridden country; it had to UK gardeners banned from using hose pipes
rely upon food aid from the UN and aid and sprinklers
organisations
Countless children were dying from severe Swimming pools around the Mediterranean
malnutrition
remain empty of water
Worst affected were nomadic herders such Such poor pastures that livestock farmers in
as the Fulani; up to 70% of their livestock France forced to start using winter stocks of
died through lack of fodder
fodder such as hay
Nomads move their animals towards available Lower electricity output from HEP stations
pastures where they come into conflict with
crop farmers for scarce resources

Information about Niger


Income per head US$250 per year
Birth rate 55 per 1000
Fertility rate 7.91 per woman

UCLES 2010

Information about Europe


Income per head US$11,800 per year
Birth rate 12 per 1000
Fertility rate 2.1 per woman

5014/12/O/N/10

551

25

BALOL (i) Describe how the effects of the droughts were different between Niger and
Europe.

For
Examiners
Use

..................................................................................................................................
..................................................................................................................................
..................................................................................................................................
..................................................................................................................................
..................................................................................................................................
............................................................................................................................. [3]
(ii)

Two main factors explain the different effects of the droughts for Niger and Europe.
What are they?
..................................................................................................................................
............................................................................................................................. [1]

(iii)

Describe how and why these led to different effects in Niger and Europe.
..................................................................................................................................
..................................................................................................................................
..................................................................................................................................
..................................................................................................................................
..................................................................................................................................
.............................................................................................................................. [3]
[Total: 40 marks]

UCLES 2010

5014/12/O/N/10

552

BALOL
UNIVERSITY OF CAMBRIDGE INTERNATIONAL EXAMINATIONS
GCE Ordinary Level

MARK SCHEME for the October/November 2010 question paper


for the guidance of teachers

5014 ENVIRONMENTAL MANAGEMENT


5014/12

Paper 1, maximum raw mark 120

This mark scheme is published as an aid to teachers and candidates, to indicate the requirements of
the examination. It shows the basis on which Examiners were instructed to award marks. It does not
indicate the details of the discussions that took place at an Examiners meeting before marking began,
which would have considered the acceptability of alternative answers.
Mark schemes must be read in conjunction with the question papers and the report on the
examination.

CIE will not enter into discussions or correspondence in connection with these mark schemes.

CIE is publishing the mark schemes for the October/November 2010 question papers for most IGCSE,
GCE Advanced Level and Advanced Subsidiary Level syllabuses and some Ordinary Level
syllabuses.

553

Page 2

Mark Scheme: Teachers version


GCE O LEVEL October/November 2010

Syllabus
5014

Paper
12

BALOL
Section A
1

(a)

(i) Two areas of igneous rock,


One area of sedimentary rock,
One area of metamorphic rock,
One area of folded rock,
One area of faulted rock,

areas 2 from B/C/D


area A / E
area F
area A
area E

All 6 correct = 3
4/5 correct = 2
2/3 correct = 1

[3]

(ii) Diagrams to show difficulty caused by two of:


Folding
Steeply dipping
Faulting
Thinning seams / wash outs
Great depth
Up to two marks for each diagram. For two marks both the geological factor and the
nature of the difficulty should be made clear by labelling. .One mark for geological factor
label and one for difficulty label. Unlabelled diagrams = 0.
[4]
(b) Credit ideas such as:
As resources deplete they increase in value
Increased / high demand results in higher price / profits
There are large reserves beyond the geological difficulty
There are good quality reserves beyond the geological difficulty
Improved technology allows easier access / exploitation
Improved / cheaper transport to the area
Provides employment (especially in areas without any other)
Great export value to earn foreign exchange
Valuable minerals / minerals with special uses

[3]
[Total: 10]

(a)

(i) Californian / Kurile

[1]

(ii) Clockwise circulation of ocean currents


Moved by main currents east and then south / named currents in the circle
Minor currents move it towards Hawaii

[2]

(iii) It cannot cross the ocean currents which surround the area
All of the currents move / push towards the centre

[1]

(b) Credit ideas such as:


(Much of it) floats
(Much of it) light so easily blown by wind
Does not biodegrade / very durable / almost indestructible
Photodegrades / disintegrates in to small pieces
Small enough to enter the food chain via birds / marine animals
Can absorb organic pollutants / DDT / PCBs / PAHs and become toxic

[3]

UCLES 2010

554

Page 3

Mark Scheme: Teachers version


GCE O LEVEL October/November 2010

Syllabus
5014

Paper
12

BALOL
(c) Credit ideas such as:
Recycle plastics
Substitute other materials or e.g. of alternative material
Enforce bans on dumping at sea
Filter / remove plastic from sewage before releasing it into the ocean
Clear beaches of plastic
Educate public about the dangers of plastic
Fines for litter dropping

[3]
[Total: 10]

(a)

(i) Glass sphere


Concentrates the suns rays on one point
Metal frame behind
Reflects heat onto the recording sheet
When sun shines the recording sheet is scorched
Recording sheet has hours marked on it
Accept any other relevant method
(ii) Varied cloud / fog amount / revolution of Earth around the sun

(b) (i) Month

September difference 0.4 hours (both needed)

(ii) Winter

[3]
[1]
[1]
[1]

(iii) Accept any way of expressing the idea that they rise to a peak in July / summer then fall
to the end of year
[1]
(iv) Accept ideas, such as:
Calgary would be more suitable than Vancouver because it has higher sunshine hours in
every month
Both suitable in summer because higher summer sunshine hours
Both unsuitable in winter because low sunshine hours
Calgary more suitable for more months because e.g. it has 6 months with over 6 hours
of sun but Vancouver has only 5
Relevant reference to / use of values
[3]
[Total: 10]
4

(a)

(i) Sparse
Widely spaced / scattered
Low / short
Scrub
Cushion shaped / tussocky
Even height / one layer
Some lighter coloured / white
No trees / all shrubs / bushes (or grass)
Small leaves
Grass
Rocky background bare of vegetation cover

[3]

UCLES 2010

555

Page 4

Mark Scheme: Teachers version


GCE O LEVEL October/November 2010

Syllabus
5014

Paper
12

BALOL
(ii) Credit adaptations with climatic reason such as:
Widely spaced to gather water from a large area after rain / competition for water
Shallow wide spreading roots to catch rain as soon as it falls / before it evaporates
Small leaves / thorns /spines minimise transpiration
Waxy / hairy leaves reduce transpiration
Thick cuticle reduces transpiration
Seeds lie dormant during dry periods
Spring to life / germinate / flower quickly / short life cycle after rain
Bulbous roots store water
Halophytic because high evaporation leads to saline soils
(b) Credit problems such as:
Low carrying capacity
Vegetation not very nutritious/ unpalatable
Easily damaged by trampling
Easily overgrazed leading to soil erosion
Little rain for drinking water for animals
Great heat unsuitable for sheep

[4]

[3]
[Total: 10]

UCLES 2010

556

Page 5

Mark Scheme: Teachers version


GCE O LEVEL October/November 2010

Syllabus
5014

Paper
12

BALOL
Section B
5

(a) (i) Process of photosynthesis.


Plants absorb carbon dioxide from the atmosphere,
give off oxygen to support life on Earth.
Correct statement of the formula.
3 @ 1 mark

[3]

(ii) Decay and decomposition of leaves on forest floor,


work of decomposers releases nutrients,
taken up by plant roots and reused for new growth
2 @ 1 mark

[2]

(iii) Evaporation of water into water vapour by heating,


feeds the rising warm air until it cools and moisture condenses,
further cooling leads to fresh precipitation on to the Earth's surface.
2 @ 1 mark

[2]

(b) (i) Canopy possible bullet points:


Stops the impact of heavy rain on the ground.
Helps to prevent soil erosion.
Reduces leaching of minerals in the soil.
Provides a habitat for birds and animals.
Two advantages such as these 2 @ 1 mark

[2]

(ii) Tree roots possible bullet points:


Anchor soil / prevent it moving and eroding (especially on slopes).
Absorb water from the soil which is transpired through the leaves.
Absorb nutrients from weathered rock.
Two advantages such as these 2 @ 1 mark

[2]

(c) (i) Early clearances were along the coast (on both sides of the mouth of the Amazon), and
in long strips following the main river valleys, whereas most recent clearances are along
the southern edges, almost whole length but more extensive in east / thinner trail in far
west.
Three points such as these 3 @ 1 mark, but a maximum of two marks for separate
statements without establishing a difference.
[3]
(ii) Likely reasons include:
Early access from the sea, using the rivers to reach inland areas and penetrate the forests,
whereas recent settlement is from the south from the more populated part of Brazil (map
evidence from the number of cities and label for the economic core of Brazil).
Trail of clearances in the south west going across river valleys suggest now following
roads rather than the rivers in order to penetrate deep into the forests.
Some of the interior forests may be protected forest areas.
Understood, with well stated reasons for the differences 3 marks
Suggests valid reasons without full coverage 2 marks
Limited understanding and progress 1 mark

[3]

UCLES 2010

557

Page 6

Mark Scheme: Teachers version


GCE O LEVEL October/November 2010

Syllabus
5014

Paper
12

BALOL
(d) (i) Journey time to reach the river port of Santarem would be much reduced especially
during the wet season / description of the problems on the dirt part.
Remembering that distances are massive, (1700km from Cuiaba).
Soybean and beef are important exports for the economy of Brazil.
Would open up / encourage more forest land to be used for economic activities.
Credit statement of advantages as well as elaboration.

[3]

(ii) Economic reasons are strong will be the easiest view to explain:
To use more fully natural resources such as timber and extend farming area,
especially in the time of high soybean prices.
One approach if economic reasons are considered to be weak would be to focus on the
short-term nature of profits before environmental damage reduces the economic benefits.
Mark according to answer worth; for both marks, the explanation must be appropriate for
the strength level suggested for the economic reasons.
[2]
(iii) Evidence from the information given can be assembled to show the potential for great
forest losses to occur, such as:
Trees have already gone from sides of unpaved sections despite the land being
government owned.
Logging already dominates at the expense of the rule of law, 'Trees, not gold are the
wealth of the Amazon'.
Already people are buying up land in anticipation that the road will improve access.
Roads are doing what rivers did in the past, by providing access into the dense
rainforests.
Also plenty of evidence of the economic opportunities (at least in the short term) for
selling hardwoods, soybeans and beef.
Comment about what could hold it back:
Greater pressure from international environmental groups.
Idea mooted for carbon payments whereby developing countries are paid to keep their
rainforests as carbon stores.
Global economic downturns.
Attitude of the government of Brazil etc.
Statements from the information given, but limited or no comment towards question
asked = 1 or 2 mark answers.
Valid comment about the question asked, well supported = 3 or 4 mark answers

[4]

(e) Greatest biodiversity on Earth.


Elaborated upon or exemplified.
Greatest mass / highest primary productivity of any forest.
Elaborated upon by references to the mass of plants filling all manner of forest niches.
More vertical layers to the forest than in any other (4 or 5 of them).
Elaborated upon by stating examples.
Identification of relevant factor = 1 mark.
Elaboration = 2nd mark.
2 + 1 marks or 3 @ 1 mark

[3]
UCLES 2010

558

Page 7

Mark Scheme: Teachers version


GCE O LEVEL October/November 2010

Syllabus
5014

Paper
12

BALOL
(f)

(i) Using / living from natural forest resources only, cannot overuse them or it leads to
starvation / loss of living.
Only a small proportion of the resources available are taken to allow natural
replacement.
They move to other areas of the forest when resources are reduced.
'No forest, no living' made clear in the answer.
Further description of individual activities to show how they match these descriptions.
Three points / comments made along these lines.

[3]

(ii) Provide a living / allow survival but nothing more than this.
Can't generate surpluses without over-using natural resources.
Subject to the vagaries of nature / little or nothing stored.
Only low levels of development.
OR
Being pushed out / way of life destroyed by invasion of outsiders.
As natural ecosystems are being exploited in previously untouched areas.
No resistance to diseases, superior technology etc.
Also population pressure from outside leading to larger scale / more intensive farming.
Points made like these that demonstrate understanding.

[2]

(iii) Sustainable logging involves using techniques such as cutting only mature trees,
isolating and felling individual trees and in some cases replacement, whereas Brazil style
logging (as elsewhere) is clearing the whole forest, using the commercially valuable
trees and leaving or burning the rest.
Known and understood = 2 marks
Some idea = 1 mark

[2]

(iv) Ecotourism takes into consideration the natural environment and needs of local people
instead of just what tourists want.
National Parks etc, help to preserve the natural forest and its wildlife, which is what the
tourists go to see.
Local people have expertise useful to tourism drive the canoes, act as guides as well
as for general tasks.
For the future of the rainforests motto 'if it pays, it stays' so forests are more likely to be
safe in tourist areas, but the returns are steady long-term as opposed to the big shortterm gains from logging and farming, which make pressures for clearance difficult to
resist.
Also allow critical references to ecotourism if placed in question context such as loss of
traditional indigenous values.
Knowledge and understanding of ecotourism up to 3 marks.
Comment about the broader theme of the question up to 3 marks.

[4]
[Total: 40]

UCLES 2010

559

Page 8

Mark Scheme: Teachers version


GCE O LEVEL October/November 2010

Syllabus
5014

Paper
12

BALOL
6

(a) (i) The three main parts (for the most obvious labels) are funnel, inner container and outer
cylinder (allow other names and descriptions).
Other labels are possible, such as 30cm for the height above the ground.
Three accurate labels = 3 marks.

[3]

(ii) To reduce / prevent evaporation of the rain water collected before it is measured,
or similar.

[1]

(iii) Water collected is emptied into a measuring cylinder, which is placed on a level surface
such as a table top to be read.
Reading taken at eye level.
Two points such as these.

2 @ 1 mark.

(iv) Best answer D = 1 mark.


In an open area on grass = 1 mark.

[2]
[2]

(v) A splash back from the hard surface / also sheltered from rain by the building /
cannot be partly buried on a hard surface.
B in shelter of building.
C under the shelter of trees which would block some of water.
2 @ 1 mark.

[2]

(b) (i) Percentages shown with high level of accuracy = 2 marks.


At least one drawn correctly = 1 mark.
Sectors labelled or shown in a key = 1 mark.

[3]

(ii) Formation.
In hot ocean areas / over the oceans near to the Equator.
Movement.
Away from the Equator towards the two tropics, towards the west (north west in northern
hemisphere, south west in the southern),
Circular patterns of movement (clockwise N, anticlockwise S of Equator).
Areas affected.
Narrow coastal strips of land,
on windward / mainly eastern coasts.
Examples of areas affected.
1 mark reserved for each heading = 3 @ 1 mark.
4th mark for further detail / elaboration for any one of them.

[4]

(iii) Dangers come from very strong winds and heavy rainfall causing floods,
also the strong winds piling up the waves and driving sea water inland.
Explanation why the winds are so strong and rainfall so heavy
Up to 3 marks.
Types of damage from high winds and / or flooding.
Difficult to build homes capable of withstanding high winds above 150km/hr, especially
as many countries in tropics are developing countries.
Four points made along the lines suggested.

[4]

UCLES 2010

560

Page 9

Mark Scheme: Teachers version


GCE O LEVEL October/November 2010

Syllabus
5014

Paper
12

BALOL
(c) (i) Landlocked however expressed or indicated.
Reference to its latitude about 1520 north of the Equator / in the tropics.
(ii) Well away from the ports on the coast / long road journeys through other countries.
Has to rely upon aid materials coming by air which is more expensive.
Also cannot bring in the same amounts of food and other supplies as by sea.
Three points made along these lines, with one mark reserved for each part.

[3]

(iii) Temperature:
Hot all year / lowest temperature 24C in January (mid-winter).
Highest temperature 34C in April and May.
Annual temperature range 10C.
Temperatures fall in the wet summer season / with the arrival of the rains.
Precipitation:
Wet season May to September / summer.
Highest rainfall 188mm in August.
Dry season October to April/May / winter.
No rain in the four months November to February.
Four descriptive points with one mark reserved for each of temperature
and precipitation.
(iv) Savanna (or one of the recognised alternative names),

[4]
[1]

(v) Points that might be made:


High temperatures all year mean high rates of evaporation.
Complete mid-winter drought.
Therefore crop farmers must rely upon the summer rains (as there is no moisture stored
in the soil).
Understood and clearly explained = 2 marks.
Some understanding = 1 mark.

[2]

(vi) Unreliable means that summer rains do not always come / wide variations in rainfall
amounts from year to year = 1 mark.
If the expected rains do not arrive, in particular for two or more years, the result is
drought everything dries up / crops fail in areas where it is not normally expected to
happen = 1 mark.
[2]
(d) (i) In Niger the drought led to deaths of people (over 3 million) and animals (70% of Fulani
livestock). Emergency food aid was needed.
In Europe the drought led to reductions and losses (in farm output, and electricity) and
inconveniences (not watering gardens and filling up swimming pools) but there was no
mention of deaths.
Effects in Niger = 1 mark.
Effects in Europe = 1 mark.
Comment on differences / different nature clearly established = 1 mark.

[3]

UCLES 2010

561

Page 10

Mark Scheme: Teachers version


GCE O LEVEL October/November 2010

Syllabus
5014

Paper
12

BALOL
(ii) Recognition that the two factors are poverty and population increase, however stated.
Both needed for the mark.
[1]
(iii) Poverty in Niger meant that there was nothing in reserve for dealing with the drought,
which is why people and animals died. Aid was needed to alleviate its effects. Made
worse by the high birth rate population growth and more and more mouths to feed so
that surpluses could be not be built up to prepare for years with bad harvests.
Contrast this with livestock farmers in France able to use stored winter fodder.
No mention of deaths and aid because shortfalls in farm output could be offset by buying
food elsewhere.
Understood and clearly stated = 3 marks.
Some progress = 1 or 2 mark answers.

[3]
[Total: 40]

UCLES 2010

562

General Certificate of Education Ordinary Level


5014 Environmental Management November 2010
Principal Examiner Report for Teachers

BALOL
ENVIRONMENTAL
MANAGEMENT

Paper 5014/12
Paper 12

General comments
In Section A, there was little difference in mark outcomes between the four questions. Among more able
candidates, full marks were obtained more frequently for Question 4 than for any of the others; marks for
Questions 3 and 4 were typically higher by one or two than for Questions 1 and 2. Among less able
candidates, the mark pattern was slightly different; Question 2 most regularly yielded the highest mark and
Question 4 the lowest. Question 4 appeared to have two hurdles for weaker candidates: one was needing
to concentrate only on description from the photograph in 4(a)(i) and the other was having to write an answer
about pastoral and not crop farming in part (b). In many cases, the total mark out of 40 for answering
Section A was similar to the candidate's average mark for the two longer questions in Section B.
Occasionally there was a big difference in standard between the answers to Sections A and B, but there
was no consistent pattern as to which one of the two was the better answered. It all came down to individual
candidate preference and performance.
Within Section B, the general distribution of marks between Questions 5 and 6 on this year's paper was
slightly different from the pattern seen in previous examinations. This year the total mark for Question 6 for
the majority of candidates was a little higher than that for Question 5. The apparent preference this year for
Question 6 over Question 5 represented a rare example of a question on the Atmosphere section of the
syllabus being more popular than one drawn from the Biosphere.
Most candidates had been well prepared for the examination. Questions covered topic areas with which the
majority of candidates showed a high degree of familiarity, particularly those related to the characteristics of
desert vegetation in 4(a)(i) and (ii), the usefulness of trees and forests in 5(a) and (b), siting the rain gauge
in 6(a, and tropical cyclones in 6(b). There was a minority who showed such limited understanding of the
main topics that they struggled throughout, able only to cover some of the basic points without any hint of
elaboration.
Time for completing the paper was an issue only for a handful of candidates. For these individuals, it was a
serious issue. Answers to the short questions in Section A regularly carried on into the empty spaces
beyond the lines left for answering. In contrast, answers to Question 6 in Section B became progressively
shorter and more like notes until some questions, at least those in part (d), were left unanswered. It seemed
to be more of an issue of exam time management for individuals rather than inadequate availability of time
for the length of paper, because other candidates completed all the questions, and some from time to time
even continued their answers on the blank pages 26 and 27. On this paper, all well prepared and
knowledgeable candidates can expect to be busy for the full 2 hours 15 minutes allowed; therefore they
should enter the exam room with a plan for managing their time efficiently. They can be recommended to
spend no longer than 45 minutes answering Section A. They can always go back and add to earlier
answers if they find that they have finished Section B with some time to spare.
Many candidates could have improved the reward for their knowledge and understanding in three different
ways in this examination, all of them related to examination technique. The first was by reading ahead to the
next related part or parts of the question, before beginning to answer the first part. For example, many
candidates wrote longer than necessary answers to Question 6(a)(iv), because they also gave the answer
needed in the next part, 6(a)(v), by stating the negative points of sites A, B and C. Doing this led one able
candidate to write 'Oh explained previously' in the margin before having to repeat the answer in (a)(v).
Secondly, some candidates were unable to distinguish whether the question set demanded only description
from the information given, or whether further explanation was what was needed. This was particularly
noticeable in many of the answers given to parts (i) and (iii) of Question 6(d). Some answers to part 6(d)(i)
were full of reasons, related to differences in level of development and population between Niger and
Europe. When part (d)(iii) was reached, the problem then for the candidate was whether or not to repeat
these reasons, which meant giving the same answer as in part (d)(i). The question instruction required

563

General Certificate of Education Ordinary Level


5014 Environmental Management November 2010
Principal Examiner Report for Teachers
candidates
BALOL to describe 'why' as well as 'how'. Some did repeat or write similar answers to gain marks for
their answers to (d)(iii), but others went back to describing how the effects were different between Niger and
Europe, the answer that had been needed in (d)(i). By reversing the answers needed, candidates lost all
marks for both parts. Quite often this poor ending was atypical of earlier candidate performance. The
principle behind many of the questions set on this paper, including the one followed here, is to require
candidates to describe first what a resource shows, before expecting explanation.
Thirdly, some answers would have benefited from a fuller and more obvious use of the source information
given. There might have been an issue in Question 5(d) because so much information about the BR 163
road was given. Certainly many answers to all three parts of (d) would have come closer to gaining
maximum marks if candidates had made fuller use of the information provided to include more variety of
points in their answers. The focus in Question 6(c)(iii) (the climate graph) was much narrower. In order to
gain all four marks, it was essential for candidates to incorporate relevant supporting values from the graph
in their answers. Candidates can be guided into what to look for when asked in examination questions to
describe the main characteristics of the climate of a place. The significant characteristics that candidates
should be guided into looking for include general temperature levels (hot, warm, cool, cold), highest and
lowest temperatures and their months, annual range of temperature, and precipitation distribution during the
year (whether seasonal, all year or virtually none) along with significant months (such as the month with
highest rainfall). As usual with photograph based questions, some candidates in 4(a)(i) did not make any
use of the photograph. The reference to hot desert regions in the question triggered off a considerable
number of answers about characteristics of desert vegetation beyond those that could be observed in the
area shown on the photograph.

Comments on individual questions


Section A
Question 1
The style of question in (a)(i) was such that candidates could write letters in the spaces provided irrespective
of whether they knew the answers or not. The frequency of entirely incorrect answers from the less able
candidates suggested a dominance of guesswork. Perhaps the two which caused most problems were C
(used for sedimentary or folded rocks instead of igneous) and F (used for the example of folded rock instead
of metamorphic). For answering (a)(ii) wise candidates made some use of the diagram of rock types in (a)(i)
as a basis for illustrating mining difficulties caused by folding and faulting. Otherwise two-mark answers
were more common than four-mark answers from candidates successfully showing difficulties associated
with mining depth, especially if they tried to indicate depth below the surface. Unfortunately a few either
ignored the theme of geological factors, or did not know what was meant, and concentrated on showing
mining problems such as tunnel collapse instead. There were several possible routes to answering part (b)
which led to many successful answers. The most frequent references were to precious metals, size of the
deposits, high export value and employment.
Question 2
In (a)(i) candidates had the choice of Kurile and Californian currents, as the only two currents flowing in the
direction from the poles towards the Equator. Understanding the needs of (a)(ii) was rarely a problem for
candidates. Whether a candidate claimed both marks or only one of them depended on how complete their
answers were. The weakest answers came from candidates who took an over-narrow view, believing that
the minor currents flowing from the Kuro Siwo alone would carry the plastic pieces to Hawaii. The majority of
answers to (a)(iii) adequately conveyed how all the currents flowed towards the centre of the North Pacific
surrounding the area without an exit. Part (b) was dominated by two-mark answers, typically for stating nonbiodegradable and being eaten by marine organisms. Some other characteristics that had been expected to
be widely known, such as plastic floating and being light so that it is easily blown by the wind and carried by
the currents, were only occasionally referred to. Likewise part (c) was dominated by two-mark answers,
either for mentioning two ways instead of three, or for failing to elaborate more fully about any of the
individual ways. Question 2 seemed to be a question for which four and five marks were much easier to
reach than those worth eight marks and more.

564

General Certificate of Education Ordinary Level


5014 Environmental Management November 2010
Principal Examiner Report for Teachers
Question
BALOL 3
From those who knew all about the sunshine recorder there were many three-mark answers to part (a)(i).
The best answers to (a)(ii) were for day to day variations in cloud cover and / or type of weather. If the
candidate took a longer, more yearly, view the answer had to be about the Earth's revolution around the sun,
not its rotation. There were quite a few incorrect 'November' and '2.0' answers to (b)(i) for candidates who
sought to give the month that was most different instead of one that was most similar. Again, 'November'
was the most common incorrect answer to (b)(ii) instead of winter season. Recognising the sunshine peak
in July was the clearest indication of candidate understanding in (b)(iii); most candidates claimed this mark.
Higher sunshine hours in every month in Calgary was the starting point for most answers to (b)(iv). For the
remaining marks, candidates needed to support their answer either with good references to supporting
values, or by some broadening of answer scope. Some did this by referring to the limited solar potential in
both places in winter. The weakest answers came from those who either talked in general about the
advantages of solar power as an alternative energy source, or believed that the presence of sunshine hours
in every month made both places highly suitable for solar power.
Question 4
If candidates concentrated on description alone from the photograph in (a)(i) there was plenty for them to
describe for both nature and distribution, and they readily mentioned at least three of the points in the mark
scheme. It was those candidates who responded more to the mention of 'hot desert area' in the question
than to the command word to describe, and tried to use knowledge instead of observation, who did less well
and sometimes failed to claim even one mark. Part (a)(ii) was one of the best answered questions on the
entire paper with desert vegetation characteristics, such as large rooting systems, thorns and spines instead
of leaves, succulent stems and widely spaced plants, being directly related to survival in the hot and dry
climate. How much progress was made in part (b) depended upon whether the candidate focused on
livestock farming or crop cultivation. The split tended to be according to candidate ability. Some less able
candidates began their answers as if for pastoral, but then switched to the poverty of desert soils and the
need for irrigation water in a crop growing context.
Section B
Question 5
Part (a) was well answered. Most candidates recognised that the process of photosynthesis was the key to
answering (a)(i). Elaboration upon it led to the award of all three marks, which happened frequently. Parts
(a)(ii) and (a)(iii) were answered equally well and consistently, even by candidates who struggled with some
of the later parts of Question 5. Slightly less well answered was (b)(i). A majority of candidates claimed at
least one mark, often for 'provides a habitat for birds and animals', although this was expressed in an
enormous number of different ways. Claiming the second mark proved to be more difficult, either because of
the repetition of the same point in a different way, or through taking too narrow a view, often solely in relation
to plant life within tropical rainforests. Again in part (ii) full-mark answers were the norm, often from one
simple statement about the roots absorbing both water and nutrients from the soil. One-mark answers were
characteristic of candidates who gave similar answers for both bullet points: essentially related to roots
preventing soil erosion.
Part (c) was perhaps the least well answered part of Question 5. Even candidates who began well by
focusing their answers on the theme of location in (c)(i) tended to stop answering once they had stated one
different feature of the location of early and recent clearances; they had not taken into account that this was
a three mark question. One-mark answers were typically more vague, referring to locations in Brazil in terms
of 'north', 'south' and 'centre'. Part (c)(ii) was intended to be a more testing question. Candidates were
asked only to 'suggest reasons'. While the attractions of riverside locations for early rainforest clearance and
settlement were widely acknowledged, only more able candidates recognised how the location of recent
clearances reflected expansion from the economic core in the south east of the country.
Although parts (d)(i), (ii) and (iii) seemed to pose few problems for candidate understanding of what was
needed, most answers fell short of the amount of content needed for all marks to be claimed. In part (d)(i),
references to the ease of export for beef and soya beans gave a sound basis to many answers. Something
else was needed for the third mark, which is where the source information could have been more fully used.
For example, infrequent use was made of the enormous distances shown on the roadside sign. Also the
problems during the wet season that would be avoided by paving the road, given below it, were little referred
to. The wording of part (ii) allowed the candidate to take the view that economic reasons are 'not very
strong'. To be effective, explanation for this view needed to focus on short-term economic gain and long-term

565

General Certificate of Education Ordinary Level


5014 Environmental Management November 2010
Principal Examiner Report for Teachers
environmental
loss. In reality, many of the answers trying to support a 'not very strong economic' viewpoint
BALOL
were too environmental. It was much easier to explain strong economic reasons, simply because so much
evidence in the source information was supporting this view. Two-mark answers were the most common
outcome in part (d)(iii), typically from references to the economic value of logging and to the amount of
illegal logging that already exists and is likely to increase after improved access from paving the road. Only
candidates who searched more carefully for relevant information from the source achieved greater breadth in
answering, which in turn increased opportunities for including meaningful comment. The weakest answers
were those which were entirely devoted to description of environmental problems resulting from forest
clearances, this was not a relevant response. It was a common type of answer from less able candidates.
The mark scheme identified three unique features of tropical rainforests for answers to part (e), namely
greatest biodiversity, greatest mass (or highest primary productivity) and largest number of vertical layers.
This tended to be the order of frequency with which they were referred to by candidates. A few candidates
identified all three. More frequently, able candidates mentioned two of them, in which case some elaboration
or exemplification was needed for the third mark to be claimed. Weaker candidates were more likely to
describe forest characteristics without specifically identifying either great biodiversity or mass and achieved
enough for a one mark answer. Some answers were dominated by references to the rainforest's unique
uniformly hot and wet climate, which was not what the question really wanted.
More careful reading of the question set in (f)(i) would have stopped a good number of candidates from
writing about the activities listed in DF as well. While Examiners ignore irrelevant answers, candidates
penalised themselves by devoting too little time to describing activities AC. In weak answers, there was an
over-reliance on repeating 'small-scale' given in activity A and the general meaning of the term 'sustainable';
in other words, there was inadequate development of answers which applied to particular activities. Overall
two and three-mark answers were the more common, from candidates who examined each activity in turn
and described how their environmental impacts were low. There were so many different but equally valid
approaches to answering (f)(ii) that these two marks were readily claimed by most candidates. A minority of
candidates (often more able ones) answered in relation to low outputs and levels of development from these
traditional activities. The majority referred to outside pressures on the forests from population growth, the
need for increased food output and the drive towards economic development. In the significantly better
answers to part (f)(iii) two ways of sustainable logging were stated upfront. Most answers began by stating
how logging in Brazil was not sustainable; and some of them never reached the point of describing
sustainable ways. Weakest of all were answers based around the words illegal and legal. Many candidates
were really trying to answer their own question 'State how the logging taking place along the sides of the BR
163 in Brazil is different from sustainable logging of hardwoods', which is not the same as the one set. The
better answers to (f)(iv), worth three and four marks, came from those candidates who followed the order of
the question and began by giving a definition of ecotourism; best of all was when this included references to
both the natural environment and local people. These candidates were then in a better position to explain
how making money out of the forests was more likely to lead to both the forests and the lifestyles of local
communities being saved. Two-mark answers were more common in those that were dominantly economic.
Answers more about tourism in general instead of ecotourism in particular were worth little.
Considering the nature of the topic, the middle part of Question 1 was not as consistently well answered as
might have been expected. The more biological parts of the question in parts (a) and (b) were well
answered in comparison with the many map and written resource-based questions in parts (d) and (e).
Questions in the four parts of (f) demanded the application of candidate knowledge and understanding,
which the more able were always going to be able to answer better.
Question 6
A few candidates ignored part (a)(i) and left the diagram unlabelled, even when some of its component parts
were referred to in later answers. The funnel was best known; for the other component parts there was
some overuse of the labels cylinder, flask, beaker and jar, sometimes without any difference between the
inner and outer containers. This meant that two-mark answers were more common than those worth three
marks. In part (ii), reducing the chances of the collected rainwater evaporating was a much better answer
than maintaining the rain gauge's stability. For the answer to part (iii), candidates could refer to occasions
when the rainwater falls into an already calibrated glass jar, instead of the more usual situation in which the
contents of the inner container are emptied into a separate measuring cylinder. If they did the former, they
were more likely to omit to mention the need to place the cylinder on a level surface in order to take accurate
readings at eye level. Those candidates who knew nothing about measurement of rainwater amount
referred instead to the general siting advantages of the rain gauge. There was some repetition of answers
between parts (a)(iv) and (v), as referred to previously under the general comments. D was widely chosen
as the best site in part (iv), although for the second explanatory mark candidates needed to do more than

566

General Certificate of Education Ordinary Level


5014 Environmental Management November 2010
Principal Examiner Report for Teachers
just
state 'open area' or 'on grass', which were marked for them on the plan. In part (v), C was perhaps the
BALOL
easiest of the sites for which to explain non-suitability, but most candidates had few problems explaining
sites A and B as well. Those who left out site letters in their answers to (iv) and (v) lost some of the marks,
quite unnecessarily.
Completing the pie graph in part (b)(i) posed few problems. Most candidates included their calculations for
the number of degrees, although given the closeness of two of the raw percentages to one third and two
thirds, doing these calculations was not absolutely necessary in order to draw the pie graph to a sufficiently
high level of accuracy for claiming the marks. The world map in (b)(ii) was used better than the one in
Question 5(c), perhaps because the pattern shown was more familiar to candidates; two and three-mark
answers were typical here. For the area of formation mark, candidates needed to put together both the
ocean source areas and the fact that they are in or near the tropics (in other words, over warm ocean
surfaces). One without the other was the main reason for losing one of the marks here. A fuller answer to
any one of the three parts could have been used to claim the floating fourth mark. Some candidates charged
into the answer to (b)(iii) with many details of how people lose their lives during and after tropical cyclones,
without stating basic causes such as very strong winds, heavy rainfall leading to flooding and landslides and
storm surges along the coast. The better answers came from candidates who adopted the alternative
approach and gave information about the characteristics of tropical storms first, as a lead into effects on
people. Although the question was a consistent mark earner, two-mark answers were more common than
those worth three or four marks because of the lack of basic physical information about cyclones.
There was an easy location mark in (c)(i) for stating between the Tropic of Cancer and the Equator. The
other marks in (c)(i) and (ii) followed from recognising that Niger was a landlocked country (however stated).
The third mark was usually awarded to candidates who explained the disadvantages resulting from not
having direct access to aid supplies from the sea. Candidates who fared best when answering (c)(iii) were
those who had been well prepared, so that they knew what to look for when asked to describe the climatic
characteristics from a climate graph. Maximum marks were easily reached by candidates willing to support
general statements, such as wet summer by reference to months of the year, and hot all year by quoting
lowest temperature value and month, or the range of temperature values between highest and lowest (with
C added as the unit, of course). Without precise references to values and/or months, few answers were
able to reach more than half marks. Despite it being a basic syllabus requirement, candidates never find it
easy to name a climate type from a climate graph, although in the case of part (c)(iv) they had the map of
location as well to help. The most popular choice was Desert, while the correct answer of Savanna came as
a poor second. The amount of precipitation in the summer season was too high for desert to be correct.
Those who made good use of the climate graph were the ones most likely to gain both marks for their
answers to (c)(v). High temperatures received many mentions; they were of greatest value to the answer
when combined with references to high rates of evaporation. Those who totally ignored the climate graph, or
believed that 554 mm indicated desert conditions, were the ones most likely to miss out on any marks. While
the term 'drought' was understood by all in part (vi), some candidates tried to answer this question too
narrowly by referring only to the dry winter months and to what was happening within the average year.
Those candidates who viewed the question in terms of variations in amount of rainfall between one year and
the next gave the best and most successful answers, and there were many.
Answers to the three parts of (d) tended to be polarised between good and bad. When the answer to (d)(i)
had been focused on reasons instead of the effects themselves, most candidates were reluctant to restate
them in (d)(iii), where they were needed. Candidates who listed effects separately for Niger and Europe in
(d)(i) could reach a maximum of two marks. Most of those following the best approach to answering
commented on the greater severity in Niger as well. The stark differences with respect to livestock were
used very effectively by some. Answering the first part in the correct manner seemed to help candidates to
identify income per head and population increase, (with wide variations in the ways of expressing them), as
the two main factors responsible for the differences. Some stated both birth rate and fertility rate, without
any reference to income and development, which was not as good an answer. For those taking note of the
command words, the final part of (d)(iii) discriminated well between those who used population and income
with only a limited amount of elaboration and those who were able to convey the greater picture. In Niger it
was a matter of life and death from persistent poverty and rapid population increase, not allowing surpluses
to be accumulated in the good years for use in times of drought. In contrast, in Europe there was little more
than economic inconvenience as farmers were forced to use their stores of winter fodder to keep animals
alive.
The total mark for Question 6 was typically, even if not always, a few marks above that for Question 5.
Until the final part (d), all the other parts were answered well across the ability range. Less able candidates
picked up marks quite consistently, even if some of their answers lacked the depth and incisive focus to
claim all the marks available for those questions worth more than two marks.

567

BALOL
UNIVERSITY OF CAMBRIDGE INTERNATIONAL EXAMINATIONS
General Certificate of Education Ordinary Level

* 8 5 2 0 3 9 7 2 2 6 *

5014/11

ENVIRONMENTAL MANAGEMENT
Paper 1

May/June 2011
2 hours 15 minutes

Candidates answer on the Question Paper.


Additional Materials:

Ruler
Protractor

READ THESE INSTRUCTIONS FIRST


Write your Centre number, candidate number and name on all the work you hand in.
Write in dark blue or black pen.
You may use a soft pencil for any diagrams, graphs or rough working.
Do not use staples, paper clips, highlighters, glue or correction fluid.
DO NOT WRITE IN ANY BARCODES.
Answer all questions.
All questions in Section A carry 10 marks.
Both questions in Section B carry 40 marks.
At the end of the examination, fasten all your work securely together.
The number of marks is given in brackets [ ] at the end of each question or part
question.

For Examiners Use


1
2
3
4
5
6
Total

This document consists of 23 printed pages and 1 blank page.


UCLES 2011

[Turn o

568

BALOL
1

Section A

(a) Look at the table, which gives information about very strong earthquakes in Indonesia
since 1960.

(i)

years

number of very
strong earthquakes

1960 69

70

1970 79

5 000

1980 89

1990 99

2 500

2000 04

228 120

2004 09

12 100

For
Examiners
Use

approximate number of
deaths from earthquakes
in Indonesia

In which 20 year period were there no very strong earthquakes in Indonesia?


.............................................................................................................................. [1]

(ii)

How did the number of very strong earthquakes in Indonesia change after 1999?
.............................................................................................................................. [1]

(iii)

Was there a relationship between the earthquake strength and the number of
deaths in Indonesia between 1960 and 2009? Support your answer with evidence
from the table above.
..................................................................................................................................
..................................................................................................................................
..................................................................................................................................
.............................................................................................................................. [2]

(b) Indonesia lies along a destructive plate boundary. Describe how this explains the
occurrence of earthquakes in Indonesia.
..........................................................................................................................................
..........................................................................................................................................
..........................................................................................................................................
..........................................................................................................................................
..........................................................................................................................................
...................................................................................................................................... [3]

UCLES 2011

5014/11/M/J/11

569

BALOL
(c) What are the difficulties for rescue services in the first few days after a destructive
earthquake?

For
Examiners
Use

..........................................................................................................................................
..........................................................................................................................................
..........................................................................................................................................
..........................................................................................................................................
..........................................................................................................................................
...................................................................................................................................... [3]

UCLES 2011

5014/11/M/J/11

[Turn over

570

population (million)

2 BALOL
(a) The bar graph shows the total numbers of people living in homes without water and
without sanitation for both urban and rural areas of South East Asia.

(i)

200
190
180
170
160
150
140
130
120
110
100
90
80
70
60
50
40
30
20
10
0

without
sanitation

without
water
key:
urban population
rural population

What percentage of people without access to water in South East Asia live in rural
areas?
%

(ii)

For
Examiners
Use

[1]

Compare the total number of people living in homes without water with those
without sanitation in South East Asia.
..................................................................................................................................
.............................................................................................................................. [1]

(iii)

Describe how access to water and sanitation in homes differs between urban and
rural areas of South East Asia and suggest one reason for the difference.
difference ..................................................................................................................
..................................................................................................................................
reason ......................................................................................................................
.............................................................................................................................. [2]

UCLES 2011

5014/11/M/J/11

571

BALOL(iv) Most of the homes without water and sanitation in urban areas in South East Asia
are on the edge of the cities. Explain why this is so.

For
Examiners
Use

..................................................................................................................................
..................................................................................................................................
..................................................................................................................................
..................................................................................................................................
..................................................................................................................................
.............................................................................................................................. [3]
(b) In some areas, householders are encouraged to catch rainwater in plastic sheets and
to send it through pipes to a tank. What are the advantages and disadvantages of this
method of water supply?
..........................................................................................................................................
..........................................................................................................................................
..........................................................................................................................................
..........................................................................................................................................
..........................................................................................................................................
...................................................................................................................................... [3]

UCLES 2011

5014/11/M/J/11

[Turn over

572

1800

1800

1700

1700

1600

1600

1500

1500

1400

1400

1300

1300

1200

1200

1100

1100

1000

1000

900

900

800

800

700

700

600

600

For
Examiners
Use

methane in the atmosphere (ppb or parts per billion)

methane in the atmosphere (ppb or parts per billion)

3 BALOL
(a) Look at the graph, which shows amounts of methane in the atmosphere since 1600.

0
0
1600 1650 1700 1750 1800 1850 1900 1950 2000
year
(i)

How much methane (ppb) did the atmosphere contain in 2010?


ppb

(ii)

[1]

Describe the main trends in the methane content of the atmosphere since 1600.
..................................................................................................................................
..................................................................................................................................
.............................................................................................................................. [2]

UCLES 2011

5014/11/M/J/11

573

BALOL
(b) Describe the main sources of methane in the atmosphere.
..........................................................................................................................................

For
Examiners
Use

..........................................................................................................................................
..........................................................................................................................................
..........................................................................................................................................
..........................................................................................................................................
...................................................................................................................................... [4]
(c) (i)

Explain why many scientists are concerned about the levels of methane in the
atmosphere.
.............................................................................................................................. [1]

(ii)

Suggest why other scientists are not greatly concerned about methane in the
atmosphere.
..................................................................................................................................
..................................................................................................................................
.............................................................................................................................. [2]

UCLES 2011

5014/11/M/J/11

[Turn over

574

8
4 BALOL
(a) Describe the density of trees within the tropical rainforest and the characteristics of trees
in the canopy layer.

For
Examiners
Use

density of trees
characteristics of trees .....................................................................................................
..........................................................................................................................................
..........................................................................................................................................
..........................................................................................................................................
...................................................................................................................................... [4]
(b) Look at the photograph of tropical rainforest in Panama.

(i)

What evidence suggests that subsistence cultivation is taking place in this area?
..................................................................................................................................
..................................................................................................................................
..................................................................................................................................
.............................................................................................................................. [2]

UCLES 2011

5014/11/M/J/11

575

BALOL(ii) Clearance has affected the vegetation in the foreground of the photograph. State
two ways in which the vegetation which has re-grown differs from natural tropical
rainforest.

For
Examiners
Use

1 ...............................................................................................................................
2 ........................................................................................................................... [2]
(c) The government wants to stop clearance of the forest. Suggest reasons why subsistence
cultivation should be allowed to continue.
..........................................................................................................................................
..........................................................................................................................................
..........................................................................................................................................
...................................................................................................................................... [2]

UCLES 2011

5014/11/M/J/11

[Turn over

576

10

BALOL
5

Section B

(a) (i)

In the spaces, name the two weather instruments and state what each one
measures.

For
Examiners
Use

Instruments for measuring the weather

name

........................................................

..........................................................

measures ........................................................

..................................................... [2]

(ii)

What is similar about how the two instruments work and where they are sited?
work ..........................................................................................................................
..................................................................................................................................
site ............................................................................................................................
.............................................................................................................................. [3]

UCLES 2011

5014/11/M/J/11

577

11

BALOL
(b) Wind can be used as a power resource.

For
Examiners
Use

Costs of electricity generation in the UK in 2005 by power sources


power source

cost
(US cents per kilowatt hour)

fossil fuels
nuclear
wind
wave and tidal
solar
(i)

5
7
8
22
72

Draw a bar graph to show these values.

[4]
(ii)

Looking at the graph, how likely is it that further use will be made of wind power to
generate electricity in the UK in the future? Explain your view.
..................................................................................................................................
..................................................................................................................................
..................................................................................................................................
..................................................................................................................................
.............................................................................................................................. [3]

UCLES 2011

5014/11/M/J/11

[Turn over

578

12

BALOL(iii) In other countries the cost of generating electricity from these sources may be
different.

For
Examiners
Use

Suggest reasons for this.


..................................................................................................................................
..................................................................................................................................
..................................................................................................................................
.............................................................................................................................. [2]
(c) The strongest winds in the world occur in cyclones. Look at the section through a tropical
cyclone.
Section through a tropical cyclone

A
(i)

In which one of the locations labelled A, B and C on the section will the winds be
strongest?
Explain your choice of location.
location
explanation ................................................................................................................
..................................................................................................................................
..................................................................................................................................
.............................................................................................................................. [3]

UCLES 2011

5014/11/M/J/11

579

13

BALOL(ii) Where, when and why do cyclones form?


where .......................................................................................................................

For
Examiners
Use

when .........................................................................................................................
why ...........................................................................................................................
..................................................................................................................................
..................................................................................................................................
.............................................................................................................................. [4]
(iii)

Describe how very strong winds in cyclones can cause loss of life and injuries to
people.
..................................................................................................................................
..................................................................................................................................
..................................................................................................................................
.............................................................................................................................. [2]

(iv)

Cyclones which affect the Caribbean are called hurricanes. Look at the summary of
the 2005 hurricane season in the Caribbean.

named storms

expected
number per
year

actual
number in
2005

difference
between
expected and
actual number
in 2005

total number at all


strengths

10

26

21
(in 1933)

number at hurricane
strength

14

12
(in 1969)

number at highest
hurricane strength
(category 5)

previous
record
number

Complete the table by filling in the differences for the number in 2005 compared
with the average number expected.
[1]
(v)

Describe what the table shows about the 2005 hurricane season in the Caribbean.
..................................................................................................................................
..................................................................................................................................
..................................................................................................................................
.............................................................................................................................. [2]

UCLES 2011

5014/11/M/J/11

[Turn over

580

14

BALOL
(d) Information about the effects of four of the hurricanes in the 2005 season is given
below.

For
Examiners
Use

2005 Hurricane season in the Caribbean


Hurricane Katrina (USA)
A direct hit on the city of New Orleans left 1321 people dead and thousands homeless;
most were poorer people who did not have cars or the money to escape inland before
Katrina arrived. Production from oil and gas fields in the Gulf of Mexico was disrupted.
But the USA as a whole was little affected because the Gulf States contribute only 2 per
cent to the total economy.
Hurricane Rita (USA)
Millions of Texans from large cities such as Houston got into their cars and tried to go
north, further inland, to escape the threat of Hurricane Rita. They caused traffic jams
more than 150km long.
Hurricane Stan (El Salvador and Guatemala)
Very heavy rains set off mudslides and avalanches, killing up to 1000 people as homes
and villages were destroyed.
Hurricane Wilma (Mexico)
The country counted the cost in lost tourist dollars as thousands of visitors from overseas
were driven home early when Wilma destroyed beaches and hotels.
(i)

Give examples of the human costs of the 2005 hurricane season using the headings
social and economic.
social ........................................................................................................................
..................................................................................................................................
economic ..................................................................................................................
.............................................................................................................................. [3]

(ii)

Describe how the evidence from the 2005 hurricane season shows that, as usual
with climatic hazards, the poor (both people and countries) were more badly
affected than the wealthy.
..................................................................................................................................
..................................................................................................................................
..................................................................................................................................
..................................................................................................................................
..................................................................................................................................
.............................................................................................................................. [3]

UCLES 2011

5014/11/M/J/11

581

15

BALOL(iii) How can the negative impact of climatic hazards be reduced? Describe strategies
that can be used before the hazard happens.

For
Examiners
Use

..................................................................................................................................
..................................................................................................................................
..................................................................................................................................
..................................................................................................................................
..................................................................................................................................
..................................................................................................................................
.............................................................................................................................. [4]
(iv)

Is it ever going to be possible to stop people dying from the effects of climatic
hazards?
Give your views on this and explain them as fully as you can.
..................................................................................................................................
..................................................................................................................................
..................................................................................................................................
..................................................................................................................................
..................................................................................................................................
..................................................................................................................................
..................................................................................................................................
.............................................................................................................................. [4]
[Total: 40 marks]

UCLES 2011

5014/11/M/J/11

[Turn over

582

16
6 BALOL
(a) (i)

Water is essential for life on Earth. Why?


..................................................................................................................................

For
Examiners
Use

..................................................................................................................................
..................................................................................................................................
.............................................................................................................................. [2]
(ii)

Look at the diagram which shows part of the water cycle.


rainfall

A
B

C
P
Q

river

Name the water cycle processes at A, B and C.


A B
C

(iii)

[3]

Rock layers P and Q have different effects on water movement underground. How
and why are they different?
..................................................................................................................................
..................................................................................................................................
.............................................................................................................................. [2]

UCLES 2011

5014/11/M/J/11

583

17

BALOL
(b) Look at the information on world population and water use.

1000

2000

3000

4000

For
Examiners
Use

world water use


(km3 per year)

world population
(billions)

year
1960
(i)

2000

By how many times did world population increase between 1960 and 2000?
.............................................................................................................................. [1]

(ii)

By how many times did world water use increase between the same dates?
.............................................................................................................................. [1]

(iii)

Why is it very likely that world water demand will increase even more between 2000
and 2025?
..................................................................................................................................
..................................................................................................................................
..................................................................................................................................
..................................................................................................................................
.............................................................................................................................. [3]

UCLES 2011

5014/11/M/J/11

[Turn over

584

18

BALOL
(c) Look at the world map of water availability. It shows

areas already at risk from severe water shortages


countries expected to be at risk of severe water shortages by 2025, likely to be bad
enough to restrict water use.

For
Examiners
Use

Water shortages

Tropic of Cancer
Equator
Tropic of Capricorn

key:
areas in 2005 already at risk from
severe water shortages
countries expected to be at
risk from severe water shortages by 2025
Describe the distribution of areas and countries with water shortages (already and
expected by 2025).
..........................................................................................................................................
..........................................................................................................................................
..........................................................................................................................................
..........................................................................................................................................
..........................................................................................................................................
..........................................................................................................................................
...................................................................................................................................... [4]

UCLES 2011

5014/11/M/J/11

585

19

BALOL
(d) Most places at risk from severe water shortages have climates which are dry for all
or part of the year. One possible source of water for farmers in these areas is from
underground stores.

For
Examiners
Use

Look at the section across part of the Sahara Desert.


rain

limestone
plateaus

oasis

wadi
sand dunes
oasis

sandstone

clay

limestone

sand

(i)

On the diagram, shade or colour in the water-bearing layer of rock (the aquifer). [1]

(ii)

Give the source of the water and explain why the water is trapped to form an
underground store.
..................................................................................................................................
..................................................................................................................................
..................................................................................................................................
..................................................................................................................................
.............................................................................................................................. [3]

(iii)

Explain why oases (areas of cultivation and settlement) are located in the areas
shown and not in other areas such as the one labelled S.
..................................................................................................................................
..................................................................................................................................
..................................................................................................................................
..................................................................................................................................
.............................................................................................................................. [3]

UCLES 2011

5014/11/M/J/11

[Turn over

586

20

BALOL(iv) Some wells are better than others for reliability and low cost water supply. Look at
the diagram showing three wells.

For
Examiners
Use

B
C

well / borehole
present day wet season water level (water table)
present day dry season water level (water table)
permanent water bearing rock
Which well is the best for water supply? Why is it better than the other two?
best well
why ...........................................................................................................................
..................................................................................................................................
..................................................................................................................................
..................................................................................................................................
..................................................................................................................................
.............................................................................................................................. [3]

UCLES 2011

5014/11/M/J/11

587

21

BALOL
(e) Many environmentalists believe that there should be improved efficiency in water use for
irrigation, before farmers look for new supplies of water.

For
Examiners
Use

Look at the sketch showing channel (furrow) irrigation.


water
channels

canal

crops
(i)

How is water lost and wasted when this method of irrigation is used?
.................................................................................................................................
..................................................................................................................................
..................................................................................................................................
.............................................................................................................................. [2]

UCLES 2011

5014/11/M/J/11

[Turn over

588

22

BALOL(ii) An environmental problem which often results from using this method of irrigation
is salinisation.

For
Examiners
Use

Below is a partly completed flow diagram to show how and why the problem of
salinisation occurs.
How and why salination occurs
over-use of irrigation water in hot climates

...........................................................................

...........................................................................

a hard crust of salt forms on the ground surface

...........................................................................

...........................................................................

crops can no longer be grown on the land


Use the statements below to fill the four empty spaces and complete the flow
diagram.
high concentrations of salt accumulate around crop roots
moisture in the soil is evaporated
most crops cannot tolerate high levels of salt
salts are drawn up to the top of the soil
[3]

UCLES 2011

5014/11/M/J/11

589

23

BALOL(iii) Draw a labelled diagram to show a different method of irrigation, which makes more
efficient use of water than channel irrigation. Describe how it makes better use of
the water available.

For
Examiners
Use

..................................................................................................................................
..................................................................................................................................
..................................................................................................................................
..................................................................................................................................
.............................................................................................................................. [4]
(iv)

Is salinisation more or less likely to occur with this method than with channel
irrigation? Explain your answer.
..................................................................................................................................
..................................................................................................................................
..................................................................................................................................
.............................................................................................................................. [2]

(v)

When supplies of irrigation water run out, what other farming methods can be used
to make sure that crops can still be grown?
..................................................................................................................................
..................................................................................................................................
..................................................................................................................................
..................................................................................................................................
.............................................................................................................................. [3]
[Total: 40 marks]

UCLES 2011

5014/11/M/J/11

590

BALOL
UNIVERSITY OF CAMBRIDGE INTERNATIONAL EXAMINATIONS
GCE Ordinary Level

MARK SCHEME for the May/June 2011 question paper


for the guidance of teachers

5014 ENVIRONMENTAL MANAGEMENT


5014/11

Paper 1, maximum raw mark 120

This mark scheme is published as an aid to teachers and candidates, to indicate the requirements of
the examination. It shows the basis on which Examiners were instructed to award marks. It does not
indicate the details of the discussions that took place at an Examiners meeting before marking began,
which would have considered the acceptability of alternative answers.
Mark schemes must be read in conjunction with the question papers and the report on the
examination.

Cambridge will not enter into discussions or correspondence in connection with these mark schemes.

Cambridge is publishing the mark schemes for the May/June 2011 question papers for most IGCSE,
GCE Advanced Level and Advanced Subsidiary Level syllabuses and some Ordinary Level
syllabuses.

591

Page 2

Mark Scheme: Teachers version


GCE O LEVEL May/June 2011

Syllabus
5014

Paper
11

BALOL
SECTION A
1

(a) (i) 197089/90;

[1]

(ii) large increase;

[1]

(iii) accept any two supporting statements for either no or yes e.g.
no relationship because:
years without (very) strong earthquakes had 5000 deaths;
one (very) strong earthquake had only 70 deaths;
4 strong earthquakes had as many as 228 120 deaths/ the highest number of deaths;
9 strong earthquakes only resulted in 12 100 deaths;
also accept yes because e.g.
there were no deaths when there were no (very) strong earthquakes; there were very
many deaths / 228 120 when there were 4 (very) strong earthquakes;
also accept comparative statements

max 2

[2]

(b) plates are moving together / colliding;


Indo-Australian against Eurasian / Philippine;
subduction / denser / oceanic plate beneath lighter / continental plate,
causes friction;
stress / pressure builds up;
until rocks fracture / break / fault;
sudden movement;
max 3

[3]

(c) answers such as:


transport disrupted / roads / railways/ports/airports damaged / blocked/traffic jams;
bridges destroyed;
telephone lines down / difficult to communicate;
fires;
water pipes fractured / difficult to find water to put fires out;
need to / difficult to get heavy lifting equipment;
shortage of fuel;
max 3

[3]
[Total: 10]

(a) (i) 80 (accept 79 and 81);

[1]

(ii) more without sanitation/fewer without water;


180 million without sanitation compared with 100 million without water;

[max 1]

(iii) less access to water and sanitation in rural areas than urban areas;
Accept any sensible reason e.g.
expense of supplying rural areas where populations are more scattered/low density;

[2]

(iv) answers such as:


movement of people to towns without money/jobs;
squatter settlements;
which are unplanned;
squatter settlements grow rapidly;
local authorities cannot keep pace with/afford the need for services;

[3]

max 3

592

Page 3

Mark Scheme: Teachers version


GCE O LEVEL May/June 2011

Syllabus
5014

Paper
11

BALOL
(b) advantages such as:
cheap / can be afforded by the poor;
easy to do;
will reduce the degree of contamination;
disadvantages such as:
depends on amount of rainfall / problems in dry spells;
at least one advantage and one disadvantage.

max 3

[3]
[Total: 10]

(a) (i) accept any between 1745 1755 inclusive;


(ii) small(er)/slow(er) increase to 1750-1805*;
large(r)/ faster rise to 1995/6/78/9;

[1]

overall increase = 1
if neither of these
points are made

levelled off since 95/6/7/8/99;


* accept any dates between and including these years

max 2

[2]

(b) decaying organic matter/wetlands;


rice paddies;
grazing animals/cattle/deer/sheep/camels, up to 2 marks if developed
termites;
animal waste;
industrial air pollution;
landfill sites;
leaks from natural gas pipelines;
drilling;
coal mines;
biomass burning;
domestic sewage treatment;
(methane hydrate) released from the ocean;
melting permafrost;
max 4

[4]

(c) (i) greenhouse gas / contributes to global warming;


(ii) it is in small concentrations in the atmosphere;
it only stays in the atmosphere a short time;

1
2

[3]
[Total: 10]

593

Page 4

Mark Scheme: Teachers version


GCE O LEVEL May/June 2011

Syllabus
5014

Paper
11

BALOL
4

(a) close together/dense; = 1


tall trees;
buttress roots;
straight trunks;
thin trunks;
branch near / at the top;
crowns length greater than width / almost oval / almost rounded;
large leaves;
drip-tip leaves;
shallow roots;
not seasonal;
trees = 3

max 4

[4]

(b) (i) fire / smoke suggests forest clearance;


by shifting cultivators/slash and burn;
clearings/small patches have been cleared;

max 2

[2]

max 2

[2]

(ii) less dense / more open;


lower;
grasses;
bushes / smaller trees;

(c) answers such as:


it is not a problem where population density is low / many years
between clearances;
traditional way of life should be allowed / preserved;
small plots so easy for the forest to regenerate;
subsistence farmers have no/little knowledge of other types of farming / jobs;
close communities should not be split up;
max 2

[2]
[Total: 10]

594

Page 5

Mark Scheme: Teachers version


GCE O LEVEL May/June 2011

Syllabus
5014

Paper
11

BALOL
SECTION B
5

(a) (i) Wind vane wind direction


Anemometer wind speed
2 @ 1 mark
One correct, or both names without correct measures, or both measures without names
= 1 mark
[2]
(ii) Work both have arms rotated by the wind, fixed high up at the top of a long pole
Site in an open space / on top of buildings, away from the shelter of trees / buildings
At least one similarity for each needed for full marks.
3 @ 1 mark

[3]

(b) (i) Axes drawn and labelled (vertical for costs and horizontal for power sources), accurate
plots = 2 marks (at least two correct = 1 mark), overall appearance / effectiveness /
completeness.
4 @ 1 mark

[4]

(ii) Wind power is much cheaper than other renewables (wave, tidal, solar), wave/tidal is
about three times more expensive and solar nine times, but more expensive than longer
established sources such as fossil fuels and nuclear, although not by large amount with
fewer environmental effects.
Conclusion costs given suggest a greater likelihood of further use than from other
renewables in the UK.
Up to two marks for valid points made about relative costs of wind power.
One mark for overall comment relating to the question.

[3]

(iii) Costs of obtaining raw materials are different e.g. local oil in the Middle East, so fossil
fuel cost of production lower = one approach to answering up to 2 marks.
Renewables listed rely upon nature other countries will have more sun / less wind than
the UK, desert countries / tropics more and hotter sunshine = another approach up to
2 marks.
One mark for each of the two approaches also possible.

[2]

595

Page 6

Mark Scheme: Teachers version


GCE O LEVEL May/June 2011

Syllabus
5014

Paper
11

BALOL
(c) (i) B = 1 mark
Place where surface winds are being drawn in fastest to be sucked up by the strong
vertical currents to replace air that is rising
Minimum 1 mark about B; 2 marks for B possible for a full explanation.
Compared with other sites, C is calm = the eye where air sinks
A is further away from the centre where the cyclone is still in the process of developing
and intensifying.
Up to 1 mark.
Incorrect choice; give 1 mark for answers which attempt to explain why winds are so
strong in a cyclone.
[3]
(ii) Where over warm ocean surfaces in the tropics / near the Equator in Atlantic, Pacific
and Indian Oceans
When late summer when the sea water temperatures are at their highest (26C+)
Why surface heating causes warm air to rise, which draws in air towards it,
establishing a moving circulation in the atmosphere capable of developing into a cyclone.
1 mark for each.
4th mark for answers which include greater detail either overall or in one part.

[4]

(iii) Falling trees / buildings destroyed so that debris falls on / hits people, in coastal areas
strong winds increase wave height and flatten everything.
One of these basic points = 1 mark Elaborated / developed = 2 marks
(iv) Table values +16, +8, +2 (all three needed with or without + sign).

[2]
[1]

(v) More active / busier than ever previously known not just compared with the average, but
it exceeded all previous records, some of them set many years ago.
Basic description = 1 mark
Strong statement as above or good use of values = 2 marks
(d) (i) Social

[2]

people dead and homeless (Katrina and Stan), people forced to leave /
evacuated from homes / traffic jams (Rita)
evidence that poor people suffered most (Katrina)

Economic oil and gas fields output disrupted (Katrina)


tourist industry in Mexico badly affected (Wilma)
3 @ 1 mark for examples arranged under correct headings
However, maximum 2 marks in answers that include examples of costs placed under the
wrong heading.
[3]

596

Page 7

Mark Scheme: Teachers version


GCE O LEVEL May/June 2011

Syllabus
5014

Paper
11

BALOL
(ii) People rich people had cars and could use the strategy of moving inland from coastal
cities to avoid the worst effects of the storm; poor people left behind
Loss of life in poor / developing countries like Guatemala and El Salvador.
Countries Bad hurricane disruption in the Gulf States caused little damage to the
overall economy of a very rich big country like the USA. Whereas the Mexican economy
felt the loss of tourist dollars (foreign exchange income)
Further comment based on the evidence such as cost of repairing damage /
infrastructure or poor people more likely to live in places vulnerable to mudslides and
avalanches.
Three points made relevant to the question. Reserve 1 mark each for people and
countries.
3 @ 1 mark
[3]
(iii) Named syllabus strategies to reduce the negative impact of climatic hazards are;
improved forecasting, appropriate settlement patterns and buildings and disaster relief.
(For this question preparations are more important such as education about what to do
when the hazard is forecast, provision of shelters, trained emergency teams.)
One strategy only described or a list of strategies rather than description = 1 or 2 marks
At least two strategies described in relevant ways for the question = 3 or 4 marks

[4]

(iv) Positive views might refer to improved weather forecasting, education and organisation
and how they are being achieved e.g. with weather forecasting how the satellite study of
cyclones has greatly increased the accuracy of forecasting a few days ahead of a
coastal hit. Greater wealth and economic development improve education and increase
chances of allocating resources to allow preparations / readiness in advance.
Negative views might focus on the strength / intensity of cyclones and other climatic
hazards, beyond what is reasonable for even rich countries / people to cope with (e.g.
damage caused by hurricanes such as Katrina in USA); some suggest that global
warming is making climatic hazards more intense and more frequent.
Limitations imposed by poverty in many countries what can they do even when they
know that it is likely to happen.
Accept single or dual views / approaches to answering.
Some comment related to needs of question but without much development; limited
coverage; view to answer question may not be very clear. 1 or 2 marks
Fuller explanation; range of points used. Clear answer given to the question about
whether it is going to be possible to stop people dying from the effects of climatic
hazards. 3 or 4 marks
[4]
[Total: 40]

597

Page 8

Mark Scheme: Teachers version


GCE O LEVEL May/June 2011

Syllabus
5014

Paper
11

BALOL
6

(a) (i) Essential for plant growth, plants are the producers for humans and other species to
consume, human body needs...
Two points made along these lines. 2 @ 1 mark
(ii) A Transpiration / Evapo-transpiration / Evaporation
B Interception
C Infiltration
3 @ 1 mark

[2]

[3]

(iii) Rock layer P is permeable / porous rock with spaces to allow water to pass through it,
whereas Q is impermeable without empty spaces preventing water passing through.
Difference with use of terms or examples of permeable / impermeable rock types
= 2 marks
Difference established in general terms or specific about only one of the rock types
= 1 mark
[2]
(b) (i) 2 times / twice / double
(ii) 3 times (or its equivalents)

[1]
[1]

(iii) Answers to previous two questions show that demand for water is increasing faster than
growth in population i.e. people are consuming more water per head.
World population has already grown since 2000 and is expected to continue to grow
further by 2025.
Economic development means that people consume more water for domestic and
industrial purposes. More food needs to be grown for more people with consequent
demands for more water for irrigation.
Points made along these lines, drawn from at least two of the three separate sections
above for a full mark answer.
3 @ 1 mark

[3]

598

Page 9

Mark Scheme: Teachers version


GCE O LEVEL May/June 2011

Syllabus
5014

Paper
11

BALOL
(c) Areas with existing water shortages include western USA / northern Mexico, Spain and
Portugal, northern Middle East (Iraq, Syria etc.), northern China in the northern hemisphere,
and Namibia and Australia in the southern hemisphere.
Countries expected to have water shortages by 2025 are concentrated in Africa, Middle East
and South Asia, with a noticeable continuous east-west belt from Morocco to India. Also
there is a belt down the eastern side of Africa from the Horn of Africa to South Africa plus
some countries in West Africa.
Elsewhere only the occasional country is shown at risk only Peru in South America and
Poland added in Europe, and South Korea in Asia.
One or two points which describe the distribution, while overall coverage remains incomplete
and haphazard or vague. The worth of some fuller descriptive answers might be reduced by
locational inaccuracies. 1 or 2 marks
Wider range of accurate locational points given, which together provide a more complete
picture of the world distribution of water shortages. 3 or 4 marks
[4]
(d) (i) Layer of sandstone rock shaded in.

[1]

(ii) Rainwater on the high ground / mountain ridge (on the left), where the layer of sandstone
rock outcrops on the surface, sandstone rock is trapped between layers of clay,
permeable rock surrounded by impermeable rocks.
3 @ 1 mark

[3]

(iii) Oasis located where water bearing rock touches or is easy to reach from the surface,
whereas a downfold in the rock takes the water well below S, which is in the middle of an
area of sand with soils no use for cultivation, other disadvantages of sand dunes such as
constantly shifting in the wind.
Three explanatory points like these. 3 @ 1 mark

[3]

(iv) Good cases can be made out for both A and B, but definitely not C. Marks awarded for
reasons why, not choice of well.
A all year water supply; not too deep a well making it cheaper to build and to take out
water. But, soon at risk if water levels fall lower.
B all year water supply; cut deeper into the dry season water bearing rock for future
use / security. But deeper and more expensive to build; more power needed to draw
water up to the surface.
C only useful in the wet season; how useful is that, even if it is the cheapest method?
Three points made along these lines for a choice of A or B. 3 @ 1 mark

[3]

(e) (i) Water is lost from the canal and channels by evaporation, it is also lost into the ground
by seepage, so that only a proportion of the water is actually available to crops for
growth.
Understood and well explained = 2 marks
Some understanding e.g. reference to one way that the water is lost = 1 mark

[2]

599

Page 10

Mark Scheme: Teachers version


GCE O LEVEL May/June 2011

Syllabus
5014

Paper
11

BALOL
(ii) Order of statements;
moisture in the soil is evaporated
salts are drawn up to the top of the soil
high concentrations of salt accumulate around plant roots
most crops cannot tolerate high levels of salt
All four correct = 3 marks
Two or three correct = 2 marks
One correct = 1 mark

[3]

(iii) Syllabus method named is trickle drip irrigation which directs irrigation water towards
plant roots and does not water the surroundings; often enclosed pipes laid between the
lines of plants with holes for water flow only near plant roots.
Also widely known is clay pot irrigation with water stored in a clay pot protecting it from
evaporation and seepage, and small holes controlling the water outflows towards crop
plants.
Reserve two marks for the diagram written text only answers maximum 2 marks.
1 mark for the diagram itself
1 mark for labelling
2 marks for written answer describing how; these can also be claimed by full labelling on
the diagram.
For other less suitable methods of irrigation, such as use of sprinklers, credit diagrams
up to 2 marks. 2 mark max. for these.
[4]
(iv) Explanation for why it is much less likely that salinisation will occur because a higher
proportion of the water provided is being used by the plant; this will reduce water losses
from evaporation from the soil, so that fewer salts will be drawn up to the surface where
plants are being grown.
Well explained for method of irrigation used in part (iii) = 2 marks
Part explanation for method, or a more general explanation = 1 mark

[2]

(v) Possibilities:
Developing and using new crop strains which need less water such as drought resistant
varieties (e.g. dry rice instead of padi rice) or GM crops modified to grow with less water /
withstand risks of drought
Replace traditional varieties of crops with high yielding varieties so that as much or more
can be produced from a smaller area of irrigated land
Different farming techniques better adapted to dry environments such as dry land
farming
Either one method well elaborated upon, or two or more methods identified with more
limited development.
Method identified = 1 mark
Elaboration = up to 2 marks

[3]
[Total: 40]

600

General Certificate of Education Ordinary Level


5014 Environmental Management June 2011
Principal Examiner Report for Teachers

BALOL
ENVIRONMENTAL
MANAGEMENT

Paper 5014/11
Paper 1

Key messages

Only short answers are expected to the four 10 mark questions in Section A. Beware of extending
answers beyond the lines left for answering. This increased the chances of candidates rushing the last
question and not answering in the same detail as they had in previous questions.
Likewise control the length of answers given to Question 5 Section B. Candidates can always go back
and add more once they have finished all of Question 6.
Read the questions carefully, read each question more than once and underline key question words
such as the command words, words which tell candidates what to do.
Question instructions most frequently ignored were Looking at the graph in 5(b)(ii) and Describe how
the evidence in 5(d)(ii).
Take careful note of the number of marks for the question. For 3, 4 and 5 mark questions it is not just a
matter of filling all the lines; it is highly likely that a variety of points needs to be made, or details about an
example given, instead of just repeating one idea.
Questions which suffered from limited candidate coverage in relation to number of marks available
included 5(b)(ii), 5(d)(iv) and 6(b)(iii), three or four mark questions to which many one mark answers
were given.

General comments
There were only minor variations in the quality of responses between the four short questions which formed
Section A. If anything, marks for Question 3 were slightly lower. Some candidates extended their answers
below the lines available for answering. While this was perfectly acceptable practice, candidates did not
always add greatly to their overall performance, often repeating points already made, without further
worthwhile elaboration or any use of examples. Also in a few cases spending too much time answering
Section A resulted in a noticeable decline in amount written in 6(e) Section B.
Despite the inevitable individual variations in performance between Centres and between candidates, total
marks for Question 5 and Question 6 in Section B were usually similar. In some cases a significantly lower
mark for Question 6 was a sign of mis-allocation of time. There is always some pressure to complete this
paper on time, which is why candidates must control their initial enthusiasm and not spend too much time on
the short questions in Section A. Questions left unattempted were few and far between, suggesting that
most candidates were comfortable with the topics covered.
Questions which were generally well answered by candidates included most parts of (a) in Questions 1 4,
5(b)(i)(drawing the bar graph), 5(d)(i) and (ii) (about the effects of hurricanes), 6(a)(i) and (ii) (about the
importance of water and the water cycle) and 6(a)(ii) and (iii) (the flow diagram for salinisation, and drawing
a labelled diagram showing either trickle drip or clay pot methods of irrigation). Conversely, the questions
which proved to be more difficult than average for the majority of candidates included 3(b) (sources of
methane in the atmosphere), 5(c)(ii) (about the formation of cyclones), 6(d)(ii) and (iii) (explaining the
underground store of water and the presence of oases in relation to the diagram) and 6(e)(v) (non-irrigation
farming methods for crop growth such as drought resistant seeds).
Candidates who followed carefully the themes of the questions were the ones most likely to give successful
answers. Often the following questions within the lettered sub-sections (a e) continued the same theme,
for example, question 5(b) with a graph showing relative costs of different power sources in (i). Relative
costs was the basis of the questions set and for the answers needed in (ii) and (iii). Some candidates lost
sight of costs and referred more to other advantages and disadvantages of these power sources. Answers
to 5(b)(i) and (ii) fed into the answer needed to (iii). Showing the position of the aquifer on the diagram in
6(d)(i) was intended to make it much easier for candidates to answer (ii) and (iii); however, a good number

2011

601

General Certificate of Education Ordinary Level


5014 Environmental Management June 2011
Principal Examiner Report for Teachers
of
candidates never used the aquifer again in their answers, having not realised its importance in answering
BALOL
why surface water was available allowing farming in some places and not others.
Many questions required the use of resource information supplied. The better the candidate used the
sources, the more likely the success of the answer. Using relative costs from the bar graph in 5(b)(i)
provided the basis for answering (b)(ii); only a minority quoted or used any values from the graph. Likewise
in the best answers to 5(d)(ii), candidates made use of named examples from the hurricanes listed to
reinforce the general points made by them, obeying the question reference to evidence from the sources.
Answers to 6(c) should have only been taken from the resource supplied, the world map. Two groups are
shown on the map, areas already at risk from drought and countries expected to be at risk. Answers gaining
full credit were only possible with named references to both.

Comments on individual questions


Section A
Question 1
Parts (a)(i) and (ii) fulfilled their role as the easy starter questions, even though a few candidates gave one of
the two 10 year periods without any very strong earthquakes instead of the 20 year period requested by the
question. Likewise (a)(iii) caused few problems because the table contained plenty of supporting evidence
for both a relationship and for no relationship, although the amount available to the candidate was greater for
the latter. In (b), candidates did not develop their answers to explain how earthquakes occur, after having
made the basic point about colliding plate boundaries. The quality of the answers given to (c) depended
upon how well focused they were on question need. Instead of concentrating on difficulties, and in the first
few days after a destructive earthquake, many candidates described strategies for reducing loss of life, both
before and after the earthquake. This meant that fewer difficulties such as disrupted communications,
broken infrastructure and power shortages (however expressed) were referred to than should have been.
Question 2
Candidates found questions (a)(i) (a)(iii) easier than (a)(iv), probably because most of the answers could
be taken from the source graph. The best answers to (iv) were from those candidates who realised that
shanty towns are commonly located on the edges of urban areas in developing world cities. Even then,
many of the answers fell short of making all the three points needed for a full answer. In particular, there
were relatively few references to the speed of growth and the sheer scale of numbers, which make it difficult
for city authorities to keep pace, even when willing to try. Part (b) was quite well answered, with a majority of
candidates trying to satisfy the needs of the question by looking to give at least one advantage and one
disadvantage. References to cheapness and ease, and to problems in dry climates or during dry seasons,
were the relevant points most frequently seen. One often repeated disadvantage was that the rainwater was
acid and too contaminated to drink, which was somewhat surprising, especially when comparing rainwater
quality with that from surface and ground supplies.
Question 3
In (a) some candidates only referred to the post 1850 part of the graph, as if the graph showed the same
trend throughout the period since 1600. Part (b) was much less well answered, especially among
candidates who spread the blame for methane in the atmosphere no further than fossil fuels. Indeed many
of the answers were more appropriate for a carbon dioxide question on the same theme. Thus there was a
shortage of references to such as rotting vegetation, animal waste and landfill sites. After correctly stating
greenhouse gas in (c)(i), candidates struggled more with (c)(ii) because many seemed unaware that
methane was present in the atmosphere in relatively small concentrations.
Question 4
Stating the density of trees was the easy starter in (a). There needed to be a range of points made about
tree characteristics; in some answers there was not much progress beyond tall trees. Given the number of
characteristics that could have been identified, and candidates general familiarity with tropical rainforests,
this was something of a surprise. The most obvious piece of evidence for subsistence cultivation in (b)(i) is
the smoke. A few candidates gained further credit by stating its significance for shifting cultivation.
Alternatively, others used the photographic evidence of small clearings. Most candidates observed the
photograph well enough to be awarded full credit in (b)(ii). Some candidates appreciated the needs of (c)

602

General Certificate of Education Ordinary Level


5014 Environmental Management June 2011
Principal Examiner Report for Teachers
better
than others, by explaining how and why subsistence cultivation can be sustainable and has been in
BALOL
some areas for thousands of years. Other answers were more descriptive of subsistence cultivation without
really addressing the question.
Section B
Question 5
Most candidates were awarded at least partial credit for (a)(i) and (ii), about the weather instruments used to
record wind. They seemed to find it easier to describe similarities in site in (a)(ii) such as in an open space,
on the top of a building and away from obstructions like trees than to state what was similar about how they
work. Expected answers for this included both with arms rotated by the wind, or both fixed high up at the
top of a long pole.
Full credit was commonly awarded in (b)(i). Many graphs were not only accurately completed, but neatly
executed as well, to give a good overall appearance. Using one large square of five small squares for 10 US
cents on the vertical axis led to the most accurate and best looking graphs. Some candidates used an
irregular scale for costs, typically with the scale for low costs larger than that for high costs; their graphs
consequently did not display the great size difference in costs of production between fossil fuels and solar.
Part (ii) was poorly answered by candidates who ignored the clear question instruction to look at the graph;
usually these candidates referred to the general advantages and disadvantages of wind power without any
reference to costs. Some candidates stopped too quickly after making only the basic point that wind power
is cheaper than the other two renewables shown in the graph. Fuller use of the graph, including relative
differences in cost such as solar power being nine times more expensive than wind, was the characteristic
which distinguished answers receiving full credit. Variations in climate and / or national supplies of raw
materials between countries were the key to the successful answers to (iii). If these were not used, answers
tended to skirt around the question, often referring instead to differences in level of economic development,
which were less relevant for this particular question.
The correct choice of B, supported by an explanation based on understanding, was restricted to more able
candidates in (c)(i). They knew that C in the eye of the storm was an area of calm and used this as part of
their choice justification. Part (ii) was perhaps the least well answered question on this years paper.
Precise knowledge and understanding of cyclones was restricted to a real minority of candidates. In (c)(iii)
candidates did not necessarily focus on strong winds. References to flooding were relevant only if placed in
the context of strong winds whipping up high waves in coastal areas increasing the flood risk on adjacent
coasts. Many of the wind based answers seemed to be more appropriate to what happens in a brief but very
fierce tornado, where warnings are more immediate and less reliable than for cyclones. Answering (iv) was
meant to be straightforward and it was. It was intended as a lead into (v) to make it easier for candidates to
recognise that 2005 was truly a record season. Somewhat surprisingly, some candidates did not come to
this conclusion, because they compared the difference they had worked out in the table with previous record
numbers stated in the final conclusion. They arrived at the erroneous conclusion that 2005 was not as bad.
Part (d) was generally well answered throughout. The weakest answers to (d)(ii) came from those
candidates who ignored the question instruction to Describe how the evidence.... These candidates gave
general answers about the importance of poverty in determining loss of life in climatic hazards, some of
which they then needed to repeat in the next two parts. Otherwise there were some really strong answers
from those who compared the effects of Hurricanes Katrina and Rita on rich and poor within the USA, or
compared the effects of Katrina and Wilma on the economies of the USA and Mexico, or recognised how
poverty played a part in the large losses of life from Hurricane Stan in developing countries like El Salvador
and Guatemala. The most successful answers to (d)(iii) were given by candidates who focused on the
question theme of climatic hazards. This made them more likely to refer to strategies that were relevant to
severe weather events, such as improved forecasting and provision of shelters with emergency food and
water supplies. Others who turned this into a question about tectonic hazards gained some credit for
referring to strategies useful when planning for all hazards, such as using trained emergency teams. It left
their answers short of references to sufficient relevant strategies for climatic hazards. As always with the
style of question used in (d)(iv), the explanation mattered more than the view expressed. Answers with
limited range or development, often with one point repeated and re-stated, were awarded little credit.
Whereas candidates who covered a wider range of points, such as suggestions that climatic hazards are
becoming stronger and more frequent due to climatic change, were able to access further credit.

603

General Certificate of Education Ordinary Level


5014 Environmental Management June 2011
Principal Examiner Report for Teachers
Question
BALOL 6
The better answers to (a)(i) referred to more than just humans. Those candidates who began with water as
essential for plant growth, and extended this in terms of plants as producers for other species to consume,
gained credit easily. Of the three water cycle processes indicated in the diagram in (ii) A (transpiration,
although evaporation was also accepted) was the best known; B (interception) was the least well known.
Candidates who recognised that P showed a permeable rock, while Q was impermeable gave the most
convincing and shortest answers to (iii). There was considerable variation in the quality of candidate
answers to (a), although overall it was quite well answered.
In (b)(i) and (ii) some candidates simply stated the answers, which was all that was needed. Others showed
the totals on which they were based. These were often more correct than the final result. For example, the
increase from 3 to 6 billion in world population led many to an answer of three instead of two. Some
candidates gave the number of times for one of the questions, and somewhat mysteriously stated the actual
difference for the other. More care in answering might have led to a higher percentage of accurate answers
here. These two questions were intended to set candidates up for answering (b)(iii). Often they did, but
only for incomplete answers about population increase causing more water use. The difference in ratio of
2:3 between population increase and water use was used by only a few. More detailed answers typically
included references to economic development with increased demands for water for irrigation and industrial
purposes.
The most common candidate approach to answering (c) was to describe first the distribution of areas already
at risk in 2005 and then to move on to the countries expected to be at risk by 2025. This was also the
approach which led to the fullest and most accurate answers, especially when specific references were
made to a range of countries and to different continents. The weakest answers came from candidates
unable or unwilling to refer to places, who attempted to describe locations only by reference to the Equator
and the two Tropics using the latitude lines drawn on the map. Their appreciation of the distribution was
shown to be very limited. As a result this question was another one which led to wide variations in the quality
of candidate performance and discriminated well in terms of candidates abilities.
Most candidates correctly identified the sandstone layer as the aquifer and shaded in the whole layer in
(d)(i); a few only shaded in that part of the layer below the oasis. Some also shaded in the surface sand
layer, which was not correct. Others shaded in one or both of the limestone layers; while this was
understandable on the basis that limestone is a permeable rock, their locations made them less suitable as
aquifers in this example. From many of the answers given to (d)(ii) it was clear that candidates understood
what an aquifer was and how it needed to be trapped in a layer of porous or permeable rock between two
layers of impermeable rocks, with access to rain for the permeable layer to be filled up. What many
candidates were unable to do was to apply their knowledge to this diagram. Few referred to the source of
rain shown; more did refer to the clay as the impermeable rock trapping the water, although by no means all.
When answering (iii), not all candidates referred back to the aquifer layer which they had shaded on the
diagram. A considerable number tried to explain the presence of an oasis in terms of nearness to surface
water flows down the mountain, instead of the closeness of the sandstone layer to the surface. The next
diagram was better used in the answers given to (iv). B was far and away the most common choice. In
good answers the advantages of B for the reliability of its water supply were contrasted with the limits of A
and C. Some candidates made a good case for A on the basis of ease and cost compared with B.
However, the small number of candidates who chose C were always going to struggle, and their answers
tended to confirm their limited understanding.
In (e)(i) candidates mentioned either evaporation or seepage into the soil, but more rarely both of them.
Reference to only one of them was sufficient for full credit to be awarded, provided that there was some
development or elaboration. From the amount of crossings out, very few candidates seemed to have made
just one attempt at fixing the statements into the flow diagram in (e)(ii). Judging by the number of reverse
arrows used, there were plenty of examples of candidates wishing to change their order. Both suggested
that candidates would have benefited from more study time before committing pen to paper. Many of the
solutions were eventually correct. The most common reversal was for high concentrations of salt
accumulate around crop roots to be placed on the second line above salts are drawn up to the top of the
soil, which was incorrectly relegated to the third space. Many successful labelled diagrams were drawn in
(e)(iii) for either trickle drip or clay pot methods of irrigation. A good diagram made justification easier and
there were many instances of the award of full credit. However, diagrams and answers based on some form
of channel irrigation were not worthy of credit because they were not allowed by the question. Between the
two extremes were answers based on use of sprinklers; usually some of the content could be rewarded,
although not as fully as for trickle drip irrigation. A good choice of irrigation method made it easier to answer
(e)(iv) well. In answers to (e)(v) the most common way suggested was some form of small scale irrigation,

604

General Certificate of Education Ordinary Level


5014 Environmental Management June 2011
Principal Examiner Report for Teachers
sometimes
BALOL down to garden level. However, the question was directed at new crop strains (19.1 in the
syllabus) and / or dry land farming (30.1). Candidates who used one or both of these gave the most
successful answers.
Overall the standard of answers given to Question 6 was comparable with that for Question 5, since the
topics examined seemed to be equally familiar to most candidates. Most candidates managed a group of
good answers to three or four consecutive questions once or twice during Question 6, which raised the
standard of their performance. As usual the strongest answers came from candidates able to maintain high
standards of answering throughout, with relevant answers related in length and depth to the number of
marks available.

605

BALOL
UNIVERSITY OF CAMBRIDGE INTERNATIONAL EXAMINATIONS
General Certificate of Education Ordinary Level

* 5 4 1 6 0 9 6 7 7 3 *

5014/12

ENVIRONMENTAL MANAGEMENT
Paper 1

May/June 2011
2 hours 15 minutes

Candidates answer on the Question Paper.


Additional Materials:

Ruler

READ THESE INSTRUCTIONS FIRST


Write your Centre number, candidate number and name on all the work you hand in.
Write in dark blue or black pen.
You may use a soft pencil for any diagrams, graphs or rough working.
Do not use staples, paper clips, highlighters, glue or correction fluid.
DO NOT WRITE IN ANY BARCODES.
Answer all questions.
All questions in Section A carry 10 marks.
Both questions in Section B carry 40 marks.
At the end of the examination, fasten all your work securely together.
The number of marks is given in brackets [ ] at the end of each question or part question.

For Examiners Use


1
2
3
4
5
6
Total
This document consists of 23 printed pages and 1 blank page.
UCLES 2011

[Turn o

606

BALOL
1

Section A

(a) Look at the photograph showing one use of solar power in a small settlement.

(i)

For
Examiners
Use

What is the solar power in the photograph being used for?


.............................................................................................................................. [1]

(ii)

Describe the power unit at the top of the post.


..................................................................................................................................
.............................................................................................................................. [1]

(iii)

Why do both units face in the same direction?


.............................................................................................................................. [1]

UCLES 2011

5014/12/M/J/11

607

BALOL(iv) What objections might people living here have raised if wind power had been used
instead of solar power?

For
Examiners
Use

..................................................................................................................................
..................................................................................................................................
..................................................................................................................................
..................................................................................................................................
.............................................................................................................................. [3]
(b) (i)

State one disadvantage of solar power.


.............................................................................................................................. [1]

(ii)

Explain the advantages of using solar power in rural areas of developing countries.
..................................................................................................................................
..................................................................................................................................
..................................................................................................................................
..................................................................................................................................
.............................................................................................................................. [3]

UCLES 2011

5014/12/M/J/11

[Turn over

608

4
2 BALOL
(a) Explain how malaria spreads in a population.
..........................................................................................................................................

For
Examiners
Use

..........................................................................................................................................
..........................................................................................................................................
..........................................................................................................................................
..........................................................................................................................................
...................................................................................................................................... [4]

65

65

60

60

55

55

50

50

45

45

40

40

35

35

30

30

25

25

20

20

15

15

10

10

malaria cases (thousand)

malaria cases (thousand)

(b) The graph shows the number of cases of malaria in South Africa from 1999 to 2008.

0
1999 2000 2001 2002 2003 2004 2005 2006 2007

2008

year

UCLES 2011

5014/12/M/J/11

609

BALOL Describe how the number of cases of malaria in South Africa changed between 1999
and 2008.

For
Examiners
Use

..........................................................................................................................................
..........................................................................................................................................
..........................................................................................................................................
..........................................................................................................................................
...................................................................................................................................... [3]
(c) In all malarial areas, the number of cases can vary from year to year. Suggest reasons
why.
..........................................................................................................................................
..........................................................................................................................................
..........................................................................................................................................
..........................................................................................................................................
..........................................................................................................................................
...................................................................................................................................... [3]

UCLES 2011

5014/12/M/J/11

[Turn over

610

6
3 BALOL
(a) Look at the sketch across part of China giving information about some problems in
Tibet. Use it to answer the questions below.
TIBET

For
Examiners
Use

CENTRAL CHINA

pastures and lakes


have dried up,
frozen ground has
thawed
4000
height above sea level
(metres)

Tibet
is 1.5C
warmer than
in 1960

rivers flow to
Central China
industrial
cities

west

not to
scale

east
key:

water enters unfrozen rocks


sources of rivers drying up

(i)

Explain how industries in Central China might contribute to rising carbon dioxide
levels.
.............................................................................................................................. [1]

(ii)

Explain how an increase of carbon dioxide in the atmosphere might lead to:
1

an increase in temperature;

..................................................................................................................................
..................................................................................................................................
.............................................................................................................................. [2]
2

a decline in pastoral farming in the semi-arid areas of Tibet.

..................................................................................................................................
..................................................................................................................................
..................................................................................................................................
.............................................................................................................................. [2]

UCLES 2011

5014/12/M/J/11

611

BALOL
(b) Using information from the diagram, suggest how central China may be affected by the
changes in Tibet.

For
Examiners
Use

..........................................................................................................................................
..........................................................................................................................................
...................................................................................................................................... [2]
(c) What difficulties does China face in cutting carbon dioxide emissions?
..........................................................................................................................................
..........................................................................................................................................
..........................................................................................................................................
..........................................................................................................................................
...................................................................................................................................... [3]

UCLES 2011

5014/12/M/J/11

[Turn over

612

8
4 BALOL
(a) The map shows the position of the Galapagos Islands, a National Park which is part of
Ecuador, and some information about them.

For
Examiners
Use

The Galapagos Islands have a tourist industry of great value


to the economy of Ecuador. Birds and animals, such as great
tortoises, not found anywhere else in the World, can be seen
there. In 2007 the United Nations listed the National Park as
one of its endangered sites.

Ecuador
Galapagos
Islands

South
America

1000

2000

3000

km
(i)

What is the approximate distance from the Galapagos Islands to the nearest point
on the coast of Ecuador?
. km

(ii)

[1]

Why are some of the birds and animals living in the Galapagos Islands found
nowhere else in the world?
..................................................................................................................................
.............................................................................................................................. [1]

UCLES 2011

5014/12/M/J/11

613

BALOL
(b) The table gives information about recent changes in the population numbers on the
Galapagos Islands.
total population in 2000

15 000

total population in 2010

30 000

number of inhabitants in 2010 who were born


in the Galapagos Islands

For
Examiners
Use

4 500

number of Galapagos inhabitants in 2010 who


are migrants
(i)

Calculate the number of migrants in the 2010 population and complete the table.
[1]

(ii)

Most migrants to the Galapagos Islands were from mainland Ecuador.


Suggest why people in developing countries, such as Ecuador, migrate from one
part of the country to another.
..................................................................................................................................
..................................................................................................................................
..................................................................................................................................
..................................................................................................................................
.............................................................................................................................. [3]

(iii)

Describe how ecosystems of small islands, such as the Galapagos, might be


damaged by large increases in population.
..................................................................................................................................
..................................................................................................................................
..................................................................................................................................
..................................................................................................................................
..................................................................................................................................
..................................................................................................................................
.............................................................................................................................. [4]

UCLES 2011

5014/12/M/J/11

[Turn over

614

10

BALOL

BLANK PAGE

UCLES 2011

5014/12/M/J/11

615

11

BALOL
5

Section B

(a) The Earth provides people with many useful natural resources in the atmosphere, on
the land surface, under the land surface and in the oceans.

For
Examiners
Use

atmosphere

ocean

land

Fill in the remaining boxes by naming two different examples of useful natural resources
for people from the atmosphere, land surface and oceans.
place

natural resources

atmosphere
......................................... .........................................
on the land surface
......................................... .........................................
under the land surface

rocks

minerals

oceans
......................................... .........................................
[3]

UCLES 2011

5014/12/M/J/11

[Turn over

616

12

BALOL
(b) An environmental organisation has attempted to measure the ecological footprint of
every country. The ecological footprint is the average amount of air, land, fresh water
and sea resources used per person in each country, measured in hectares. World
average is about 2 hectares per person.

For
Examiners
Use

Look at the world map showing the locations of countries with ecological footprints
greater and lower than the world average.
Ecological footprint of countries

above 6 hectares per person


4 6 hectares per person
2 4 hectares per person
under 2 hectares per person below
the world average
(i)

Describe the location of countries with greater than average ecological footprints.
..................................................................................................................................
..................................................................................................................................
..................................................................................................................................
..................................................................................................................................
..................................................................................................................................
..................................................................................................................................

UCLES 2011

5014/12/M/J/11

617

13

BALOL(ii) How is the distribution of countries with lower than average ecological footprints
different from that of countries which are greater than average?

For
Examiners
Use

..................................................................................................................................
..................................................................................................................................
..................................................................................................................................
.............................................................................................................................. [5]
(iii)

On the world map, clearly mark and name any two countries with different ecological
footprints, one above average and one below average.
[2]

(iv)

Give reasons for the different ecological footprints of these two countries.
..................................................................................................................................
..................................................................................................................................
..................................................................................................................................
..................................................................................................................................
..................................................................................................................................
..................................................................................................................................
..................................................................................................................................
.............................................................................................................................. [4]

(v)

A report in 2007 by another environmental organisation calculated that humans are


using 30% more resources each year than the Earth can replace.
Why is this use unsustainable? Explain referring to examples of natural resources.
..................................................................................................................................
..................................................................................................................................
..................................................................................................................................
..................................................................................................................................
..................................................................................................................................
.............................................................................................................................. [3]

UCLES 2011

5014/12/M/J/11

[Turn over

618

14

BALOL
(c) World population growth is a major cause of the unsustainable use of natural resources.

(i)

year

total world population


actual and expected (million)

1800
1850
1900
1950
2000
2050

980
1260
1660
2500
6160
9800

For
Examiners
Use

By how many times is world population expected to have increased in the 250
years between 1800 and 2050?
.............................................................................................................................. [1]

(ii)

Draw a line graph to show actual and expected world population numbers between
1800 and 2050.
World population growth

world population (million)

10,000

5000

0
1800

1850

1900

1950

2000

2050

year
[3]

UCLES 2011

5014/12/M/J/11

619

15

BALOL(iii) How does the graph suggest that pressure on the Earths natural resources will
continue to increase?

For
Examiners
Use

..................................................................................................................................
..................................................................................................................................
..................................................................................................................................
.............................................................................................................................. [2]
(d) Look at the population information for Nigeria, the country in Africa with most people.
total population
(million)
2005
2050 (expected)
(i)

127
250

birth and death rates


in 2005 (per 1000)
birth rate
death rate

39
18

population structure
in 2005 (%)
under 15
over 60

44%
5%

How many more people is Nigeria expected to have in 2050 compared with 2005?
.............................................................................................................................. [1]

(ii)

Calculate the rate of natural increase per 1000 in Nigeria in 2005.


.............................................................................................................................. [1]

(iii)

Describe how the population structure of Nigeria suggests that its population will
continue to grow for many more years.
..................................................................................................................................
..................................................................................................................................
.............................................................................................................................. [2]

(iv)

More widespread use of family planning would help to reduce the high rates of
population growth in Nigeria and many other countries in Africa, Asia and Central
America.
Explain why some countries have been slow to manage their population growth.
..................................................................................................................................
..................................................................................................................................
..................................................................................................................................
..................................................................................................................................
..................................................................................................................................
..................................................................................................................................
..................................................................................................................................
.............................................................................................................................. [4]

UCLES 2011

5014/12/M/J/11

[Turn over

620

16

BALOL
(e) Some people say that a new type of economics is needed one that puts a money
value on the services that natural ecosystems provide free for humans.
Look at some of the advantages for humans of conserving natural forests.

conservation of the
biodiversity of plant and
animal species

preventing
soil erosion

For
Examiners
Use

reducing
flooding

advantages of the conservation


of natural forest

(i)

Complete the spider diagram by adding three more advantages for humans.

[3]

(ii)

Explain why conservation of biodiversity of plant and animal species is important to


humans now and in the future.
..................................................................................................................................
..................................................................................................................................
..................................................................................................................................
..................................................................................................................................
..................................................................................................................................
.............................................................................................................................. [3]

(iii)

Why are people continuing to destroy and clear natural forests despite all these
advantages?
..................................................................................................................................
..................................................................................................................................
..................................................................................................................................
..................................................................................................................................
..................................................................................................................................
.............................................................................................................................. [3]
[Total: 40 marks]

UCLES 2011

5014/12/M/J/11

621

17
6 BALOL
(a) Rocks and minerals have many uses for people. Here is a list of nine useful rocks and
minerals.
bauxite
limestone
(i)

(ii)

coal

diamonds

oil (petroleum)

iron ore
phosphates

For
Examiners
Use

lead
uranium

From the list, choose the rock or mineral for each of the uses named below.
use

rock / mineral

concrete and cement

....................................................

plastics and synthetic fibres

....................................................

steel girders

....................................................

nuclear power

....................................................

[2]

Choose any two of the other five rocks and minerals in the list, which were not used
in answering part (i). Give a use for each of them.
rock / mineral
1 ...........................................

use
.............................................................................
.............................................................................

2 ...........................................

.............................................................................
.............................................................................
[2]

UCLES 2011

5014/12/M/J/11

[Turn over

622

18

BALOL
(b) Look at the diagram which shows rock formations in a mining area.

For
Examiners
Use

mining
town
mine

mines
surface

depth (metres)

100
200

300
C

400
500
key:

(i)

mineral bearing layer of rock

Name the type of mining used to take minerals out of rock layer A.
.............................................................................................................................. [1]

(ii)

Describe the methods of mining used to take minerals out of rock layer B.
..................................................................................................................................
..................................................................................................................................
..................................................................................................................................
..................................................................................................................................
..................................................................................................................................
.............................................................................................................................. [3]

UCLES 2011

5014/12/M/J/11

623

19

BALOL(iii) Explain why four mines are being used to take the minerals out of rock layer A,
compared with only one for rock layer B.

For
Examiners
Use

..................................................................................................................................
..................................................................................................................................
..................................................................................................................................
..................................................................................................................................
.............................................................................................................................. [3]
(iv)

All mining causes environmental problems. Would you expect the environmental
problems to be greater from mining rock layer A or B? Explain your answer.
..................................................................................................................................
..................................................................................................................................
..................................................................................................................................
.............................................................................................................................. [2]

(v)

When mining finishes at A and B, the mining company will need to look at rock
layers C and D. Describe how the problems for mining layers C and D are likely to
be greater than they were for A and B.
..................................................................................................................................
..................................................................................................................................
..................................................................................................................................
..................................................................................................................................
..................................................................................................................................
.............................................................................................................................. [3]

(vi)

Which rock layer would you expect them to mine first, C or D? Explain your answer.
..................................................................................................................................
..................................................................................................................................
..................................................................................................................................
.............................................................................................................................. [2]

UCLES 2011

5014/12/M/J/11

[Turn over

624

20

BALOL
(c) Cerro de Pasco is a mining town in the Andes of Peru. At a height of 4,380 metres
above sea level, mining is the only reason for the existence of the town. Silver, lead and
zinc have been mined here for over 400 years from a large open pit mine in the centre
of town. The town clings to the edges of the 380 metre deep pit, as the map below
shows. The mine produces 60,000 tonnes of lead and 150,000 tonnes of zinc a year
and reserves are plentiful. The streets of poor houses, with their corrugated iron roofs
black with mining dust, suddenly stop at the edge of the pit. Houses near the edge of the
pit show many cracks.

For
Examiners
Use

Cerro de Pasco

built up area
with 65.000 inhabitants

edge of city
OPEN
PIT
MINE

old city

lake

N
scale:

0.5

1.0

1.5

2.5

km
key:
(i)

waste heaps

Look at the map and its scale. Describe how it shows the large size of the mine.
..................................................................................................................................
..................................................................................................................................
..................................................................................................................................
.............................................................................................................................. [2]

UCLES 2011

5014/12/M/J/11

625

21

BALOL(ii) Describe the location of the mine.


..................................................................................................................................

For
Examiners
Use

..................................................................................................................................
..................................................................................................................................
.............................................................................................................................. [2]
(iii)

Suggest a reason for the large number of cracks reported in the houses near the
edge of the pit.
..................................................................................................................................
.............................................................................................................................. [1]

(iv)

Where does the waste from the mine go?


.............................................................................................................................. [1]

(v)

A health report in 2007 showed that over 90% of children and 80% of women of
child-bearing age had high blood levels of toxic substances like lead. Diseases of
lungs and heart were found to be common in older residents.
Explain how the mining here can cause great health problems like these for the
inhabitants of Cerro de Pasco.
..................................................................................................................................
..................................................................................................................................
..................................................................................................................................
..................................................................................................................................
..................................................................................................................................
..................................................................................................................................
.............................................................................................................................. [4]

UCLES 2011

5014/12/M/J/11

[Turn over

626

22

BALOL
(d) The mining company wants to increase the size of the open pit to mine in the area under
the old city. This will involve the destruction of the main church, historical buildings and
many houses.

For
Examiners
Use

There are two plans.


Plan 1 The big move

Plan 2 Local resettlement by the


mining company

Build a new town for 70,000


people 35 km away, along
the main road
Cost estimates range from
US$500 million to US$3500
billion; who will pay?
Expected time for doing this
1015 years

Build a new church, public


buildings and houses not far
from the mine
Cost estimates are US$5-10
million
Expected time for doing this
23 years

Views of residents

A
Growth of the mine should
be stopped until there is a fair
plan for everyone to live in a
healthy place.

B
The mining company
is only interested in shortterm profits, not sustainable
development.

C
The mine gives people
work, but the price in terms of bad
health and poor living conditions
is high.

(i)

What are the advantages of Plan 1 compared with Plan 2?


..................................................................................................................................
..................................................................................................................................
..................................................................................................................................
..................................................................................................................................
..................................................................................................................................
..................................................................................................................................

UCLES 2011

5014/12/M/J/11

627

23

BALOL(ii) How likely is it that Plan 1 will ever be put into effect? Explain your view.
..................................................................................................................................

For
Examiners
Use

..................................................................................................................................
..................................................................................................................................
..................................................................................................................................
.............................................................................................................................. [5]
(e) Some countries depend upon mineral exports for most of their income. One example is
Zambia, a poor landlocked country in Africa.
Zambia the country

Zambia minerals

population: 11 million
income per head: US$750
birth rate: 42 per 1000

Africas largest copper


producer
exports: copper 85% of total
platinum 10% of total
1 in 10 paid jobs in mining

World copper price


the London Metal Exchange
10000
9000
8000
7000
6000
US $
5000
per
tonne 4000
3000
2000
1000
0
Oct
2006

(i)

Oct
2008

How big was the difference in the copper price between October 2006 and 2008?
.............................................................................................................................. [1]

(ii)

A market stall holder in Chingola, the main town in Zambias copper belt, said
Everyone in town gets worried when copper prices fall in London.
Describe the likely effects of the big drop in copper price between 2006 and 2008
on local people living in Zambias copper belt.
..................................................................................................................................
..................................................................................................................................
..................................................................................................................................
..................................................................................................................................
..................................................................................................................................
..................................................................................................................................
..................................................................................................................................
.............................................................................................................................. [4]

UCLES 2011

5014/12/M/J/11

[Turn over

628

24

BALOL(iii) The main cause of the drop in world copper price was the recession in developed
world countries. Why would a producer of copper (used in electrical wiring) and
platinum (used in catalytic converters), located more than 12,000 km away like
Zambia, be so badly affected?

For
Examiners
Use

..................................................................................................................................
..................................................................................................................................
..................................................................................................................................
.............................................................................................................................. [2]
[Total: 40 marks]

629

BALOL
UNIVERSITY OF CAMBRIDGE INTERNATIONAL EXAMINATIONS
GCE Ordinary Level

MARK SCHEME for the May/June 2011 question paper


for the guidance of teachers

5014 ENVIRONMENTAL MANAGEMENT


5014/12

Paper 1, maximum raw mark 120

This mark scheme is published as an aid to teachers and candidates, to indicate the requirements of
the examination. It shows the basis on which Examiners were instructed to award marks. It does not
indicate the details of the discussions that took place at an Examiners meeting before marking began,
which would have considered the acceptability of alternative answers.
Mark schemes must be read in conjunction with the question papers and the report on the
examination.

Cambridge will not enter into discussions or correspondence in connection with these mark schemes.

Cambridge is publishing the mark schemes for the May/June 2011 question papers for most IGCSE,
GCE Advanced Level and Advanced Subsidiary Level syllabuses and some Ordinary Level
syllabuses.

630

Page 2

Mark Scheme: Teachers version


GCE O LEVEL May/June 2011

Syllabus
5014

Paper
12

BALOL
SECTION A
1

(a) (i) street light / lamp / light

[1]

(ii) photovoltaic,
panel divided into cells,
square panel at the top of a pole.
Any one.
(iii) to face the sun / catch the maximum amount of sunlight

[1]
[1]

(iv) ugly / unsightly / spoil the appearance of the neighbourhood,


noise from the wind turbines turning,
no power on calm days / days without wind,
possible dangers to birds,
interference with TV / radio reception,
danger from blades falling off in very high winds.
Any three along these lines.
3 @ 1 mark

[3]

(b) (i) high costs of installation,


high cost of electricity provided compared with from fossil fuels,
low output on days with much cloud / in places with short hours of daylight.
Any one.

[1]

(ii) Advantages in rural areas of developing countries include:


renewable / long-term energy source,
making use of a natural resource that is readily available in the tropics,
once installed cheap and easy to operate / little maintenance needed,
widely available in places remote from other power sources,
can be small scale and therefore local in its supply,
non polluting / causes no local environmental damage,
improves peoples' quality of life,
further elaboration about how it does this.
Any three along these lines.
3 @ 1 mark

[3]
[Total: 10]

631

Page 3

Mark Scheme: Teachers version


GCE O LEVEL May/June 2011

Syllabus
5014

Paper
12

BALOL
2

(a) female (anopheles) mosquito,


usually bites an affected person at night,
sucks blood containing parasites,
parasites transmitted to bloodstream of the next person it bites,
parasites multiply in the person's liver / blood,
mosquitoes breed in stagnant water pools / channels,
this is why malaria is an example of a water-bred disease.
Four points made along these lines.
4 @ 1 mark

[4]

(b) overall decrease in number of cases from 1999 to 2008,


supported by use of values e.g. from over 50,000 to under 10,000,
fastest / greatest decrease from 2000 to 2001,
supported by values e.g. almost 40,000 decrease,
slower / more gradual decrease from 2002 to 2008,
some ups and downs / variable numbers from year to year,
examples of increases such as from 1999 to 2000 / 2005 to 2006 / 2007 to 2008.
Can be all statements for full marks, but reward also references using numbers of malaria
cases and sometimes years, provided that they support the statement in line with the
suggestions / examples shown above.
3 @ 1 mark

[3]

(c) Suggested reasons for variations from year to year include:


weather / climate variations with more cases in wetter years / fewer in drier years,
related to extent of areas of surface water available for breeding.
up to 2 marks.
Human factors such as increased precautions such as sleeping under nets, new nets treated
with insecticide, spraying in and around homes, government programmes to spray or drain
breeding grounds, also possibly more use of anti-malarial drugs.
Also possible effects of political turmoil / wars / influx of refugees.
up to 3 marks.
Any combination of climate and / or human reasons.
Three valid reasons or two reasons with added explanation / elaboration.
3 @ 1 mark

[3]
[Total: 10]

632

Page 4

Mark Scheme: Teachers version


GCE O LEVEL May/June 2011

Syllabus
5014

Paper
12

BALOL
3

(a) (i) use of fossil fuels / coal is the main fuel for electricity production

[1]

(ii) 1 recognition that carbon dioxide is a greenhouse gas,


some explanation about how the greenhouse effect works such as reduces heat
loss to space / traps heat from the surface,
absorbs outgoing (long wave / heat) radiation from the Earth's surface.
Two explanatory points made along these lines. 2 @ 1 mark

[2]

2 higher temperatures increase rates of evaporation / dry out the soil,


pastures deteriorate reducing the carrying capacity of the land,
land deteriorates further due to overgrazing / soil erosion / desertification.
Some understanding = 1 mark
Good / fuller understanding of the likely relationship between higher evaporation and
poorer pastures = 2 marks
[2]
(b) diagram shows more water infiltrating the ground instead of surface run-off,
higher temperatures increase evaporation at the expense of run-off,
therefore rivers will carry less water to Central China,
leading to possible water shortages.
1 point needed from each part.
2 @ 1 mark
Alternative 1 mark answer based on Tibet warming up, leading to snow and frost melting and
possible floods.
[2]
(c) Suggestions for reduction difficulties include:
reliance on / heavy use of fossil fuels,
China's great economic growth,
China has large coal deposits of its own,
examples of advantages to a country of using its own natural resources,
cost of changing from the long established use of fossil fuels / difficulties of implementation,
expense of / lack of progress in developing alternative energy sources,
many Chinese unconvinced of need to reduce carbon emissions,
government view that developed countries who have polluted longer should share more of
the reductions.
Three points made along these lines.
3 @ 1 mark

[3]
[Total: 10]

633

Page 5

Mark Scheme: Teachers version


GCE O LEVEL May/June 2011

Syllabus
5014

Paper
12

BALOL
4

(a) (i) Any value between 850 and 1300.

[1]

(ii) Islands isolated from the continental mainland of South America,


in the middle of the (Pacific) ocean too far away from other land ecosystems.
One idea along these lines.
(b) (i) 25,500

[1]
[1]

(ii) Valid suggestions include:


push factors such as poverty, low standards of living, poor services and lack of work,
pull factors such as opportunities for more jobs, higher family income and improved
quality of life,
something related to the source such as mention of the great value of the tourist industry
in the Galapagos which suggests job possibilities,
some elaboration about why this tourist potential exists in the Galapagos islands,
similar approach followed to candidate's own choice of example related to developing
countries such as from rural areas to big cities,
some specific elaboration and / or exemplification.
Note likely maximum of 2 marks for just stating examples of push and pull factors,
especially if some are opposites of the others without a proper framework, or support by
references to real places.
Three valid points made in line with the above. 3 @ 1 mark

[3]

(iii) Possible ways of ecosystem destruction from large population increases:


habitat destruction for food supply, economic development and growth of settlements;
also need to exploit natural resources; illegal activities and exploitation in protected
areas such as National Parks, poaching and capture of native species of birds and
animals.
introduction of plants and animals from outside which damage / replace endemic
species; examples include rats and domestic pets such as cats etc.
pollution from humans and their activities both at sea and around the coast, such as
waste disposal, oil spills, and on land, such as litter, non-treated waste disposal,
landfill.
All the ways can come from one of the three sections if accompanied by appropriate
elaboration, explanation and /or exemplification. Or there can be mention of four clearly
different ways with the minimum elaboration and anything in between.
4 @ 1 mark

[4]
[Total: 10]

634

Page 6

Mark Scheme: Teachers version


GCE O LEVEL May/June 2011

Syllabus
5014

Paper
12

BALOL
Section B
5

(a) Atmosphere sunlight (solar power), precipitation (rainfall), oxygen, wind energy
On land surface soil, plants (vegetation), wild animals (fauna)
Oceans fish, salt, other minerals (notably oil & gas)
These are the likely examples; many others are acceptable provided that they are different
e.g. from plants, an example of a food and raw material (rice and rubber)
from animals, an example of a food producing and draught animal (chicken and camel)
from oceans such as tuna and sponges / pearls
Assess on the basis of being sufficiently different when individuals rather than groups are
named by candidate.
Basically half marks for each answer, but the final sub-section mark must always be a whole
mark. This might allow breathing space for the assessment of less obvious answers.
In general, 1 or 2 valid answers = 1 mark
3, 4 or 5 valid answers = 2 marks
all 6 valid answers = 3 marks
[3]
(b) (i) Overwhelmingly concentrated in three continents in the developed world / temperate
lands,
Europe, North America and Australasia (Oceania),
the highest of all above 6 per person are concentrated in North America and Australasia,
many countries in north and west Europe in the 4 to 6 per person range,
elsewhere above average are well scattered in Africa,
large block in east and south of South America,
but with some Middle Eastern countries showing up as well above average,
references to specific examples which contrast with neighbours e.g. UAE, Japan.
These are some of many possibilities.
For guidance only but most likely minimum 3 marks, maximum 4 marks for this part.
(ii) Countries overwhelmingly concentrated in two of the three developing world continents,
in Africa and Asia (and to a lesser extent Central & South America),
large continuous blocks of countries in south/east Asia and central Africa below average.
These are only the most obvious differences.
One mark reserved for a clearly identified difference.
For guidance only, but most likely minimum 1 mark, maximum 2 marks for this part.
Mark the worth of the answer overall since the two parts are closely related / not
mutually distinctive.
[5]
(iii) Countries need to be correctly located and named for the marks.
2 @ 1 mark

[2]

635

Page 7

Mark Scheme: Teachers version


GCE O LEVEL May/June 2011

Syllabus
5014

Paper
12

BALOL
(iv) Reasons for differences in ecological footprint levels include:
wealth and level of economic development; why these are associated with higher
levels of resource use per head for industry, transport, domestic consumption etc.
These are the reasons about which candidates are likely be able to write the most
and explain most successfully.
great size of the home resource base; around the oil rich Gulf countries (e.g. UAE,
Kuwait on world map); Australia and Canada are great mineral exporting countries.
countries with some difficult environmental conditions where high / comfortable living
standards are costly to achieve such as Middle East and Australia (desert heat and
water shortages), Canada and Norway winter cold and daylight shortage
reverse for countries with small footprints; the majority are poor subsistence / rural
based economies, too poor to afford to import resources needed for development.
The emphasis here is on giving reasons and explaining rather than giving a variety of
different reasons. Mark the worth of the answer on this basis.
Some reasons given, but simple and without development. Statements about the country
with the low footprint may merely be the reverse of those for the one with the high
footprint or differences may not be made clear = 1 or 2 marks
Fuller reasons which make clear the differences between the two countries in terms of
ecological footprint. For all four marks something specifically related to at least one of
the two identified countries is needed = 3 or 4 marks.
[4]
(v) Unsustainable because there are going to be fewer and fewer resources for future
generations to use (i.e. the prime need for a sustainable resource),
will get worse year by year as the size of the resource gap will widen / increase,
examples of unsustainable natural resources are fossil fuels and most other minerals,
further explanation about why they are non-sustainable in relation to their formation or
their value to humans or scale with which they are used or mineral 'life expectancies',
other normally sustainable natural resources remain sustainable only for as long as they
are used within the time needed for replenishment e.g. references to fish stocks etc.
Basic answer about non-sustainability = 1 mark (possibly worth 2 marks)
Explanation with reference to examples of natural resources = 2 marks
3 @ 1 mark
(c) (i) 10 times

[3]
[1]

(ii) All correct plots = 2 marks


Solid line used to link the points to 2000 and a broken line or line of different colour for
20002050 = 2 marks
Fall-back positions;
3 plots correct = 1 mark
Continuous line of the same type connecting all plots = 1 mark
[3]

636

Page 8

Mark Scheme: Teachers version


GCE O LEVEL May/June 2011

Syllabus
5014

Paper
12

BALOL
(iii) The fast growth trend from 1950 onwards is clear on the graph,
expected part of line remains as steep / population growth similarly rapid to 2050,
therefore even more people for the Earth's finite natural resource base.
Graph evidence used correctly and explained / understood in terms of Earth's natural
resources = 2 marks
Part answer = 1 mark
[2]
(d) (i) 123 million
(ii) 21 per 1000

[1]
[1]

(iii) Predominantly a young / youthful population,


almost half of them yet to reach child bearing / producing ages,
who will marry and have their own families continuing population growth.
Understood and well stated = 2 marks
Some recognition but not fully described = 1 mark

[2]

(iv) Factors which help to explain low take-up of family planning in countries:
poverty; national costs of implementation of programmes and policies
political; low priority, resources spent in other ways, weak government
religion; not normally part of state policy in Muslim and Catholic countries
attitude; socially, culturally and economically unacceptable
infrastructure; problems of implementation in remote regions
Explanatory points can be made following on from the factors suggested above.
One factor only; range of factors with little explanation; focus on individuals rather than
countries = 1 or 2 mark answers
References included to at least two factors that are clearly different; attempts to explain
the significance of factors; focus is on countries = 3 or 4 mark answers
[4]
(e) (i) Possible advantages of conservation of forests to be added to the spider diagram:
carbon store / plants take carbon dioxide out of atmosphere
potential sources of new medicines / crops from plants
home, food and materials source for indigenous tribes / people
products useful to outsiders such as Brazil nuts and natural rubber
oxygen supply to the atmosphere during process of photosynthesis
growth of tourism as an economic activity in conserved forests
used for selective logging (i.e. an example of something specific with forests
conserved)
Three advantages such as these.
3 @ 1 mark

[3]

637

Page 9

Mark Scheme: Teachers version


GCE O LEVEL May/June 2011

Syllabus
5014

Paper
12

BALOL
(ii) Has created unique species and living environments in harmony with a wide variety of
natural environments,
biodiversity is a genetic resource for humans,
wild plants and animals are the source of all cultivated crops and domesticated animals
used for human food supply,
the wider the genetic pool the more chance of finding new strains of crops for particular
purposes / to suit particular growing conditions,
many present day / likely future drugs are derived from natural plants,
specific examples to illustrate any of these.
3 explanatory points made along these lines.
3 @ 1 mark

[3]

(iii) Essentially because the short-term financial gains are so great,


money from logging, extracting minerals and use of cleared land for farming,
for economic development by governments in developing countries,
landless people able to have their own land to farm once forests are cleared,
population pressure leading to great use of forest resources for fuel etc.,
all the advantages are long-term and income from them is less obvious,
benefits of conservation are likely to be as great / greater for people living in other
countries / developed world which have consumed own forest resources.
Three points made along these lines which directly answer the question.
3 @ 1 mark

[3]
[Total: 40]

(a) (i) Limestone


Oil (petroleum)
Iron ore
Uranium
All 4 correct = 2 marks
2 or 3 correct = 1 mark

[2]

(ii) Bauxite aluminium


Coal electricity, heating / smelting
Diamonds jewellery (or individual items), industrial cutting
Lead batteries, cables, roofing
Phosphates fertilisers
Also allow the minerals which formed the answers in part (i) provided they were not used
by the candidate in (i) and they give another use which is correct.
Only one use needed for the mark; answers which list more than one use are credited as
well provided that all uses are valid.
2 @ 1 mark

[2]

638

Page 10

Mark Scheme: Teachers version


GCE O LEVEL May/June 2011

Syllabus
5014

Paper
12

BALOL
(b) (i) Opencast mining (or directly equivalent such as open pit mining).

[1]

(ii) Vertical shaft cut from the surface down to rock layer B,
horizontal tunnels made as cutters work along to remove mineral,
mineral (and waste) lifted up to the surface through the shaft,
further detail of mining processes such as mechanical cutters in modern mines.
Points made along these lines.
3 @ 1 mark

[3]

(iii) Easier and cheaper to build mines on the surface,


some description of techniques such as scraping off surface vegetation and soil and
then using giant cranes and diggers,
can target directly places where the mineral is most available,
technically more difficult to build and operate an underground mine,
layer B is quite deep at 200 metres which makes it more difficult,
much time taken up each day by miners getting to and back from work place.
Points made along these lines which show understanding.
Credit quality of explanation as well.
3 @ 1 mark

[3]

(iv) The answer A can be justified by extent of the damage on the surface from four mines
instead of just one underground mine; also choice can be explained with reference to the
techniques used, the scale of the operations and to the direct destruction of surface
vegetation and wildlife habitats. Whereas waste heaps from mine B will be piled up in
one place and the rest of area might be untouched.
An answer B is harder to support; deep mining in general is more dangerous for humans
but less damaging for the surface environment. Much of the dust and dirt remains
underground. A deep mine like this one is less likely to have other effects on surface
such as underground water polluting surface water courses, subsidence etc.
The two marks are for justification, not choice itself.
Well supported choice 2 marks
Some support 1 mark

[2]

(v) Greater problems for mining rock layer C:


deeper (more than 400m below surface compared with 200m)
mineral bearing rock layer is folded / upfolded instead of being horizontal
plus detail about the extra mining problems / expense these will cause
Greater problems for mining rock layer D:
more to do with its location than the slight folding
higher up on a steep part of the hill side with access problems this might cause
Comment along these lines. Maximum 2 marks for answers without valid points for both
C and D.
3 points @ 1 mark
[3]

639

Page 11

Mark Scheme: Teachers version


GCE O LEVEL May/June 2011

Syllabus
5014

Paper
12

BALOL
(vi) Greater mining problems at C perhaps partly offset by the greater thickness and length
of the mineral bearing layer; likely to be able to mine much more of the mineral from C
than from D.
Greater mining problems at D perhaps partly offset by being able to tunnel into the
mineral layer, and work from the surface instead of deep underground; this is easier,
safer and cheaper than deep mining.
There is not an obviously better answer here. All the marks are for explanation.
2 mark answers are ones in which explanation for one rock layer is followed by the
equivalent inferior / weaker position of the other.
1 mark for a narrow answer which explains for only the chosen mineral layer.
[2]
(c) (i) Best answer measure / use the scale to give some idea of pit size;
almost 2km / 1.9 km at its longest and almost 1.5km /1.4km at its widest.
Also credit attempts to calculate total area such as just under 3 sq.km.
Worth 2 marks; less well done 1 mark
Also worth at least one mark would be to use the map to compare the size of the open
pit mine with that of the built-up area of Cerro de Pasco; and suggest how it fills about
one third of the area within the city boundary.
The two approaches (measurement and description) can be mixed to make a
2 mark answer.

[2]

(ii) Mine is in middle of the built-up area / surrounded on all sides by the city,
it lies on the (northern) edge of the old city,
closer to the western edge / later growth has been eastwards from the mine.
2 points made along these lines.
2 @ 1 mark

[2]

(iii) Suggestions include from the blasts / dynamite used to break up the rocks in the pit for
mining, or mining extending outwards under some of the houses on the edge of the pit
(undermining them).
One valid suggestion along these lines.

[1]

(iv) Answered from the map waste heaps on the western / south western side just beyond
the edge of the built-up area.
[1]
(v) Toxic mining substances (like lead in this example) enter underground water courses,
may be present in water that people drink, are present in the waste tips where local
people may scavenge for pieces of metal, or where children may play, or where animals
that people later eat may search for food. Older residents have been exposed for a
longer time. References to the possibly polluted water in the lake.
Mining work creates dust such as blasting/dynamiting the rock, digging it out and
transporting it, especially from open pit mining where all goes directly into the air.
Everyone in Cerro de Pasco lives so close to the mine; in most mining settlements there
is separation between the mine and homes, but here the pit in the middle of town. This
helps to explain why the health problems are so severe here.

640

Page 12

Mark Scheme: Teachers version


GCE O LEVEL May/June 2011

Syllabus
5014

Paper
12

BALOL
Points made along these lines. A general answer about how mining causes health
problems can gain all four marks, but most are likely to make some use of the given
information for Cerro de Pasco. Mention of health problems associated with lead would
also be relevant.
Some explanation limited in range and/or development. Over-reliance on information
already provided = 1 or 2 mark answers
More developed answers; relationships between mining and health explained and
elaborated upon = 3 or 4 marks
[4]
(d) (i) The big advantage of Plan 1 is that for the first time it will separate people and mining,
which should bring massive benefits for the health and quality of life of the inhabitants.
The new settlement will still have easy access (to mine and elsewhere) because it will be
along the side of the main road. This is more likely to be a sustainable long-term solution
and then the mining company can extend as much as it needs to with resulting
economic benefits for it, for the local people and amount of work available, and for the
economy of Peru. Plan 2 seems to be a cheap, short-term option which benefits the
mining company and no one else.
(ii) Never / highly unlikely is the easier view to explain mainly on basis of cost;
the long time for completion is also a factor. Mining companies tend to have more power
than what might be best for workers / people. Acting in a sustainable way always costs
more money. Developing world full of plans never executed.
The good chance view would depend upon outside pressure groups or perhaps the
government stepping in and compelling the company to relocate people well away from
the mine. Higher world mineral prices could make the high cost of movement more
viable.
Credit the explanation rather than view expressed.
Mark the two parts together.
General statements, mainly simple and short, many strongly based on information
already given or from comments made without much candidate input or development.
Breadth of coverage with some balance between the two parts may lift the answer to the
top of this mark range.
[13 marks]
Precise statements, elaborated upon or explained; good range of points made;
reasonable balance between the two for full marks. One part exceedingly well answered
with little of worth for the other, up to 4 marks.
[45 marks]
(e) (i) US$5000 / more than double in 2006 that of 2008

[1]

(ii) Mine owners lay off workers when world copper prices fall,
spending power of workers reduced,
income for market stall holders, shop owners, service providers goes down,
less money in town to spend on improvements /maintenance,
contrasts with when mine work expands and investment pours in.
Basic points made about falling employment, incomes etc, but with limited
development about consequences for local people
[1 or 2 marks]
Developed points made with possible knock-on effects on local people recognised and
elaborated upon for a broader perspective
[3 or 4 marks]

641

Page 13

Mark Scheme: Teachers version


GCE O LEVEL May/June 2011

Syllabus
5014

Paper
12

BALOL
(iii) Type of points that can be made;
countries like Zambia are raw material producers,
their minerals are made into manufactured goods in the developed world,
part of the long established world / global pattern of world trade,
recession in Europe / North America meant lower demand for cars etc.,
fewer minerals needed and prices fell,
nothing to do with availability / amounts of copper mined in Zambia.
Some basic understanding shown = 1 mark
Understood and clearly explained = 2 marks

[2]
[Total: 40]

642

General Certificate of Education Ordinary Level


5014 Environmental Management June 2011
Principal Examiner Report for Teachers

BALOL
ENVIRONMENTAL
MANAGEMENT

Paper 5014/12
Paper 1

Key messages

Only short answers are expected to the four 10 mark questions in Section A. Beware of extending
answers beyond the lines left for answering. This increased the chances of candidates rushing the
last question and not answering in the same detail as they had in previous questions.
Likewise control the length of answers given to Question 5 in Section B. Candidates can go back
and add more once they have finished all of Question 6 if they have time spare.
Read the questions carefully, read each question more than once, and underline key question words,
such as the command words, words which tell candidates what to do.
Question instructions most frequently ignored were How does the graph .... in 5(c)(iii) and Describe
how the population structure ... in 5(d)(iii).
Take careful note of the number of marks for the question. For 3, 4 and 5 mark questions it is not
just a matter of filling all the lines; it is highly likely that a variety of points need to be made, or details
about an example given, instead of just repeating one idea.
Questions which suffered from limited candidate coverage in relation to number of marks available
included 5(b)(iv), 5(d)(iv) and 6(c)(v), four mark questions to which many one and two mark
answers were given.

General comments
Questions 1 and 2 were much better answered than Questions 3 and 4 in Section A. Despite this, only
occasionally, and for relatively few candidates, was there a noticeable difference between the total mark out
of 40 for Section A and the individual total marks out of 40 for each of Questions 5 and 6 in Section B.
Additionally, total marks for Question 5 and 6 were themselves usually similar; if anything, a slightly higher
mark for Question 6 was more common, the reverse of what has happened in most previous years papers.
Pressure of time to complete this paper was an issue only for a few candidates; however it was significant for
those who spent too long giving exceedingly full answers to the short questions in Section A. One or two
also did the same for Question 5. The unfortunate result was a marked decline in the amount written or
number of questions left unanswered in (c) to (e) of Question 6. Any extra credit gained from writing those
very full answers, often filling spaces below the lines left for answering, was unlikely to have offset the credit
unavailable in Question 6. Questions left unattempted were few and far between, which suggested that
most candidates were comfortable with the topics covered, apart from the ones mentioned in the second half
of Question 6 caused by candidates time mis-allocations.
Although there was a definite pattern of performance in Section A, in Section B there was less of a pattern
than previously. This made it more difficult than usual to identify questions which were either persistently
well answered by candidates, or consistently answered badly. The nearest to the former was drawing the
line graph in 5(c)(ii), for which the vast majority of candidates were awarded full credit. The nearest to the
latter was 5(b)(iv), for which candidates did not seem to know what to say; most resorted to using what was
given in the stem at the start of (b); e.g. High average amount of use of air, land, fresh water and resources
in developed countries and low use in developing countries was stated without any attempt to give any
reasons why. Otherwise, varying standards of performance on each question seemed to reflect the
understanding of individual candidates more than inherent question difficulty.
A variety of question resources was used. The better the use made of these, the better the answer. The
labels on the sketch in 3(a) provided useful starting points for answers to (a)(ii) and (b). Using the shape
and gradient of the line graph in 5(c)(ii) led to more effective answers in (c)(iii) than quoting population totals
from the table. The graph had the advantage of showing in a clear way the change in speed of population
growth from 1950, by the marked change in steepness of the line, a key point referred to by only a few in
answers to (c)(iii). In 5(d)(iii), birth and death rates could show continued population growth into the future,

643

General Certificate of Education Ordinary Level


5014 Environmental Management June 2011
Principal Examiner Report for Teachers
but
this question asked for population structure to be used; the information needed for answering was in the
BALOL
next box in the table. Answers to 6(b)(v) and (vi) could only be taken from the diagram of rock formations.
Candidates who looked for more than one piece of evidence gave the best answers. Although most
commented on the depth of layer C, many fewer referred to its folds in (v), and even fewer referred to the
size and thickness of this mineral bearing layer compared with all the others shown, in answers to (vi).

Comments on specific questions


Section A
Question 1
Parts (i) (iii) of (a) fulfilled their role of gentle starter questions. Electricity alone was insufficient in (i) and
more than just solar cells was needed for description in (ii). The disadvantages of wind power needed in
(iv) were well known, and most candidates made points that were sufficiently different for them to be
awarded full credit. One objection quite frequently mentioned in answers, needing more space than solar,
was not considered valid in the context of this question. Mentioning it, however, did not prevent a candidate
from gaining full credit, provided that three other acceptable objections were stated. High cost on its own
was not enough in (b)(i) until something to qualify it, such as installation, was added. A long list of
advantages was considered acceptable in (b)(ii). Sometimes explanation was inadequate, but many
candidates answered by using a range of points to secure full credit. Most candidates were very comfortable
answering the questions and on average performed very well.
Question 2
How malaria spreads was well known to the majority of candidates in (a). Some good answers fell short of a
complete explanation. In a few cases, candidates lost their way by believing that malaria was a waterspread disease, passed on to other people through contaminated water. There were so many different
points that candidates could validly describe from the graph in (b) that reaching a total of three, with the
support of relevant values and dates from the graph, proved to be a relatively easy task. Part (c) proved to
be more testing. Some did not focus upon variations from year to year; instead these candidates looked at
variations between places and countries with different climates. Others gave entirely climatic or entirely
human prevention answers, which limited the breadth of their responses. Some tried to structure their
answers around preventative measures, but without either mentioning specific measures or by referring to
some that are not at present possible for malaria, such as vaccination. Those who gave answers of the type
expected by the question typically referred to differences between wetter and drier years, and to variations in
how seriously precautions are pursued, such as spraying, draining stagnant water bodies and provision of
insecticide treated nets. Again, candidates appeared to be comfortable throughout with this question.
Question 3
Mention of fossil fuels in general or coal in particular was necessary for the award of credit in (a)(i).
Candidates who began their answers to (a)(ii) by stating that carbon dioxide was a greenhouse gas usually
went on to give the most complete and accurate answers about why the world is heating up. Similarly, those
who began their answers to (a)(iii) with references to how higher temperatures increase rates of evaporation
and dry out the soil, then found it easier to explain the decline in pastoral farming in areas that were already
semi-arid. Once again, those who began their answers to (b) by stating what the diagram showed, such as
factors for reducing the run-off into rivers flowing to Central China, were the ones most likely to receive full
credit. In (c), some candidates showed awareness of the great importance of coal burning in China.
However, that was not essential knowledge, since there were many other points that could be made: in
relation to the importance of industry to economic growth and development in China, to the high costs of
replacing fossil fuels by alternatives that apply everywhere, and to the belief in developing countries that they
should not be made to pay for the damage already done by developed countries.

644

General Certificate of Education Ordinary Level


5014 Environmental Management June 2011
Principal Examiner Report for Teachers
Question
BALOL 4
Any value between the maximum and minimum distances of 1300 and 850 km was credited in (a)(i). Most
answers to (i) were correct; in contrast, few answers to (ii) were credited. Instead of trying to suggest why
much of the wildlife in the Galapagos was unique on the basis of an island location separated by too great a
distance from the nearest landmass, most gave answers about their preservation in the National Park.
There were many answers of 10,500 in (b)(i) rather than the correct answer of 25,500, after having used
total population for 2000 instead of 2010 as the starting point for their calculation. Part (b)(ii) was the best
answered part of this question; most answers included both push and pull factors. Additionally some made
use of the introductory information in the question about the growing tourist industry in the Galapagos. It was
rare for candidates not to be awarded at least partial credit for answering (b)(iii), typically either for
references to different types of pollution, or for the use of more and more land at the expense of vegetation
and wildlife. Answers receiving full credit gave a wider range of precise references, which sometimes
included other valid points such as poaching or the introduction of exotic plants and animals. These answers
had greater breadth.
Section B
Question 5
The three natural resources named most frequently in (a) were sunlight, plants and fish (but with some
variations in wording). The most commonly named resource that was not credited was water for oceans,
unless its use was specified say for transport or for desalination into fresh water. Fish, salt and minerals
such as oil and gas were considered to be the best answers for oceans; any others were assessed on their
merits.
There were immense variations in the quality of candidates answers given to (b)(i) and (ii). Answers
awarded little credit gave location and distribution examined only in terms of nearness or otherwise to the
Equator and tropics, and/or in terms of the North-South dividing line between the developed and developing
worlds. For further credit answers included names of continents and countries. In order to gain full credit
there needed to be more detailed references to locations within continents, or by a good use of the scale
separating out locations for the very high footprints above 6 from those between 2 and 6. In (b)(iii)
candidates most frequently marked and named their home country plus either the USA or Australia, although
the choices made were almost limitless. A few candidates (from quite different ranges of ability) merely
labelled above average and below average on the map without adding any country names. Most
candidates only received partial credit for (b)(iv), usually for stating some of the differences between a
developed and a developing country, but without starting to give any reasons for these differences. Bearing
in mind that the values quoted were per person, some candidates did make perceptive points about how
countries with very large populations such as India and China inevitably had low footprints per head because
of numbers; this was despite recent progress towards higher levels of economic development. Good
reasons to explain the differences, which were given by only a small minority of candidates, included higher
levels of economic development leading to greater resource consumption per head for industry, transport
and domestic purposes, the great size of the home resource base as in the oil rich countries of the Middle
East, and the subsistence rural based economies still dominant in many developing countries. Part (b)(v)
was much better answered, especially when candidates referred to the fossil fuels and their short life
expectancies compared with the millions of years needed for their formation. The concept of sustainability
was well understood; however, without useful references to such as fossil fuels, minerals or overfishing
further credit was often not awarded.
Perhaps as many as one in three of the candidates stated the difference in millions (8820) instead of the
number of times greater (10) when answering (c)(i). The line graph in (ii) was usually accurately drawn and
full credit awarded. Some candidates displayed even higher levels of understanding by showing the
expected line between 2000 and 2050 in a different way from the line showing known population up to 2000.
This was not essential, but it did allow candidates some leeway should one or more of their values not have
been plotted slightly accurately on the graph. A very small number of candidates incorrectly drew bar
graphs. In (c)(iii) there was often a lack of evidence of graph use; a question requirement. What the
candidates own line graphs clearly showed was the fast and persistent population growth from 1950, in
marked contrast to earlier speed of growth. Most candidates relied instead on stating values from the table.
Part (d) was generally well answered throughout. 123 million in (i) and 21 per 1000 in (ii) were easy enough
to work out from table data, although a few candidates made mistakes. In order to gain full credit in (d)(iii)
candidates needed to state that Nigeria was shown to have a dominantly young population under 15, who
were coming up to reproductive age and could soon be expected to start their own families. Some

645

General Certificate of Education Ordinary Level


5014 Environmental Management June 2011
Principal Examiner Report for Teachers
candidates
BALOL instead referred to birth and death rates; these do suggest that the population will continue to
grow for many more years, but the question focus was population structure. Many full answers were given to
(d)(iv). In the best answers, candidates looked for a range of factors to explain high birth rates; some
increased answer quality further by giving named examples of countries to illustrate the factors referred to.
Less able candidates tended to repeat just one or two factors, often expressed in an imprecise way, such as
lack of education and lack of family planning. A few candidates tried to approach the answer by using an
example of a country with a population policy (usually China) or by stating what was different in developed
countries. These answers tended to skirt around the question set and were mostly unsuccessful. The fact
that the question mentioned Africa, Asia and Central America meant that answers needed to be focused on
developing world countries, preferably without successful population policies.
A lot of the advantages suggested by candidates for the spider diagram in (e)(i) were really already covered
by labels already given. One common label was preserving habitats (or similar), already covered by
conservation of the biodiversity of plant and animal species. The three that were acceptable and most
commonly used by candidates were carbon store, oxygen supply and growth of eco-tourism (however
expressed, and they did come in many different forms). There were another four suggestions in the mark
scheme, which meant that candidates had plenty of potential choices. The most successful answers to
(e)(ii) came from candidates who homed in on the question theme of biodiversity. Less successful were
those who gave broader answers referring to the advantages of conservation of the forests, which merely led
to repetition of answers already given in (e)(i). Some answers to (e)(iii) were not much more than a list of
uses of land after forest clearance. Much better were answers from candidates who created a more
structured framework around reasons such as financial gain for big companies, national needs for economic
development and great population growth.
Varied was a suitable general summary for the performance of most candidates when answering Question
5. However, the distribution of these strong and weak answers varied greatly between individual candidates,
often even those from within the same Centre. The two exceptions were consistently better responses to
1(c)(ii) and worse in 1(b)(iv).
Question 6
The two uses which virtually all candidates knew in (a)(i) were limestone for cement and concrete and
uranium for nuclear power. The least well known was oil (petroleum) for plastics and synthetic fibres.
Bauxite was the mineral option wrongly substituted most often for oil. In (ii) candidates who chose coal, and
those who used oil because they had not already used it in (a)(i), needed to write more than simply a source
of energy to be awarded credit. Bauxite was widely used in this part by those who did know that it was the
raw material for aluminium.
The term opencast mining was the almost universal answer to (b)(i). A few candidates continued with
opencast mining into the next part, (b)(ii). However, deep shaft mining, and the methods employed, were
widely known. A good number of the answers given to (b)(iii) were more descriptive of methods of mining,
than explanatory for why it was easier and cheaper to use four mines on the surface as opposed to only one
underground. The choice of A in (iv) made this question much easier to answer. The specification of
environmental problems in the question largely cut out the choice of B. Human problems are greater than in
A, but apart from dangers of land subsidence and pollution of ground water supplies, environmental
problems are usually less noticeable in underground mining. Part (b)(v) was effectively answered by most,
especially those who recognised that the mineral bearing seams were more folded. In (b)(vi), the choice of
C could be justified by the thickness of the seam, which was likely to be sufficient to offset the higher costs of
mining deep; the choice of D could be explained by ease of access into the mineral bearing seam from the
surface, despite the steep slopes. Both lines of reasoning were regularly used. A significant number of
candidates, however, gained no credit because of unrealistic explanations, like only having to dig down lower
to layer C from layer B once mining finished in B.
Some said what needed to be done using the key and the map to answer the question in (c)(i) without
actually doing it. Others gave direct answers in the order of 2 km long and 1.5 km wide for maximum sizes,
although some tolerance was allowed for measurements, recognising that much depended on which part of
the mine was being measured. Credit was also given to those who tried to relate mine size to city size with
estimates of between a quarter and a half gaining further credit. Parts (c)(ii) to (c)(iv) were straightforward,
although reference to the location of the houses alone, without some reference to mining techniques, was
not enough for credit in (c)(iii). There was plenty of repetition of what was given in the question in answers
to (c)(v). Candidates needed to extend their answers by referring to dust from mining causing air pollution,
to the dangers of toxic substances entering water courses, and to the proximity of everyone in Cerro de
Pasco to the mine. Credit was also given for syllabus knowledge about health problems caused by lead,

646

General Certificate of Education Ordinary Level


5014 Environmental Management June 2011
Principal Examiner Report for Teachers
such
as brain damage in young people, and by dust, such as bronchitis and asthma. In other words, the
BALOL
more candidates moved away from the information in the question and supplemented it with their own
knowledge, the better their answers, and the more credit they gained.
Candidates who showed selective use of the information given with a small amount of extra comment added
to it usually were awarded partial credit. The award of full credit was reserved for wider ranging answers, in
which candidates recognised that Plan 1 was a more sustainable longer-term solution, although this made
the chances of it being implemented lower. This was because sustainable solutions cost money; people and
companies tend to be more interested in short-term profits.
Candidates needed to state the actual difference of US$5000 in (e)(i), and nearly all of them did. The main
issue which held back many of the answers to (e)(ii) was scale. The clear focus of the question was local
people living in Zambias copper belt. The focus for more than half the answers appeared to be loss of
income in Zambia as a whole, or how the national economy was being affected. This approach was
sometimes made more relevant when candidates related it to the lack of money for implementation of
government services badly affecting local people. However, those answers which began with loss of jobs in
the copper mines, followed by mention of the economic and social effects on their families, and the
worsening of the poverty trap, were awarded more credit more quickly. Only a tiny few took note of the
comment from the market stall holder in Chingola by referring to the knock-on effects of lower mine worker
earnings on local service providers who depended on their spending. Candidates who showed an
appreciation of the general pattern of world trade, between developing countries as exporters of primary
products and developed countries as importers for processing and manufacturing, were the ones most likely
to gain full credit in (e)(iii). Globalisation is not in the syllabus, but over the last few years the term is being
used more and more in the worlds media. Candidates who understood were in essence referring to the
effects of globalisation without, of course, stating explicitly that they were, as they did not need to do.
Overall the standard of answers given to Question 6 were at least as good as those given to Question 5.
Candidates kept working well to the end of the question, with any signs of tailing off restricted to a tiny
minority. Candidates showed good familiarity with the topic areas examined, and there were few questions
that they felt they could not try to answer.

647

BALOL
UNIVERSITY OF CAMBRIDGE INTERNATIONAL EXAMINATIONS
General Certificate of Education Ordinary Level

*8322734003*

5014/11

ENVIRONMENTAL MANAGEMENT
Paper 1

October/November 2010
2 hours 15 minutes

Candidates answer on the Question Paper.


Additional Materials:

Ruler
Protractor

READ THESE INSTRUCTIONS FIRST


Write your Centre number, candidate number and name on all the work you hand in.
Write in dark blue or black pen.
You may use a soft pencil for any diagrams, graphs or rough working.
Do not use staples, paper clips, highlighters, glue or correction fluid.
DO NOT WRITE IN ANY BARCODES.
Answer all questions.
All questions in Section A carry 10 marks.
Both questions in Section B carry 40 marks.
At the end of the examination, fasten all your work securely together.
The number of marks is given in brackets [ ] at the end of each question or part
question.

For Examiners Use


1
2
3
4
5
6
Total

This document consists of 24 printed pages.

UCLES 2010

[Turn o

648

BALOL
1

Section A

(a) Look at the photograph, which shows part of the largest copper mine in the world,
located in the Rocky Mountains, USA.

(i)

For
Examiners
Use

Use evidence from the photograph to describe the mining method shown.
..................................................................................................................................
..................................................................................................................................
..................................................................................................................................
..................................................................................................................................
..................................................................................................................................
............................................................................................................................ [4]

UCLES 2010

5014/11/O/N/10

649

BALOL(ii) The ore from the mine contains only 1% copper. What problems will result from
this?

For
Examiners
Use

..................................................................................................................................
..................................................................................................................................
..................................................................................................................................
..................................................................................................................................
............................................................................................................................ [3]
(b) What do you think should be done with such a large hole after mining has finished?
Give reasons for your answer.
..........................................................................................................................................
..........................................................................................................................................
..........................................................................................................................................
..........................................................................................................................................
.................................................................................................................................... [3]

UCLES 2010

5014/11/O/N/10

[Turn over

650

4
2 BALOL
(a) Look at the graph showing changes in the number of dead zones in the oceans. A
dead zone is found in very shallow, coastal waters. These areas used to have healthy
ecosystems, but not much life can now survive in them.
number of
dead zones
in oceans

(i)

number of
dead zones
in oceans

500

500

400

400

300

300

200

200

100

100

0
1965

1975

1985

1995

0
2005

How many dead zones were known in 1965 and 2005?


1965 .................................. 2005 .....................................

(ii)

[2]

Which ten-year period had the largest increase in dead zones?


...................................................................

UCLES 2010

For
Examiners
Use

[1]

5014/11/O/N/10

651

BALOL
(b) (i) Name two heavy metals.
............................................................................................................................ [1]
(ii)

For
Examiners
Use

Explain how heavy metals and plastics can cause the death of marine animals.
..................................................................................................................................
..................................................................................................................................
..................................................................................................................................
..................................................................................................................................
............................................................................................................................ [3]

(c) Why is it difficult to prevent the formation of dead zones?


..........................................................................................................................................
..........................................................................................................................................
..........................................................................................................................................
..........................................................................................................................................
.................................................................................................................................... [3]

UCLES 2010

5014/11/O/N/10

[Turn over

652

6
3 BALOL
Look at the maps showing the extent of ice cover north of the tundra zone in September
1979 and September 2007.

For
Examiners
Use

1979
Greenland

North America

Europe
Arctic Ocean

key
ice
sea water
land not
covered
by ice

Asia

2007
Greenland

North America

Europe
Arctic Ocean

Asia

UCLES 2010

5014/11/O/N/10

653

BALOL
(a) (i) Describe one main similarity and one main difference in the extent of the ice cover
in 2007 compared with 1979.

For
Examiners
Use

similarity ...................................................................................................................
difference ............................................................................................................ [2]
(ii)

On the 2007 map write T to show a land area with a tundra climate.

[1]

(b) Describe and explain:


(i)

the effect of snow and ice on the incoming rays from the Sun,
..................................................................................................................................
..................................................................................................................................
............................................................................................................................ [2]

(ii)

why insolation is low in high latitudes.


..................................................................................................................................
..................................................................................................................................
............................................................................................................................ [2]

(c) Can the changes in ice cover shown on the maps be linked to climate change?
You should give reasons for agreeing or disagreeing, or both.
..........................................................................................................................................
..........................................................................................................................................
..........................................................................................................................................
.................................................................................................................................... [3]

UCLES 2010

5014/11/O/N/10

[Turn over

654

8
4 BALOL
(a) Look at the diagram, which shows part of the food web in Yellowstone National Park,
USA.

black
bear

wolf

grizzly
bear

coyote

cougar
(mountain
lion)

elk

moose

For
Examiners
Use

plants

Many wolves once lived in Yellowstone National Park but there were none living there by
the 1970s. Wolves from Canada have recently been released in the area.
(i)

What is likely to happen to the wolf population over time?


............................................................................................................................ [1]

(ii)

Which animal can help to control the population of wolves?


...................................................................

(iii)

How many feeding (trophic) levels are shown?


...................................................................

(iv)

[1]

[1]

Use an example from the food web diagram to explain the meaning of the following
terms:
predator ....................................................................................................................
..................................................................................................................................
competition ...............................................................................................................
..................................................................................................................................
producer ...................................................................................................................
..................................................................................................................................
food chain .................................................................................................................
............................................................................................................................ [4]

UCLES 2010

5014/11/O/N/10

655

BALOL
(b) Fires which start naturally in Yellowstone National Park are allowed to burn.

For
Examiners
Use

Explain the likely effects of this policy on animals in any ecosystem.


..........................................................................................................................................
..........................................................................................................................................
..........................................................................................................................................
..........................................................................................................................................
.................................................................................................................................... [3]

UCLES 2010

5014/11/O/N/10

[Turn over

656

10

BALOL
5

Section B

(a) Look at the map which shows major plate boundaries in Europe, Africa and Asia.

For
Examiners
Use

Iceland

Eurasian
Plate
North
American
Plate

Q
Pacific
Plate

African
Plate
Q
P
Indo-Australian
Plate
South
American
Plate

Key:
Plate boundaries
Direction of plate movement
Volcano Vesuvius
(i)

State the direction of plate movement along the boundary marked P.


............................................................................................................................ [1]

(ii)

Name the type of plate boundary at P.


............................................................................................................................ [1]

(iii)

Volcanic eruptions are frequent in Iceland. Explain why.


..................................................................................................................................
..................................................................................................................................
..................................................................................................................................
............................................................................................................................ [2]

UCLES 2010

5014/11/O/N/10

657

11

BALOL(iv) A high percentage of Icelands energy needs come from renewable alternative
sources. Describe how volcanic activity can be harnessed for electric power and
heating.

For
Examiners
Use

..................................................................................................................................
..................................................................................................................................
..................................................................................................................................
..................................................................................................................................
..................................................................................................................................
............................................................................................................................ [3]
(v)

The earthquake risk is high in countries located along plate boundary Q. Explain
why earthquakes often occur along plate boundaries of this type.
..................................................................................................................................
..................................................................................................................................
..................................................................................................................................
..................................................................................................................................
..................................................................................................................................
..................................................................................................................................
............................................................................................................................ [4]

UCLES 2010

5014/11/O/N/10

[Turn over

658

12

BALOL
(b) Governments of countries with a high earthquake risk can prepare for future earthquakes.
Describe strategies they can use that will help to
(i)

For
Examiners
Use

stop buildings from collapsing in an earthquake,


..................................................................................................................................
..................................................................................................................................
..................................................................................................................................
..................................................................................................................................
..................................................................................................................................

(ii)

reduce the number of deaths after the earthquake.


..................................................................................................................................
..................................................................................................................................
..................................................................................................................................
..................................................................................................................................
..................................................................................................................................
[5]

UCLES 2010

5014/11/O/N/10

659

13

BALOL
(c) Look at the table which shows strong earthquakes along plate boundary Q during the
nine years between 2000 and 2008.

For
Examiners
Use

Strong earthquakes above 6.0 on the Richter Scale along plate boundary Q (20002008)
Date

Richter scale

Location

Estimated deaths

2000
2001 January

7.9

Gujarat, India

20,000

7.0

Algeria

December

6.5

Bam, Iran

2004 December

8.9

Off the coast of Indonesia

2005 February

6.4

Kerman, Iran

March

8.7

Off the coast of Indonesia

October

7.6

Northern Pakistan and Kashmir

2006 April

6.0

Western Iran

May

6.2

Yogyakarta, Indonesia

5,000

6.5

Baluchistan, Pakistan

300

2002
2003 May

2,000
30,000
250,000
300
1,300
79,000
70

2007
2008 October
(i)

Estimated deaths is the heading for the last column in the table. Why is this used
instead of just Deaths?
..................................................................................................................................
..................................................................................................................................
............................................................................................................................ [2]

(ii)

Name the two countries with the largest number of strong earthquakes during
these nine years.
............................................................................................................................ [1]

(iii)

Describe the pattern of strong earthquakes during the nine years shown in the
table.
..................................................................................................................................
..................................................................................................................................
..................................................................................................................................
............................................................................................................................ [2]

UCLES 2010

5014/11/O/N/10

[Turn over

660

14

BALOL(iv) What does the information in the table suggest about the chances of people
predicting when and where earthquakes will occur along plate boundary Q?

For
Examiners
Use

..................................................................................................................................
..................................................................................................................................
..................................................................................................................................
............................................................................................................................ [2]
(v)

The stronger the earthquake, the greater the number of deaths. Describe evidence
in the table for and against this statement.
..................................................................................................................................
..................................................................................................................................
..................................................................................................................................
..................................................................................................................................
..................................................................................................................................
..................................................................................................................................
..................................................................................................................................
............................................................................................................................ [4]

(vi)

In your view, how strong is the evidence for this statement? Explain your answer.
..................................................................................................................................
..................................................................................................................................
..................................................................................................................................
............................................................................................................................ [2]

UCLES 2010

5014/11/O/N/10

661

15

BALOL
(d) Look at the timeline showing dates of major eruptions of the volcano Vesuvius in

1850

1800

1900

For
Examiners
Use

19
44

19
06

18
5
18 0
5
18 5
61
18
72

18
22
18
35

southern Italy.

1950

2000

eruption and date


(i)

Describe differences in the pattern of eruptions between the nineteenth (18001900)


and twentieth centuries (19002000).
..................................................................................................................................
..................................................................................................................................
..................................................................................................................................
............................................................................................................................ [2]

(ii)

More than half a million people live in the area around Vesuvius. It is the most
densely populated part of southern Italy.
State the most likely reason why so many people live near this volcano.
............................................................................................................................ [1]

UCLES 2010

5014/11/O/N/10

[Turn over

662

16

BALOL(iii) Scientists keep a close watch on Vesuvius in the hope of predicting the next
eruption and warning people before it happens. Four ways scientists watch and
study volcanoes are given in the spider diagram below.
thermometers to measure
temperatures in the crater

For
Examiners
Use

seismographs to
record small earthquake
shocks

watching and studying


volcanoes

observations of
emissions of gas
and steam

tilt meters to
record small changes in
ground shape

Describe how these ways are useful for predicting the next volcanic eruption.
..................................................................................................................................
..................................................................................................................................
..................................................................................................................................
..................................................................................................................................
..................................................................................................................................
............................................................................................................................ [3]
(iv)

Suggest why the scientists might not always make correct predictions about
volcanic eruptions.
..................................................................................................................................
..................................................................................................................................
..................................................................................................................................
............................................................................................................................ [2]

UCLES 2010

5014/11/O/N/10

663

17

BALOL(v) When scientists predict that Vesuvius is about to erupt again, more than half a
million people will need to be evacuated from their homes. How easy or difficult will
it be to put this strategy into action? Answer as fully as you can.

For
Examiners
Use

..................................................................................................................................
..................................................................................................................................
..................................................................................................................................
..................................................................................................................................
..................................................................................................................................
............................................................................................................................ [3]
[Total: 40 marks]

UCLES 2010

5014/11/O/N/10

[Turn over

664

18
6 BALOL
(a) The table below shows the composition of the lower atmosphere.
gas

percentage

nitrogen

78%

oxygen

21%

other gases
(i)

For
Examiners
Use

1%

In the frame below, draw a divided bar graph to show these percentages and
complete the key.
key:
nitrogen
oxygen

100

percentage

other gases

[3]
(ii)

Among the other gases are water vapour, carbon dioxide and ozone.
Explain the importance of each of these gases for life on Earth.
Water vapour ............................................................................................................
..................................................................................................................................
..................................................................................................................................
..................................................................................................................................
Carbon dioxide .........................................................................................................
..................................................................................................................................
..................................................................................................................................
..................................................................................................................................
Ozone .......................................................................................................................
............................................................................................................................ [6]

(iii)

How is the natural balance of the gases, nitrogen, oxygen and carbon dioxide,
maintained in the atmosphere?
..................................................................................................................................
..................................................................................................................................
..................................................................................................................................
..................................................................................................................................
..................................................................................................................................
............................................................................................................................ [3]

UCLES 2010

5014/11/O/N/10

665

19

BALOL
(b) Some human actions are disrupting this natural balance.

For
Examiners
Use

The graph below shows total world carbon emissions since 1860.
400

billions of tonnes of carbon

350
300
250
200
150
100
50
0 1860 1880 1900 1920 1940 1960 1980 2000
year
(i)

On the graph, draw a summary line to show the change between 1860 and 2000.
[1]

(ii)

Describe the trends shown before and after 1960 and quote values to support your
answer.
..................................................................................................................................
..................................................................................................................................
..................................................................................................................................
..................................................................................................................................
............................................................................................................................ [3]

(iii)

Explain why many people believe that increased emissions of carbon dioxide are
causing global warming.
..................................................................................................................................
..................................................................................................................................
..................................................................................................................................
..................................................................................................................................
..................................................................................................................................
............................................................................................................................ [3]

UCLES 2010

5014/11/O/N/10

[Turn over

666

20

BALOL
(c) Information about the two countries in the world with the greatest carbon dioxide
emissions in 2007 is given below.

For
Examiners
Use

USA
China

percentage of total world


carbon dioxide emissions
in 2007

percentage of total world


carbon dioxide emissions
in 2007
Key
% share of world
CO2 emissions
emissions per head,
1tonne per head

carbon dioxide emissions


per head (2007)

(i)

carbon dioxide emissions


per head (2007)

Describe what the information above shows about the share of total world emissions
from these two countries.
..................................................................................................................................
..................................................................................................................................
..................................................................................................................................
..................................................................................................................................
............................................................................................................................ [3]

UCLES 2010

5014/11/O/N/10

667

21

BALOL(ii) The cartoon below is trying to explain why carbon dioxide emissions are increasing
so fast in China.

For
Examiners
Use

an Companies invest in Ch
Americ
in a

Factory
clothes,
trainers
,
toys

Explain what the cartoon shows about the reasons for carbon dioxide emissions
increasing in China.
..................................................................................................................................
..................................................................................................................................
..................................................................................................................................
..................................................................................................................................
............................................................................................................................ [3]

UCLES 2010

5014/11/O/N/10

[Turn over

668

22

BALOL
(d) When complete, the flow diagram below will show the worldwide effects of global
warming.

For
Examiners
Use

worldwide effects of global warming

ice sheets melting

economic losses for farmers, house


owners, companies and governments

Fill in the boxes by choosing the best comment for each box from the list below.
* sea defences breached

* previously populated areas abandoned

* rising sea levels

* flooding of low lying coastal areas such as deltas


[3]

UCLES 2010

5014/11/O/N/10

669

23

BALOL
(e) In November 2008 the new President in the Maldives said that he intended to set up a
wealth fund from tourist income. This fund would be used to buy land in another country
where his people could move, should global warming be worse than expected.
Global sea levels are expected to rise 2558 cm by 2100.

For
Examiners
Use

Read the information about the Maldives.


The Maldives
Population

380,000
capital city Mal 100,000

Geography

coral islands in the middle of the Indian Ocean


about 1200 islands, of which about 250 are populated
highest point 2.4 m above sea level; average 1.5 m

Economy

main income from tourism with about 500,000 visitors a year


average income about US$ 4,600 per head

(i)

Explain why the President of the Maldives is more worried than leaders in most
other countries about possible threats from global warming.
..................................................................................................................................
..................................................................................................................................
..................................................................................................................................
............................................................................................................................ [2]

(ii)

The President of the Maldives said We can do nothing to stop climate change on
our own.
How true is this statement? Explain your answer.
..................................................................................................................................
..................................................................................................................................
..................................................................................................................................
............................................................................................................................ [2]

(iii)

The President plans to buy land in another country for his people to go to if sea
levels rise. Describe one likely problem with this plan.
..................................................................................................................................
..................................................................................................................................
..................................................................................................................................
............................................................................................................................ [2]

UCLES 2010

5014/11/O/N/10

[Turn over

670

24

BALOL
(f) Read the information below.

For
Examiners
Use

2008 a bad year for climatic hazards


millions face drought in Ethiopia
11 million affected by monsoon floods in India
128,000 killed by a cyclone that struck Myanmar (Burma)

A I blame climate
change for all
these disasters

(i)

B these are
just normal climate
events

C large numbers
are affected because
of poverty

Explain the viewpoints of persons B and C.


..................................................................................................................................
..................................................................................................................................
..................................................................................................................................
..................................................................................................................................
..................................................................................................................................
..................................................................................................................................
..................................................................................................................................
............................................................................................................................ [4]

(ii)

How far do you agree with person A? Explain your answer.


..................................................................................................................................
..................................................................................................................................
..................................................................................................................................
............................................................................................................................ [2]
[Total: 40 marks]

671

BALOL
UNIVERSITY OF CAMBRIDGE INTERNATIONAL EXAMINATIONS
GCE Ordinary Level

MARK SCHEME for the October/November 2011 question paper


for the guidance of teachers

5014 ENVIRONMENTAL MANAGEMENT


5014/11

Paper 1, maximum raw mark 120

This mark scheme is published as an aid to teachers and candidates, to indicate the requirements of
the examination. It shows the basis on which Examiners were instructed to award marks. It does not
indicate the details of the discussions that took place at an Examiners meeting before marking began,
which would have considered the acceptability of alternative answers.
Mark schemes must be read in conjunction with the question papers and the report on the
examination.

Cambridge will not enter into discussions or correspondence in connection with these mark schemes.

Cambridge is publishing the mark schemes for the October/November 2011 question papers for most
IGCSE, GCE Advanced Level and Advanced Subsidiary Level syllabuses and some Ordinary Level
syllabuses.

672

Page 2

Mark Scheme: Teachers version


GCE O LEVEL October/November 2011

Syllabus
5014

Paper
11

BALOL
Notes on application of the mark scheme
Marking points are separated by semi-colons. Each line usually represents one mark.
Oblique lines separate ideas which are alternatives.
Ideas in brackets are not essential to the answer but anything underlined is.
Reward any equivalent way of expressing the ideas in the mark scheme.
Reward any valid answer which is not in the mark scheme.
Section A
1

(a) (i) A sedimentary;


B igneous;
C metamorphic;
2 or 3 correct = 2, 1 correct = 1

[2]

(ii) sedimentary = 1;
because heat / pressure would destroy trees / carbon;
because trees grew in sediments;
Accept any sensible suggestion = 1

[2]

(b) e.g.
limestone cement / concrete / flux etc.;
clay brick making / pottery;
chalk cement;
sand(stone) glass;
Interpret industrial use widely e.g. allow construction.
Names of two valid rocks = 1
2 uses @ 1 = 2 (can be for one type of rock if well developed)
(c) Answers such as concerns about:
visual pollution;
noise;
dust;
damage to roads from heavy lorries;
damage to scenery;
destruction of habitats / wildlife scared away;

[3]

[3]

Any 3.
[Total: 10]

673

Page 3

Mark Scheme: Teachers version


GCE O LEVEL October/November 2011

Syllabus
5014

Paper
11

BALOL
2

(a) (i) 98 (mm);

[1]

(ii) 50 (mm);

[1]

(iii) precipitation was higher than evaporation leaving water to infiltrate;

[1]

(iv) dry soil / insufficient soil moisture / drought;


crops need artificial watering to survive / crops die without water,
evaporation greater than precipitation;
high evaporation continues to evaporate water from the soil.
rain quickly evaporated;
Mark as a unit and accept points where they come but for max. need one from each
group.
[3]
(b) plants take water in through their roots; = 1
transpire moisture through their leaves;
through pores / stomata;
leaves intercept rain;
evaporates;

[4]

Any 3.
[Total: 10]
3

(a) (i) 6/7/8 (%);

[1]

(ii) increase in commercial and decrease in subsistence;


Allow use of percentages to make the point.
(b) (i) 361;
(ii) ideas such as:
commercial use (more) fertiliser;
(more) insecticides / pesticides / herbicides;
(more advanced) machinery;
(more) scientific methods / crop rotation or other e.g. of;
(likely to be on) more fertile soils;
(more) skilled labour force;
better seeds / HYVs;
use of irrigation;

[1]
[1]

[4]

Any 4.

674

Page 4

Mark Scheme: Teachers version


GCE O LEVEL October/November 2011

Syllabus
5014

Paper
11

BALOL
(c) answers such as:
commercial because:
(more) likely to lead to water / land pollution by overuse of inorganic fertilisers;
(more) likely to cause air pollution by spraying insecticide / herbicide;
resulting effect on wildlife of water / air pollution;
(more likely to) use monoculture damaging the soil;
effect on nearby crops if use of GM seeds;
reduction of gene pool;
irrigation can cause salinisation of soils;
Any 3.
The candidate may choose subsistence farming. If so, give credit for any sensible reason
given e.g. lack of knowledge of consequences of ploughing down slopes;
ploughing down slope leads to soil erosion;
over-cultivation by shifting cultivators if population pressure;
[3]
[Total: 10]
4

(a) (i) farming in dry areas without irrigation /


where little/insufficient rain without irrigation;

[1]

(ii) mulch prevents evaporation by shading the soil;


allows dew to trickle through and be shaded from the sun;
saves the rain from two years for the crop to use in the second year;
strips of grass help to keep soil from blowing away;
keep soil from washing away/impede water movement;
Any 3.
Allow a well developed answer to score 2 marks for one method.
(b) built a wall :
protection from wind;
wind cannot dry plant/soil;
shade from sun reduces water loss;
3 @ 1 mark each.
(c) Credit ideas such as:
soil erosion;
desertification;
soil infertility/exhaustion;
soil structure deteriorates/soil more friable;
bare/loose/dry soil easily blown away;
bare/loose/dry soil easily washed away;

[3]

[3]

[3]

Any 3.
[Total: 10]

675

Page 5

Mark Scheme: Teachers version


GCE O LEVEL October/November 2011

Syllabus
5014

Paper
11

BALOL
Section B
5

(a) (i) at least 80 % of the gently sloping ocean area next to the coast shaded in;

[1]

(ii) flatter / gently sloping;


shallow water / less deep;
One of these or similar.

[1]

(iii) from magma (from the mantle / inside of the Earth) which reaches the surface;
associated with constructive plate boundaries;
some parts built up by volcanoes / lava flows;
Two points such as these. 2 @ 1 mark.

[2]

(iv) more light penetrates the water because it is shallow;


more nutrients to support plant and animal life;
some carried from the land in river sediments;
some brought by ocean currents (especially cold currents);
richest where cold currents upwell from deeps / warm and cold currents meet;
related example used;
food chain / web supporting other life in the oceans;
Points made like these which lead to effective explanation. 4 @ 1 mark each.

[4]

(v) depth of water cheaper and easier to exploit resources in shallow waters;
more difficult to discover deep water resources;
increased distance from shore to provide equipment and supplies for workers;
Two factors such as these. 2 @ 1 mark each.
(b) (i) 75m tonnes;

[2]
[1]

(ii) growth in demand either from growing world population;


or the value of fish in the human diet as a source of protein;
improved technology for discovering fish shoals;
example of improved / larger scale methods of fishing;
Two human reasons like these. 2 @ 1 mark.

[2]

(iii) fish stocks are plentiful when the cold Peruvian current upwells near the coast;
cold waters rich in plankton on which anchovy feed;
shoals of anchovy migrate from coast in years when warm equatorial current from the
west brings warmer less nutrient-rich water to the coast;
periodic climatic change which causes stronger winds from the west blowing the warm
waters;
Understood and reasonably complete explanation = 3 marks.
Outline understanding without complete explanation = 2 marks.
Some understanding, perhaps misunderstandings and inaccuracies as well = 1 mark. [3]

676

Page 6

Mark Scheme: Teachers version


GCE O LEVEL October/November 2011

Syllabus
5014

Paper
11

BALOL
(iv) natural factors only likely if new fishing grounds in the ocean are being exploited as a
result of new technology, and in sustainable quantities;
limit to the amount of fish that the natural ecosystems can support;
plentiful evidence of overfishing and formerly rich fishing grounds producing less such as
the Grand Banks off North America and the North Sea in Europe;
human factors improvements in technology for locating big shoals of fish (such as
sonar), for catching fish (bigger nets and larger boats) and for preserving and processing
fish caught (such as factory ships);
NB some clues in later question information in part (d).
widespread evidence of overfishing strongly suggests human factors are more
responsible than physical ones;
Clear conclusion with relevant supporting detail = 2 or 3 marks according to amount and
quality of explanation.
Comment but without a clear answer to the question = 1 mark.
Answer to question without relevant support = 0 marks.
[3]
(c) (i) fish stocks with fish of all ages, including young fish which will grow to maturity;
so that overall numbers will be maintained / may even increase;
Understood and clear explanation = 2 marks.
Some understanding = 1 mark.

[2]

(ii) 17% over-fished + 8% depleted / exhausted; Working = 1 mark.


25%; Answer = 2nd mark.

[2]

(iii) from the Atlantic Ocean;

[1]

(iv) mainly near the coast of Africa (rather than Europe);


largest breeding grounds are towards the eastern side of the Mediterranean (off Libya
and Egypt);
closer detail about the location of one or more of the four breeding grounds marked;
Two general points (along the lines of the first two), or one general and one that is more
local = 2 marks.
Only description of separate locations for individual breeding grounds = maximum
1 mark.
[2]
(v) two exhausted breeding grounds off Spain and Italy suggesting overfishing has
occurred;
[1]
(vi) accurate plots = 2 marks.
OR at least 4 correct = 1 mark.
plots linked by a line = 1 mark.

[3]

(vii) big drop in breeding age tuna between 1970 and 2005 by 130,000 tonnes;
peak was in 1975 and lowest in 2005 (by 170,000 tonnes / under one quarter);
a big drop in the 10 years since 1995 strongly suggesting overfishing;
comment about the significance of these being fish of breeding age;
3 relevant points along these lines without necessarily being the same as these.
3 @ 1 mark, but a maximum 2 marks for answers without the use (as opposed to
mere statement / repetition) of values.

[3]

677

Page 7

Mark Scheme: Teachers version


GCE O LEVEL October/November 2011

Syllabus
5014

Paper
11

BALOL
(d) (i) differences in size and age of the boats;
local fishermen as opposed to multi-national companies as operators;
method of fishing trapping with nets compared with large nets and hi-tech equipment;
further comment about the significance of individual differences;
Stating information from source without adaptation to question need = 0 marks.
Valid difference(s) using information but without comment to question = 1 mark.
Differences adapted to question need and commented on = 2 or 3 marks.

[3]

(ii) Traditional fishermen are going back to coastal ports where catch can be monitored;
whereas modern boats take the catch to fish farms in the Mediterranean without going
into ports;
then loaded straight on to boats for export so that amount is impossible to control fully.
Variations on this line of argument are possible credit answers according to validity.
Understood and clearly explained = 2 marks.
Some understanding = 1 mark.
[2]
(iii) industrial suggests 'factory' / also commercial business practice;
'tuna ranches' in the information suggests fish farming on a large scale;
it is the type of organisation of an industry that would be expected of large companies;
Understood and clearly explained = 2 marks.
Some understanding = 1 mark.

[2]
[Total: 40]

(a) (i) 50 metres/m (allow 46 to 52 metres);


(ii) around 30 m/metres / between 25 and 35 metres;

[1]
[1]

(iii) have buttresses above the surface / on the forest floor;


shallow root systems below;
2 @ 1 mark each.

[2]

(iv) high density with four or five different layers;


plants like Iianas / creepers occupy spaces between the trees;
lack of branches on trees until canopy is reached due to competition for sunlight;
hot and wet all year creating ideal conditions for plant growth;
typical temperature around 27C all year (well above minimum for plant growth);
high annual rainfall above 1500mm and lack of wet season;
Two marks for describe and two for explain, but allow three for strong explanation and
weak description.
[4]
(v) niche fill spaces between the tall trees using the trees for support;
using the tall forest trees for support allows them to reach the sunlight above the mass of
forest vegetation while having their roots anchored in the ground;
Full answer = 2 marks.
Part answer = 1 mark.

[2]

678

Page 8

Mark Scheme: Teachers version


GCE O LEVEL October/November 2011

Syllabus
5014

Paper
11

BALOL
(b) (i) likely choices of habitat:
canopy / middle and higher levels in the forest in the crowns and among the branches
where food supplies include leaves, fruits, nuts and berries for birds such as toucans
and animals like monkeys.
forest floor ground vegetation such as ferns, less rich food supply from plants directly
but fruits, berries etc. that have fallen to forest floor. Some animals are vegetarian like
the tapir, many are carnivores eating smaller creatures, such as jaguars and snakes.
Habitats identified and differences between them stated up to 2 marks.
Related to forest creatures present and the differences between them up to 3 marks.
[4]
(ii) producers fruit, berries, leaves;
At least two named for 1 mark.
primary consumers toucan, tapir, monkey (also frogs, birds, butterflies and insects);
At least two named for 1 mark.
secondary consumer jaguar / snakes / insects and birds (only if specified such as birds
of prey);
One named for 1 mark.
Fourth mark for completeness and accuracy of the food web overall with arrows used to
link the different layers from producer to primary consumer to secondary consumer;
4 @ 1 mark each.

[4]

(iii) consume both plants and animals / wide variety of available food sources;
Indian tribes hunt, fish and collect and gather forest products;
humans have the technology / know-how to kill creatures and use all food sources;
Understood and well explained = 2 marks.
One or more points made to show some understanding = 1 mark.
(iv) (the) decomposers;
(c) (i) plate tectonics / continental drift;

[2]
[1]
[1]

(ii) at the destructive margin where the Indian plate meets another plate (Eurasian Plate);
sediments folded up / rocks melted in the subduction zone cause volcanic activity;
led to formation of the Himalaya (if 'where' is answered from knowledge);
Two points along these lines. 2 @ 1 mark each.

[2]

679

Page 9

Mark Scheme: Teachers version


GCE O LEVEL October/November 2011

Syllabus
5014

Paper
11

BALOL
(d) (i) diversity of species found nowhere else on Earth;
seen as part of the great natural biodiversity that exists on the Earth's surface;
explanation of the importance of this biodiversity to people plants as a genetic pool for
crops, for medicines etc.;
comment about the long term advantages of keeping the natural forest and species as
opposed to the short term financial advantages from mining and logging;
Minimum answer remains close to what is already provided in the introduction to the
question.
A little explanation beyond is likely to make the answer worth two marks instead of one.
1 or 2 marks.
Explanation developed in relation to the importance of biodiversity and / or advantages
of maintaining rich and varied ecosystems, especially those that are unique as in
Madagascar.
3 or 4 marks.
[4]
(ii) IUCN The World Conservation Union; link organisation between governments,
government agencies and many different non-governmental organisations;
Its slogan is 'The Green Web';
WWF World Wide Fund for Nature; uses the slogan 'Taking action for a living planet';
Charity / NGO funded by supporters focuses on conservation of wildlife and their
habitats, as well as the wider implications of man's activities on the environment; Funds
particular conservation projects such as tigers in India;
CITES Convention on International Trade in Endangered Species of Wild Flora and
Fauna; an international agreement between countries to ensure that the international
trade in specimens of wild animals and plants does not threaten their survival;
High profile examples include trade in elephant ivory and rhino horns;
Description of work:
Basic knowledge =1 mark.
Fuller description = 2 marks.
Description gives a good idea of work undertaken by it = 3 marks.

[3]

(e) (i) Tourism that is environmentally and ecologically sound i.e. it takes into account
needs of natural environments, habitats and species as well as local communities,
ensuring that their ways of life and traditions are maintained.
Some understanding perhaps the environmental without the social = 1 mark.
Well understood and stated = 2 marks.

[2]

(ii) tourists will only come if the forests and their wildlife are preserved since these are what
they are coming to see / the attractions;
by giving local people income and involving them, they become less likely to clear the
forests and capture animal species;
they are very poor people and need an income to stop them doing this;
Two points made along these lines. 2 @ 1 mark each.

[2]

680

Page 10

Mark Scheme: Teachers version


GCE O LEVEL October/November 2011

Syllabus
5014

Paper
11

BALOL
(f)

(i) forests are carbon stores, trees trap carbon dioxide as part of the process of
photosynthesis, when cleared and burnt the carbon dioxide is released into the
atmosphere contributing to the 'greenhouse effect' and global warming;
locally forests contribute to high rates of evapo-transpiration which maintains water
sources in the atmosphere for condensation and rainfall;
(ii) possible advantages developing countries receive an income / foreign exchange;
Instead of selling logs, mining and using the land for agriculture, all of which involve
forest clearances, with all the advantages that maintaining natural forests brings globally;
financial incentive for governments to conserve forests will exist;
possible disadvantages may be difficult to monitor with much clearance continuing
because many of these areas are not under direct government control; existence of
corrupt local officials, money may also be siphoned off by corrupt politicians;
perhaps unlikely that local people will see any financial benefits;
Only general comment throughout, little development for either description or
explanation.
One part may be a lot better answered than the other = 12 marks.
Fuller responses, meaningful description and explanation, perhaps a lack of balance in
the strength to the two parts = 34 marks.
Full responses and well balanced = 5 marks.

[5]
[Total: 40]

681

BALOL
UNIVERSITY OF CAMBRIDGE INTERNATIONAL EXAMINATIONS
General Certificate of Education Ordinary Level

* 0 8 6 5 5 4 0 1 2 9 *

5014/12

ENVIRONMENTAL MANAGEMENT
Paper 1

October/November 2011
2 hours 15 minutes

Candidates answer on the Question Paper.


Additional Materials:

Ruler
Protractor
1 Insert

READ THESE INSTRUCTIONS FIRST


Write your Centre number, candidate number and name on all the work you hand in.
Write in dark blue or black pen.
You may use a soft pencil for any diagrams, graphs or rough working.
Do not use staples, paper clips, highlighters, glue or correction fluid.
DO NOT WRITE IN ANY BARCODES.
Answer all questions.
All questions in Section A carry 10 marks.
Both questions in Section B carry 40 marks.
The Insert contains the photograph needed for Question 4.
DO NOT WRITE ON THE INSERT.
At the end of the examination, fasten all your work securely together. But keep
the Insert separate from the question paper it is not needed by the Examiner.
The number of marks is given in brackets [ ] at the end of each question or
part question.

For Examiners Use


1
2
3
4
5
6
Total

This document consists of 25 printed pages and 3 blank pages and 1 insert.
UCLES 2011

[Turn o

682

BALOL

Section A

For
Examiners
Use

(a) The diagram shows a soil with numbered layers of different sized mineral particles above
solid rock.
Soil surface
with vegetation
layer 1
layer 2
layer 3

layer 4
solid rock
(i)

How does the mineral particle size change with depth?


............................................................................................................................. [1]

(ii)

In which layer are the particles least rounded?


............................................................................................................................. [1]

(iii)

What was the source of the mineral particles shown in the diagram?
............................................................................................................................. [1]

(iv)

Explain the change in particle size with depth.


..................................................................................................................................
..................................................................................................................................
............................................................................................................................. [2]

(v)

Describe the pore space, drainage and likely air content of a sandy soil with a
coarse texture.
pore space ...............................................................................................................
drainage ...................................................................................................................
air content ........................................................................................................... [3]

UCLES 2011

5014/12/O/N/11

683

BALOL
(b) How can a farmer improve land where the soil is too wet for farming?
..........................................................................................................................................

For
Examiners
Use

..........................................................................................................................................
..................................................................................................................................... [2]

UCLES 2011

5014/12/O/N/11

[Turn over

684

4
2 BALOL
(a) Explain why large numbers of fish live in sea areas near the coast.
..........................................................................................................................................

For
Examiners
Use

..........................................................................................................................................
..........................................................................................................................................
..........................................................................................................................................
..........................................................................................................................................
..........................................................................................................................................
..........................................................................................................................................
..................................................................................................................................... [4]
(b) Look at the pie chart which shows the percentage of the World fish catch which is useful
and the percentage which is bycatch (the percentage which is unused or wasted).
0
90

10

80

20

key:
bycatch
useful fish catch

70

30

40

60
50

What percentage of the total World catch is bycatch?


................................. %

UCLES 2011

[1]

5014/12/O/N/11

685

BALOL
(c) The table shows the weight of the total catch and of the bycatch for four fishing areas.

area
North East
Atlantic

bycatch
(million metric tonnes)

13.5

2.7

Mediterranean
and Black Sea

1.5

0.3

Caribbean

0.4

0.25

10.0

7.0

Africa

(i)

total catch
(million metric tonnes)

What percentage of Africas total catch is bycatch?


.............................................. %

(ii)

For
Examiners
Use

[1]

How does Africas bycatch percentage differ from all the other areas in the table?
............................................................................................................................. [1]

(d) Suggest why many people are very concerned about high bycatch levels.
..........................................................................................................................................
..........................................................................................................................................
..........................................................................................................................................
..........................................................................................................................................
..........................................................................................................................................
..................................................................................................................................... [3]

UCLES 2011

5014/12/O/N/11

[Turn over

686

6
3 BALOL
(a) The map shows a cyclone in the Pacific Ocean approaching Luzon, the most populated
island in the Philippines.

China
Sea

Luzon

For
Examiners
Use

Pacific
Ocean

Manila

key:
expected path of cyclone
capital city
no cloud
light cloud
dense cloud

very dense cloud


(i)

300

600

km

In which compass direction was the cyclone expected to travel?


............................................................................................................................. [1]

(ii)

How many kilometres is the leading edge of the cloud from the nearest coast of
Luzon?
........................................... km

(iii)

[1]

Describe the cloud pattern of the cyclone.


..................................................................................................................................
..................................................................................................................................
............................................................................................................................. [2]

(iv)

Describe and explain the amount of pressure in the centre of a cyclone.


..................................................................................................................................
..................................................................................................................................
..................................................................................................................................
............................................................................................................................. [2]

UCLES 2011

5014/12/O/N/11

687

BALOL
(b) Suggest what weather warnings and advice would have been issued to the people of
Luzon by weather forecasters as the cyclone approached.

For
Examiners
Use

weather warnings .............................................................................................................


..........................................................................................................................................
..........................................................................................................................................
advice ..............................................................................................................................
..........................................................................................................................................
..........................................................................................................................................
..........................................................................................................................................
..................................................................................................................................... [4]

UCLES 2011

5014/12/O/N/11

[Turn over

688

8
4 BALOL
(a) Look at the photograph (Insert).
(i)

The natural vegetation of the area is shown in the foreground of the photograph.
Describe the natural vegetation and how it differs from the planted forest in the
background which has replaced it.

For
Examiners
Use

..................................................................................................................................
..................................................................................................................................
..................................................................................................................................
..................................................................................................................................
..................................................................................................................................
............................................................................................................................. [3]
(ii)

One area on the photograph has been cleared of coniferous forest for timber.
Explain why this could lead to a poorer soil environment.
..................................................................................................................................
..................................................................................................................................
..................................................................................................................................
..................................................................................................................................
..................................................................................................................................
............................................................................................................................. [3]

UCLES 2011

5014/12/O/N/11

689

BALOL
(b) Describe methods of managing forests more sustainably.
..........................................................................................................................................

For
Examiners
Use

..........................................................................................................................................
..........................................................................................................................................
..........................................................................................................................................
..........................................................................................................................................
..........................................................................................................................................
..........................................................................................................................................
..................................................................................................................................... [4]

UCLES 2011

5014/12/O/N/11

[Turn over

690

10

BALOL
5

Section B

(a) Look at the map which shows major plate boundaries in the Atlantic Ocean, Europe and
Africa.

For
Examiners
Use

EUROPE

AFRICA

ATLANTIC
OCEAN
key:
plate boundaries
direction of plate movement
Italy
0

km

5000

(i)

On the map, shade or colour in the course of the constructive plate boundary.

(ii)

In which part of Africa is the earthquake risk highest? Explain your answer.

[1]

..................................................................................................................................
..................................................................................................................................
..................................................................................................................................
..................................................................................................................................
............................................................................................................................. [3]

UCLES 2011

5014/12/O/N/11

691

11

BALOL(iii) State what happens at a constructive plate boundary.


..................................................................................................................................

For
Examiners
Use

..................................................................................................................................
..................................................................................................................................
............................................................................................................................. [2]
(b) The map gives more information about tectonic activity in Italy.

ALPS

NE

NI

EN

AP
Rome

LAquila
MO
UN
TA
INS
Naples

Eurasian Tectonic
Plate

African
Tectonic
Plate

key:
major plate boundary
fault lines (large cracks)
cities
Mt. Vesuvius volcano

location of strong earthquakes since 1908

UCLES 2011

5014/12/O/N/11

[Turn over

692

12

BALOL (i) From the map, state the evidence which shows that the earthquake risk is high in
many areas of Italy.

For
Examiners
Use

..................................................................................................................................
..................................................................................................................................
..................................................................................................................................
..................................................................................................................................
............................................................................................................................. [3]
(ii)

Describe how earthquakes can cause great loss of life both immediately after the
main earthquake shock, and in the following days and weeks.
..................................................................................................................................
..................................................................................................................................
..................................................................................................................................
..................................................................................................................................
..................................................................................................................................
..................................................................................................................................
..................................................................................................................................
............................................................................................................................. [4]

(c) The most recent of the strong earthquakes marked on the map of Italy was centred in
the city of LAquila.
(i)

Why was the earthquake risk near LAquila particularly high?


..................................................................................................................................
............................................................................................................................. [1]

UCLES 2011

5014/12/O/N/11

693

13

BALOL(ii) Summary information about this earthquake is given in the box below.

For
Examiners
Use

LAquila earthquake
Date and time

April 6th 2009: 03.32 hrs local time

Strength

Richter Scale 6.3

Effects

* 294 dead; 1200 injured


* estimated 30,000 left homeless
* 15,000 buildings destroyed or damaged beyond repair
* churches and houses in the old centre were most badly damaged
* insurance companies estimated their losses at US$ 4bn.

Responses

* A massive search and rescue effort involving 1700 rescuers and aid
workers.
Civil Protection staff brought in sniffer dogs and heavy lifting gear.
The Air Force delivered blood plasma and flew out the wounded.
* Within two days, 31 tented cities with chemical toilets were giving
shelter to 18,000 homeless people. Train sleeper carriages were
brought and parked in railway sidings. Bus companies from other areas
sent 70 coaches to transport people to go to stay with relatives and
friends in other areas of Italy.
* An emergency fund of US$ 40 million was set up by the Government.

Where was the damage to buildings greatest? Suggest a reason why the buildings
here were so badly damaged.
..................................................................................................................................
..................................................................................................................................
............................................................................................................................. [2]
(iii)

Give reasons why sniffer dogs and chemical toilets were used in rescue and relief
efforts.
sniffer dogs
..................................................................................................................................
..................................................................................................................................
chemical toilets
..................................................................................................................................
............................................................................................................................. [2]

UCLES 2011

5014/12/O/N/11

[Turn over

694

14

BALOL(iv) Italy is a developed country. Were the effects of the earthquake and peoples
responses to it more like those of a developed or developing country?

For
Examiners
Use

With the help of the information given, explain as fully as you can.
..................................................................................................................................
..................................................................................................................................
..................................................................................................................................
..................................................................................................................................
..................................................................................................................................
..................................................................................................................................
..................................................................................................................................
............................................................................................................................. [4]

UCLES 2011

5014/12/O/N/11

695

15

BALOL
(d) Many survivors of the LAquila earthquake were angry about the amount of damage to
their homes. Look at some of the comments they made.

A Why did modern


buildings like the citys newest
hospital collapse? Please ask the
builders why they used substandard concrete and iron.

For
Examiners
Use

B We felt tremors for


weeks, and they were getting
stronger. Last week a severe jolt led to
schools being closed for two days, but
the Authorities said that it was just
normal tectonic activity.

C Weeks ago a
geologist put a message on
the Internet that there would soon be
an earthquake in LAquila, based on his
measurements of fault movements. The
Authorities accused him of spreading fear
and forced him to remove it, saying that
earthquakes cannot be predicted.

D You know, here


in Italy we have earthquakes,
we have laws to make all buildings
earthquake proof, but we also forget
about them. It is not in our culture
to build in an appropriate way in
earthquake-prone areas.

(i)

E In California, an
earthquake like this would not
have killed a single person.

Describe the methods used to reduce the risk of buildings collapsing and killing
people in places such as California.
..................................................................................................................................
..................................................................................................................................
..................................................................................................................................
..................................................................................................................................
............................................................................................................................. [3]

UCLES 2011

5014/12/O/N/11

[Turn over

696

16

BALOL(ii) Give reasons why damage to buildings still happens in earthquakes, even in a
developed country such as Italy.

For
Examiners
Use

..................................................................................................................................
..................................................................................................................................
..................................................................................................................................
..................................................................................................................................
..................................................................................................................................
..................................................................................................................................
..................................................................................................................................
............................................................................................................................. [4]
(iii)

Were the Authorities in Italy correct when they said that earthquakes cannot be
predicted? Explain your answer.
..................................................................................................................................
..................................................................................................................................
..................................................................................................................................
............................................................................................................................. [2]

(e) In the south of Italy, just 11 km east of Naples, is the volcano Vesuvius (look back to the
map of Italy for its location). The most famous eruption was almost 2000 years ago in
year 79 when it destroyed the Roman cities of Pompeii and Herculaneum, killing about
16,000 people. Today, there are up to 20 towns around Naples, with a total population of
over half a million people, who are living in the area at risk from another big eruption.
* The good news volcanoes often give warning signs before erupting
* The bad news predicting when an eruption will occur and how big it will be is not an
exact science
(i)

Large numbers of people live in some areas close to active volcanoes, such as
around the volcano Vesuvius. Suggest reasons for this.
..................................................................................................................................
..................................................................................................................................
..................................................................................................................................
..................................................................................................................................
..................................................................................................................................
............................................................................................................................. [3]

UCLES 2011

5014/12/O/N/11

697

17

BALOL
(ii)

Give an example of a warning sign which suggests a volcano might be about to


erupt.

For
Examiners
Use

..................................................................................................................................
............................................................................................................................. [1]

(f)

Choices facing the Authorities in Naples and the area around it


What to do if Vesuvius gives warning signs of a possible eruption

A
Evacuate half a million people in advance
perhaps for days, if not for weeks
(i)

B
Leave people to try to escape when the
big eruption happens

What are the disadvantages of each of policies A and B?


..................................................................................................................................
..................................................................................................................................
..................................................................................................................................
..................................................................................................................................
..................................................................................................................................

(ii)

Is one policy better than the other? Explain what you think.
..................................................................................................................................
..................................................................................................................................
..................................................................................................................................
..................................................................................................................................
............................................................................................................................. [5]
[Total: 40 marks]

UCLES 2011

5014/12/O/N/11

[Turn over

698

18
6 BALOL
(a) Information about world average water use is given below.

For
Examiners
Use

Water use world averages


water use per head
per year
(cubic metres)

water use by sector


(percentages)
domestic
8
(i)

industry
22

agriculture
70

626

Show the values for water use by sector on a pie graph.


World average water use, % by sector

key:

[3]
(ii)

Show the value for water use per head on the pictograph.
World average water use, per head per year

key:
100 cubic
metres
[1]

UCLES 2011

5014/12/O/N/11

699

19

BALOL
(b) World averages like these hide differences in water use between different continents.
Water use in five continents
mainly developing countries
water use

mainly developed countries


water use

domestic
%

industry
%

agriculture
%

Africa

88

Asia

85

12

16

72

continents

Central &
South
America
(i)

For
Examiners
Use

domestic
%

industry
%

agriculture
%

Europe

14

55

31

North
America

13

47

40

continents

Use the values to describe the main differences in water use between countries in
developing and developed continents.
..................................................................................................................................
..................................................................................................................................
..................................................................................................................................
..................................................................................................................................
............................................................................................................................. [3]

(ii)

Using values from the table, state the evidence for the importance of water for
agricultural use in continents which have mainly developing countries.
..................................................................................................................................
............................................................................................................................. [1]

(iii)

Suggest reasons for the great importance of water use for agriculture in the
developing world continents.
..................................................................................................................................
..................................................................................................................................
..................................................................................................................................
..................................................................................................................................
..................................................................................................................................
............................................................................................................................. [3]

UCLES 2011

5014/12/O/N/11

[Turn over

700

20

BALOL
(c) Farmers in all continents make use of underground water supplies. Two examples of
underground water stores are shown in the diagrams below.

For
Examiners
Use

Artesian basins
well

water
table

well

water table
key:
Impermeable rocks
Clay

Underground water store

Granite
Permeable rocks
Limestone
Sandstone
(i)

The water table is the level below which the rocks are saturated with water (full of
water). On both diagrams, shade in the areas of rocks where underground water is
stored. Complete the key.
[2]

(ii)

Describe how in both diagrams the layout of the rocks leads to the formation of
underground water stores.
..................................................................................................................................
..................................................................................................................................
..................................................................................................................................
..................................................................................................................................
..................................................................................................................................
..................................................................................................................................
............................................................................................................................. [4]

UCLES 2011

5014/12/O/N/11

701

21

BALOL(iii) Explain why the rock outcrops marked A are essential for the formation of these
underground water stores and for continued water use by farmers.

For
Examiners
Use

..................................................................................................................................
..................................................................................................................................
..................................................................................................................................
............................................................................................................................. [2]
(iv)

Are the two wells marked on the diagrams located in the best positions for farmers
to obtain water from these underground stores? Explain as fully as you can.
..................................................................................................................................
..................................................................................................................................
..................................................................................................................................
..................................................................................................................................
............................................................................................................................. [3]

UCLES 2011

5014/12/O/N/11

[Turn over

702

22

BALOL
(d) Look at the information about water supply on a farm in northern India.
Ashok is a vegetable farmer in the Punjab with one hectare of land. It is a
family farm. During his lifetime he has seen many changes. One of these
is water supply for the farm and family. In 2007 Ashok invested Rs 100,000
(about US$ 2000) building a new bore well and installing a diesel pump. Many
rice farmers near Ashok are doing the same.
1970
Familys old well

For
Examiners
Use

2007
Ashoks new well
ground surface

0
water table

depth below surface (metres)

10

20

water table

30

40

50
key:
60

water table
well

(i)

By how many metres has the level of the water table dropped between 1970 and
2007?
............................................................................................................................. [1]

UCLES 2011

5014/12/O/N/11

703

23

BALOL(ii) Why has the cost of obtaining underground water increased greatly for Ashok and
other farmers in this area?

For
Examiners
Use

..................................................................................................................................
..................................................................................................................................
..................................................................................................................................
............................................................................................................................. [2]
(iii)

Is this an example of sustainable or unsustainable use of underground water


supplies? Explain your answer.
..................................................................................................................................
..................................................................................................................................
..................................................................................................................................
............................................................................................................................. [2]

(e) In summer 2009 the monsoon rains in many parts of India were poor, well below average.
Water levels in many reservoirs fell to 11 per cent of total storage capacity, compared
with expected water levels of about 26 per cent at this time of year.
One way of increasing water storage in countries such as India is by building new
dams.
(i)

State the physical conditions needed for building a large dam and reservoir.
..................................................................................................................................
..................................................................................................................................
..................................................................................................................................
..................................................................................................................................
..................................................................................................................................
............................................................................................................................. [3]

UCLES 2011

5014/12/O/N/11

[Turn over

704

24

BALOL(ii) Often people living in the area are not in favour of new large dams being built.
Explain why.

For
Examiners
Use

..................................................................................................................................
..................................................................................................................................
..................................................................................................................................
..................................................................................................................................
..................................................................................................................................
............................................................................................................................. [3]
(iii)

Look at the information about population and water resources in India.

Population

River run-off

Renewable water
resources per head

Water use

Employment

1,100 million

4% of world
total water flow

1800 cubic metres

93% for
agriculture

Two thirds of
Indias population
depends on farming

14% of total
world population

World average
6900 cubic metres

Natural increase
15.3 per 1000
Some experts are predicting that India will face a major water crisis by 2025. How
strongly does the information support this prediction?
..................................................................................................................................
..................................................................................................................................
..................................................................................................................................
..................................................................................................................................
..................................................................................................................................
............................................................................................................................. [3]

UCLES 2011

5014/12/O/N/11

705

25

BALOL(iv) Describe what farmers can do to reduce water use while trying to maintain levels of
food output from their farms.

For
Examiners
Use

..................................................................................................................................
..................................................................................................................................
..................................................................................................................................
..................................................................................................................................
..................................................................................................................................
..................................................................................................................................
..................................................................................................................................
............................................................................................................................. [4]
[Total: 40 marks]

UCLES 2011

5014/12/O/N/11

706

BALOL
UNIVERSITY OF CAMBRIDGE INTERNATIONAL EXAMINATIONS
GCE Ordinary Level

MARK SCHEME for the October/November 2011 question paper


for the guidance of teachers

5014 ENVIRONMENTAL MANAGEMENT


5014/12

Paper 1, maximum raw mark 120

This mark scheme is published as an aid to teachers and candidates, to indicate the requirements of
the examination. It shows the basis on which Examiners were instructed to award marks. It does not
indicate the details of the discussions that took place at an Examiners meeting before marking began,
which would have considered the acceptability of alternative answers.
Mark schemes must be read in conjunction with the question papers and the report on the
examination.

Cambridge will not enter into discussions or correspondence in connection with these mark schemes.

Cambridge is publishing the mark schemes for the October/November 2011 question papers for most
IGCSE, GCE Advanced Level and Advanced Subsidiary Level syllabuses and some Ordinary Level
syllabuses.

707

Page 2

Mark Scheme: Teachers version


GCE O LEVEL October/November 2011

Syllabus
5014

Paper
12

BALOL
Section A
1

(a) (i) becomes larger with depth / smaller near the surface;

[1]

(ii) layer 4;

[1]

(iii) the (solid) rock / weathered from the rock;

[1]

(iv) rock broken up by weathering;


more time / greater length for weathering nearer the surface;

[2]

(v) pore space large;


drainage good / free / fast;
air content (usually) high / large;
3 @ 1 mark

[3]

(b) drainage; details of method;


mix in lighter materials; such as sand;
farmers working on the surface by methods such as; forking, harrowing, ploughing;
Answers such as these (method plus description).

[2]
[Total: 10]

(a) wide continental shelf;


shallow waters with available light (for photosynthesis);
waters contain minerals;
some brought from the land by rivers;
some carried in ocean currents;
especially where upwelling or meeting of currents occurs;
makes them rich in plankton;
plankton are food for fish;
Four points made along these lines. 4 @ 1 mark

[4]

(b) answer within 4244 (%);

[1]

(c) (i) 70(%);

[1]

(ii) highest / much greater than all the others;

[1]

(d) great / wasteful loss of fish life;


not sustainable;
includes young fish / small fish reducing remaining fish stocks;
endangered species also caught;
deaths may lead to species extinction;
bycatch threatens income / jobs / food supply for the future;
Three points made such as these. 3 @ 1 mark

[3]
[Total: 10]

708

Page 3

Mark Scheme: Teachers version


GCE O LEVEL October/November 2011

Syllabus
5014

Paper
12

BALOL
3

(a) (i) towards the west / west;


(ii) 800900 (km);

[1]
[1]

(iii) overall almost circular / oval shape to cloud cover;


no cloud in the eye / in the centre;
becomes less dense away from the centre;
overall pattern of concentric zones of different density;
2 @ 1 mark

[2]

(iv) pressure very low / low / suggested value;


warm air rises;
2 @ 1 mark

[2]

(b) weather warnings


strong / damaging / dangerous winds;
heavy rain / large amounts of rain;
winds leading to high tides / big waves;
leading to high risk of flooding (especially in coastal areas);
Maximum 2 marks for this part.
advice
board up windows;
evacuate from coastal areas / to areas of higher ground;
move family to cyclone shelters / stronger brick or concrete buildings;
store essentials such as food, water, candles;
listen to radio for further advice;
fishermen not to go out to sea;
farmers to move animals to higher ground;
Maximum 3 marks for this part.
4 @ 1 mark

[4]
[Total: 10]

(a) (i) greater variety of vegetation in foreground;


compared with coniferous forest of all the same trees and same height;
different shades of green compared with all dark green colour of conifers;
some trees without leaf / deciduous compared with all in leaf / evergreen;
further description about types of vegetation in foreground contributing towards variety;
further description of coniferous trees based on what can be seen on the photograph;
Points such as these. 3 @ 1 mark

[3]

709

Page 4

Mark Scheme: Teachers version


GCE O LEVEL October/November 2011

Syllabus
5014

Paper
12

BALOL
(ii) bare soil / ground easily eroded;
rapid run-off on these steep slopes;
no plant roots to hold soil in place;
no vegetation to intercept rain / reduce surface impact;
no longer any nutrient recycling to keep soil fertile;
open to wind and water erosion made worse by steep relief;
Points such as these. 3 @ 1 mark

[3]

(b) replanting immediately after felling;


selective logging of mature / valuable trees;
practising agro-forestry;
community forestry;
planting fast growing trees for fuelwood;
active management / regulations protecting against illegal logging;
Points such as these (credit use of examples and further description).
4 @ 1 mark

[4]
[Total: 10]
Section B

(a) (i) attempt to shade or colour in the plate boundary in the middle of the Atlantic Ocean;

[1]

(ii) in the north / countries next to the Mediterranean Sea;


closest to the destructive plate boundary;
further information about what happens at destructive boundaries to cause earthquakes;
most of Africa located in the middle of the plates away from activity zones;
Three points made along these lines. 3 @ 1 mark

[3]

(iii) magma from the mantle reaches the surface;


in the crack / fault / space between plates as they move apart;
builds up mountain ranges / forms volcanoes / rift valleys;
new rocks / new land formed (hence the name constructive);
Two points like these showing understanding and no confusion with destructive.
2 @ 1 mark
[2]
(b) (i) direct map evidence relevant to the answer
to the south is the major plate boundary where Eurasian and African plates meet;
fault lines run north-south and east-west through the Apennine mountains;
locations shown for six major earthquakes since 1908 / in the last 100 years;
shown in most parts of Italy / widespread throughout the country;
Three relevant statements like these. 3 @ 1 mark

[3]

710

Page 5

Mark Scheme: Teachers version


GCE O LEVEL October/November 2011

Syllabus
5014

Paper
12

BALOL
(ii) immediately after the main shock
people hit by falling debris from collapsing buildings, bridges etc.;
people trapped alive but unable to be rescued under weight of fallen debris;
fires spread due to explosions from gas pipes / damage to electrical wires;
mud flows / landslides triggered off engulfing houses / villages / all in their path;
out at sea trigger off tsunamis which destroy low lying coastal regions;
in the following days and weeks
further after shocks may cause already weakened structure to collapse;
spread of diseases from disruption of water supplies / sewage systems;
examples of water related diseases leading to epidemics such as cholera, typhoid;
inadequate food / relief operations outside major urban areas;
Four points such as these. Minimum one mark for each of immediate and following.
4 @ 1 mark
[4]
(c) (i) at the place where north/south and east/west fault lines cross;

[1]

(ii) where the old centre of L'Aquila;


why old buildings either frail because of their age or unlikely to have been built using
techniques to make buildings earthquake-proof;
2 @ 1 mark

[2]

(iii) sniffer dogs to smell people still alive but trapped under the debris, so that rescuers
know where to concentrate efforts;
chemical toilets to reduce the risk of the spread of disease, high with so many people
(18 000) concentrated in one place;
One mark for each.

[2]

711

Page 6

Mark Scheme: Teachers version


GCE O LEVEL October/November 2011

Syllabus
5014

Paper
12

BALOL
(iv) Responses to earthquake are much more like those of a developed country
massive scale of the search and rescue operation, ready with what was needed for
an earthquake such as sniffer dogs, heavy lifting gear, blood plasma and planes
great scale of relief operations, for as many as 18,000 people; accommodation, and
transport elsewhere, soon provided
government could afford to set an emergency fund as big as US$40 million.
Comment all of these suggest what would be expected from a developed country,
ready and prepared, which had the organisation / infrastructure and money to put plans
into effect. Indications could be given about how things would be different in developing
countries.
Effects of earthquake possible to argue that these were sufficiently severe for a 6.3
earthquake to be more like those expected in a developing country.
Mainly direct use of information about responses, with minimum of comment in terms of
likelihood for developing and developed countries, or an entirely developing country
answer with support = 1 or 2 marks.
Information selected forms the basis for comment, well directed towards differences
between developed and developing world earthquake responses (i.e. the theme of the
question) = 3 or 4 marks.
[4]
(d) (i) steel structures within the frame of the building for reinforcement;
damping and bracing systems to help absorb shocks;
foundation piles made out of alternate layers of steel and rubber for flexibility;
tall skyscrapers built in pyramid-like shapes;
use of high quality steel and building materials;
existence of building regulations which are enforced;
Three points made along these lines. 3 @ 1 mark

[3]

(ii) Can have all the know-how about making buildings earthquake-proof, but unless they
are put into practice by builders and monitored by the authorities, people will try to save
money, because earthquake-proof methods involve extra costs of construction. This is
why with side by side buildings, one collapses, the other remains standing, all due to
human nature and greed. One comment suggests that doing things by the rules is not
the norm, even in a developed country like Italy. People believing that it won't happen to
them is a common human attitude.
Some earthquakes can be so strong, especially those above 7.5 magnitude, that some
damage will occur anyway, even in places with well constructed buildings. Also buildings
directly on fault lines where the ground opens up / splits apart stand less chance of
surviving big movements.
Many old buildings still exist in developed world cities; these may have been weakened
by age or previous earth movements.
Although buildings are well constructed, they may not have been effectively zoned from
other land uses, e.g. industrial units like oil refineries which explode / catch fire, which
spreads to affect them.
Reasons used such as these; reward elaboration and answer development. At least two
different reasons needed for answers worth more than three marks.
[4]

712

Page 7

Mark Scheme: Teachers version


GCE O LEVEL October/November 2011

Syllabus
5014

Paper
12

BALOL
(iii) points that can be made:
it is impossible to predict exact location / time / strength of an earthquake
sometimes light shocks relieve the tension of movement along the faults and no
major quake occurs; in this example they seemed to indicate something bigger was
going to happen but who knows?
however, it is possible to predict places at high risk of a major earthquake like
L'Aquila, which is perhaps why the Italian rescue and relief authorities were
prepared.
Marks for explanation rather than candidate opinion.
Well explained / good understanding = 2 marks.
Some explanation, but not full or clear enough for both marks = 1 mark.

[2]

(e) (i) likely reasons and possible elaborations:


fertile soils; lava / ash constantly being weathered to release new minerals;
mineral deposits such as sulphur;
tourist potential: hot springs, mud pools; special attractions such as preserved old
Roman cities;
geothermal power; industrial potential for smelting metals;
Reasons stated and elaborated on along these lines. Fertile soils is the most important.
Generally a minimum of two reasons needed for full marks, but a well elaborated and
exampled soil answer might be worth all three marks.
[3]
(ii) warning signs such as:
temperature increases in and around the cone;
increased emissions of steam and gases from cone / top of volcano;
small earthquake shocks felt in surrounding areas;
Any one.
(f)

[1]

(i) Disadvantages of Policy A could go to enormous cost / create massive disruption to


normal life, all for nothing. Houses etc. left behind might be looted. Getting people to
cooperate before an event is very difficult. Can the expense and inconvenience be
justified just by warning signs, not a definite prediction?
Disadvantages of Policy B half a million people is too many to move within hours, and
to provide rescue and relief services (numbers can be compared with the 18 000
homeless in L'Aquila earthquake).
(ii) Policy A it should save lives and stop a repeat of the massive losses of life that
resulted after the eruption in year 79. Loss of life could be much greater with half a
million people now living here.
Policy B only responding after the known event, and can prepare for evacuation with
practice drills and education / increasing awareness of residents in the area.
Mark both parts together.
Briefly stated advantages and disadvantages. View about which one is better weakly
explained, whatever the view = 13 marks.
At least one advantage clearly stated for each of policies A and B. View clear and well
supported with explanation = 45 marks.
[5]
[Total: 40]

713

Page 8

Mark Scheme: Teachers version


GCE O LEVEL October/November 2011

Syllabus
5014

Paper
12

BALOL
6

(a) (i) Accurate plot of pie graph = 2 marks.


Only one (or two) sectors correct = 1 mark.
Key completed to match pie graph = 1 mark.
(ii) 6 figures, or 6 figures and about one quarter of another, filled in = 1 mark.

[3]
[1]

(b) (i) differences


main water use: developing continents agriculture, developed industry;
more domestic consumption in developed although size of difference between
developed and developing is less marked than for agriculture;
significantly greater industry use in developed than developing;
Two differences = 2 marks.
Supporting and relevant use of values for at least one of them = 1 mark.

[3]

(ii) for example in Africa and Asia only up to 15% of water is consumed in other sectors or
similar;
Any use of values for the three continents together, or for one individual continent, which
emphasises dominance of use for agriculture for 1 mark.
[1]
(iii) many people / more than half workforce in many countries are farmers;
most are subsistence farmers needing to feed big / increasing families;
need to grow two or more crops per year including one in the dry season;
some of the developing continents have large desert areas (e.g. Sahara in Africa);
economies of many developing countries rely on export of crops to developed world;
population pressure / droughts increasing the need to use more irrigation water;
any named examples which support points mentioned above;
Three points made along these lines. 3 @ 1 mark

[3]

(c) (i) Attempt to shade in the areas of sandstone and limestone below the indicated water
table levels on both diagrams = 1 mark.
Shading / colouring used on diagrams shown in the box in the key, provided that some
part of the shading on the diagram matches question need = 1 mark.
[2]
(ii) Permeable rocks (limestone and sandstone) are surrounded by impermeable rocks
(clay, and clay and granite) / permeable have impermeable rocks above and below
them,
significance of this explained such as water stored in the spaces in permeable rocks
cannot escape because of layers of rocks which do not allow water to pass through.
Layers of permeable rock outcrop on the surface so that they can be filled with rainwater,
perhaps a suggestion that they are higher / upland areas likely to receive more rainfall.
Rocks are downfolded or slope at an angle allowing rainwater to flow deeper
underground through the spaces (until trapped by impermeable layers).
One mark for identifying a way in which the rock lay out favours water store formation.
Further mark (or possibly marks) for describing more fully how for each one. Any
combination of one and two mark answers about lay out up to four marks.
[4]

714

Page 9

Mark Scheme: Teachers version


GCE O LEVEL October/November 2011

Syllabus
5014

Paper
12

BALOL
(iii) it is the only way that the permeable layers can be filled up / replenished;
elsewhere rainwater falling on the impermeable rocks will runoff over surface;
so water table levels would keep falling with use without this new rainwater;
Understood and clearly explained = 2 marks.
Some understanding without a full explanation = 1 mark.

[2]

(iv) The general answer is yes so all the marks are for explanation.
yes because:
at the deepest water point in the rock in the centre of the downfold;
almost at deepest point in the second diagram but not too deep;
well would need to be longer / deeper if sunk at the end of permeable layer;
water taken out will flow down to bottom / will stay below the water table longest;
relatively shallow wells in relation to amount / reliability of water supply obtained;
Points made along these lines. 3 @ 1 mark
(d) (i) 25 metres;

[3]
[1]

(ii) new well is 40 metres deeper than the old well;


more expensive to construct (100 000 rupees spent in 2007);
also more costly to obtain the water out of the well because of its depth which is why a
diesel pump needed to be installed;
Two explanatory points along these lines demonstrating understanding.

[2]

(iii) Most of the evidence points towards unsustainable:


having to go deeper and deeper to reach water supplies suggesting rate of water
use is greater than rate of replenishment = basic answer
if many other rice farmers are doing the same, then how long will the new well last?
there could also be comment about the high water needs of growing a crop such as
rice.
An answer of sustainable can achieve at best 1 mark for comment on the great depth
which might provide enough water for at least the next generation.
Answer clearly explained showing understanding = 2 marks.
Some, but not full, explanation for chosen answer = 1 mark.

[2]

(e) (i) relief narrow, deep, steep sided valley is ideal, often found in the mountains, natural
lake present;
climate wet, high total rainfall, heavy rainfall, melting ice / snow for surface runoff / low
rates of evaporation;
river large river with a much water from its many tributaries;
Three physical advantages stated all three could be for relief or climate. 3 @ 1 mark [3]

715

Page 10

Mark Scheme: Teachers version


GCE O LEVEL October/November 2011

Syllabus
5014

Paper
12

BALOL
(ii) best farmland / houses / villages found on valley floor which will be flooded;
economic disadvantages such as loss of best farmland / livelihoods;
social disadvantages such as forced movement / break-up of communities;
promises about new homes, farms etc. not always kept;
dam and its water supply are often of no advantage to people living nearby;
Accept other possible lines of answering provided that they relate to local people, e.g.
loss of ecosystems for human use but not for wildlife or broader environmental concerns.
Three points made along these lines. 3 @ 1 mark

[3]

(iii) information capable of being used to support the prediction:


India for its population size is not water rich (14.5% of world population compared with
only 4% of world total water flows);
another statistic which reflects this is that it has only about a quarter of the renewable
water resources per head compared with the world average;
yet its water use at 93% for agriculture is higher than all the previously quoted sector
percentages for continents at the start of the question;
and the bulk of its population which continues to increase relies on farming;
Strong case made with appropriate comment for indicated prediction strength = 3 marks.
Less strong case but still with appropriate comment = 2 marks.
Reliance on information given without meaningful comment = 1 mark.
[3]
(iv) one way is use of irrigation methods that consume less water such as trickle drip and
clay pot With further details can be worth up to 3 marks.
another is to change to crops which need less water, e.g. from rice towards other cereals
or dry rice, or to use seeds which need less water to grow, e.g. drought GM varieties
Well described up to 3 marks.
use some dry farming techniques so that available water can be spread to cultivate
crops over a wider area With further details up to 3 marks.
Mark other suggestions about ways on their merit, provided that they match question needs
some may only be worth only up to 2 marks. Breadth from two ways needed for four
marks. Point mark for ways and information about them up to three marks per way.
[4]
[Total: 40]

716

BALOL
UNIVERSITY OF CAMBRIDGE INTERNATIONAL EXAMINATIONS
General Certificate of Education Ordinary Level

* 8 5 1 9 9 3 0 1 5 0 *

5014/12

ENVIRONMENTAL MANAGEMENT
Paper 1

May/June 2012
2 hours 15 minutes

Candidates answer on the Question Paper.


Additional Materials:

Ruler

READ THESE INSTRUCTIONS FIRST


Write your Centre number, candidate number and name on all the work you hand in.
Write in dark blue or black pen.
You may use a soft pencil for any diagrams, graphs or rough working.
Do not use staples, paper clips, highlighters, glue or correction fluid.
DO NOT WRITE IN ANY BARCODES.
Answer all questions.
All questions in Section A carry 10 marks.
Both questions in Section B carry 40 marks.
At the end of the examination, fasten all your work securely together.
The number of marks is given in brackets [ ] at the end of each question or
part question.

For Examiners Use


1
2
3
4
5
6
Total

This document consists of 23 printed pages and 1 blank page.


UCLES 2012

[Turn o

717

BALOL

Section A

For
Examiners
Use

(a) Look at the map showing the location of Iceland on a constructive (divergent) plate
boundary.
66N

66N
Arctic Circle

Iceland

North
American
Plate

Eurasian Plate
N
Atlantic
Ocean
0

50

100

150

200

km
Key
plate boundary

Arctic Circle

active volcano

Hveragerdi

(i)

On the map, draw arrows to show the likely directions of movement of the two
named plates.
[1]

(ii)

At Hveragerdi, tropical fruit and vegetables are grown in hothouses heated by


steam and hot water which rise from hot rocks. Explain why there are hot rocks
near the surface in Iceland.
..................................................................................................................................
..................................................................................................................................
..................................................................................................................................
.............................................................................................................................. [2]

(iii)

Iceland is located close to the Arctic Circle. How many kilometres due south of the
Arctic Circle is Hveragerdi? Circle the nearest answer to your measurement.
225

UCLES 2012

250

275

300

[1]

5014/12/M/J/12

718

BALOL(iv) Why is the use of rising hot water in hothouses of great economic benefit at this
high latitude?

For
Examiners
Use

..................................................................................................................................
.............................................................................................................................. [1]
(v)

Suggest two other different uses of the hot water and steam.
1 ...............................................................................................................................
2 ........................................................................................................................... [2]

(b) Describe the possible disadvantages of living near a plate boundary.


..........................................................................................................................................
..........................................................................................................................................
..........................................................................................................................................
..........................................................................................................................................
..........................................................................................................................................
...................................................................................................................................... [3]

UCLES 2012

5014/12/M/J/12

[Turn over

719

4
2 BALOL
(a) The diagram shows the results of an explosion at an oil well on the sea bed in the Gulf
of Mexico in April 2010.
coast with
wetlands,
tourist beaches
and fishing ports

1000

sea surface

0
oil spill

sea

1000

sea bed

2000

2000
rock

3000

3000

oil bearing rock


4000

4000
rock

5000

5000

depth below sea level (metres)

depth below sea level (metres)

wind
0

For
Examiners
Use

site of explosion
(i)

How did the location of this well make this oil leak very difficult to stop?
..................................................................................................................................
.............................................................................................................................. [1]

(ii)

Explain how the oil spill from the sea bed reached the coast.
..................................................................................................................................
..................................................................................................................................
..................................................................................................................................
.............................................................................................................................. [2]

(iii)

The divided bar graph shows the estimated costs which resulted from this oil spill.
cost (billion US $)

key
7

cost to:
fisheries
tourism
restoring
wetlands

What is the estimated cost of restoring the wetland ecosystems?


................................. billion US $

UCLES 2012

[1]

5014/12/M/J/12

720

BALOL
(b) (i) Animal plankton take in the oil. Add arrows to the diagram below to show how the
oil spreads to other organisms in the food chain.
oil

large fish

plankton

shrimps

For
Examiners
Use

[1]
(ii)

State two other ways in which ecosystems are damaged by oil spills.
1 ...............................................................................................................................
2 ........................................................................................................................... [2]

(c) Look at the diagram showing methods used to prevent this oil spill from reaching the
coasts of the USA.
spraying chemicals on the spill

setting fire to the oil spill

methods
used

skimming the oil up

putting floating barriers near the coast

For three of these methods suggest how each can help to reduce the impact of an oil
spill.
..........................................................................................................................................
..........................................................................................................................................
..........................................................................................................................................
..........................................................................................................................................
..........................................................................................................................................
...................................................................................................................................... [3]

UCLES 2012

5014/12/M/J/12

[Turn over

721

6
3 BALOL
(a) The bar graph shows amounts of nitrous oxide in the atmosphere at one hundred year
intervals from 1700.
The expected amount of nitrous oxide in the atmosphere in 2030 is 340 parts per
billion (ppb).
Add a bar to the diagram to show this.
[1]

340

340

335

335

330

330

325

325

320

320

315

315

310

310

305

305

300

300

295

295

290

290

285

285

280

280

0
1700

1750

1800

1850

1900

1950

2000

nitrous oxide in the atmosphere (ppb)

nitrous oxide in the atmosphere (ppb)

(i)

For
Examiners
Use

0
2050

year
(ii)

What is significant about what the graph shows?


..................................................................................................................................
.............................................................................................................................. [1]

UCLES 2012

5014/12/M/J/12

722

BALOL(iii) Natural sources supply 76% of the nitrous oxide in the atmosphere and the
remainder is from human activities. Show this on the pie graph below.

For
Examiners
Use

[1]
(b) Explain different ways in which oxides of nitrogen can reach the atmosphere as a result
of:
(i)

commercial agriculture .............................................................................................


.............................................................................................................................. [1]

(ii)

transport ...................................................................................................................
.............................................................................................................................. [1]

(c) Look at the diagram of different temperature conditions in the atmosphere.


A

B
layer of warm air

layer of cold air

layer of cold air

layer of warm air


Earths surface

(i)

Which diagram shows a temperature inversion? Explain your answer.


diagram letter ............
reason .................................................................................................................. [1]

(ii)

Explain how a temperature inversion can lead to an increase in atmospheric


pollution.
..................................................................................................................................
.............................................................................................................................. [1]

UCLES 2012

5014/12/M/J/12

[Turn over

723

BALOL
(d) Suggest reasons why farmers may prefer to use inorganic fertilisers rather than organic
ones.

For
Examiners
Use

..........................................................................................................................................
..........................................................................................................................................
..........................................................................................................................................
..........................................................................................................................................
..........................................................................................................................................
...................................................................................................................................... [3]

(a) The map shows the distribution of tundra vegetation which is in an area where the
environment is difficult for plant growth.

X
66 N

Arctic Circle

23 N
0
23 S

tundra vegetation

Describe the distribution of tundra vegetation shown on the map.


..........................................................................................................................................
..........................................................................................................................................
..........................................................................................................................................
..........................................................................................................................................
..........................................................................................................................................
...................................................................................................................................... [3]

UCLES 2012

5014/12/M/J/12

724

BALOL
(b) Look at the cross section of a valley in the tundra showing vegetation in the summer.
North

lichens, mosses
and flowering
plants

heather

cotton
grass

South
lichens, mosses
and grass

For
Examiners
Use

thin layer
thaws in
summer

lake
permanently frozen rock

(i)

Use the information on the diagram to explain the term plant community.
..................................................................................................................................
.............................................................................................................................. [1]

(ii)

How and why is the vegetation on the south facing slope different from that on the
north facing slope?
..................................................................................................................................
..................................................................................................................................
..................................................................................................................................
.............................................................................................................................. [2]

(iii)

Describe the roots of plants in the tundra.


.............................................................................................................................. [1]

(c) For centuries, nomadic pastoralists have roamed with their reindeer herds in area X on
the world map on page 8. Tourism is now increasing in the western part of area X.
Suggest why the nomadic pastoralists might have a variety of opinions about the
increase of tourism.
..........................................................................................................................................
..........................................................................................................................................
..........................................................................................................................................
..........................................................................................................................................
..........................................................................................................................................
...................................................................................................................................... [3]

UCLES 2012

5014/12/M/J/12

[Turn over

725

10

BALOL

Section B

For
Examiners
Use

(a) Look at the sketch of a weather station.


A
B

D
C

(i)

Name the weather elements recorded by the instruments marked AD.


A ...............................................................................
B ...............................................................................
C ...............................................................................
D ................................................................................

(ii)

[4]

Describe how the highest and lowest temperatures of the day are measured and
recorded at a weather station.
..................................................................................................................................
..................................................................................................................................
..................................................................................................................................
..................................................................................................................................
.............................................................................................................................. [3]

UCLES 2012

5014/12/M/J/12

726

11

BALOL
(b) Look at the climate graph for Manila in the Philippines in East Asia.

For
Examiners
Use

precipitation (mm)

temperature (C)

Average temperature and precipitation in Manila (latitude 14 N)


30

30

25

25

20

20

15

15

10

10

500

500

400

400

300

300

200

200

100

100

(i)

Complete the climate graph by plotting these monthly temperature values for Manila
using a line graph.

J
25

F
26

M
27

A
29

M
29

J
28

J
27

A
27

S
27

O
27

N
26

D
25
[3]

(ii)

State the annual temperature range in Manila (the difference between the highest
and lowest monthly temperatures).
.............................................................................................................................. [1]

(iii)

Describe what the graph shows about the distribution of rainfall during the year in
Manila.
..................................................................................................................................
..................................................................................................................................
.............................................................................................................................. [2]

UCLES 2012

5014/12/M/J/12

[Turn over

727

12

BALOL(iv) Suggest a reason why the temperature in June and July is lower than in April,
although the sun is shining from a higher angle in the sky in mid-summer.

For
Examiners
Use

..................................................................................................................................
.............................................................................................................................. [1]
(c) (i)

Farming in the Philippines is dominated by crop growing. Describe how the climate
is favourable for crop growing.
..................................................................................................................................
..................................................................................................................................
..................................................................................................................................
..................................................................................................................................
.............................................................................................................................. [3]

(ii)

Rice is the most important food crop grown in the Philippines, the staple food in
most of the country. Along with corn (maize) it accounts for half of the total crop
area. Another quarter of cropland is planted with coconuts, an important export
earner. Smaller amounts of sugar cane and pineapples are also exported.
From the crops named, give one example of a subsistence crop and one example
of a commercial crop in the Philippines.
subsistence crop ...............................................
commercial crop ................................................

(iii)

[1]

What are the main differences between subsistence and commercial types of
farming?
..................................................................................................................................
..................................................................................................................................
..................................................................................................................................
.............................................................................................................................. [2]

(iv)

The system of farming often used for growing crops such as coconuts and sugar
cane is plantation farming.
Describe some of the distinctive characteristics of plantation farming.
..................................................................................................................................
..................................................................................................................................
..................................................................................................................................
..................................................................................................................................
.............................................................................................................................. [3]

UCLES 2012

5014/12/M/J/12

728

13

BALOL
(d) Look at the world map showing main areas of formation and tracks (directions of
movement) of cyclones.

For
Examiners
Use

Cyclones: main areas of formation and tracks followed

Key:

(i)

Main areas of formation


of cyclones

Main tracks followed

Describe where the main areas of cyclone formation are located.


..................................................................................................................................
..................................................................................................................................
..................................................................................................................................
..................................................................................................................................
.............................................................................................................................. [3]

(ii)

What is similar about the tracks followed by cyclones as they move away from the
areas of formation?
..................................................................................................................................
..................................................................................................................................
.............................................................................................................................. [2]

(iii)

The main months of the year for the formation of tropical cyclones in the northern
hemisphere are between July and November. Give the reasons for this.
..................................................................................................................................
..................................................................................................................................
..................................................................................................................................
.............................................................................................................................. [2]

UCLES 2012

5014/12/M/J/12

[Turn over

729

14

BALOL
(e) Read the information about two cyclones in East Asia in 2009. Cyclones which form in
the South China Sea are called typhoons.
Philippines average income per head per
year US$1,040

Japan average income per head per year


US$36,170

Typhoon Ketsana September 2009


nearly 300 deaths
heaviest rains for over 40 years
floods affected over 80 percent of
Manila, the capital city
large parts of Laguna in eastern Manila,
where more than 300,000 people live in
shanty towns, were still flooded 12 days
later

Typhoon Melor October 2009


2 deaths from falling trees
at least 27 people injured
gusts of wind up to 198kph were
recorded
violent winds damaged homes and
uprooted trees; heavy rains increased
the risk of landslides

Comment vast flooding was largely


the result of insufficient and inadequate
drainage

Comment after warnings from the Weather


Office, many people were evacuated into
shelters by the Disaster Management
Agency before the cyclone arrived

General comment high numbers of


deaths every year from cyclones creates
a cycle of poverty; people are constantly
recovering from previous cyclones, making
it more difficult for them to afford to take
preventative measures ready for the next
one

General comment the threat of natural


disasters in developed countries like Japan
encourages technological improvement to
make the effects of the next cyclone less
damaging than those of previous cyclones

(i)

For
Examiners
Use

Explain why cyclones, like these two Asian typhoons, are dangerous for people and
can lead to considerable loss of life and injury.
..................................................................................................................................
..................................................................................................................................
..................................................................................................................................
..................................................................................................................................
..................................................................................................................................
.............................................................................................................................. [3]

UCLES 2012

5014/12/M/J/12

730

15

BALOL(ii) The UN (United Nations) estimates that

total numbers of people affected by cyclones each year in the Philippines and
Japan are similar,
17 times more people die in the Philippines from cyclones than in Japan.

For
Examiners
Use

With the help of the information given in the boxes, explain as fully as you can why
more people die in the Philippines from the effects of cyclones than in Japan.
..................................................................................................................................
..................................................................................................................................
..................................................................................................................................
..................................................................................................................................
..................................................................................................................................
..................................................................................................................................
..................................................................................................................................
..................................................................................................................................
..................................................................................................................................
.............................................................................................................................. [5]
(iii)

Is it ever going to be likely that all deaths from cyclones can be avoided? Explain
your view about this.
..................................................................................................................................
..................................................................................................................................
..................................................................................................................................
.............................................................................................................................. [2]
[Total: 40 marks]

UCLES 2012

5014/12/M/J/12

[Turn over

731

16
6 BALOL
(a) Look at the graph showing the importance of different energy sources in 2009.

For
Examiners
Use

World energy consumption by source (2009)


hydro electric and
other renewables
nuclear
biomass
and waste

oil

natural
gas

coal
(i)

On the graph, shade in the sectors showing energy from fossil fuels.

[1]

(ii)

Describe what the graph shows about the importance of fossil fuels for world
energy consumption in 2009.
..................................................................................................................................
..................................................................................................................................
..................................................................................................................................
.............................................................................................................................. [2]

(iii)

Approximately what percentage of total world energy consumption in 2009 came


from coal?
.............................................................................................................................. [1]

UCLES 2012

5014/12/M/J/12

732

17

BALOL
(b) Look at the diagrams which show one method used for mining coal.

For
Examiners
Use

Coal mine

surface
towers
A
other rocks

coal seam
shafts

train
coal cutter

Coal cutter

(i)

What is the purpose of the shafts and towers shown on diagram A?


..................................................................................................................................
..................................................................................................................................
.............................................................................................................................. [2]

(ii)

Using both diagrams, describe how the coal is being mined.


..................................................................................................................................
..................................................................................................................................
..................................................................................................................................
..................................................................................................................................
.............................................................................................................................. [3]

UCLES 2012

5014/12/M/J/12

[Turn over

733

18

BALOL(iii) Would you describe this as an old or a modern coal mine? Explain your answer.
..................................................................................................................................

For
Examiners
Use

..................................................................................................................................
..................................................................................................................................
.............................................................................................................................. [2]
(c) Mining is dangerous work. Read this newspaper report about what happened in a
Chinese coal mine in March 2010.

123 workers trapped by flooding


The vast Wangjialing coal mine
in northern China is estimated to
have 2.3bn tonnes of coal reserves,
including over 1 billion tonnes
of proved reserves. Yesterday
underground water rushed into
the mine where 261 people were
working. Only 138 managed to
escape the flood waters.
Chinas coal mines are well known
for being some of the worlds most
dangerous. Earlier this month,
(i)

25 people died in a coal mine fire


in central China. Last November,
108 men were killed when an
explosion blasted through a coal
mine belonging to another state
owned company. 2009 was a bad
year; there were two other explosions
which killed more than 50 workers.
Safety standards are often ignored
to try to meet the ever rising demand
for coal. Coal supplies 70 per cent of
Chinas energy needs.

State the four different reasons for the loss of life in Chinas coal mines, mentioned
in the newspaper report.
..................................................................................................................................
..................................................................................................................................
.............................................................................................................................. [2]

(ii)

Explain why the dangers of working in opencast coal mines are less than in deep
mines.
..................................................................................................................................
..................................................................................................................................
..................................................................................................................................
..................................................................................................................................
..................................................................................................................................
.............................................................................................................................. [3]

UCLES 2012

5014/12/M/J/12

734

19

BALOL(iii) Suggest reasons why mining safety is much better in some countries than in others.
..................................................................................................................................

For
Examiners
Use

..................................................................................................................................
..................................................................................................................................
..................................................................................................................................
..................................................................................................................................
..................................................................................................................................
.............................................................................................................................. [4]

UCLES 2012

5014/12/M/J/12

[Turn over

735

20

BALOL
(d) (i) Coal is often said to be a dirty fuel compared with other fuels. One type of pollution,
for which coal fired power stations are blamed most, is acid rain.
Name the gases from coal fired power stations which cause acid rain.

For
Examiners
Use

.............................................................................................................................. [2]
(ii)

The map shows acid rain and its effects in part of Europe. It was most serious in
the 1970s.
Acid rain in northern Europe
ICELAND

Norwegian Sea

SWEDEN
North Atlantic
Ocean

NORWAY
North Sea

nd
wi
UNITED
n
ai
KINGDOM m

on
cti
e
r
di

Baltic
Sea

NETHERLANDS

FRANCE

GERMANY

How does the map show that acid rain can be an international problem?
..................................................................................................................................
.............................................................................................................................. [1]
(iii)

Explain fully why the trees in the north of the UK on the map are shown in a different
way from those in Sweden.
..................................................................................................................................
..................................................................................................................................
..................................................................................................................................
..................................................................................................................................
..................................................................................................................................
.............................................................................................................................. [3]

UCLES 2012

5014/12/M/J/12

736

21

BALOL
(e) (i) The problem of acid rain in northern Europe is less now than it was in the 1970s.
Describe what has been done to reduce the problem of acid rain pollution from coal
fired power stations.

For
Examiners
Use

..................................................................................................................................
..................................................................................................................................
..................................................................................................................................
.............................................................................................................................. [2]
(ii)

Why is finding a solution to acid rain and other types of air pollution slower because
they are international problems instead of just being a national problem?
..................................................................................................................................
..................................................................................................................................
..................................................................................................................................
..................................................................................................................................
..................................................................................................................................
.............................................................................................................................. [3]

UCLES 2012

5014/12/M/J/12

[Turn over

737

22

BALOL
(f) Look at the divided bar graphs showing how electricity is produced in three north
European countries. (They are named on the map of acid rain).

For
Examiners
Use

Percentage of electricity production by source in Iceland, Norway and Sweden (2008)


100
90
80
Key:
70

HEP (hydro-electric)
geothermal

60

natural gas
percentage

50

nuclear

40

wind and other renewables

30
20
10
0
Iceland
(i)

Norway

Sweden

How much of Icelands electricity comes from renewables?


.............................................................................................................................. [1]

(ii) Look back to the pie graph of world energy consumption in part (a).
How important are renewables for electricity production in these three north European
countries compared with their importance in total world energy consumption?
..................................................................................................................................
..................................................................................................................................
..................................................................................................................................
..................................................................................................................................
.............................................................................................................................. [3]

UCLES 2012

5014/12/M/J/12

738

23

BALOL(iii) Give reasons why types of renewable energy sources, and amounts used, vary
greatly between different countries.

For
Examiners
Use

..................................................................................................................................
..................................................................................................................................
..................................................................................................................................
..................................................................................................................................
..................................................................................................................................
..................................................................................................................................
..................................................................................................................................
..................................................................................................................................
..................................................................................................................................
.............................................................................................................................. [5]
[Total: 40 marks]

UCLES 2012

5014/12/M/J/12

739

BALOL
UNIVERSITY OF CAMBRIDGE INTERNATIONAL EXAMINATIONS
GCE Ordinary Level

MARK SCHEME for the May/June 2012 question paper


for the guidance of teachers

5014 ENVIRONMENTAL MANAGEMENT


5014/12

Paper 1, maximum raw mark 120

This mark scheme is published as an aid to teachers and candidates, to indicate the requirements of
the examination. It shows the basis on which Examiners were instructed to award marks. It does not
indicate the details of the discussions that took place at an Examiners meeting before marking began,
which would have considered the acceptability of alternative answers.
Mark schemes must be read in conjunction with the question papers and the report on the
examination.

Cambridge will not enter into discussions or correspondence in connection with these mark schemes.

Cambridge is publishing the mark schemes for the May/June 2012 question papers for most IGCSE,
GCE Advanced Level and Advanced Subsidiary Level syllabuses and some Ordinary Level
syllabuses.

740

Page 2

Mark Scheme: Teachers version


GCE O LEVEL May/June 2012

Syllabus
5014

Paper
12

BALOL
General notes
Symbols used in Environmental Management mark schemes.
/

separates alternatives for a marking point other valid ways of expressing the same
idea are also credited

separates points for the award of a mark

[3]

indicates the number of marks available

[max 3]

the number shows the maximum number of marks available for the question where there
are more marking points than total marks available

[max 3]

when part of the marks of a question must come from part of the mark scheme, this is
indicated by non-bold marks showing the internal maxima for different parts of the
question
these non-bold marks are also used to show marks for bands where banded mark
schemes are used

italic

indicates that this is information about the marking points and is not required to gain
credit
italic text is also used for comments about alternatives that should be accepted, ignored
or rejected

ora

or reverse argument shows that an argument from an alternative viewpoint will be


credited

AW

alternative wording, sometimes called or words to that effect


AW is used when there are many different ways of expressing the same idea

( )

the word / phrase in brackets is not required to gain marks but sets the context of the
response for credit
e.g. (nuclear) waste nuclear is not needed but if it was described as a domestic waste
then no mark is awarded

volcanic

underlined words the answer must contain exactly this word

ecf

error carried forward if an incorrect answer is given to part of a question, and this
answer is subsequently used by a candidate in later parts of the question, this indicates
that the candidates incorrect answer will be used as a starting point for marking the later
parts of the question

741

Page 3

Mark Scheme: Teachers version


GCE O LEVEL May/June 2012

Syllabus
5014

Paper
12

BALOL
Section A
1

(a) (i) arrow on each plate pointing away from the plate boundary;

[1]

(ii) magma rises at this type of plate boundary;


Hveragerdi is close to / between two plate boundaries / near active volcanoes;
new land with many hot springs heated by magma reservoir below / large
reservoir of molten magma below the surface;
(iii) 275 or 300;

[max 2]
[1]

(iv) crops can be grown which can only grow (naturally) in hotter climates / the
climate in Iceland is too cold for such crops;
saves cost of importing (tropical) the crops / provides cheaper crops;

[max 1]

(v) geothermal energy;


heat / hot water for houses;
(outdoor) bathing pools / steam baths;
industries with high energy uses / smelting metals / aluminium;

[max 2]

(b) disadvantages include:


danger from / damage by earthquakes;
danger from / damage by volcanic eruptions;
both can occur without much warning / both unpredictable in behaviour;
economic costs of preparation for tectonic events;
detail of one of the above;

[max 3]
[Total: 10]

(a) (i) it was at great depth / it was about 1500 m below the surface;
visibility very poor (at great depth) / other problems such as water pressure;
(ii) oil rises because it is less dense than water;
forced out by release of pressure from within the rock;
winds blew the slick towards the coast;
moved by flow of ocean currents / tides;
(iii) 4;
(b) (i) arrowheads pointing from oil to plankton to shrimps to large fish;
(ii) ways include:
death of fauna;
oil coats birds preventing flight;
some elements in oil are toxic;
oil covers feeding areas;
death of plants;
oil clogs stomata;

[max 1]

[max 2]
[1]
[1]

[max 2]

742

Page 4

Mark Scheme: Teachers version


GCE O LEVEL May/June 2012

Syllabus
5014

Paper
12

BALOL
(c) setting fire to the spill burns the oil before it reaches the coast / affects wildlife;
spraying chemicals on the spill to disperse the oil / break up the slick making it easier for
currents / tides to carry oil way;
skimming the oil up removes it from the sea / allows it to be disposed of safely elsewhere;
floating barriers near the coast prevent to prevent slicks from reaching the populated areas /
coastal areas rich in fauna and flora;
[max 3]
[Total: 10]
3

(a) (i) plotting of bar to 340 ppb at 2030;


(ii) rapid / great increase / much higher in 2030;

[1]
[1]

(iii) pie graph showing 76% natural sources and 24% from human activity, and sectors
identified;
[1]
(b) (i) application of fertiliser;
farm vehicle / tractor exhausts;
(ii) from vehicle exhausts;

[max 1]
[1]

(c) (i) A because normally air is cooler with increased altitude;


A because the warmer air is above cooler air (which is not the usual situation); [max 1]
(ii) pollutants are trapped as (cold) air cannot rise into warmer air;
(d) reasons include:
cheaper;
easier to obtain;
easier to apply lower mass;
more pleasant / less smell from them / quicker release;
does not keep animals;
does not wish to rotate crops / grows few crop types;
can target nutrients needed;

[1]

[max 3]
[Total: 10]

743

Page 5

Mark Scheme: Teachers version


GCE O LEVEL May/June 2012

Syllabus
5014

Paper
12

BALOL
4

(a) Northern hemisphere (only);


north coasts of continents / of North America / of Asia;
many areas found north of the Arctic Circle;
more specific detail about locations;
supported by the of named locations such as Iceland and Southern Greenland;

[max 3]

(b) (i) plants growing together such as lichens, mosses and flowering plants /
lichens, mosses and grass;

[1]

(ii) How?
more varied plant life / has heather / has flowering plants / zones at different heights;
Why?
because warmer / sunnier / better aspect / sheltered from cold winds;
(iii) short / close to surface;

[2]
[1]

(c) opinions such as:


concerned because tourism
may lead to destruction / deterioration of pastures;
reduce the land area they can roam in;
may dilute their culture / bring bad influences;

[max 2]

in favour of tourism because


may gain income from work associated with tourism such as guides;
may sell more reindeer meat to hotels;
tourists may pay to visit them and their herds;
other opportunities such as selling handicrafts to them;

[max 2]

[max 3]
[Total: 10]

744

Page 6

Mark Scheme: Teachers version


GCE O LEVEL May/June 2012

Syllabus
5014

Paper
12

BALOL
Section B
5

(a) (i) A
B
C
D

wind direction;
wind speed;
rainfall / precipitation;
sunshine;

[4]

(ii) using maximum and minimum thermometers / Six's thermometer;


taking readings from the bottom of the metal indices in both thermometers;
doing this once a day and resetting the indices on top of the mercury and alcohol
columns;
placing the thermometers in the shade in the Stevenson's screen / white wooden box;
AVP e.g. further details about the maximum and minimum thermometers and how they
work and are read;
[3]
(b) (i) at least 10 accurate plots;
12 accurate plots centred;
Line used to link the values plotted;

[3]

(ii) 4 C;

[1]

(iii) descriptive material wet or very wet from May to November / in summer AND
(reasonably) dry/much drier from December / January to April / in winter;
clear recognition of two seasons;
no marks are separately reserved for quoting of monthly rainfall totals, but their inclusion
could help confirm recognition of the two seasons
[2]
(iv) more cloud cover / rainfall so less direct sunlight;
accept a more indirect answer referring to monthly rainfall totals (33 mm compared
with 254 mm and 432 mm)
(c) (i) all year growing season/high temperatures above 25 C all year;
huge amounts of summer rainfall (over 2000 mm);
ref. to storing some of this for crop use during the drier winter;
ref. to the importance of heat and water for crop growth;
ref. to potential for two or three crops a year;
(ii) subsistence crops rice / corn (maize) AND
commercial crops coconuts / sugar cane / pineapples;
both needed for the mark.

[1]

[max 3]

[1]

(iii)
subsistence
mainly for own consumption
small-scale / small farms
more reliance on human and animal
power
wider variety of crops / mixed farming
with animals
low inputs / investments

commercial
is for sale;
large-scale / large farms;
mechanised;
more specialised / perhaps one crop
plantations / monoculture;
high inputs / large investments;
[max 2]

it is possible that there will be two differences within one full two sided statement

745

Page 7

Mark Scheme: Teachers version


GCE O LEVEL May/June 2012

Syllabus
5014

Paper
12

BALOL
(iv) mainly grow only one crop / monoculture;
two or more examples of typical plantation crops e.g. bananas, sugar cane, pineapples,
coffee, tea;
large scale / cover big areas of land;
many owned by big companies / examples of foreign companies / multinationals;
ref. to high inputs including mechanisation / irrigation / use of chemical fertilisers /
pesticide / insecticide sprays;;
export orientated;
[max 3]
(d) (i) all form within / around the tropics;
in the Pacific Ocean off the coasts of SE Asia (or some named countries ) and Australia
/ AW;
in the Indian Ocean north of the Equator in Bay of Bengal and Arabian Sea / AW;
more extensive area of formation south of the Equator between Australia and Africa
/ AW;
in the Atlantic to the east the Caribbean and the south east of the USA / AW;
[max 3]
(ii) early direction of movement is mostly from east to west;
then curved tracks out of the tropics / towards more temperate latitudes,
all finish by tracking northwards in the northern hemisphere and southwards in the
southern hemisphere / towards the poles;
[max 2]
(iii) Sea water temperatures in areas of formation are at their highest (at least 25 C);
constantly rising warm moist air in the low pressure is what drives and sustains cyclones
/ more evaporation of water leading to cyclone formation;
[2]
(e) (i) Strong and violent winds and heavy rains accompany cyclones / AW;
high winds damage buildings which can injure / kill people;
high winds bring down trees which can injure / kill people
heavy rains cause flooding so people drown;
heavy rains cause landslides on steep slopes so that houses / people are buried by soil /
mud / rocks;
[max 3]
(ii) answers which go little further than identifying appropriate information given in
the boxes
general answers relying upon just one or two valid points
[max 2]
better answers use the information and explain more fully the factors responsible for the
differences
some answers may be unbalanced with more written about one of the two countries than
the other
[max 4]
good answers which are well written covering a range of relevant factors
differences between the two countries made very clear
[max 5]
Helpful information in the boxes
Philippines
'flooding largely the result of insufficient and inadequate drainage'
'cyclones create a cycle of poverty' which makes 'it more difficult for them to afford to
take preventative measures ready for the next one'
Japan
'after warnings from the Weather Office, many people were evacuated into shelters by
the disaster management agency before the cyclone arrived.'

746

Page 8

Mark Scheme: Teachers version


GCE O LEVEL May/June 2012

Syllabus
5014

Paper
12

BALOL
'the threat of natural disasters in developed countries like Japan encourages
technological improvement'
factors, therefore, which help to account for differences in loss of life from
cyclones between Philippines and Japan are human and include
poverty and wealth
level of technology
degree of preparedness
administrative efficiency and organisation
all of these are shown to be positive and high in Japan
these can be supported by references to what can be done to alleviate the effects of
cyclones; the syllabus mentions
improved forecasting
appropriate settlement patterns and buildings
disaster relief
[max 5]
(iii) marks for view explained
candidate takes the view that this is unlikely / impossible
strength, power and force of very strong cyclones make it highly unlikely humans can
ever fully defeat the immense power and fury of nature
unpredictability
cyclones can strike big cities with millions of people, from which a full evacuation would
be impossible
some people are always unwilling to leave homes, often from fear of looters
candidate takes the view that this is likely / possible
technology is improving all the time
weather satellites and computer models are becoming more sophisticated at tracking
and predicting cyclones
shelters stocked with drinking water and food can prevent all loss of life from cyclones
better built / concrete housing
[max 2]
[Total: 40]

747

Page 9

Mark Scheme: Teachers version


GCE O LEVEL May/June 2012

Syllabus
5014

Paper
12

BALOL
6

(a) (i) shading of all three sectors for oil, coal and natural gas only;

[1]

(ii) they are the top three / three largest;


accounting for about 80 % of the total energy consumption;
allow ecf from (a)(i) for [max 1]

[2]

(iii) / quarter / 2527%;

[1]

(b) (i) surface towers lifting gear / AW; ventilation pumps / AW;
[max 1]
shafts route for miners to reach the coal seam; for coal / waste to be removed /
ventilation path;
[max 1]
[2]
(ii) the coal cutter digs the coal from the seam;
the cutter has giant mechanical teeth to bite into the coal / AW;
ref. mechanical / metal pit props to support the tunnel roof;
the loose coal is carried away by train;

[max 3]

(iii) the best answers will refer to the characteristics of both old and modern coal mines in
order to support the choice of modern mine
highly mechanised;
details e.g. machinery instead of men doing the work underground / in an old mine men
would be cutting into the coal with picks ands shovels;
ref. recent/modern looking buildings on the surface;
mine is not in the middle of a mining settlement / mine is shown in the middle of open
countryside;
[max 2]
(c) (i) flood or equivalent / fire / explosions / safety standards often ignored;;
four
two or three
(ii) opencast mine all work is done in the open air / on the surface;
more of the work can be done by machines;
roof collapses do not exist;
not possible to get build-ups of gas leading to explosions and fires;
if there is an accident it is easier for emergency treatment to be provided;

[2]
[1]

[2]

[max 3]

(iii) safety rules vary from country to country;


variable degree to which safety standards are enforced;
greater health and safety culture in some countries;
safety costs countries money;
richer / developed countries can better afford the safety supervision;
if a country depends on minerals for export, the emphasis can be on production rather
than safety;
age and condition of the mines;
extent to which they have been modernised;
type of mining / physical conditions;
how deep underground the mines extend;
named example e.g. gold mines in South Africa are the world's deepest;
extent of underground geological problems;
whether the mining is official or unofficial;
named example e.g. illegal mining in frontier regions of Brazil;

[max 4]

748

Page 10

Mark Scheme: Teachers version


GCE O LEVEL May/June 2012

Syllabus
5014

Paper
12

BALOL
(d) (i) sulphur dioxide;
oxides of nitrogen;

(accept named i.e. nitric / nitrous oxide / nitrogen dioxide)

[2]

(ii) pollution from the UK / Germany / one group of countries, being carried by the wind to
other countries / Norway / Sweden / Scandinavia, making it an international problem
/ AW;
[1]
(iii) main wind direction is south west / from south west to north east;
pollution from coal fired stations is carried away from the UK;
so trees in northern UK unaffected by the acid rain;
acid rain (in Sweden) increases soil acidity;
causes faster leaching of soil nutrients / calcium / potassium;
manganese / aluminium released from soils and harm roots;
long-term causes trees shed their leaves / needles and die / AW;

[max 3]

(e) (i) flue gases from chimneys can be 'scrubbed' / ref. filters / AW;
ref. flue gas desulphurisation / FGD;
details of FGD e.g. removing sulphur by using a mixture of limestone and water;
nitrogen oxides removed by catalytic reaction with ammonia / equivalent;
[max 2]
(ii) problems are less in the producing country;
reducing gas emissions costs money and increases the cost of electricity;
reaching agreements between countries is difficult because each has its own national
agenda / AW;
illustrated by the limited success of recent climate change world summits;
many countries in Asia wish to develop economically leading to an increase in air
pollution emissions;
objections of developing countries to being restricted because of pollution already
caused by developed countries;
developed countries like the US need to reduce their high emissions but there is a
lot of public and political opposition;
[max 3]
(f)

(i) 100 % / all of it;

[1]

(ii) Explanation of the theme of much greater importance in the three northern countries
compared with world consumption
renewables made up only about 4% of world energy consumption / a tiny percentage
compared with fossil fuels;
in these three north European countries the situation is reversed with renewables
dominating and fossil fuels making up a tiny percentage / AW;
use of comparative figures e.g. ratios between renewables : fossil fuels
Iceland 100 : 0 Norway 97 : 3 Sweden 53 : 4 / other comparatives e.g. percentages;
Sweden, (of the three, the country that uses least renewables) has a much higher
nuclear sector than the world average, instead of using fossil fuels / AW;
total energy consumption and electricity consumption are not quite the same thing;
[max 3]

749

Page 11

Mark Scheme: Teachers version


GCE O LEVEL May/June 2012

Syllabus
5014

Paper
12

BALOL
(iii) One or two relevant reasoned comments, but limited progress towards answering both
elements in the question
[max 2]
Fuller coverage; wider range of points; likely to touch on both amount and types, but may
be unbalanced between the two elements
[max 4]
Good range of reasons, perhaps supported by use of named examples
[5]
ref. relationship to a country's own national resources
countries with plentiful deposits of oil / coal / gas, amount likely to be dominated by fossil
fuels these are cheaper to use the technology is more developed / traditional than
renewables so there is less incentive to look for alternatives ora for countries without
fossil fuels
ref. examples such as coal use in China and India, or oil use in the Middle East
ref. related factors e.g. degree of economic development and economic needs
type of renewables depends a lot on physical possibilities
renewables are not necessarily able to be afforded by all countries with favourable
natural conditions
potential examples of renewables for discussion might include
mountainous countries with good rainfall have the best prospects for HEP e.g. Norway
in the example used here / alternative HEP is most widely used renewable technology
geothermal power most available in areas of volcanic activity e.g. Iceland in this
example on the plate boundary in the middle of the Atlantic Ocean / alternative
flat or mountainous and windy countries, especially islands, lend themselves to wind
power e.g. Netherlands / alternative but technology expensive and therefore mostly
used in developed countries
tropical and subtropical countries / named example, are best for solar power but the
technology is still developing to make solar more economically competitive therefore
mostly used in developed countries despite their relative lack of insolation
biomass should be globally available but requires investment and large land area
conflict with food production Brazil has currently made most progress
[max 5]
[Total: 40]

750

General Certificate of Education Ordinary Level


5014 Environmental Management June 2012
Principal Examiner Report for Teachers

BALOL
ENVIRONMENTAL
MANAGEMENT

Paper 5014/12
Paper 12

Key messages

Candidates should plan their time carefully in the examination. No more than 45 minutes should be
spent answering the four short questions in Section A; in Section B, no more than 45 minutes should
be spent on Question 5, so that 45 minutes are left to answer Question 6.

It is advisable to read each question more than once and underline key question words, especially the
command words which tell candidates what to do. Question instructions most frequently ignored in this
examination were use the information on the diagram in 4(b)(i), what is similar about the tracks
followed by cyclones in 5(d)(ii), and using both diagrams in 6(b)(ii).

Candidates should take careful note of the amount of credit available for the question, as this is a good
guide as to how much information is expected; it is highly likely that a variety of points or reference to an
example are needed, rather than simply repeating one idea or theme. Examples of questions for which
candidates did not provide adequate answers as suggested by the credit allowance were 6(f)(ii) and
6(f)(iii). Many candidates gave very brief answers, without any supporting use of percentages from the
graphs. Always support descriptions from graphs by referring to relevant values.

General comments
Most candidates scored at least as well in the short questions in Section A as they did in each of Questions
5 and 6. This showed that candidates tended to continue the standard set in answering the early questions
into the later questions. Only occasionally did a significant difference in candidate topic knowledge result in
much higher credit being awarded for either of the questions in Section B.
There were fewer instances than previously of candidates failing to complete all the later parts of
Question 6. Pressure of time appeared to be much less of an issue this year, suggesting that many
candidates planned the use of their time more effectively. Fewer answers than previously seen continued
into the blank spaces beyond the answer lines in the short questions in Section A.
There seemed to be few differences in degree of difficulty between the four questions in Section A, with
candidates scoring equally well in each of Questions 1, 2, 3 and 4. Candidates had most difficulty with
3(c)(i) and (ii) about inversion of temperature.
In Section B, most candidates were able to give answers to the majority of the questions, and completely
incorrect answers were rarely seen. The only exception to this was Question 5(b)(i), where a few
candidates chose to draw a bar graph to show temperature instead of a line graph as instructed by the
question. Some candidates misinterpreted Question 5(a)(i) or failed to give the depth or breadth in their
answers that were required in Question 5(e)(ii).
The main discriminator between candidates was the extent to which they extended their answers in line with
the amount of credit available. For those who failed to develop their answers, minimal credit was most
frequently awarded in Questions 6(a)(ii), 6(d)(ii) and 6(f)(ii).

2012

751

General Certificate of Education Ordinary Level


5014 Environmental Management June 2012
Principal Examiner Report for Teachers
Comments
BALOL on Specific Questions
Section A
Question 1
(a) (i)

Arrows drawn on the map needed to show plates moving away from each other in general westerly
and easterly directions, although it was not important which plate boundary (or boundaries) were
marked with arrows. Most attempts were successful.

(ii) The presence of active volcanoes on the map was typically the trigger for explaining the presence
of hot rocks. The best and most convincing answers came from those who understood that
magma from the mantle reaches the surface at plate boundaries.
(iii) Many candidates arrived at an acceptable answer and circled 275 (which is closest to the precise
answer) or 300 (which was credited in this case as an answer closer to this could just be obtained
from plausible measurements). Some candidates left this blank and others gave one of the
incorrect answers.
(iv) There was a clear reference to given in part (ii) to tropical fruit and vegetables being grown in
hothouses. In this question, repetition of the word hothouses triggered appropriate answers about
the economic uses in terms of crop growth in a minority of cases. other answers tended to be
vague.
(v) Geothermal energy and heating inside buildings were the most common answers seen here. Since
the question referred to uses of hot water and steam, the basic answer of for tourism was
inadequate unless a specific tourist use was stated, such as bathing pools.
(b)

Many answers gave danger and damage from earthquakes and volcanoes. The highest credit was
scored by those candidates who introduced breadth to their answers, by referring to the
unpredictability of earthquakes and volcanic eruptions and to the economic costs of being prepared
for them, for example in more expensive building codes.

Question 2
(a) (i)

Credit-worthy answers to this referred to the great depth of the well (about 1500 m below the
surface). Other candidates answered by referring to some of the problems associated with the
great depth, for example the poor visibility or high water pressure.

(ii) Frequent references were made to winds, ocean currents and to the lightness of oil compared with
water. Many candidates received full credit.
(iii) Most candidates read the graph accurately to give the correct answer.
(b) (i)

Most candidates correctly added arrows, pointing in an anti-clockwise direction, to show the food
chain. A few candidates incorrectly drew a link from large fish to oil; this circular link was invalid.

(ii) Many candidates focused on damage and death for fauna and plants and gained credit. A few
candidates did not read the question and stated the other causes of oil leaks, such as tanker spills,
rather than answering the question.
(c)

Most candidates gained credit by giving acceptable suggestions for three methods as asked for by
the question.

Question 3
(a) (i)

The majority of candidates plotted the bar in the correct position on the graph for the year 2030.

(ii) A number of candidates simply stated that that there was an increase in amounts of nitrous oxide.
Other candidates correctly qualified the increase more strongly, emphasising the rapid, or great
increase.

752

General Certificate of Education Ordinary Level


5014 Environmental Management June 2012
Principal Examiner Report for Teachers
(iii) Most candidates divided the pie graph into 76% and 24%, but some incorrectly did not also add
BALOL
labels to indicate which sector represented which source of nitrous oxide.

(b) (i)

The most common answer here was fertilisers, although some candidates appropriately suggested
agricultural machinery exhausts. If exhaust was not linked to agricultural machinery or similar then
credit could not be awarded.

(ii) Vehicle exhaust was the most frequently seen answer in this part.
(c) (i)

Most candidates made the correct choice, A, but some did not give a valid reason which was
required in order to be credited.

(ii) The minority of candidates who had incorrectly selected B in part (c)(i) generally scored credit in
this part by making reference to pollutants being trapped in the lower atmosphere. The majority of
candidates who correctly chose A went on to give a valid explanation.
(d)

The majority of candidates were able to suggest two reasons here, but three were needed in order
to score full credit. Weaker candidates often over-relied on the use of the word better to describe
inorganic fertilisers without clearly explaining how they were superior to organic ones.

Question 4
(a)

The majority of candidates attempted to give descriptions which were sufficiently detailed. Most
candidates concentrated on the northern hemisphere, particularly on northerly locations mostly
within the Arctic Circle. They usually gave support by use of names of continents and countries.
This part was consistently well answered.

(b) (i)

While it was clear from answers that the term plant community was widely known and understood,
many candidates failed to gain credit because they did not follow the instructions in the questions
which told them to make use of the information on the diagram in their answers.

(ii) Good answers to this began by stating how vegetation on the south facing slope was different.
Those who noted the presence of a greater variety of plants were more likely to find a valid
physical explanation. Acceptable explanations included references to the south facing slope being
warmer and sunnier and also more sheltered from cold (northerly) winds.
(iii) Most candidates gained credit for stating that tundra plants can only have short roots.
(c)

There were many excellent answers, gaining full credit in this part. A few candidates concentrated
on one opinion only, which restricted the amount of credit that could be awarded. Candidates who
strayed from the question by referring to possible advantages and attractions of the tundra for
tourists could not be credited.

Section B
Question 5
Most important in Question 5 was consistent performance and therefore regular accumulation of credit. The
greatest loss of credit was often seen in part (d). Candidates showed good familiarity with the topics being
examined.

(a) (i)

Careless reading of the question led to many candidates concentrating on naming the weather
instruments instead of the weather elements recorded. While credit could not be awarded for wind
vane or anemometer, because they did not specifically mention wind direction and wind speed,
credit was awarded for rain gauge and sunshine recorder, because the weather element was part
of the name of the instrument. Plenty of others gave the short accurate answers of wind direction,
wind speed, rainfall and sunshine hours. Of these, sunshine hours was the one most
frequently omitted.

(ii) The clearest answers almost invariably began with the name of the instrument, either maximum
and minimum or Sixs thermometer. Whether or not the answers earned full credit depended on
the amount of further detail given about the thermometers, and on the accuracy of the description

753

General Certificate of Education Ordinary Level


5014 Environmental Management June 2012
Principal Examiner Report for Teachers

BALOL of how readings from them were taken. Answers obtaining less credit came from candidates who
concentrated irrelevantly on describing how the highest and lowest temperatures of the day are
used in further calculations of daily mean and range.

(b) (i)

The most accurate line graphs were drawn by those candidates who placed the dot for temperature
in the centre of the column for each of the months. Those who placed the values on the lines to
the left or the right drew graphs which were not quite as accurate and were only awarded partial
credit, provided that all the plots were accurate, which almost all were, and linked by a line. A
number of candidates drew a bar graph to show temperature, which was not only more time
consuming than drawing a line graph, but also incorrect.

(ii) Most answers here were correct, although a few candidates stated the extremes (25 C and 29 C)
instead of working out the range, 4 C.
(iii) Answers worth most credit came from candidates who clearly recognised the two season rainfall
regime, wet summer and dry winter. Answers in which candidates worked through the year from
January to December describing when and whether it was dry or wet were worth less credit.
Answers in which candidates did not do much more than pick out the two months with lowest and
highest rainfall were not worthy of credit.
(iv) Most candidates showed themselves to be aware of higher rainfall and more cloud reducing
temperatures in June and July compared with April.
(c) (i)

Those candidates who used both temperature and rainfall, and directly related these to
opportunities for crop growing, gave the best answers. For example, some related high summer
rainfall to the possibilities of storing water for the dry season and / or to the good conditions for rice
cultivation. Less convincing were answers based on high rainfall all year being good for crop
growing, since rainfall amounts were clearly inadequate for crop growing during the months of
January to April, especially with these high tropical temperatures.

(ii) Most gave correct answers here, although a few reversed their choices.
(iii) Most candidates showed good understanding of differences between subsistence and commercial
farming and gave accurate answers in this part. The amount of credit available meant that more
than the basic difference of for the family and for sale was needed; most elaborated further about
differences in the size or scale of farming operations between them.
(iv) Monoculture, for export, and ownership by large companies were the three characteristics of
plantation farming mentioned most in answers. Some supported their answers by naming
examples of crops, or some of the many inputs. Less credit-worthy answers were more about
commercial farming in general than plantation farming in particular.
(d)

The majority of candidates found part (d) more difficult than part (c).
(i)

Many candidates started by describing the areas as being in the tropics. The best descriptions
included references to both oceans and names of adjacent countries; references to cyclone
formation in the Indian and Pacific Oceans were usually more precise than to those in the Atlantic.

(ii) Candidates needed to work harder for credit in this part because they were asked to focus on what
was similar about the tracks followed. Most missed the basic point that initial movement was from
east to west; more commented on movement away from the Equator or towards the poles.
Correctly stating clockwise movement in the northern hemisphere and anticlockwise in the
southern hemisphere did not meet the similar theme of the question; those who referred to a
circular movement, or to tracks turning or bending with movement away from the Equator,
successfully adapted their answers to meet the question theme.
(iii) The best answers here came from candidates who recognised that the key to answering was high
sea water temperatures at the end of the northern summer. Any further elaboration about why this
led to air rising enabled full credit to be awarded. Some answers were more about the formation of
the Asian monsoon, and differences in pressure between land and sea, than about individual
cyclone formation.

754

General Certificate of Education Ordinary Level


5014 Environmental Management June 2012
Principal Examiner Report for Teachers
(e)
(i) Those candidates who began their answers by stating that cyclones bring strong winds and heavy
BALOL
rainfall gave consistently the most credit-worthy answers. Such candidates tended to follow up
with the resulting dangers for humans from the destruction of buildings and trees, and from flooding
and landslides. Those candidates who made no mention of the physical problems could still gain
credit for stating how loss of life and injuries occurred. Many of the answers which took this
approach lacked depth and precision. Even so, the majority of candidates gained some credit on
this question by making good use of the information provided.
(ii) The factors which accounted for the differences in loss of life from cyclones between the
Philippines and Japan included poverty and wealth, level of technology, degree of preparedness
and organisational efficiency, although most of these were inter-related. Candidates needed to
include references to at least two of them to gain full credit. The best style of answer was to make
direct comparisons between the two countries throughout. Answers which concentrated on only
one of the countries rarely reached more than half the available credit. Answers in which
differences were implied without being fully stated gained less credit.
(iii) A clearly stated view with full and appropriate reasons was the main feature of the most creditworthy answers here. Either opinion was equally acceptable provided that it was backed up by
good justification. This justification was usually stronger for one clear view, because general
statement such as improvements in technology could be elaborated upon by reference to key
material such as greater coverage by weather satellites, or improved computer models, or more
efficient communications with people in areas likely to be affected.

Question 6
Overall the standard of answers given to Question 6 tended to be consistent throughout all parts of the
question and almost all received about the same amount of credit in both Questions 5 and 6. The strongest
performances, as usual, were from candidates who gave full and relevant answers throughout.
(a) (i)

Most candidates knew that oil, coal and natural gas were the fossil fuels in the pie graph. Some
candidates left natural gas un-shaded or incorrectly shaded nuclear power.

(ii) Having stated a percentage between 75% and 100% for fossil fuels, most candidates failed to look
for further description that could have led to full credit.
(iii) Answers within the range 25%-27% were accepted in this part.
(b) (i)

The purpose of the shafts in a coal mine were better known than for the towers. Surface towers
hold the lifting gear and supply the mine with ventilation and other essential services. Some
candidates knew this, but a significant number incorrectly believed that the towers were used for
either storing or processing the coal.

(ii) Answers which gained almost full credit, based on what could be seen in diagram A, were
predominant. The instruction in the question to make use of both diagrams was missed in many
cases.
(iii) Diagram B was sometimes used to support the answer in part (iii). Those who thought that the
diagram showed an old coal mine found it difficult to explain given the presence of so much large
machinery.
(c) (i)

Many candidates gave four reasons in (c)(i); if only three different reasons were found, safety
standards ignored was the one most likely to be missed.

(ii) For an effective explanation candidates needed to demonstrate clear knowledge of opencast
mining, so that they could emphasise effectively why the problems referred to in the newspaper
report either did not apply or applied less than in deep mining.
(iii) Those candidates who incorporated breadth into their answers to part (iii), by referring to factors
other than safety standards such as age and physical condition of mines, gained more credit.
Answers based solely on safety standards and their enforcement tended to gain partial credit.
Nevertheless, with sufficient depth it was possible to gain full credit using this line of reasoning,
especially if examples of countries were given in support.

755

General Certificate of Education Ordinary Level


5014 Environmental Management June 2012
Principal Examiner Report for Teachers

BALOL

(d) (i)

Sulfur dioxide and oxides of nitrogen were the gases needed for answering (d)(i). They were
widely known. Carbon dioxide was incorrectly included in quite a number of answers acid rain as
an environmental issue only includes additional acids to the natural carbonic acid produced by
carbon dioxide in the air.

(ii) Carried by the wind to other countries was the most frequent and correct line of answering to
(d)(ii).
(iii) The best answers to part (iii) began with a clear statement about how the trees in Sweden were
shown to be different from those in the UK. Following from this was an impressive range of
reasons, referring to wind carried pollution from coal fired power stations, causing increased soil
acidity, faster leaching of soil nutrients such as calcium and potassium, and their replacement by
more harmful manganese and aluminium. Incorrect responses were made by candidates who
attempted to explain the greater presence of trees in terms of differences in climate and biomes
between the warm UK and cold Sweden.
(e) (i)

Candidates who covered flue gas desulfurisation and/or the use of catalysts to remove nitrogen
gained credit in (e)(i). Some candidates tried to go wider than the question set by answering in
terms of renewable sources replacing coal which earned little or no credit.

(ii) Many candidates relied only upon making the basic point about the difficulty of reaching
agreements between countries, without further elaboration about differences in national interests or
wealth between countries. There were some more perceptive answers, in which a broader look
was taken at attempted international agreements; these included references to the limited success
of any agreements reached at climate change summits.
(f)

(i)

Many candidates gave the correct answer, but some gave 75%, having not recognised geothermal
power as one of the renewable sources. Occasionally the answer given was 0% which was
surprising.

(ii) Almost all candidates began by establishing the greater importance of renewables in these three
north European countries compared with total world energy consumption. Many filled most or all of
the answer lines, but then made no attempt to elaborate more fully, taking into account the credit
allocation for this part. Others were more successful when they used percentages from the graph
to support the 100% for Iceland already stated in the first part of the question, with 97% for Norway
and 53% for Sweden. Also a few successfully approached the answer by discussing the limited
fossil fuel use for electricity in Norway and Sweden, with natural gas use at only 3% and 4%
respectively. The graphs could have been more fully used to earn more credit.
(iii) Most candidates followed one line of answering. Sometimes it was related to the higher cost of
renewables compared with fossil fuels, and they related this to countries levels of development.
The theme used most frequently was of specific physical needs, supported by outline references to
two or three examples of renewables. Answers in which candidates used a variety of reasons,
such as the two mentioned above plus references to a countrys own fossil fuel resources, stood
out as being clearly superior. Even better were those in which candidates regularly included
names of countries as examples, which allowed them to make their points even more strongly. The
result was that lower scoring answers were much more common than those which gained higher or
full credit, even though virtually all candidates understood the type of answer needed, and knew
something about three or four different types of renewables.

756

BALOL
UNIVERSITY OF CAMBRIDGE INTERNATIONAL EXAMINATIONS
General Certificate of Education Ordinary Level

* 1 3 5 2 1 9 0 4 9 0 *

5014/11

ENVIRONMENTAL MANAGEMENT
Paper 1

October/November 2012
2 hours 15 minutes

Candidates answer on the Question Paper.


Additional Materials:

Ruler

READ THESE INSTRUCTIONS FIRST


Write your Centre number, candidate number and name on all the work you hand in.
Write in dark blue or black pen.
You may use a soft pencil for any diagrams, graphs or rough working.
Do not use staples, paper clips, highlighters, glue or correction fluid.
DO NOT WRITE IN ANY BARCODES.
Answer all questions.
All questions in Section A carry 10 marks.
Both questions in Section B carry 40 marks.
At the end of the examination, fasten all your work securely together.
The number of marks is given in brackets [ ] at the end of each question or part
question.

For Examiners Use


1
2
3
4
5
6
Total

757

BALOL
1

Section A

(a) Look at the map showing the location of an opencast iron ore mine in northern Europe.
The railway and settlements were built for the mine.
to the
ocean
with warm
ocean
current
10 km

For
Examiners
Use

Key:
port
settlement

railway for transport


of iron ore
road
sea
opencast iron ore mine
heaps of soil, rock and waste
settlement

workers
settlement

processing plant
1

km

190 km to Arctic Circle


(i)

How long is the railway line from the mine to the coast?
.............................................................................................................................. [1]

(ii)

Why is it easy to export iron ore from this mine all year?
..................................................................................................................................
.............................................................................................................................. [1]

(iii)

Describe the impact of this mine on the environment.


..................................................................................................................................
.............................................................................................................................. [1]

UCLES 2012

5014/11/O/N/12

758

BALOL(iv) The mine is at approximately 70 North in an area of tundra climate. What are the
problems for opencast mining at high latitudes?

For
Examiners
Use

..................................................................................................................................
..................................................................................................................................
..................................................................................................................................
..................................................................................................................................
..................................................................................................................................
.............................................................................................................................. [3]
(b) (i)

The iron ore is about 35% pure. Explain what this means.
.............................................................................................................................. [1]

(ii)

Explain why the ore is partially processed at the mine, before being transported to
market.
..................................................................................................................................
.............................................................................................................................. [1]

(c) The mine closed in 1996 and re-opened in 2009. Suggest reasons why some mines
re-open.
..........................................................................................................................................
..........................................................................................................................................
..........................................................................................................................................
...................................................................................................................................... [2]

UCLES 2012

5014/11/O/N/12

[Turn over

759

4
2 BALOL
(a) Look at the diagram showing how nitrate fertiliser can reach a river.
nitrogen

For
Examiners
Use

nitrate
fertiliser
X
Y

uptake
by plant
roots

(i)

denitrification

river

The arrows labelled X and Y on the diagram show two ways in which pollutants can
move to the river as part of the water cycle. Name the processes.
X ...............................................................................................................................
Y ........................................................................................................................... [2]

(ii)

From the diagram, state two ways in which nitrates can be prevented from reaching
the river.
1 ...............................................................................................................................
..................................................................................................................................
2 ...............................................................................................................................
.............................................................................................................................. [2]

(iii)

What is produced during denitrification?


.............................................................................................................................. [1]

(b) Algae and water plants grow quickly when large amounts of nitrate reach rivers. Explain
two effects of this increased growth.
..........................................................................................................................................
..........................................................................................................................................
..........................................................................................................................................
...................................................................................................................................... [2]

UCLES 2012

5014/11/O/N/12

760

BALOL
(c) Suggest:
(i)

how strips of vegetation along the sides of rivers, as shown on the diagram, may
help to reduce pollution of the river during the spraying of pesticides,

For
Examiners
Use

..................................................................................................................................
.............................................................................................................................. [1]
(ii)

a weather condition in which farmers should not spray harmful chemicals,


..................................................................................................................................
.............................................................................................................................. [1]

(iii)

an advantage to wildlife of growing a variety of different types of vegetation in these


strips.
..................................................................................................................................
.............................................................................................................................. [1]

UCLES 2012

5014/11/O/N/12

[Turn over

761

6
3 BALOL
(a) Look at the climate graph for a Swedish city with a cool temperate interior climate.

temperature
/ C

15

15

10

10

10

10

15

J
J
month

80

60

60

precipitation
40
/ mm

40

20

20
F

J
J
month

temperature
/ C

15

80

For
Examiners
Use

precipitation
/ mm

Use the temperature figures in the table to complete the climate graph. Plot the figures
as a line graph.
[3]
month

temperature C

11 12

11

15

12

10

(b) Coniferous forest grows in this climate. In Sweden the forests have been badly affected
by acid rain.
(i)

Name the two main pollutants that cause acid rain.


..................................................................................................................................
.............................................................................................................................. [1]

(ii)

Explain how acid rain damages forests.


..................................................................................................................................
..................................................................................................................................
..................................................................................................................................
.............................................................................................................................. [3]

UCLES 2012

5014/11/O/N/12

762

BALOL
(c) Suggest why it is often difficult to reduce the problem of acid rain in a country.
..........................................................................................................................................

For
Examiners
Use

..........................................................................................................................................
..........................................................................................................................................
..........................................................................................................................................
...................................................................................................................................... [3]

UCLES 2012

5014/11/O/N/12

[Turn over

763

8
4 BALOL
(a) Look at the population pyramid, which shows the population of Rwanda in 2010.

For
Examiners
Use

age range
male

80 and over
7579
7074
6569
6064
5559
5054
4549
4044
3539
3034
2529
2024
1519
1014
59
04

female

1.0 0.9 0.8 0.7 0.6 0.5 0.4 0.3 0.2 0.1 0
0 0.1 0.2 0.3 0.4 0.5 0.6 0.7 0.8 0.9 1.0
population (in millions)
(i)

What is the total number of children in Rwanda aged 0 to 9 years old?


.......................................................... million

(ii)

[1]

How does the number of people in older age groups differ from that in younger age
groups?
.............................................................................................................................. [1]

(iii)

Migration into a country can affect the shape of its population pyramid. State one
age group in Rwanda which appears to have been affected by this.
.............................................................................................................................. [1]

(b) The increase in population in rural areas leads to rural-urban migration. This leads to
loss of many middle-aged people from rural areas.
Suggest the problems caused for rural areas by the loss of middle-aged people.
..........................................................................................................................................
..........................................................................................................................................
..........................................................................................................................................
..........................................................................................................................................
..........................................................................................................................................
...................................................................................................................................... [3]

UCLES 2012

5014/11/O/N/12

764

BALOL
(c) Describe the different ways in which city authorities have attempted to deal with the
problem of housing large numbers of migrants.

For
Examiners
Use

..........................................................................................................................................
..........................................................................................................................................
..........................................................................................................................................
..........................................................................................................................................
..........................................................................................................................................
..........................................................................................................................................
..........................................................................................................................................
...................................................................................................................................... [4]

UCLES 2012

5014/11/O/N/12

[Turn over

765

10

BALOL
5

Section B

(a) Over the past 200 years countries have passed through the stages of the Demographic
Transition Model at different rates. The diagram shows the stages in the Demographic
Transition Model for a country.
50

50

birth and 40
death rate 30
per 1000
20
people
per year
10

40

For
Examiners
Use

30
20
10
stage 1

stage 2

stage 3

stage 4

stage 5

Key
birth rate
death rate
natural increase of population
natural decrease of population
(i)

(ii)

Look at the diagram. Give the stage number which matches each of these two
descriptions:
fastest decrease in birth rate

.........................................

fastest increase in rate of population growth

.........................................

[2]

State one similarity and one difference between stage 1 and stage 4.
similarity ...................................................................................................................
..................................................................................................................................
difference ..................................................................................................................
.............................................................................................................................. [2]

(iii)

UCLES 2012

On the graph, use two different types of shading to show where there are natural
increases and natural decreases of population. Complete the key for the types of
shading used.
[3]

5014/11/O/N/12

766

11

BALOL
(b) The spider diagram shows factors which can decrease the rate of population growth in a
country.
family planning

For
Examiners
Use

urban growth

factors for decreasing rate of


population growth

education of women
(i)

very high death rates

Choose three of these factors. For each factor, describe how it can decrease the
rate of population growth in a country.
1 ...............................................................................................................................
..................................................................................................................................
..................................................................................................................................
2 ...............................................................................................................................
..................................................................................................................................
..................................................................................................................................
3 ...............................................................................................................................
..................................................................................................................................
.............................................................................................................................. [6]

UCLES 2012

5014/11/O/N/12

[Turn over

767

12

BALOL(ii) Another factor which can affect the rate of population growth in a country is the
policy of the government.

For
Examiners
Use

Look at the information box showing how national population policy changed in Iran
between the early and late 1980s.
Population policy in Iran
Early 1980s
Islamic Revolution
Ayatollah Khomeinis regime said that it was
the duty of all Iranian citizens to have children
and expand the population.

Population in 1980
Population estimate made in
1980 for 2005

Late 1980s
Regime realised that it had made a mistake.
Instead it introduced a policy to reduce
population growth
* contraception was made more widely
available
* couples were forced to attend family
planning clinics before marriage
* child benefits were stopped after three
children

40 million
100 million

Actual population in 2005

70 million

How and why did the population policy of the government change in Iran between
the early and late 1980s?
..................................................................................................................................
..................................................................................................................................
..................................................................................................................................
.............................................................................................................................. [2]
(iii)

As a result of this change in policy, how big was the decrease in population between
the number estimated in 1980 for 2005 and the actual population in 2005?
.............................................................................................................................. [1]

UCLES 2012

5014/11/O/N/12

768

13

BALOL(iv) Look at the data for fertility rates (average number of children per woman over her
lifetime) in Iran between 1980 and 2010.
year

1980

1985

1990

1995

2000

2005

2010

6.5

6.6

5.6

4.3

2.5

2.1

2.1

fertility rate

For
Examiners
Use

Draw a line graph to show the change in fertility rates in Iran from 1980 to 2010.
7
6
5
4
fertility
rate 3
2
1
0
1980

1985

1990

1995

2000

2005

2010

year
[3]
(v)

Describe how the data and your graph show that fertility rates in Iran decreased
most quickly during the 1990s.
..................................................................................................................................
..................................................................................................................................
..................................................................................................................................
.............................................................................................................................. [2]

(vi)

Suggest reasons why fertility rates decreased faster during the 1990s than after
2000.
..................................................................................................................................
..................................................................................................................................
..................................................................................................................................
.............................................................................................................................. [2]

UCLES 2012

5014/11/O/N/12

[Turn over

769

14

BALOL
(c) Why do national population policies have a big effect on the size of population growth
in countries? Explain as fully as you can. Use examples of countries with and without
population policies to illustrate your answer.

For
Examiners
Use

..........................................................................................................................................
..........................................................................................................................................
..........................................................................................................................................
..........................................................................................................................................
..........................................................................................................................................
..........................................................................................................................................
..........................................................................................................................................
..........................................................................................................................................
..........................................................................................................................................
...................................................................................................................................... [5]
(d)

Population data for Iran (2005)


birth rate

20.3 per 1000

death rate

5.2 per 1000

population under 15 years


population over 60 years
(i)

28.7 per cent


6.4 per cent

What was the rate of natural increase per 1000 in Iran in 2005?
.............................................................................................................................. [1]

(ii)

Look again at the Demographic Transition Model in part (a).


In which stage does the example of Iran fit best? Explain your choice.
..................................................................................................................................
..................................................................................................................................
.............................................................................................................................. [2]

UCLES 2012

5014/11/O/N/12

770

15

BALOL(iii) Describe some of the economic and environmental effects of continued population
growth for countries such as Iran.

For
Examiners
Use

economic ..................................................................................................................
..................................................................................................................................
..................................................................................................................................
..................................................................................................................................
environmental ...........................................................................................................
..................................................................................................................................
..................................................................................................................................
.............................................................................................................................. [4]
(e) Many people believe that the continued growth of world population is unsustainable.
(i)

Why do they think this?


..................................................................................................................................
..................................................................................................................................
..................................................................................................................................
..................................................................................................................................
..................................................................................................................................
..................................................................................................................................

(ii)

Do you agree? Explain your views on this.


..................................................................................................................................
..................................................................................................................................
..................................................................................................................................
..................................................................................................................................
..................................................................................................................................
[5]
[Total: 40]

UCLES 2012

5014/11/O/N/12

[Turn over

771

16
6 BALOL
(a) Look at the chart. It shows percentages of people without access to clean (safe) water
in five regions of the developing world for 1975 and 2005.
100
90
80
percentage 70
of people 60
without
50
access to
40
clean water
30
20
10
0

Key
1975
2005

South
America
(i)

For
Examiners
Use

Sub-Saharan
Africa

South East
Asia

South
Asia

Middle
East

From the chart, name the world region in which:


1.

over 80 per cent of the people were without access to clean water in 1975;
...........................................................................................................................

2.

the greatest percentage of people were without access to clean water in 2005;
...........................................................................................................................

3.

there was the largest percentage increase of people with access to clean
water between 1975 and 2005.
...........................................................................................................................
[3]

(ii)

Using the chart, state how the Middle East is different from the other four regions.
..................................................................................................................................
.............................................................................................................................. [1]

(iii)

Suggest reasons why rates of improvement in access to clean water between 1975
and 2005 varied so greatly between developing world regions.
..................................................................................................................................
..................................................................................................................................
..................................................................................................................................
..................................................................................................................................
..................................................................................................................................
.............................................................................................................................. [3]

UCLES 2012

5014/11/O/N/12

772

17

BALOL
(b) People without access to clean water are the ones most at risk from water-related
diseases.

For
Examiners
Use

The most widespread water-related diseases are:


bilharzia
(i)

cholera

malaria

Which two of these diseases are water-borne, caught by people drinking unclean
(polluted) water?
.............................................................

(ii)

typhoid

.............................................................[1]

Describe how the other two named water-related diseases are different from the
water-borne ones.
..................................................................................................................................
..................................................................................................................................
..................................................................................................................................
.............................................................................................................................. [2]

(iii)

Some people die from water-related diseases. Which people and groups of people
are most at risk and why?
..................................................................................................................................
..................................................................................................................................
..................................................................................................................................
..................................................................................................................................
..................................................................................................................................
..................................................................................................................................
..................................................................................................................................
.............................................................................................................................. [4]

UCLES 2012

5014/11/O/N/12

[Turn over

773

18

BALOL(iv) Many other people feel ill and weak for long periods of the year. This contributes to
the family poverty cycle.

For
Examiners
Use

family poverty

unclean drinking water

diseases are common


and widespread

people become
ill

Complete the poverty cycle by filling in the two remaining boxes with suitable labels.
[2]
(c) (i)

Some people struggle to find enough water, and the water that they do find is rarely
clean.
Read this report from Practical Action, a NGO (non-governmental organisation)
working in developing countries.
For years, the people of Turkana in northern Kenya have suffered persistent
periods of drought. In great heat, the women have to walk up to 10 km to driedup river beds to find water for their families. The women are in danger of being
attacked along the way. When they reach the river beds, they dig scoop holes
with their bare hands, uncovering small amounts of dirty, polluted water. They
have no choice but to drink it.
Explain why the people of Turkana suffer from problems of both water shortage and
water quality.
water shortage .........................................................................................................
..................................................................................................................................
..................................................................................................................................
water quality .............................................................................................................
..................................................................................................................................
.............................................................................................................................. [3]

UCLES 2012

5014/11/O/N/12

774

19

BALOL(ii) Look at the diagram showing Practical Actions solution to this problem.

For
Examiners
Use

water storage
water point
stock
watering

photovoltaic (solar) panel

power unit
water level
pump
motor

Describe how the solution works to provide poor communities with water.
..................................................................................................................................
..................................................................................................................................
..................................................................................................................................
..................................................................................................................................
.............................................................................................................................. [3]
(iii)

Is this a sustainable solution to the problems of lack of water and poor water quality
for poor people living in developing countries in the tropics?
Explain as fully as you can.
..................................................................................................................................
..................................................................................................................................
..................................................................................................................................
..................................................................................................................................
..................................................................................................................................
..................................................................................................................................
..................................................................................................................................
.............................................................................................................................. [4]

UCLES 2012

5014/11/O/N/12

[Turn over

775

20

BALOL
(d) Look at the map of the River Ganges, Indias largest river.

For
Examiners
Use

Course of the River Ganges


N

7817

Delhi

am

an

MA

L AY

AS

8848

ge
s

un

HI

a
Kanpur

G ange

Bengal
Bay of
Key

8848

(i)

land above 2000 m

Tehri Dam

mountain peak
/ metres

city
0

100 200 300 400 km

The map shows the location of the Tehri Dam, one of the largest in India,
265 m high with a reservoir 75 km long behind it.
Suggest reasons why this is a good location to build a large dam.
..................................................................................................................................
..................................................................................................................................
..................................................................................................................................
..................................................................................................................................
.............................................................................................................................. [3]

UCLES 2012

5014/11/O/N/12

776

21

BALOL(ii) One of the main reasons for building the dam was to supply Delhi with clean
drinking water.

For
Examiners
Use

Delhi is the capital city with a population of 16 million people.


Approximately how far is Delhi from the dam?
.............................................................................................................................. [1]
(iii)

After the construction of the dam, the town of Tehri and many villages were flooded.
More than 100 farming villages still remain in the area around the Tehri Dam. People
living in these villages have found that the construction of the dam has interfered
with the natural springs, their main source of water supply for living and farming.

Farmer in the
village of Pipola

For the first time ever, we are


now short of water. We cannot
get water from the reservoir itself
because its high sides are made
of loose gravel and are too steep.
Please give us time. In the due
course of time, we will be able to
help the villagers. We are already
sending a daily water tanker.

Local state official

What choice have we


but to migrate to Delhi?
Villager
These villagers must look at the
bigger picture. The government
must think of the national need.
India needs to develop
into a big power.

Indian government
official in Delhi

Explain what the comments show about the economic and social disadvantages for
local people of building large dams.
..................................................................................................................................
..................................................................................................................................
..................................................................................................................................
..................................................................................................................................
..................................................................................................................................
.............................................................................................................................. [3]

UCLES 2012

5014/11/O/N/12

[Turn over

777

22

BALOL(iv) Why are the views of local people usually ignored when large dams are being
planned?

For
Examiners
Use

..................................................................................................................................
..................................................................................................................................
..................................................................................................................................
.............................................................................................................................. [2]
(v)

Is the migration of people to Delhi a good way of dealing with the water problems of
villages around the Tehri Dam? Explain your view on this.
..................................................................................................................................
..................................................................................................................................
..................................................................................................................................
.............................................................................................................................. [2]

(vi)

Between Tehri and Kanpur two large canals take river water from the Ganges for
farm use. Kanpur is an industrial city with a population of over 3 million people. It
is best known for tanning leather. Its 400 leather tanneries release 30 million litres
of waste water back into the Ganges every year. This water is contaminated with
chromium and chemical waste.
Building the Tehri dam has made environmental pollution worse in the River Ganges
at and below Kanpur. Suggest reasons for this.
..................................................................................................................................
..................................................................................................................................
..................................................................................................................................
..................................................................................................................................
.............................................................................................................................. [3]
[Total: 40]

UCLES 2012

5014/11/O/N/12

778

BALOL
CAMBRIDGE INTERNATIONAL EXAMINATIONS
GCE Ordinary Level

MARK SCHEME for the October/November 2012 series

5014 ENVIRONMENTAL MANAGEMENT


5014/11

Paper 1, maximum raw mark 120

This mark scheme is published as an aid to teachers and candidates, to indicate the requirements of
the examination. It shows the basis on which Examiners were instructed to award marks. It does not
indicate the details of the discussions that took place at an Examiners meeting before marking began,
which would have considered the acceptability of alternative answers.
Mark schemes should be read in conjunction with the question paper and the Principal Examiner
Report for Teachers.

Cambridge will not enter into discussions about these mark schemes.

Cambridge is publishing the mark schemes for the October/November 2012 series for most IGCSE,
GCE Advanced Level and Advanced Subsidiary Level components and some Ordinary Level
components.

779

Page 2

Mark Scheme
GCE O LEVEL October/November 2012

Syllabus
5014

Paper
11

BALOL
Notes on application of the mark scheme for Section A
marks are separated by commas. Each line usually represents one mark
oblique lines separate ideas which are alternatives
ideas in brackets are not essential to the answer but anything underlined is
reward any equivalent way of expressing the ideas in the mark scheme
reward any valid answer which is not in the mark scheme
Section A
1

(a) (i) accept from 9 to 10.5 km

[1]

(ii) sea is ice free all year

[1]

(iii) waste heaps destroy habitats/scenery/ugly


causes a big hole in the ground
dust

[1]

(iv) very cold winters


cold will give difficult working conditions for the miners
frozen ground
will be hard/difficult to excavate
snow cover
winter/months of darkness
will need artificial lighting
short summers/period when mining is easy

[3]

(b) (i) it contains 35% iron/is the amount of iron in the ore
(ii) (to reduce its weight/bulk) to make it cheaper to transport
(c) increase in price paid/value on the market
increased market demand
improved technology
change of ownership

[1]
[1]

[2]
[Total: 10]

(a) (i) X runoff


Y infiltration

[2]

(ii) taken up by plant roots


by harvesting the plants
denitrification by bacteria in the soil

[2]

(iii) nitrogen

[1]

(b) depletion of oxygen by algae/plants (at night) by respiration


fish die because of lack of oxygen
algal/plant decomposition uses up oxygen
algae/plants shade plants below preventing photosynthesis

[2]

780

Page 3

Mark Scheme
GCE O LEVEL October/November 2012

Syllabus
5014

Paper
11

BALOL
(c) (i) by trapping the spray

[1]

(ii) strong winds

[1]

(iii) credit ideas such as:


variety/increase of habitats
shelter
provide corridors for movement of wildlife
good hunting areas for predators
alternative food sources
variety of foods

[1]
[Total: 10]

(a) plots for temperature all correct = 3


all correct but no lines joining = 2
lines but 1 incorrect = 2
lines but 2 incorrect = 1
no lines and 2 incorrect/3 incorrect = 0

[3]

(b) (i) sulfur dioxide


nitric oxide
nitrous oxide
oxides of nitrogen (if neither specified)

[1]

(ii) damages leaves


trees lose their leaves
trees become prone to disease/drought/frost
acid soil water increases leaching/soil becomes more acidic
reduces soil fertility/plant nutrients/calcium/potassium
manganese/aluminium increase damaging the roots of trees

[3]

(c) credit reasons such as:


pollutants often originate in another country
carried by winds
from power stations/industries/transport important to that country
costs a lot to reduce pollutants
adds cost to consumers/users

[3]
[Total: 10]

781

Page 4

Mark Scheme
GCE O LEVEL October/November 2012

Syllabus
5014

Paper
11

BALOL
4

(a) (i) accept 3.1 to 3.5 million

[1]

(ii) higher age groups have fewer people/are smaller

[1]

(iii) 20 24/50 54

[1]

(b) credit any relevant problems, such as:


loss of workers
farm productivity will reduce
fewer to support/care for elderly/young
split families
loss of best educated/professional people

[3]

(c) credit any relevant attempts, such as:


supply squatter areas with services
provide temporary shelters
build low cost/basic homes
self-help housing schemes
loans
community involvement
apartment blocks/flats
building new homes in satellites outside the city

[4]
[Total: 10]

782

Page 5

Mark Scheme
GCE O LEVEL October/November 2012

Syllabus
5014

Paper
11

BALOL
Section B
5

(a) (i) Fastest birth rate decrease Stage 3


Fastest increase in population growth Stage 2
2 @ 1 mark

[2]

(ii) Similarity birth and death rates almost the same in both/low rates of natural population
increase
Difference birth and death rates much higher in Stage 1 (or vice versa for Stage 4),
typical rates between 45 and 50 in Stage 1 compared with 10 to 15 in Stage 4. Or others
such as fluctuations in Stage 1 compared with birth rate above death rate in Stage 4.
1 + 1 = 2 marks
[2]
(iii) Natural increase
Natural decrease

main/largest area in and across Stages 2 and 3


much smaller areas in Stages 1 and 4
main area in Stage 5
smaller areas within Stage 1

Some areas shaded that are correct for natural increase = 1 mark
Some areas shaded that are correct for natural decrease = 1 mark
More complete for natural increase and decrease e.g. in Stage 1 = 1 mark
or absolute precision in shading in Stages 2 and 3, and 5 with shading matching the key
3 @ 1 mark
Some parts correct but others incorrect then 1 mark provided that the correctly
shaded areas are larger than those that are incorrect.

[3]

(b) (i) Family planning advice about limiting family size, easy availability of or free
contraception
Urban growth child labour less obviously useful in urban than on farms in rural areas,
increases in wealth, change in cultural attitude/further away from rural traditions, greater
availability of family planning
Education of women makes women more career orientated, greater opportunities for
learning of family planning, encourages a change in the role of women in society
Very high death rates increase to a size that is above the birth rate, references to
causes such as wars, spread of AIDs
Essentially 3 @ 2 marks for each chosen factor.
1 or 2 marks per factor depending on amount of elaboration for description limited to one
element, or breadth in number of elements of the answer covered.
In cases of exceptional breadth/depth/use of an example, allow 3 marks to become a 3 +
2 + 1 route to all 6 marks.
[6]
(ii) How change from encouraging population increase, to a population policy encouraging
a slow down in population growth
Why government realised that it had made a mistake; great size of the predicted
population of 100 million for 2005 (a 2.5 fold increase)
1 + 1 mark
(iii) 30 million

[2]
[1]

783

Page 6

Mark Scheme
GCE O LEVEL October/November 2012

Syllabus
5014

Paper
11

BALOL
(iv) No more than two mistakes = 1 mark
Accurate plot of all seven values = 2 marks
Line drawn to link plots = 1 mark

[3]

(v) Data between 1990 and 2000 the rate was cut by more than half from 5.6 to 2.5, or
difference stated such as 3.1 decrease from 1990 to 2000.
Graph this rapid decline is shown by the line being steepest during the 1990s.
References to both needed for 2 marks
2 @ 1 mark
[2]
(vi) Big effect of the policy change had now passed rate down to an average of only 2
children per family with social comment on the significance of this or demographic
comment if it is known that 2.1 is the stable population replacement fertility rate.
1 point made showing part understanding = 1 mark
2 points made along these lines showing understanding = 2 marks

[2]

(c) Attitude of government matters a lot for setting up family planning clinics, funding
contraceptives, in poor developing countries where people couldn't otherwise afford and
would not know about modern contraception. Government advertising/use of public
notices/TV advertisements and promotional programmes are part of most successful national
policies e.g. in Asian countries such as Thailand. It is the only one in a position to see the
broader picture of the relationship to economic growth and development.
Sometimes government propaganda is used to increase population numbers with incentives,
for motives such as increasing regional influence or world importance of the country.
Example of China and its one child population policy shows what a strong government,
determined to reduce population growth, can do. This is an extreme example. Most other
countries used more carrot than both carrot and stick.
Earlier Iran example shows what a difference a change in government policy can make.
Weak explanation; only one or two simple points made
Perhaps no more progress than repeating points already made about Iran, without
broadening the response = 1 or 2 marks
Fuller and broader response focused on the effects of national policies on growth
Might use China and its one child policy without much in the way of broader references = 3 or
4 marks
Well focused, fully explained
Good use of relevant examples, including references to countries without meaningful national
policies such as many African countries (e.g. Niger) and many Muslim countries (e.g. Saudi
Arabia) = 5 marks
[5]

784

Page 7

Mark Scheme
GCE O LEVEL October/November 2012

Syllabus
5014

Paper
11

BALOL
(d) (i) 15.1 per 1000

[1]

(ii) Stage 2 (late) or Stage 3 (early) but no separate marks for stating the stage without
some qualification of position in the stage, either here or later in the explanation.
Up to 2 marks for explanation for relating to the graph in terms of still having quite a high
birth rate (well below the 45 50 in Stage 1, but above around the 10 mark in Stages 2
and 3). The very low death rate could fit any of Stages 2 4, but it is the quite large
natural increase that makes it fit either Stage 2 or Stage 3.
Points along these lines related to what the DTM shows.

[2]

(iii) Economic large numbers of children cost money for health care, schooling; they are
economic dependents. May later swell the labour market increasing pool if the
unemployed. Great numbers of people without formal employment. Pressure of numbers
increases cost of other infrastructure services.
Environmental increased pressure on natural resources; clearing forests for farmland
for food, overuse of farmland leading to soil erosion, increased water and energy
consumption etc.
Points made along these lines. Reserve one mark for each of economic and
environmental allocated to the correct heading.

[4]

(e) (i) People, often environmentalists, who believe that the world population growth is
unsustainable point to evidence of overuse of Earth's natural resources, such as:
Continued clearance of forests, especially rainforests, with the accompanying losses of
biodiversity
Land being destroyed/environmental degradation shown by soil erosion, desertification
and salinisation
Water stress and water shortages affecting up to half the world's countries
Progressive exhaustion of non renewable energy resources and other minerals
Overall assessment along the lines that the net resource deficit each year is estimated at
about 30% which, if correct, cannot carry on for ever, and will be worsened by the
predicted continued world growth of population (even if at lower rates than previously).

785

Page 8

Mark Scheme
GCE O LEVEL October/November 2012

Syllabus
5014

Paper
11

BALOL
(ii) View does not matter for the marking; all marks are for the explanation.
Agree continuation and extension of arguments put forward in part (i); references to
named areas and specific examples would strengthen the support.
Don't agree inventiveness of humans, increases in technology, it will hasten the switch
to use of renewables (e.g. to replace fossil fuels)
Further increases in agricultural output previously Green Revolution, now possibly GM.
These are merely suggestions of the routes answers are likely to follow.
The two parts can be marked as an overall answer.
Unsustainable explained in a simple or limited way.
View either imprecise or weakly explained = 1 or 2 marks
Fuller explanation for unsustainable either from a broader range of references, or one or
two unsustainable elements dealt with in more detail.
View likely to be clearer with some support = 3 or 4 marks
Unsustainable view well understood, perhaps with some attempt at an overview.
View made clear and effectively supported = 5 marks

[5]

[Total: 40]
6

(a) (i) 1 South East Asia


2 Sub-Saharan Africa
3 South East Asia
(ii) Smallest percentage change between the two dates, or lowest percentage without
access to clean water in 1975.
One difference stated.

[3]

[1]

(iii) The answer could be based on area with greatest change SE Asia reflects growth in
wealth and economic development, much of it associated with growth of manufacturing
industry, that allows better public infrastructure plus the rising demands for a better
standards of living and improved quality of life.
The area with least change Sub-Saharan Africa reflects more economic stagnation,
most countries still rural societies where improved access is more expensive and difficult
to provide. Political factors/attitudes of governments along with frequency of wars and
corruption.
Or the answer could be based on factors wealth & economic development, wealth of
resources (e.g. oil in Middle Eastern countries), political stability, attractiveness to Aid
providers.
Three points made that include valid reasons.
Maximum two marks if theme of great variations between developing world regions is not
directly addressed.
[3]
(b) (i) Cholera and typhoid uniquely identified.

[1]

(ii) Bilharzia is water-based, since the carrier (water snails) live in water

786

Page 9

Mark Scheme
GCE O LEVEL October/November 2012

Syllabus
5014

Paper
11

BALOL
Malaria is water-bred, since the carrier (mosquitoes) breed in water.
Water-based and water-bred only (stated or by description) = 1 mark
Further basic details about differences = 2 marks

[2]

(iii) Age groups young because they have not acquired sufficiently high levels of
resistance; more likely to play in and around surface water/elderly particularly those
who are already infirm.
Poverty people too poor to seek medical assistance once infected, or unable to afford
to take measures of protection to reduce likelihood of catching the disease, or unable to
have access to safe/clean water because of area's poverty.
Farmers and rural dwellers live and work closer to surface water stores, least likely to
have access to piped clean water compared with urban dwellers, plus reasons why.
Farmers at their busiest in the wet season when the risk from water related diseases are
highest.
This is not exhaustive there are other possibilities, such as people affected by natural
disasters like earthquakes/flooding.
For identifying groups/types if three different ones are mentioned = 1 mark merely for
identification.
Rest of marks for explaining why; full 4 mark answers need references to at least two
clearly different types/groups. If well done, answers can still be given all the marks.
Point mark within these guidelines 4 @ 1 mark

[4]

(iv) The most obvious sequence would be


less able/too weak to work (or words to the same effect) followed by less food grown (or
words to the same effect such as 'less well nourished')
Others may follow a more medical sequence.
Mark according to worth
one worthy answer, or an illogical sequence of otherwise worthy answers = 1 mark
similar sequences to the above and logical = 2 marks

[2]

(c) (i) Water shortage live in an area that suffers from persistent periods of drought, great
heat dries up river beds, women have to walk up to 10km which limits amount that can
be carried to be used.
Water quality have to take water from surface scoop holes, suggestions why these are
likely to be dirty and polluted.
1 mark for each of water shortage and water quality taken from the report = up to 2
marks
[3]
3rd mark for some additional explanatory comment for at least one of them.

787

Page 10

Mark Scheme
GCE O LEVEL October/November 2012

Syllabus
5014

Paper
11

BALOL
(ii) Solar power/photovoltaic panel drives the motor
the motor operates the pump
placed within the underground water layer
the pump lifts water to the surface (no manual work involved)
where it is stored in a water tower later to be distributed and used
flows by force of gravity to the water points.
Three points made like these preferably stated in sequence
3 @ 1 mark
Limit of 1 mark for answers about what is there rather than how it all works
Limit of 2 marks for non-logical answers without an overall picture of how it works.

[3]

(iii) Points that can be made supporting sustainability


Solar power is a renewable resource; evidence from earlier references to the weather
that the sun is hot, and if a drought affected area, then there will be no clouds to block
the sunlight. Thus this seems a suitable renewable energy source for this area.
Underground water store, provided that it has not been over-exploited, will provide water
from earlier rains, stored where it cannot evaporate in the heat. People no longer tied to
the vagaries of rainfall for amount of water supply.
Underground water stores are less likely to be polluted than surface stores; water was
cleaned as it filtered down through the permeable rocks
Fact that women no longer have to walk up to 10km means that they can undertake
more productive work, increasing family income and food supply, improving health etc.
Points that may be made in an unsustainable context
Motor and pump might break down; issues such as cost of repairs and know-how.
More efficient pumping of water could increase the speed at which water is taken from
the underground store, making rate of use greater than rate of replenishment, with
adverse long-term consequences.
More food, more water, fewer people die from malnutrition, more live births, more people
leading to greater pressure on the land and land degradation.
Note this is trying to cover the range of possible lines of reasoning; individual
candidate answers will be much narrower.
Answer may rely upon just one line of reasoning (limited supporting breadth)
Not much in the way of explanation beyond stating candidate's view = 1 or 2 marks
Broader references to a rage of different reasons; these can be either for sustainable or
unsustainable or both. One or two of the reasons given may be further explained = 3 or 4
marks
[4]

788

Page 11

Mark Scheme
GCE O LEVEL October/November 2012

Syllabus
5014

Paper
11

BALOL
(d) (i) Just below/to the south of the Himalayas which are the water source
Dam is below the point where two rivers meet to increase amount of water
Further details about how good a water source the Himalayas are relating to the great
heights of the peaks, the high likelihood of stores of ice and snow
Or making the point that numerous rivers are shown on the map starting in the
Himalayas before flowing south
And possibly that being so close to the mountains the valley is likely to be steep sided
and good for building a dam/storing water.
Three points along these lines, either definitely shown on the map, or reasonably
interpreted from it or from candidate knowledge of high mountain areas for water
storage.
3 @ 1 mark
[3]
(ii) 200km

[1]

(iii) Evidence for economic disadvantages:


For the first time, farmers are short of water for their crops because building the dam has
interfered with natural springs, their main source of water. Will lead to loss of income,
perhaps loss of livelihoods
Sending a daily water tanker is not going to help much with amount of water needed for
farming
Evidence for social disadvantages:
No other options but to migrate to Delhi, and social consequences of this
Also other possible social consequences from loss of income
Three points made along these lines, but for full marks the answer must include a valid
reference to a social disadvantage and must include some explanatory comment beyond
comments stated.
[3]
(iv) One clue is in the comment from the Indian government official about the need to look at
the bigger national picture, under the banner of economic development.
Further explanation either related to this or using understanding from the study of other
large dams.
Some understanding; limited or simply expressed = 1 mark
More complete understanding shown by clear explanation = 2 marks

[2]

(v) Not a good way is perhaps easier to justify, using reasons such as:
Will reduce food output in rural areas and will increase the urban problems of Delhi
i.e. problem of poverty is simply transferred from rural to urban areas, where the
resources do not exist to provide for all new city migrants.
A good way can be justified, perhaps along the lines:
Growth in rural areas is stagnant/rural areas are backward
Modern services are more expensive and difficult to provide in rural areas
Cities are the places where modern economic growth is concentrated.
Mark the explanation, not the view expressed.
Some explanation but incomplete or unconvincing = 1 mark
View well explained using either one, or more than one point = 2 marks

[2]

789

Page 12

Mark Scheme
GCE O LEVEL October/November 2012

Syllabus
5014

Paper
11

BALOL
(vi) Comment suggests that volume of water in the river is reduced by storage in the Tehri
dam and by water distribution to farms,
explanation of possible consequences of this such as less water available in Kanpur to
carry/flush away the waste or use in treatment works,
leading to higher concentrations of toxic pollutants from the tanneries like chromium and
chemical by-products in the Ganges at and below Kanpur,
with consequences for river life (plants and fish) and habitats,
reference to water pollution contributions from farming.
Heavy reliance on the source information with limited development = 1 mark
Reasonable understanding, but the answer may be more general rather than referring to
this example = 2 marks
Well arranged and focused answer along these lines = 3 marks
[3]
[Total: 40]

790

Cambridge General Certificate of Education Ordinary Level


5014 Environmental Management November 2012
Principal Examiner Report for Teachers

BALOL
ENVIRONMENTAL
MANAGEMENT

Paper 5014/11
Paper 11

Key messages
Only short answers are expected to the four 10 mark questions in Section A. Beware of extending answers
beyond the lines left for answering. This increased the chances of candidates rushing the last question and
failing to answer in the same detail as in earlier questions.
Read the questions carefully; read each question more than once; underline key question words, such as the
command words, words which tell candidates what to do.
Take careful note of the number of marks for the question. For 3, 4 and 5 mark questions it is not just matter
of filling all the lines; it is highly likely that a variety of points need to be made, or details about an example
given, rather than just repeating one idea.
Avoid over-using vague terms such as pollution. Always specify the type of environmental pollution by at
least adding air, water or land before it as well as something about its nature and origin e.g. when referring to
pollution from a damaged oil rig, do not just write pollution, but preferably write something more detailed e.g.
water pollution by oil from a damaged rig.
General comments
There was no evidence of lack of time for candidates of all abilities. The standard of answers was similar in
Sections A and B.
Only minor variations in the quality of responses between the four short questions which formed Section A
were seen. Some candidates extended their answers below the lines available for answering. Candidates
are often repeating points already made, without further worthwhile elaboration or any use of examples.
Such candidates, in a few cases, spent too much time answering Section A resulting in a noticeable decline
in amount written in the various parts of 6(d) over the last three pages in Section B.
In Section B, questions which were generally well answered by candidates included 5(a)(i) and (ii) (using
the Demographic Transition Model), 5b)(iv) (drawing the line graph), 6(a)(i) and (ii) (using the information
about access to clean water in some world regions) and 6(b)(i) and (ii) (about water-related diseases).
In contrast, the questions which proved to be more difficult than average for the majority of candidates
included 5e)(i) and (ii) (unsustainable world population growth), and 6(a)(iii) (reasons for different rates in
improvement in access to clean water between developing world regions).
Questions which suffered from limited candidate coverage in relation to number of marks available included
5(b)(i), 5(d)(iv) and 6(d)(i).
Many questions required the use of resource information supplied. The better the candidate used the
sources, the more likely the success of the answer. Question instructions most frequently ignored were
describing from the graph as well as the data in 5(b)(v) and explaining in the context of After the
construction of the dam .. in 6(d)(iii).
Using the data provided and what the candidates own line graph showed in 5(b)(iv) were needed for
answering (b)(v) well; only a minority used (rather than repeated) data from the table, as well as describing
what the shape of the line in the graph showed.
Similarly, in the best answers to 6(d)(iii) and (iv) it was clear that candidates had made a careful study of all
comments before starting to write their answers. Answers to 6(d)(i) could only be based upon the resource
map supplied. A careful study of the map was needed, following the rivers southwards from their sources in

791

Cambridge General Certificate of Education Ordinary Level


5014 Environmental Management November 2012
Principal Examiner Report for Teachers
the
Himalayas. Without having done this, some candidates focused their attention and answers on rivers
BALOL
and cities to the south of the Tehri Dam, neither of which had any relevance to the question.
Comments on specific questions
Section A
Question 1
A distance between 9.0 and 10.5 km was considered an acceptable answer in part (a)(i): many candidates
gave a value comfortably within this range.
In answers to (a)(ii) the majority of candidates failed to appreciate the importance of all year in the question.
They answered in terms of the transport links from mine to port, rather than making use of the map
information showing that the sea is ice free all year, despite the location 190 km north of the Arctic Circle.
Evidence for the environmental impact of the mine was plentiful on the map which meant that most
candidates gained credit in (a)(iii).
In most answers to (a)(iv) little knowledge of the tundra climate was shown; many answers lacked any
climatic references, which made it difficult for them to be awarded credit. The significance of the cold tundra
climate in restricting human activities such as mining in these high latitudes was not appreciated.
More candidates gained credit in part (b)(i) with greater numbers understanding that 35% was the amount of
iron ore in the rock, and that the rest was waste.
Such points were often used in valid answers in (b)(ii) about the need to reduce bulk, earning many of these
candidates credit. Fewer candidates continued, mentioning the importance of this reduced mass for
reducing transport costs.
Increased market demand was the most common answer given to part (c) gaining partial credit. Relatively
few candidates referred to higher prices or improved technology to gain full credit.
Question 2
Some candidates realised that they needed to use of their water cycle knowledge to answer (a)(i). Runoff for
X was given more often than infiltration for Y.
(a)(ii) was often well answered with many candidates gaining full or partial credit, and with taken up by plant
roots the most common individual answer.
Those candidates who correctly included dentrification as one of the ways in part (a)(ii) were the ones most
likely to know that the process produced nitrogen when answering (a)(iii).
Part (b) was the best answered part of the question; the effects of algae in shading, and depletion of oxygen
by decomposition, with the low oxygen concentration killing fish earning credit for many candidates.
Few candidates appreciated that that vegetation could trap some of the spray in (c)(i).
In (c)(ii) fewer candidates gave the correct answer of strong winds than the incorrect references to rainfall.
There was more choice of answer in (c)(iii); Some candidates took note of a variety of different types of
vegetation in the question and stressed the importance of increasing habitats, or providing alternative food
sources, earning credit.
Question 3
Most of the attempts to plot the temperature values and link them with a line in part (a) were correct, earning
credit. A few candidates drew a bar graph, despite the instruction in the question to use a line graph. Also
temperature, as an example of continuous data, should only be shown in a line graph.

792

Cambridge General Certificate of Education Ordinary Level


5014 Environmental Management November 2012
Principal Examiner Report for Teachers
In
part (b)(i) candidates either knew that sulfur dioxide and oxides of nitrogen were the gases which caused
BALOL
acid rain, gaining credit, or they did not.
Their effects on forests in (b)(ii) were well known. Many candidates did not give the coverage and
elaboration needed for full credit. Partial credit was given for answers that were about trees losing their
leaves and eventually dying. Such answers typically lacked any references to the soil and the effect on trees
of its increased acidity.
In part (c) some candidates gained full credit, realising that the pollutants often originate in another country
and are carried by winds, which means that affected countries are unable to control the source of the
pollution, gaining full credit. Other candidates gave parts of this, gaining only partial credit.
Question 4
Values between 3.1 and 3.5 million, given by most candidates, were accepted in (a)(i). Some candidates
incorrectly gave values about half this number for males or females only.
In (a)(ii) most candidates gained credit.
For (a)(iii), the graph required careful study and thus fewer candidates gained credit. The age group 20 to
24 gave the clearest evidence of migration into Rwanda and was seen most often.
Part (b) was consistently well answered with many candidates giving references to both economic problems
caused by the loss of the most active work groups, and social problems from families being split up and
gaining full credit. Some candidates developed their answers less thoroughly and gained partial credit.
Some answers to part (c) gained full credit, where candidates had focused fully on the question theme of the
problem of housing, and gave evidence of their understanding of the role of city authorities in providing
services to poor housing areas. Such answers moved on to the way that the interest of city authorities
encouraged community involvement and self-help housing schemes. Most candidates did not provide the
amount of content needed for full credit.
Section B
Question 5
Most candidates gave at least one of the correct stages and many understood the Demographic Transition
Model well enough to give both.
In (a)(ii) most candidates gained credit for correctly giving similarities and differences between the stages..
Question (a)(iii) placed a greater demand on understanding. Many candidates correctly showed natural
increase in stages 2 and 3 and natural decrease in stage 5, even if the shading was not always confined to
the spaces between the lines for birth rate and death rate. .
In part (b)(i) most candidates showed a basic understanding of how the factors in the spider diagram could
decrease a countrys rate of population growth, gaining partial credit. Few candidates elaborated more fully
upon each basic statement in order to gain full credit. For example, there were many partial credit answers
along the lines that with better education women had greater awareness of methods of family planning,
without incorporating another aspect associated with education of women, such as encouraging later
marriages or becoming more career orientated which were required for full credit.
In part (ii) most candidates gained partial credit by answering the why part of the question. Some candidate
did not attempt to answer the how half of the question as well.
In (b)(iii) the correct answer was given by most candidates.
A few candidates encountered problems plotting the data for fertility rates using a line graph in (b)(iv)
although most gained full credit. The occasional inaccuracy in plotting led to some answers gaining partial
credit. The most common reason for no credit was drawing bar graphs, which also consumed more time.
Most answers to (b)(v) gained only partial credit. The main reason for not gaining full credit was that
candidates used data only to describe, without any obvious references to the graph as well, as demanded by

793

Cambridge General Certificate of Education Ordinary Level


5014 Environmental Management November 2012
Principal Examiner Report for Teachers
the
question. The best answers suggested, for example, that the decline in fertility rate was fastest (at 3.1)
BALOL
in the 1990s, shown by the noticeably steeper gradient of the line on the graph.
Part (b)(vi) demanded more understanding. It was pleasing that many candidates linked fast decrease
during the 1990s back to the earlier box information about contraception being more widely available. Full
credit was given to candidates who recognised the greater population stability, associated with a fertility rate
of 2.1 (stable population replacement fertility rate) after social attitudes to population numbers had changed.
Most candidates answers to part (c) began with a sentence about the how national population policies in
general can affect a countrys population growth. This was followed by reference to a country with a
population policy. China was the dominant choice. Next a country without a population policy was named as
a contrasting example. For this the choice of country was wider, but it was often the candidates home
country. This was a sensible choice since it made it more likely that the candidate would be able to give
meaningful elaboration. The amount of credit awarded reflected the amount and accuracy of the supporting
detail. Less credit was awarded for the smaller number of general answers without any named countries, or
no more than passing references to a country without any more information about it. In a few cases Japan
was obviously being confused with China. There were very few answers in which there was no discernible
knowledge of what was meant by a national population policy.
In part (d)(i), most candidates knew how to work out natural increase and gave the correct answer of 15.1.
The few candidates who did not know how to do this typically restated the birth rate from the table.
Most candidates suggested either stage 2 or stage 3 in their answers to (d)(ii). There was no credit for
naming the stage. Many candidates gained partial credit for a simple explanation which matched the stage
chosen. Full credit was given to some candidates who showed very clearly that they had studied the birth
and death rates for Iran in relation to the earlier diagram of the Demographic Transition Model. Some did
this by comparing the size of the birth and death rates per 1000 with those in the graph; others did it by
explaining why it could not be the other stages such as stages 1 or 4. A few candidates suggested a stage
that was clearly wrong such as 4 or 5, or avoided answering the question.
In general the answers for environmental in part (d)(iii) were better than those for economic. Many
candidates mentioned unemployment as an economic pressure. Some others described economic
pressures for the provision of essential services. Deforestation was the most common starting point for the
environmental effects. The amount of credit was determined by the amount of detail given in the
descriptions.
In part (e)(i) and (e)(ii), for those taking the unsustainable view, the most credit-worthy answers came from
candidates who concentrated on the pressure of population growth upon the Earths natural resources. Less
credit was awarded to those who pursued the theme of environmental damage (perhaps following on from
the previous part of the question), sometimes without any mention of natural resources. Little credit was
earned by entirely demographic answers about controlling population growth because most or all of their
content was irrelevant. Only a few candidates took the alternative view that world population growth could
be sustainable due to further increases in technology, leading to a switch in dependence from non-renewable
to renewable energy sources and to higher food output from GM crops and other new seeds. Full credit was
possible for those taking this viewpoint. In either case, the amount of credit reflected the amount and
precision of the supporting detail. Overall, the highest credit in part (e) was earned by those candidates who
had the understanding, as well as the knowledge, to give answers that were focused on what the question
was about.
Overall in Question 5 what discriminated most between answer quality and candidate performance was the
amount of supporting detail in the questions worth 4 marks or more, notably (b)(i), (c), (d)(iii) and (e).
Question 6
In answers to part (a)(i) many candidates gained full credit. Among candidates who made one or more
mistakes, careless reading of the question rather than an inability to read and understand the graph
appeared to be the greater problem. For example, Middle East was the most common incorrect answer to
number 3, most likely because the candidate had not homed in on the key question words greatest and
increase.
Most candidates gained credit for correctly stating one of the two differences between the Middle East and
the four other regions for the mark in (ii).

794

Cambridge General Certificate of Education Ordinary Level


5014 Environmental Management November 2012
Principal Examiner Report for Teachers
In
part (iii) the question actually set was about differences in rates of improvement between developing world
BALOL
regions. Many answers gained little credit as they were incorrectly about differences between developed
and developing countries. A few candidates gave answers correctly focused on differences in wealth and
levels of economic development between industrialising South east Asia and oil rich countries in the Middle
East, compared with the economic stagnation and political instability in Sub-Saharan Africa.
Both cholera and typhoid needed to separated out from the other diseases for credit in (b)(i) to be awarded,
as was successfully done by many candidates. This left bilharzia (water-based using the syllabus
description) and malaria (water-bred) to be described in (b)(ii). Most candidates gained full credit here.
Partial credit was gained by a few candidates in the second part (following a partly correct answer in the first
part) for accurate description of either bilharzia or malaria.
The answers to part (b)(iii) that gained most credit came from candidates who acted on the question
requirement to identify people and groups of people. By including references to poor people, people in rural
areas, young and old people such candidates were able to offer a range of different reasons for the high
risks from water-related diseases. Partial credit was gained by more general answers about developing
countries and their lack of access to clean water, which had been referred to at the start of Question 2.
Without references to people, candidates found it difficult to earn much credit.
What was most likely to be included in the flow diagram about family poverty in (b)(iv) was too weak to work
followed by less food grown or less income (or similar), gaining full credit. All other reasonable possibilities
were accepted, provided that they followed and continued the sequence to lead into family poverty. A
significant number of candidates used unable to afford medical treatment or similar in the first box, followed
by unable to work in the second box, again gaining full credit. A minority of candidates who put people die
in the first box, struggled to keep the flow going in the second box to lead into family poverty; thus gaining
partial credit. Some such candidates referred in the second box referred to the problems for their survivors
and gained full credit. Most candidates gained full credit..
Most candidates used the information given in the report to answer part (c)(i) and gained most or all of the
credit available.
The best answers to (c)(ii) were given by those who used a logical order to describe, basically beginning
with the solar panel driving the motor and working towards the storage and distribution. Candidates who did
not adopt this approach tended to give incomplete answers and gain only partial credit.
In (c)(iii) some candidates gained most or all of the credit by referring to both sustainable (usually renewable
energy source and water store topped up by tropical rains) and unsustainable (possible mechanical
breakdown and water overuse)aspects of the solution. Some candidates tried to base their responses purely
around the word sustainable without deeper consideration, gaining only partial credit. For example, solar
power was stated as sustainable without any further explanation as to what made it sustainable, such as
being a renewable energy source given that sunlight is natural and is always going to be available.
The answers to (d)(i) that gained most credit were based on what was shown in the map and what could be
interpreted from it, using knowledge from the syllabus about large dams. Such answers, for example,
suggested that the Himalayas, shown with peaks above 7000 metres high, would be a very large water
source from high rainfall or ice and snow melt. These answers also noted that, being so close to land above
2000 metres it was also likely that river valleys would be deep and steep. Finally, such answers suggested
that the confluence of two rivers just above the dam position reinforces either of the preceding ideas. Many
candidates gained partial credit for partial answers containing some of these ideas. Some candidates
gained full credit for more complete answers.
Full credit was achieved by many candidates in (d)(ii).
When answering part (iii), the candidates who gained most credit appreciated the question context of after
the construction of the dam. Such answers included references to both economic and social disadvantages,
gaining only partial credit if one or other was omitted. The candidates who gained most credit normally
added explanatory comments to the statements made by villagers. Some other candidates assumed that the
question was about displacement of local people to allow dam construction and thus earned little or no credit.
In (d)(iv) most credit was awarded to candidates who used information from the comment by the government
official and integrated this with understanding gained from having studied examples of large dams
elsewhere. Partial credit was gained by candidates who, having realised that the government officials
comment was the starting point, were then unable to take the explanation further.

795

Cambridge General Certificate of Education Ordinary Level


5014 Environmental Management November 2012
Principal Examiner Report for Teachers

BALOL

Part (d)(v) generated a wide range of answers, most supporting the view. Some candidates moved the
discussion away from water supply issues in Delhi and concentrated either on urban problems in Delhi that
had been made worse, or on the risks from reduced food output in rural areas, gaining full credit. Many
candidates gained little or no credit for discussions about water supply in Delhi. A few candidates gained
credit for opposing the view by arguing that urban areas were better placed for economic growth than rural
areas. Such candidates pointed out that essential services could be provided more cheaply and easily than
in remote rural areas.
Many candidates showed in (d)(vi) that they understood how a reduction in the volume of river water led to
higher concentrations of pollutants, gained much of the credit. Some of these candidates gained full credit
for further detail, either by looking for other possible sources of pollutants such from an extension of farming
after irrigation, or by examining the consequences of reduced water flow in more detail. Little credit was
gained by candidates who relied heavily upon the source information so that any elaboration was limited.
Overall the standard of answers given to Question 6 was comparable with that for Question 5 and Section
A. Again, the longer questions proved more demanding for less well prepared candidates.

796

BALOL
UNIVERSITY OF CAMBRIDGE INTERNATIONAL EXAMINATIONS
General Certificate of Education Ordinary Level

* 1 3 3 6 5 7 4 7 3 4 *

5014/12

ENVIRONMENTAL MANAGEMENT
Paper 1

October/November 2012
2 hours 15 minutes

Candidates answer on the Question Paper.


Additional Materials:

Ruler

READ THESE INSTRUCTIONS FIRST


Write your Centre number, candidate number and name on all the work you hand in.
Write in dark blue or black pen.
You may use a soft pencil for any diagrams, graphs or rough working.
Do not use staples, paper clips, highlighters, glue or correction fluid.
DO NOT WRITE IN ANY BARCODES.
Answer all questions.
All questions in Section A carry 10 marks.
Both questions in Section B carry 40 marks.
The number of marks is given in brackets [ ] at the end of each question or part question.
For Examiners Use
1
2
3
4
5
6
Total

797

BALOL

Section A

For
Examiners
Use

(a) Look at the pie chart showing production of HEP (hydro-electric power) in regions of the
world in 2009 as a percentage of total world HEP production.
0
90

10

production of HEP as a percentage


of total world HEP production
World region
Asia Pacific

20

80

Europe and Eurasia


North America
South and Central America
70

30

60

40
50

(i)

Africa and the Middle East

Source: BP Statistical Review


of World Energy 2010

Complete the pie chart by using the percentages in the table.

region

production of HEP as a percentage


of total world HEP production

Asia Pacific

29

Europe and Eurasia

25
[2]

(ii)

State the approximate percentage for the region which produces the least HEP.
........................................................ %

UCLES 2012

[1]

5014/12/O/N/12

798

BALOL
(b) Complete the table to show conditions which favour HEP production. Write a favourable
condition in each box.
factor

relief (height, slope, shape


of land)

For
Examiners
Use

favourable conditions for the production of HEP


....................................................................................
....................................................................................

....................................................................................
rivers and lakes
....................................................................................

....................................................................................
geology (nature of the rocks)
....................................................................................

....................................................................................
weather and climate
....................................................................................
[4]
(c) Why might people object to plans for a new HEP scheme in their area?
..........................................................................................................................................
..........................................................................................................................................
..........................................................................................................................................
..........................................................................................................................................
..........................................................................................................................................
...................................................................................................................................... [3]

UCLES 2012

5014/12/O/N/12

[Turn over

799

4
2 BALOL
(a) The total weight of fish actually caught in the worlds seas is believed to be much greater
than the catch that is recorded. The table gives estimated information about this for
2010.
recorded fish
catch / tonnes

90 million
(i)

estimated fish catch from

subsistence fishing

by-catch (fish caught during commercial


fishing and thrown away)

illegal fishing
/ tonnes
150 million

For
Examiners
Use

total recorded and


estimated fish
catch / tonnes

240 million

Draw a bar graph to show the information in the table.

[2]

UCLES 2012

5014/12/O/N/12

800

BALOL(ii) Suggest how regulations for commercial fishing can result in fish being thrown
away.

For
Examiners
Use

..................................................................................................................................
..................................................................................................................................
..................................................................................................................................
.............................................................................................................................. [2]
(b) (i)

Explain why subsistence fishing is an important activity in many parts of the world.
..................................................................................................................................
..................................................................................................................................
..................................................................................................................................
.............................................................................................................................. [2]

(ii)

Why is it difficult to prevent illegal fishing?


..................................................................................................................................
.............................................................................................................................. [1]

(c) Explain how problems are caused by overfishing.


..........................................................................................................................................
..........................................................................................................................................
..........................................................................................................................................
..........................................................................................................................................
..........................................................................................................................................
...................................................................................................................................... [3]

UCLES 2012

5014/12/O/N/12

[Turn over

801

6
3 BALOL
(a) Circle the name of the instrument used to measure atmospheric pressure.
anemometer

barometer

hygrometer

Sixs thermometer

[1]

For
Examiners
Use

(b) Look at the graph, which shows pressure recordings over 45 days at a tropical weather
station.

pressure / mb

1050

1050

1000

1000

950

950

900

900
0

10

15

20

25

30

35

40

pressure / mb

45

days
State on which days the area was affected by:
(i)

a cyclone ....................................

(ii)

a period with high pressure ....................................

(c) (i)

[2]

Complete the flow diagram to show why high pressure often leads to dry, sunny
weather, using the 3 correct words from the list:

condenses

cools

evaporates

rises

sinks

warms

high pressure

air in the atmosphere


..................................

the temperature of the


air
..................................

water in clouds
..................................

dry, sunny weather


[3]

UCLES 2012

5014/12/O/N/12

802

BALOL(ii) Describe the difficulties which long periods of drought may cause for people living
in rural areas without access to water supplies.

For
Examiners
Use

..................................................................................................................................
..................................................................................................................................
..................................................................................................................................
..................................................................................................................................
..................................................................................................................................
..................................................................................................................................
..................................................................................................................................
.............................................................................................................................. [4]

UCLES 2012

5014/12/O/N/12

[Turn over

803

8
4 BALOL
Look at the table, which gives information about Honduras and Sweden.
fact

Honduras

percentage population growth rate

2.0

For
Examiners
Use

Sweden
0.2

percentage of population 014 years

38

16

percentage of population 1564 years

58

66

18

4100

36 600

59

percentage of population 65 years and over


annual average income per person (US$)
percentage of population below the poverty level

(a) How many times greater is the population growth rate in Honduras than in Sweden?
...................................................................................................................................... [1]
(b) Use the information in the table to explain why the population pyramid of Honduras will
have a wide base and a triangular shape.
..........................................................................................................................................
..........................................................................................................................................
..........................................................................................................................................
..........................................................................................................................................
..........................................................................................................................................
..........................................................................................................................................
..........................................................................................................................................
...................................................................................................................................... [4]

UCLES 2012

5014/12/O/N/12

804

BALOL
(c) Explain why Swedens population structure might lead to problems now and in the
future.

For
Examiners
Use

..........................................................................................................................................
..........................................................................................................................................
..........................................................................................................................................
..........................................................................................................................................
..........................................................................................................................................
...................................................................................................................................... [3]
(d) State two different ways in which richer countries could help to reduce the problems
which result from poverty in countries like Honduras.
1. ......................................................................................................................................
..........................................................................................................................................
2. ......................................................................................................................................
...................................................................................................................................... [2]

UCLES 2012

5014/12/O/N/12

[Turn over

805

10

BALOL

Section B

For
Examiners
Use

(a) Look at the graph which shows world energy demand by type for 1990 and 2010, with
an estimate for 2030.
World energy demand 1990, 2010 and an estimate for 2030
18
17
16
15
14

billion tonnes oil equivalent

13
Key:

12
11

other renewables

10

hydro-electricity

biomass and waste

8
nuclear energy

7
6

coal

natural gas

oil

3
2
1
0

1990

2010

2030
(estimate)

year
(i)

What was the total world energy demand in 2010 (in billion tonnes of oil equivalent)?
............................................................................................................................. [1]

(ii)

By how many billion tonnes did total world energy demand increase between 1990
and 2010?
............................................................................................................................. [1]

UCLES 2012

5014/12/O/N/12

806

11

BALOL(iii) Describe what the graph shows about the importance of oil in the past, now and in
the future.

For
Examiners
Use

..................................................................................................................................
..................................................................................................................................
..................................................................................................................................
..................................................................................................................................
..................................................................................................................................
............................................................................................................................. [3]
(b) The world in 2010 depended on oil for:

90% of its transport needs,


10% of electricity production.

Explain why world dependence on oil


(i)

is so high for transport,


..................................................................................................................................
..................................................................................................................................
..................................................................................................................................
............................................................................................................................. [2]

(ii)

is much lower for electricity.


..................................................................................................................................
..................................................................................................................................
..................................................................................................................................
..................................................................................................................................
..................................................................................................................................
............................................................................................................................. [3]

UCLES 2012

5014/12/O/N/12

[Turn over

807

12

BALOL
(c) Unfortunately cars, trucks and buses are major causes of atmospheric pollution.
The sketch below shows the materials that come from vehicle exhausts and some of the
effects that they cause.

For
Examiners
Use

..................................
greenhouse gas
carbon
dioxide

...............................
soot &
dirt

plant and lake


life dies
acid rain

nitrogen
oxides
.........................

Three labels are missing from the sketch. The information below will enable you to write
in suitable labels to complete the sketch.
On the sketch, write in the missing labels for:
(i)

the effect of carbon dioxide emissions,

(ii)

one effect on people of emissions of soot, dirt and nitrogen oxides,

(iii)

another gas that causes acid rain.


[3]

UCLES 2012

5014/12/O/N/12

808

13

BALOL(iv) Explain how the atmospheric pollution from vehicle exhausts shown on the sketch
has effects which are local, international and global.

For
Examiners
Use

..................................................................................................................................
..................................................................................................................................
..................................................................................................................................
..................................................................................................................................
..................................................................................................................................
..................................................................................................................................
..................................................................................................................................
............................................................................................................................. [4]
(v)

Look at this message which was displayed on the side of a bus in Dubai, part of the
UAE in the oil-rich Middle East.

This bus is more environmentally friendly than a car


40 cars = 230,000 kg of CO2 per year

This bus = 3,200 kg of CO2 per year

What is the message? What is the government of Dubai trying to persuade people
to do?
..................................................................................................................................
..................................................................................................................................
..................................................................................................................................
............................................................................................................................. [2]
(vi)

Describe another way in which air pollution from traffic can be reduced.
..................................................................................................................................
..................................................................................................................................
..................................................................................................................................
............................................................................................................................. [2]

UCLES 2012

5014/12/O/N/12

[Turn over

809

14

BALOL
(d) Atmospheric pollution is a much greater problem in some countries and cities than in
others.
(i)

For
Examiners
Use

Name a country or city where air pollution is a major problem, with high levels
frequently recorded.
..................................................................................................................................

(ii)

Give reasons why air pollution levels are higher in the country or city named in
(d)(i) than in most other places.
..................................................................................................................................
..................................................................................................................................
..................................................................................................................................
..................................................................................................................................
..................................................................................................................................
..................................................................................................................................
..................................................................................................................................
..................................................................................................................................
..................................................................................................................................
............................................................................................................................. [5]

(e) Pressure is increasing on all governments to reduce their dependence on oil and other
fossil fuels.
(i)

Look back to the graph in part (a) on page 10. Describe what the estimate for 2030
shows about how important the use of fossil fuels is expected to be, compared with
what it was in 1990 and 2010.
..................................................................................................................................
..................................................................................................................................
..................................................................................................................................
..................................................................................................................................
..................................................................................................................................
............................................................................................................................. [3]

UCLES 2012

5014/12/O/N/12

810

15

BALOL(ii) Look at the graph which shows costs of producing electricity from newly built power
stations in relation to world oil prices. The higher the oil price, the more economic it
becomes to build electricity power stations using other energy sources.

For
Examiners
Use

nuclear power

onshore wind

offshore wind

40

60

80
100
120
world price of oil per barrel / US$

140

160

In August 2010 the average world price of oil was US$75 per barrel.
Draw a vertical line down the graph to show the world oil price in August 2010. [1]
(iii)

Suggest reasons why a range of costs (instead of just one) is given for the
production of electricity from wind and nuclear power to produce electricity.
..................................................................................................................................
..................................................................................................................................
..................................................................................................................................
............................................................................................................................. [2]

(iv)

How likely were governments and companies to start building new electricity power
stations in 2010 using wind and nuclear power? Explain your answer, using both
the graph and your knowledge of wind and nuclear power.
..................................................................................................................................
..................................................................................................................................
..................................................................................................................................
..................................................................................................................................
..................................................................................................................................
..................................................................................................................................
..................................................................................................................................

UCLES 2012

............................................................................................................................. [4]
5014/12/O/N/12
[Turn over

811

16

BALOL
(f) A student was asked to assess the costs and benefits of one type of energy. The results
are given below.

For
Examiners
Use

Energy assessment
renewable
no carbon dioxide emissions
safe
cheap
known technology
simple technology for use in developing countries
always available, not weather dependent
fully sustainable
(i)

Which type of energy was the student assessing? Circle one answer.
oil

(ii)

8
9
8
8
9
8
9
8

nuclear

wind

[1]

Explain your choice.


..................................................................................................................................
..................................................................................................................................
..................................................................................................................................
..................................................................................................................................
..................................................................................................................................
............................................................................................................................. [3]
[Total: 40 marks]

(a) Look at the pie charts showing total world urban population between 1950 and 2010.
2010
1990
1970
1950

770 million

1690 million

2310 million

3600 million

Key:
developed countries
developing countires
(i)

In which 20 year period was the increase in urban numbers greatest?


............................................................................................................................. [1]

UCLES 2012

5014/12/O/N/12

812

17

BALOL(ii) Describe how the percentage distribution of urban population between developed
and developing countries changed over the years from 1950 to 2010.

For
Examiners
Use

..................................................................................................................................
..................................................................................................................................
..................................................................................................................................
..................................................................................................................................
..................................................................................................................................
............................................................................................................................. [3]
(iii)

The table shows the total world population in millions between 1950 and 2010.
year

1950

1970

1990

2010

population

2800

3800

5250

6750

Complete the bar graph by showing world population totals, and the numbers living
in urban areas stated in part (a), for 1970, 1990 and 2010.
[3]
7000
6000
5000
world
4000
population
/ millions
3000

Key:
urban
population

2000
1000
0

1950

1970

1990

2010

year
(iv)

Describe how the bar graph shows widespread world urbanisation since 1950.
..................................................................................................................................
..................................................................................................................................
..................................................................................................................................
............................................................................................................................. [2]

UCLES 2012

5014/12/O/N/12

[Turn over

813

18

BALOL
(b) The two main causes of rapid urban growth in developing countries are high rates of
natural increase and large scale rural to urban migration.

For
Examiners
Use

Explain why both of these are higher and greater in developing countries than in
developed countries.
..........................................................................................................................................
..........................................................................................................................................
..........................................................................................................................................
..........................................................................................................................................
..........................................................................................................................................
..........................................................................................................................................
..........................................................................................................................................
..........................................................................................................................................
..................................................................................................................................... [5]

UCLES 2012

5014/12/O/N/12

814

19

BALOL
(c) Housing is a major urban problem in many of the big cities in developing countries.

For
Examiners
Use

Look at the cross-section through a developing world city.


FLW\FHQWUH
KRPHVIRU
ULFKSHRSOH

ROG
KRXVLQJ

IDFWRULHV

ULYHU

(i)

Poor migrants from rural areas build their own homes, creating shanty towns.
On the cross section, mark S for a likely location of a shanty town.

(ii)

[1]

Explain the advantages and disadvantages of this location for new arrivals in the
city.
..................................................................................................................................
..................................................................................................................................
..................................................................................................................................
..................................................................................................................................
..................................................................................................................................
............................................................................................................................. [3]

UCLES 2012

5014/12/O/N/12

[Turn over

815

20

BALOL
(d) Read the newspaper report describing how Diadema, home for 400 000 people, has
changed. It lies on the southern edge of Sao Paulo, Brazils largest city.

For
Examiners
Use

Jimmy da Silva remembers what Diadema was like when he was


young. There was row upon row of ramshackle houses, made from
wood and corrugated iron, separated by open sewers. Drug dealers
ruled the streets. Everyone was afraid of the regular police raids.
Murder rates were among the highest in Brazil.
Twenty years later Diadema is a different place, after the Workers
Party took control of local administration. They paved the roads, set up
street lighting and built libraries and cultural centres. With the support
of NGOs (non-governmental organisations) and local businesses,
police cleared out the drug dealers.
The area is still poor. However, brightly painted cheap cafes, fruit
stalls and beauty parlours line its main street. Buses link Diadema with
the citys underground rail system so that Sao Paulo city centre is less
than an hour away. Children play football and swim at the new sports
centre, which was paid for by local businesses.
As a summary, it can be said that, over 20 years, Diadema has changed
from a dangerous shanty town to a stable low-income neighbourhood.
(i)

Describe what Diadema used to be like, that made it a typical shanty town similar
to many others in developing world cities.
..................................................................................................................................
..................................................................................................................................
..................................................................................................................................
............................................................................................................................. [2]

(ii)

Diadema is now described as a stable low-income neighbourhood. State the


evidence which supports this summary statement of Diadema.
..................................................................................................................................
..................................................................................................................................
..................................................................................................................................
..................................................................................................................................
..................................................................................................................................
............................................................................................................................. [3]

UCLES 2012

5014/12/O/N/12

816

21

BALOL(iii) What were the two main reasons why this big change in Diadema took place?
..................................................................................................................................

For
Examiners
Use

..................................................................................................................................
..................................................................................................................................
............................................................................................................................. [2]
(e) (i)

Rapid urban growth in the cities in developing countries is unsustainable because


of damage to the environment.
Describe three types of environmental damage caused by urban growth.
1 ...............................................................................................................................
..................................................................................................................................
..................................................................................................................................
2 ...............................................................................................................................
..................................................................................................................................
..................................................................................................................................
3 ...............................................................................................................................
..................................................................................................................................
............................................................................................................................. [5]

UCLES 2012

5014/12/O/N/12

[Turn over

817

22

BALOL(ii) Look at the information for Freiburg, a city in southern Germany.

For
Examiners
Use

Changes in Freiburg 19922007

population

CO2 emissions
15

10

decrease

0
percentage
change

10
increase

15

Describe how the graph shows that Freiburg is a more sustainable city than most
others.
..................................................................................................................................
..................................................................................................................................
..................................................................................................................................
............................................................................................................................. [2]
(f)

An eco-city is one that is designed to be sustainable with as little environmental impact


as possible. Some of the characteristics of an eco-city are shown on the sketch below.
train or tram
link to other
cities
cycle tracks
and walkways
shopping
centre
renewable
energy sources
on roofs
lake

water treatment
works

UCLES 2012

for water
supply and
recreation

plentiful green
spaces around
houses

eco-friendly
buildings

energy
efficient
buildings

5014/12/O/N/12

818

23

BALOL (i) Choose two of the eco-city characteristics shown. Describe fully how they make
city living more sustainable.

For
Examiners
Use

1 ...............................................................................................................................
..................................................................................................................................
..................................................................................................................................
..................................................................................................................................
2 ...............................................................................................................................
..................................................................................................................................
..................................................................................................................................
............................................................................................................................. [4]
(ii)

Explain what can be done with the waste produced by people living in the eco-city
in order to reduce impacts on the environment.
..................................................................................................................................
..................................................................................................................................
..................................................................................................................................
..................................................................................................................................
..................................................................................................................................
..................................................................................................................................
..................................................................................................................................
............................................................................................................................. [4]
[Total: 40 marks]

UCLES 2012

5014/12/O/N/12

819

BALOL
CAMBRIDGE INTERNATIONAL EXAMINATIONS
GCE Ordinary Level

MARK SCHEME for the October/November 2012 series

5014 ENVIRONMENTAL MANAGEMENT


5014/12

Paper 1, maximum raw mark 120

This mark scheme is published as an aid to teachers and candidates, to indicate the requirements of
the examination. It shows the basis on which Examiners were instructed to award marks. It does not
indicate the details of the discussions that took place at an Examiners meeting before marking began,
which would have considered the acceptability of alternative answers.
Mark schemes should be read in conjunction with the question paper and the Principal Examiner
Report for Teachers.

Cambridge will not enter into discussions about these mark schemes.

Cambridge is publishing the mark schemes for the October/November 2012 series for most IGCSE,
GCE Advanced Level and Advanced Subsidiary Level components and some Ordinary Level
components.

820

Page 2

BALOL

Mark Scheme: Teachers version


GCE O LEVEL October/November 2011

Syllabus
5014

Paper
12

Notes on application of the mark scheme for Section A


marks are separated by commas. Each line usually represents one mark
oblique lines separate ideas which are alternatives
ideas in brackets are not essential to the answer but anything underlined is
reward any equivalent way of expressing the ideas in the mark scheme
reward any valid answer which is not in the mark scheme
Section A
1

(a) (i) correct plot = 1 mark


Attempt to use shading as in the key = 1 mark
(ii) value between 2.5 and 3.5 %

[2]
[1]

(iii) most likely answers include:


relief: mountainous / steep slopes / narrow steep sided valleys.
rivers and lakes: waterfalls / fast flowing / great volume of water / natural lakes for water
storage, water flow all year.
geology: impermeable rock / hard rock for building dam wall / away from earthquake
risk / absence of faults.
weather and climate: high precipitation / precipitation all year / no freezing in winter / cool
summers keeping rates of evaporation low.
4 @ 1 mark (one for each factor)

[4]

(c) reasons such as:


loss of farm land,
often land on valley floors is the best land for farming,
people forced to leave homes / settlements,
uncertain future with potential social / economic consequences for them,
loss of habitats / natural environments,
area becomes less attractive (e.g. ugly dam wall, power lines),
increased risk of water-related diseases,
dam water going to be of no benefit to them for water supply / power,
disturbance during the construction phase.
three valid reasons = 3 @ 1 mark
credit good elaboration of one of the reasons up to 2 marks.

[3]
[Total: 10]

821

Page 3

BALOL
2

Mark Scheme: Teachers version


GCE O LEVEL October/November 2011

Syllabus
5014

Paper
12

(a) (i) plots at 90, 150 and (240) for a valid regular scale = 1 mark
individual bars or sectors within a divided bar identified = 1 mark

[2]

(ii) suggestions why fishermen discard some of their catch:


catch is greater than the quota,
fish caught are smaller than the regulations allow,
type of fish caught is not permitted,
types of fish caught are of limited commercial value.
two suggestions along these lines.
2 @ 1 mark

[2]

(b) (i) important food source for the family / community,


high in protein increasing its food value,
allows settlement / life in areas where agriculture is difficult / impossible,
example of areas / peoples such as tundra Arctic regions.
points made along these lines.
2 @ 1 mark

[2]

(ii) reasons include:


large areas of sea / ocean to patrol,
expensive to do checks both in the ports and at sea,
authorities in many countries lack the resources / technology needed,
corruption of officials / authorities turning a blind eye (especially in remote areas),
uncooperative fishermen.
one reason such as these.

[1]

(c) consequences of overfishing include:


insufficient fish / no young fish left in the sea to breed,
fish numbers fall over the years so that commercial fishing cannot be sustained,leads to
income / job losses in coastal communities,
fishermen more likely to search out new fishing grounds,
more and more pressure on ocean resources resulting in less fish caught to eat,
disrupts the food chain.
three consequences such as these leading to problems.
3 @ 1 mark

[3]
[Total: 10]

822

Page 4

BALOL
3

Mark Scheme: Teachers version


GCE O LEVEL October/November 2011

Syllabus
5014

Paper
12

(a) barometer circled (or otherwise clearly identified)

[1]

(b) (i) cyclone allow day 13 or 14


(ii) period of high pressure from day 21 to about day 40; at least a 5 day period is needed
within this time span.
2 @ 1 mark

[2]

(c) (i) air in the atmosphere sinks


air temperature warms / allow rises
water in the clouds evaporates.
3 @ 1 mark

[3]

(ii) difficulties include:


need to walk / travel further to find water supplies,
more likely to have to drink dirty water with effects on health,
crops die / yields are reduced,
lack of pastures means their animals lose condition / die,
results in malnutrition / hunger for people,
less strong and less able to work,
no seeds for the next harvest,
thus the effects multiply with the length of drought,
may be forced to leave their lands / homes and migrate to refugee camps,
desertification leading to barren land for future use by people.
difficulties such as these credit both identification and development, as well as any
reference to examples.
4 @ 1 mark
[4]
[Total: 10]
4

(a) 10

[1]

(b) a large young population (38 %) for the wide base,


population growth rate of 2 % will help to maintain base width,
a small old population (only 4 % are 65 and over),
therefore the pyramid has a narrow top to it,
middle age group is the largest (58 %) but this is spread over more years (about 40).
points made like these with comment and use of data up to 4 marks
simple repeat of wide base / triangular shape as in the question, without support = 0 marks
[4]

823

Page 5

BALOL

Mark Scheme: Teachers version


GCE O LEVEL October/November 2011

Syllabus
5014

Paper
12

(c) ideas such as:


information shows that it has an ageing population,
the young age group is so small (only 16 %) that there will be fewer workers in future,
means that less funding will be available from to taxes to support elderly people,
in other words the dependency ration will increase,
how and why the elderly are an expensive group to provide for their needs,
further comment about pensions / healthcare costs etc.
three points made along these lines.
3 @ 1 mark

[3]

(d) aid references can claim both marks if different types of aid are identified and elaborated.
trade that is more fair increased trade in commodities that the developed world will buy;
Fair Trade to help communities as well as producers, plus leading to improved infrastructure.
new sources of income such as tourist income from attractions for people to visit from richer
developed world countries, earning foreign exchange; ecotourism spreads the benefits.
two headings for ways only, without elaboration = 1 mark
two ways identified with support = 2 marks
each of the identified ways is capable of leading to two marks, provided that two clearly
different ways are stated.
[2]
[Total: 10]

824

Page 6

Mark Scheme: Teachers version


GCE O LEVEL October/November 2011

BALOL

Syllabus
5014

Paper
12

Section B
5

(a) (i) 12

[1]

(ii) 3.2

[1]

(iii) The most used energy source at all three dates (however expressed), actual values
stated from the graph to show high use (3.3 in 1990, 4.0 in 2010 and 4.5 in 2030) up to
2 marks with comment / proper context / more than a list.
Some idea of size of oil use relative to the total / about one third of total, percentage of
total expected to reduce slightly by 2030 (closer to one quarter than to one third of the
total).
Three points such as these.
3 @ 1 mark

[3]

(b) (i) Transport;


Oil is the basis for petrol / diesel / jet fuel used for motor vehicles, trains and aircraft,
despite electric trains, some electric cars and some biofuels there is no substitute as
widely available,
Cheap and easy to use,
Modern transport growth directly linked with oil based fuels.
Understood and clearly explained (whatever line of argument is used) = 2 marks
Some understanding but less strong explanation = 1 mark

[2]

(ii) Other alternatives for making electricity are more widely available and some have been
in use for a long time,
Before oil electricity was widely generated from burning coal,
Coal most widely used in countries with large deposits of the mineral such as China,
Gas is cleaner to burn than coal and oil which explains increasing use for electricity,
Good physical conditions exist for even cleaner renewables such as HEP in some
countries,
Some are more environmentally friendly / sustainable ways of generating electricity.
Valid reasons such as these.
3 @ 1 mark
Maximum of 2 marks for answers without some comment included within them towards
the general theme of much lower for electricity than transport.
[3]
(c) (i) Global warming
(ii) Breathing problems / specific example such as asthma
(iii) Sulfur dioxide
3 @ 1 mark

[3]

825

Page 7

BALOL

Mark Scheme: Teachers version


GCE O LEVEL October/November 2011

Syllabus
5014

Paper
12

(iv) Examples of local effects soot and dirt leading to breathing problems, also discolouring
brickwork and stonework, some acid rain is precipitated in the local area with damaging
effects on plant life / rotting brickwork and stonework.
Examples of international effects gases and acid rain can be transferred by winds to
other countries / don't stop at country borders, especially where large industrial areas
are located close to national borders.
Examples of global effects most obvious is the increasing accumulation of carbon
dioxide in the atmosphere, with subsequent effects of global warming such as rising sea
levels, melting glaciers and increases in natural climatic hazards, examples of locations
most at risk from the global impacts.
1 mark for distinguishing between examples of effects at all three levels / appreciation of
the three different scales.
Remaining 3 marks for further description of how the effects operate at the different
scales.
[4]
(v) One bus can carry the same number of people as 40 cars with much lower carbon
dioxide emissions into the atmosphere (only 1.5 %)
Message to get people to give up their cars and use public transport.
Clear answers for both questions, showing a full understanding = 2 marks
Answers more dependent on direct use of information given, enough to display a basic
understanding = 1 mark.
[2]
(vi) 'Another way' means that the option of more use of public transport cannot be used,
unless it is clearly different from the Dubai example used in the question, such as
underground metro systems to replace surface traffic e.g. in Cairo.
Choice of other ways includes:
Reducing vehicle emissions either by using catalytic converters on exhausts, using
cleaner fuels like CNG, biofuels, CBG (cleaner burning gasoline in LA);
description could take the form of examples such as CNG to drive Indian tuk-tuks.
Traffic management schemes such as banning vehicles from city centres, prohibiting
entry of cars on certain days according to registration numbers.
One other valid way such as these identified and described = 2 marks.

[2]

(d) (i) Obvious choices are China among the countries and Los Angeles, Santiago and Beijing
among the cities.
Award one separate mark for naming a country or city if it is a well known air pollution
hotspot.

826

Page 8

BALOL

Mark Scheme: Teachers version


GCE O LEVEL October/November 2011

Syllabus
5014

Paper
12

(ii) Factors which influence the level of air pollution include:


Physical such as dominance of high pressure and sinking air, low average wind speeds,
little rainfall, and valley / plateau between higher mountain peaks where inversions of
temperature can help trap the pollutants
Existence of or level of regulation for control of emissions from factories and vehicles
degree of enforcement of these regulations
Wealth of the country since reducing emissions costs money for individuals, companies
and governments; also amount of industrial development and economic activity.
Valid name in (i) = 1 mark
Otherwise mark the two parts of the answer together.
General answers; general factors without anything specific for the named country or city
or even none named. Maximum of 3 marks for answers about factors which cause some
places to be more badly affected by air pollution than others. Three mark answers are
most likely for candidates who refer to both physical and human factors = 1 to 3 marks
Answers with something specific for a named country or city; reference to both physical
and human factors = 4 to 5 marks
[5]
(e) (i) Use of fossil fuels values taken from the graph
1990 7.2 out of 8.8 = 82 % of total
2010 9.7 out of 12.0 = 81 % of total
2030 12.8 out of 16.8 = 76 % of total
Clear recognition (either from a statement or from graph use) that oil, coal and natural
gas are the fossil fuels, realisation that their relative importance is expected to fall slightly
by 2030, as a result of slight increases in each of the other four alternative sources, but
without fossil fuels losing their great importance / dominance, use of values from the
graph to support statements made along these lines.
3 @ 1 mark in line with the above guidance for marking.
(ii) Vertical line accurately placed half way between 70 and 80 on graph.

[3]
[1]

(iii) Costs of setting up power plants can vary for a variety of reasons such as:
How good are the natural advantages of the location
How easy is access for construction and operation
Costs of labour and materials in one country compared with another
Need or otherwise to import the materials, equipment and technology.
The difference in costs shown between offshore and onshore wind electricity production
suggests ease of access and operation for land wind farms.
Some basic understanding why the cost of setting up power plants is not going to be the
same everywhere (in all countries and types of locations) = 1 mark
Valid reasons for this suggested along the lines indicated above = 2 marks
[2]

827

Page 9

BALOL

Mark Scheme
GCE O LEVEL October/November 2012

Syllabus
5014

Paper
12

(iv) The graph helps by showing that:


Of the three power types, only onshore wind is cost competitive with oil at the world oil
price prevailing in mid-2010
Most of them are significantly more costly; for example, offshore wind production can be
double the cost
Building new nuclear power stations needs an oil price above 90 to make it cost
competitive.
Knowledge of wind power and nuclear might be used to make points such as these:
There are no carbon dioxide emissions from nuclear and wind power, and the
importance of this
Wind in particular is a renewable and environmentally friendly alternative to fossil fuels
Some governments are working to targets to reduce their emissions so they offer
financial support to companies, thereby reducing their costs of construction
Not everywhere is suitable for wind power and there can be local opposition
Not every country has the technology and money needed to set up nuclear power
stations.
The evidence suggests that companies are not too likely to be setting up large numbers
of wind and nuclear power plants with a world oil price of 75 dollars, as in mid-2010.
The financial incentive is not there.
However, a candidate can come to another conclusion provided it is justified, such as by
stressing that economic factors can be less important than political factors, or referring to
variations in oil prices and possible / likely future increases in prices.
Answer not well organised and/or incomplete. Basic points only made.
Any views expressed weakly supported by graph use or knowledge = 1 or 2 marks
Clear focus on the likelihood or otherwise of building new wind and nuclear power
plants; explanation based upon both graph evidence and knowledge = 3 or 4 marks [4]
(f)

(i) Nuclear indicated in a clear way to the exclusion of the others

[1]

(ii) Possible to answer on an individual point basis, either directly for nuclear or
comparatively for how it is different from one or both of the others.
Examples:
Not renewable since uses uranium as its raw material, whereas wind is because it uses
the weather
Not safe because dangers of radiation and its consequences for living things
Wind is weather dependent which rules it out
Oil has carbon dioxide emissions which rules it out.
Also possible to answer it on a general basis, for instance giving additional details of the
characteristics of nuclear power which match up with the assessment.
3 @ 1 mark for explanation

[3]
[Total: 40]

828

Page 10

BALOL
6

Mark Scheme
GCE O LEVEL October/November 2012

Syllabus
5014

(a) (i) 1990 2010

Paper
12
[1]

(ii) Basic change from a higher percentage in developed in 1950 to a much higher
percentage in developing in 2010,
References to percentages to illustrate this such as 1950 ratio 62%:38%, compared with
25%:75% by 2010,
Change in distribution between developed and developing occurred between 1970 and
1990,
Every 20 year period shows an increased percentage in developing / smaller percentage
in developed.
Three points made along these lines.
3 @ 1 mark

[3]

(iii) Bar graph accurately completed in terms of numbers plotted and attempt to use the
same type of shading.
3 @ 1 mark each for 1970, 1990 and 2010

[3]

(iv) The best definition for urbanisation is the increasing percentage of people living in urban
areas; therefore the most conclusive evidence from the graphs is the persistent increase
in the proportions of total world population living in cities, to the point where by 2010
more than half the world's population was urban.
Answer along these lines showing understanding = 2 marks
Part answer emphasising urban growth rather than proportions = 1 mark

[2]

(b) High rates of natural increase;


Big differences between high birth rates and low death rates,
most likely supported by reasons for high birth rates such as poverty, traditions, value of
children as workers, role of women.
Possible also to mention why death rates have gone gone down due to spread of
rudimentary health care and extinction of major killer diseases like smallpox.
Rural to urban migration:
Big differences in wealth, service provision and perceived opportunities between rural and
urban areas, much bigger than in developed countries where personal mobility is greater and
essential services such as water and electricity are supplied in all but the most remote rural
areas.
Essentially 2 + 3 marks (for either element).
In all cases limit of 3 marks for answers without some comparative references to why higher
and greater in developing than in developed world countries.
[5]

829

Page 11

BALOL

Mark Scheme
GCE O LEVEL October/November 2012

Syllabus
5014

Paper
12

(c) (i) S marked on the cross section in one of the empty spaces either on the steep sloped
left of the river, or on the flat land next to it on the right, or on the steeper land on the
right side.
[1]
(ii) Advantages provides the unused urban or near urban land on which poor rural can
build shelters/homes; new in-migrants cannot afford the rents for already built city
housing. Comment could also be made about the closeness of the location to the city
and the opportunities to find work (formal or informal). Depending on the location used,
this could be for city centre jobs or nearness to the factories. A location chosen left of the
river might be less good in this respect.
Disadvantages steep locations at risk from landslides etc, especially after heavy rain,
or events such as earthquakes. Floodplain location prone to flooding; also stagnant
water might be breeding grounds for mosquitoes. In other words, land not already built
on was probably left for a good reason. All will lack essential services, at least at first,
because they are not part of the already built-up area. Increasing distances to work
would be an additional factor for locations higher up the slope on the right.
No valid location on section; general points only = maximum 1 mark
Location valid, but limited range of points; valid points may be all advantages or all
disadvantages = 2 marks
Answer well related to the chosen location, with a healthy mixture of advantages and
disadvantages = 3 marks
[3]
(d) (i) Relevant content from the newspaper account ramshackle houses, made from wood
and corrugated iron, separated by open sewers, drug dealers ruling streets, police raids,
high murder rates In other words, typical appearance and the same social problems that
are widespread in shanty towns.
One or two descriptive points = 1 marks
Fuller choice of points and / or with a short comment directing the answer towards the
question theme = 2 marks
[2]
(ii) Services provided / infrastructure improved paved roads, street lighting, libraries and
cultural centre built, sports centre, bus services with a link to the rail system.
Drug dealers cleared out.
Commercial life cafes, fruit stalls and beauty parlours along the main street.
Evidence up to 2 marks based on completeness or how well it is arranged.
Some comment about how these shows a stable low income neighbourhood for the 3rd
mark.
2 + 1 mark = 3 marks

[3]

830

Page 12

BALOL

Mark Scheme
GCE O LEVEL October/November 2012

Syllabus
5014

Paper
12

(iii) Workers' Party took control of the local administration and provided essential services.
NGOs and local businesses supported the changes and financed some new amenities
such as the sports centre.
The police took an interest and cleared out the drug dealers.
These are main reasons directly stated in the newspaper account. A reasonable
inference would also be that people are more likely to be in work from improved
transport links to Sao Paulo city centre.
Any two of these.

[2]

(e) (i) Rivers polluted, used as dust bins for human waste and litter
Polluted lakes and seas from lack of sanitation, destroying all signs of plant and fish life
Overuse of underground water stores, present use much greater than rate of natural
replenishment
City growth and sprawl destroy woodland and natural habitats
Air pollution from congested roads / factories.
Three types of environmental damage identified; allow other valid suggestions = 3 @ 1
mark.
4th and 5th marks for some further description either related to either cause or effects,
and / or use of an example.
[5]
(ii) The basic point is that despite increasing its population by 10 %, Freiburg has managed
to reduce its carbon dioxide emissions by an even larger 14 %.
Here population growth is not leading to increased emissions and a greater contribution
to world atmospheric pollution, which is why its growth can be described as more
sustainable.
Well understood, and probably answered with the support of values from the graph = 2
marks
Some understanding, but description how could have been more complete = 1 mark [2]
(f)

(i) The sketch includes:


Provision of public transport links to other cities to reduce use of private car for work etc.
Plenty of cycle tracks and walkways. linking to the shopping centre, again avoiding need
for car use
Plentiful green spaces around houses and lake provide open spaces for recreation and
play, improve appearance, offer natural habitats, absorb rain water reducing risk of
excessive runoff leading to flooding, improve quality of life
Eco-friendly / energy efficient buildings with insulation etc. to stop either heat loss or
reduce outside heat from entering. Eco-friendly could also include the use of natural or
recycled building materials
Renewable energy sources on roof sketch shows wind turbines and solar panels for
clean, renewable energy supplies
Water treatment works sustainable cleaning / re-use of water.
Mark amount and quality of the description towards the theme of sustainable living. Most
likely is a divide 2 + 2 marks, but allow 3 + 1 if deserved.
[4]

831

Page 13

BALOL

Mark Scheme
GCE O LEVEL October/November 2012

Syllabus
5014

Paper
12

(ii) One clue in the sketch to trigger off the answer is the presence of the water treatment
works on one side of the lake. Prevents all the disadvantages to the environment from
the disposal of raw sewage, plus can supply recycled clean water for reuse; water is a
precious resource for many cities.
For other wastes such as domestic, commercial and industrial it is best if using landfill
can be avoided with its high environmental impacts in favour of reusing (e.g. glass
bottles), and recycling (e.g. paper and plastics) as well as reducing waste by separating
out organic waste for composting or bio-digesting.
Expect most of the answer to be about how impacts on the environment can be reduced,
but allow some credit for references to environmental impacts of untreated wastes.
Four valid points relevant to the question theme.
4 @ 1 mark

[4]
[Total: 40]

832

Cambridge General Certificate of Education Ordinary Level


5014 Environmental Management November 2012
Principal Examiner Report for Teachers

BALOL
ENVIRONMENTAL
MANAGEMENT

Paper 5014/12
Paper 12

Key messages
Plan carefully your use of examination time. Spend no more than 45 minutes answering the four short
questions in Section A; in Section B, spend no more than 45 minutes answering Question 5, so that 45
minutes remain to answer Question 6.
Only short, precise answers are needed to questions in Section A. Try to make the number of different
points or reasons to the number of marks for the question. Do not extend answers beyond the number of
lines left for answering to reduce the chances of running out of time before finishing all parts of
Question 6.
Longer answers, supported by elaboration and / or examples may be expected to some questions in Section
B. Always be guided by the number of marks as to the amount of detail needed.
Read each question more than once; underline key question words, especially the command words, the
words which tell candidates what to do.
Make sure that all parts of the question are covered in the answer.
Do not begin by repeating the question. Sometimes this filled the first two lines of the answer. Begin the
answer straight away. This makes it more likely that a full answer to the question has been given by the time
all the lines for answering have been filled.
General comments
There was no evidence of lack of time for candidates of all abilities. The standard of answers was similar in
Sections A and B.
There were a few cases of candidates failing to complete all the later parts of Question 6. In such cases,
the short questions in Section A and some parts of Question 5 in Section B contained too much detail for
the nature of the questions and number of marks available before these candidates belatedly became aware
of time pressures. Careful planning of use of examination time is essential, especially by more able
candidates with good understanding and plentiful knowledge.
Within Section B, candidates were most comfortable with all the questions in 5(c) (pages 12 and 13), and in
6(d)(f) (pages 20 to 23). Parts of 5(e) and (f) proved to be more challenging. Perhaps Questions 5(e)(iii)
and (iv) were the two most difficult questions of all. Many candidates tried to relate their answers in 5(e)(iii)
to the world price of oil. In the next part, 5(e)(iv), most made too little use of the graph; they found it difficult
to appreciate how the world price of oil could affect the likelihood or otherwise of alternative ways of
electricity generation being used more widely. While the vast majority identified nuclear as the energy being
assessed in 5(f)(i), many did no more than repeat everything from the table beginning with not renewable,
no carbon emissions, without ever adding any further explanatory comments.

833

Cambridge General Certificate of Education Ordinary Level


5014 Environmental Management November 2012
Principal Examiner Report for Teachers
Comments
BALOL on specific questions
Section A
Question 1
Sectors drawn on the map in (a)(i) were nearly always correct. A few candidates, who for ease and
convenience showed the 25% for Europe and Eurasia as the first sector after the 12 oclock line, then failed
to reverse the order of shading from the key, gaining only partial credit.
In part (ii), any value between 2.5% and 3.5% was accepted, since the question asked for approximate
percentage.
Within each of the factors for HEP production named in the table in (b) there was a wide choice of
acceptable answers. This meant that answers earning full credit were quite common. In answers gaining
partial credit, it tended to be geology which was known least well.
Candidates had many valid reasons to choose from, both human and environmental, for explaining why
people object to plans for new HEP schemes in their area in (c). Many looked for and stated three reasons
having taken into account that the question was worth three marks. A few candidates unnecessarily gave
many more than three.
Question 2
Candidates who studied the values to be plotted before deciding upon the size of their vertical scale in (a)(i)
most often gained full credit, by making one large square represent 30 million; this allowed all three bars to
stop on a bold line on the graph paper. Limited credit could be gained by candidates who either tried to draw
a graph with an uneven scale, or began their scale at a value close to 90 rather than from zero.
Full credit was gained in (a)(ii) by candidates giving any two from: quotas; fish that were too small; noncommercial species. Most candidates attempted to give two reasons although not all gained full credit.
In (b)(i), most candidates gained partial credit by linking subsistence fishing and fish as an important source
of food for the fisherman and his family. However, such candidates continued with irrelevant material such
as referring to selling surpluses and bringing in an income. More relevant answers which gained more credit
were either about high protein value of fish or areas where alternatives to fishing, such as farming, are
difficult.
Nearly all candidates could give a reason for why it is difficult to prevent illegal fishing in (b)(ii). Size of the
oceans and seas, high costs of enforcement and non-cooperation from fishermen were the answers seen
most often.
Problems caused by overfishing were well known and understood in (c) in which many candidates gained
credit. As in the final part of the previous question, a few candidates wasted time giving farm more detail
than the mark allocation demanded.
Question 3
In (a), most candidates gained credit.
Most candidates gained credit for day 13 or 14 for the cyclone in (b)(i).
Fewer candidates gained credit for recognising a period with high pressure in (b)(ii), largely by stating only
one day rather than a period of days. The period of high pressure extended from about day 21 to day 40;
any period of five or more days within this period gained credit.
To gain credit in part (c)(i) required understanding in order to give the correct sequence in the flow diagram.
Candidates with less complete understanding gained partial credit for getting some items correctly
sequenced.
In part (c)(ii) a wide range of different answers gained credit. Answers including identification of the
difficulties and their elaboration or exemplification gained most credit. Relevant difficulties included social,
economic and environmental. Many candidates concentrated on a limited range of difficulties, such as some

834

Cambridge General Certificate of Education Ordinary Level


5014 Environmental Management November 2012
Principal Examiner Report for Teachers
relating
BALOL to agriculture or to health, without looking more widely, gaining partial credit. There were fewer
examples of over-extended answers.
Question 4
In (a), many candidates gained credit, but other candidates incorrectly answered 1.8, the difference instead
of How many times.
In (b) most credit was gained by candidates who made direct links between the percentage of population
values for different age groups given in the table and the shape of the population pyramid for Honduras. For
example: 38% under 14 caused the wide base or only 4% over 64 led to the narrow top of the pyramid.
Candidates gaining full credit were most likely to note that the middle age group was the largest (58%), but
that it was spread over more years. Some candidates tried to base their answers upon other data in the
table, such as high growth rate and low income, earning little credit. Other candidates gave general answers
about population pyramids, with little or no reference to the population data for Honduras, again earning little
credit.
Most candidates realised that the data for Sweden suggested an ageing population, even if this was not
always explicitly stated in part (c). Candidates who began by stating this were the ones most likely to give
sufficient supporting explanation to gain full credit. In contrast, many other candidates concentrated their
answers around the 66% in the age group between 15 and 64 years, commenting on the plentiful number of
current workers suggested by this, gaining, at best, partial credit since little or nothing was said about the in
the future indicated by the question.
In answers gaining full credit in part (d), ways were separately identified, such as from various types of aid,
more fair trade or new sources of income such as tourism. Without the structure to answering given by doing
this, answers tended to be made up of individual ways to help, which were typically worth only partial credit.
Section B
Question 5
Many candidates correctly gave the total world energy demand in (a)(i). Other candidates careless reading
of the question led to some candidates incorrectly giving the total value for oil in 2010 for whicgh they
received no credit.
In (a)(ii) Many candidates gave the correct answer with very few incorrect answers.
There were some really good answers to part (a)(iii) from candidates who focused both on describing from
the graph, and on the question theme of the importance of oil. Not only did such candidates realise that oil
was the most used individual energy source at all three dates, but they also recognised that its relative
importance within the total was declining as use of other sources increased. Sometimes such candidates
used ratios or percentages in support as a way of relating amount of oil used to total world energy demand.
Less credit was given to other candidates who concentrated on the increased use of oil and used values
from the graph in support. Most candidates gave the bare minimum graph description and relied too heavily
on vague statements unsupported by graph evidence, gaining only partial credit.. A few candidates referred
to total energy demand rather than oil, which usually resulted in no credit being awarded.
Full credit was given in (b)(i) for some candidates who referred to petrol and diesel as the basis for most
modern forms of land and sea transport, followed by one of its many advantages for transport over other
energy sources, such as its cheapness, ease of use as a liquid, or the fact that modern forms of transport
have been developed to run on oil based fuels. Many candidates concentrated instead on the great increase
in transport worldwide and did not cover the main material of the question gaining, at best, partial credit.
Most candidates gained credit in (b)(ii) by establishing that more alternatives were available that were
suitable for generating electricity. Some of these answers were continued by giving advantages of using
alternatives such as lower levels of air pollution gaining full credit. The importance of coal in generating
electricity in many countries was widely known by candidates.
Many candidates gained full credit in (c)(i) (c)(iii). Some candidates incorrectly labelled hole in the ozone
layer instead of global warming in part (i), gaining no credit.

835

Cambridge General Certificate of Education Ordinary Level


5014 Environmental Management November 2012
Principal Examiner Report for Teachers
In
(c)(iv), most candidates understood what the question wanted and gained most or all of the credit by
BALOL
explaining in the same order as in the question, namely local, then international, and finally global effects.
Other candidates took a less organized approach and gained less credit.
In (c)(v) virtually all candidates, in their own words, emphasised how greatly traffic emissions can be reduced
by using buses instead of cars and recognised that the government of Dubai wanted people to give up their
cars and travel by public transport.
Another way, other than public transport, was needed in part (c)(vi). The most popular choice was catalytic
converters on vehicle exhausts; there was a good level of knowledge about how these worked to reduce
dangerous emission giving many candidates full credit. Use of cleaner fuels such as compressed natural
gas (CNG) was another popular choice through which some candidates gained full credit. Some other
candidates referred to traffic management schemes, such as restrictions on city car entry or car use.
A good choice of city or country in (d)(i), allowed specific and detailed answers to (d)(ii). Many such
candidates were able to gain full credit by giving precise reasons, including both physical and human, for the
notoriously high levels of air pollution in the selected city. The choice of Los Angeles, for example, led many
candidates for gain all or most of the credit as candidates referred to high levels of car ownership, dry climate
dominated by high pressure and regular temperature inversions, and lowland relief surrounded by
mountains.
The choice of China, or a major city within the candidates own country permitted other
candidates to gain most of the available credit. In such answers, the details given clearly matched the choice
of city, but a common omission was lack of a reference to a physical factor favouring air pollution. Answers
with limited coverage, or with a mis-match between reasons given and the choice of city or country were
worth less credit.
In (e)(i) many candidates made it clear, either from a statement or from values quoted from the graph, that
they knew that oil, coal and natural gas were the three fossil fuels. For such candidates, all relevant
statements could then be credited. Many of these candidates did in fact make productive use of the graph of
world energy demand, for example, giving percentage contributions to the total from fossil fuels which
showed a progressive decline from 1990 to 2030. Some candidates were unable to identify the fossil fuels
and gained no credit. Some answers were about all the energy sources in the table, others were about oil
only. A few included references to only two of the three fossil fuels and could gain only partial credit.
Iin (e)(ii) candidates gained credit for drawing a line at 75 from top to bottom in the graph. Shorter vertical
lines at 75 were also given credit. Some candidates gained no credit either by leaving the graph blank or by
giving incorrect or ambiguous marks on the graph.
Question (e)(iii) proved, as anticipated, demanding for many candidates. Full credit was gained by
candidates who understood the idea, referring to factors such as location, ease of access, costs of labour
and materials, and technology and its availability. Some of the answers about wind power were, incorrectly,
more about day to day variations in wind availability rather than variable long-term suitability of locations for
high and regular wind speeds. Many answers based on nuclear power were about local raw material
supplies, which only form a very small proportion of the total costs of nuclear power generation.
Some of the answers to (e)(iv) were given without any references to the graph during the discussion of
possibilities for more wind and nuclear use. These answers were given partial credit because the question
asked for use of knowledge as well as the graph. Some candidates showed that they understood that from
the graph, only onshore wind could be competitive with oil at a world price of $75 per barrel. Such
candidates were in a strong position to answer the question, whatever their view about how likely it was that
new wind and nuclear power stations would be built and such answers gained most or all of the available
credit. A view expressed as unlikely led to answers dominated by economic reasoning. A view expressed
as likely tended to be based on other factors, such as less polluting and longer lasting.
Most candidates gained credit in (f)(i).
In (f)(ii) candidates who stated first the assessments which meant that it could not be wind (such as being
weather dependent) and oil (such as emissions of carbon dioxide) gained credit. The candidates who then
added to the information already given to explain why it was nuclear power (such as not fully sustainable
because uranium is needed or not safe because of radiation) gained more credit up to the maximum
available. Candidates, who re-stated without explanation or elaboration, the terms given in the Energy
assessment table, gained little or no credit.

10

836

Cambridge General Certificate of Education Ordinary Level


5014 Environmental Management November 2012
Principal Examiner Report for Teachers
Question
BALOL 6
Most candidates gained credit in (a)(i) with an answer of 1990-2010. Those who did not gain credit typically
stated only one of the dates (1990 or 2010) rather than the 20 year period asked for.
There were many clear, precise answers to (a)(ii) supported by the use of key percentages gaining much
credit. Some candidates made it clear that the higher percentage of urban population that had existed in
developed countries in 1950 had been replaced by a much higher percentage concentration in developing
countries in 2010, gaining full credit. Other candidates lost credit by never actually mentioning the term
urban when referring to the population. Reasons for the changes gained no credit as they were irrelevant
because the question asks for a description rather than an explanation.
Plotting the world population totals correctly for the three dates in part (iii) earned credit for almost all
candidates. Most candidates gained credit for plotting the urban populations although some made errors
and so did not gain full credit.
Partial credit for answers urban growth were more common than full credit answers increased percentage
of the population living in urban areas in part (a)(iv), even though candidates had just plotted these
percentages on the graph. Once again there were quite a number of answers in which urban was never
mentioned, reducing the potential credit.
Candidates had plenty of choice of content for answering part (b). Those candidates who included natural
increase and migration, developing and developed in their answers had little trouble gaining full credit. Many
candidates were familiar with reasons for high birth rates and rural to urban migration in developing
countries. Some candidates made no reference to one of either natural increase or to rural to urban
migration which limited credit. A few candidates misread the question and referred to push and pull factors
for migration from developing to developed countries which limited credit. Others who correctly addressed
both question themes referred only to what was happening in developing countries, ignoring differences with
developed countries again limiting credit available.
For the mark in (c)(i) the letter S needed to be placed in one of the obviously empty spaces, on either side of
the river, or on the slopes beyond the factories.
For a full explanation in (c)(ii) candidates needed to mention at least one advantage and one disadvantage.
Some candidates referred to one without the other, limiting credit. Most of credit-worthy answers were
explanations of the advantages and disadvantages of living near the river or near the factories. Stating near
the factories as an advantage needed to be qualified by its significance, for example in terms of nearness for
finding a job, or low transport costs to reach work in order to gain credit.
In, (d)(i) and (d)(ii), candidates were expected to select appropriate answers from the newspaper report,
adding some additional comment towards the description of stable low-income neighbourhood in (ii). Most
candidates gained all or most of the credit.
In (d)(iii) many candidates had few problems identifying and isolating two reasons for the big change, such
as the Workers Party, supportive NGOs and local businesses, and police clearing out drug dealers. Some
candidates gained little credit for continuing with the same answers as part (ii).
In part (e)(i), answers based on deforestation and high levels of pollution were most common. Differences in
credit mainly reflected amount of elaboration, and degree of precision about the types of pollution and their
effects. Many candidates concentrated on identifying three different types of environmental damage, with
nothing more than a basic statement about each one, gaining only partial credit. Some candidates gave
more context to their examples as well as content, gained more credit. References to pollution were only
effective when the type of pollution was specified. Some candidates narrowly named only air, water and land
pollution, mostly then giving answers that were short on real detail, thus limiting the amount of credit
available.
Pleasingly, many candidates gained full credit in part (e)(ii) for expressing the idea that when a population
increases, its carbon dioxide emissions can be expected to increase as well, the opposite of what has
happened in Freiburg however that understanding is expressed. Some candidates did no more than state
what the graph showed, for both population and carbon dioxide and these were given partial credit.
There were plentiful choices on the map for candidates to describe in (f)(i). When describing such material,
candidates need to go beyond stating the terms seen on the map and in the question, such as eco-friendly,

11

837

Cambridge General Certificate of Education Ordinary Level


5014 Environmental Management November 2012
Principal Examiner Report for Teachers
renewable
BALOL and sustainable, in order to gain credit. Most candidates had sufficient understanding to gain at
least partial credit for each of their chosen characteristics. Full credit was given to candidates who described
more fully, as demanded in the question.
A wide range of options for dealing with urban wastes were named and explained in (f)(ii). Some candidates
worked from the trigger on the sketch and referred to waste water treatment first. Others began with
recycling. Many of those who referred to landfill went out of their way to describe how the operations could
be made more sanitary. The range and amount of content were the main determinants for the amount of
credit given. Some answers stopped short of what was needed for full credit. A few candidates referred to
methods of waste disposal forgetting that they were supposed to reduce environmental impacts, limiting the
credit available. Answers from such candidates were typically landfill based. Taking the rubbish well away
from the eco-city to be dumped is not an environmentally friendly solution and could earn no credit.

12

838

BALOL
UNIVERSITY OF CAMBRIDGE INTERNATIONAL EXAMINATIONS
General Certificate of Education Ordinary Level

* 2 2 6 1 3 2 4 9 4 2 *

5014/11

ENVIRONMENTAL MANAGEMENT
Paper 1

May/June 2013
2 hours 15 minutes

Candidates answer on the Question Paper.


Additional Materials:

Ruler

READ THESE INSTRUCTIONS FIRST


Write your Centre number, candidate number and name on all the work you hand in.
Write in dark blue or black pen.
You may use a soft pencil for any diagrams, graphs or rough working.
Do not use staples, paper clips, highlighters, glue or correction fluid.
DO NOT WRITE IN ANY BARCODES.
Electronic calculators may be used.
Answer all questions.
All questions in Section A carry 10 marks.
Both questions in Section B carry 40 marks.
At the end of the examination, fasten all your work securely together.
The number of marks is given in brackets [ ] at the end of each question or
part question.

For Examiners Use


1
2
3
4
5
6
Total

839

BALOL

Section A

For
Examiners
Use

Answer all the questions.

Look at the diagram below. M, N, O and P show one method of storing nuclear waste.
One method of storing nuclear waste
solid waste
sealed in drum

rock
layers

N
drums in
carrying
crate

secure
leak-free
container
carrying
crates
(a) (i)

put in
vault
(a space
in the rock)

detail of leak-free
container in vault

How many coverings are put over the nuclear waste before it is put in the vault?
.............................................................................................................................. [1]

(ii)

From the diagram, describe the location of the vault where the waste will be finally
stored.
..................................................................................................................................
..................................................................................................................................
..................................................................................................................................
.............................................................................................................................. [2]

(iii)

Explain why it is necessary to store nuclear waste in this way.


..................................................................................................................................
..................................................................................................................................
..................................................................................................................................
..................................................................................................................................
..................................................................................................................................
.............................................................................................................................. [3]

UCLES 2013

5014/11/M/J/13

840

BALOL
(b) State one reason why some people think that nuclear power stations should not be built
near plate boundaries.

For
Examiners
Use

..........................................................................................................................................
...................................................................................................................................... [1]
(c) How can leaks from nuclear power stations in one country cause problems in another
country?
..........................................................................................................................................
..........................................................................................................................................
..........................................................................................................................................
..........................................................................................................................................
..........................................................................................................................................
...................................................................................................................................... [3]
[Total: 10]

UCLES 2013

5014/11/M/J/13

[Turn over

841

4
2 BALOL
Look at the map which shows currents in the Atlantic Ocean. Use the map to answer the
following questions.

For
Examiners
Use

North Atlantic Ocean Currents


QG

(
DVW
&X *UHH
UUH QO
QW D

Greenland
Nanortalik
60 N

rift

lan







t
hA


DP
H
WU

D
tic

R
S

.H\

ort

51RUZD\
S6KHWODQG
,VODQG

Canaries
Current

*XOI
6

Canada

Iceland

1ERXQGDU\RI
1RUZD\
Canary Islands
Africa



(a) (i)

(TXDWRU

South
America

How does the map indicate that the Gulf Stream is a warm ocean current?
..................................................................................................................................
.............................................................................................................................. [1]

(ii)

The Canary Current and East Greenland Current are both cold currents. The East
Greenland Current is colder. Explain why.
..................................................................................................................................
.............................................................................................................................. [1]

(iii)

Circle the letter T, U, or V to show which one of the following statements is most
likely to be true.
T

Nanortalik will be colder than the Shetland Islands in winter.

The Canary Islands will be colder than Nanortalik in winter.

The Shetland Islands will be colder than Iceland in winter.


[1]

UCLES 2013

5014/11/M/J/13

842

BALOL
(b) Insert the correct words to explain how and why the Canary Current affects rainfall on
the nearby west coast of Africa. Choose one word from each of the following pairs:
cooled
warmed

condensation
evaporation

For
Examiners
Use

decreases
increases

The moist wind from the sea is ................................................. as it crosses the Canary
Current.
This causes ................................................. over the current.
In this way the Canary Current ................................................. rainfall on the coast of
Africa.
[3]
(c) Refer to the map of ocean currents on page 4.
(i)

Explain why sea fishing takes place all year round off the coast of Norway but only
in summer off Greenland.
..................................................................................................................................
..................................................................................................................................
..................................................................................................................................
.............................................................................................................................. [2]

(ii)

Suggest why people on the beaches of the Canary Islands sometimes pick up
plastic waste from South America.
..................................................................................................................................
..................................................................................................................................
..................................................................................................................................
.............................................................................................................................. [2]
[Total: 10]

UCLES 2013

5014/11/M/J/13

[Turn over

843

6
3 BALOL
(a) Look at the graph of variations in wheat production and export in Russia from 2000 to
2010.

For
Examiners
Use

70

70

60

60

50

50

40

40

30

30

20

20

10

10

2000 2001 2002 2003 2004 2005 2006 2007 2008 2009 2010
(i)

million tonnes of wheat

million tonnes of wheat

Russia: wheat production and export 2000 2010

Key
total wheat
production
wheat used
in the country
the amount of
wheat exported

The table gives information for 2005. Complete the graph.


wheat production (million tonnes)

48

exports (million tonnes)

10
[1]

(ii)

How does the graph show that wheat production in Russia has, in general,
increased over the period shown? Use values to support your answer.
..................................................................................................................................
..................................................................................................................................
..................................................................................................................................
.............................................................................................................................. [2]

UCLES 2013

5014/11/M/J/13

844

BALOL(iii) Agricultural production has increased like this in many countries. Explain why.
..................................................................................................................................

For
Examiners
Use

..................................................................................................................................
..................................................................................................................................
..................................................................................................................................
..................................................................................................................................
..................................................................................................................................
..................................................................................................................................
.............................................................................................................................. [4]
(iv)

In 2010 drought decreased the total wheat crop by more than 20%. Use the graph
to suggest one other year between 2001 and 2009 in which drought had an effect
on wheat production.
.............................................................................................................................. [1]

(b) The graph shows that wheat exports vary over the period shown. Suggest economic
reasons to explain this.
..........................................................................................................................................
..........................................................................................................................................
..........................................................................................................................................
...................................................................................................................................... [2]
[Total: 10]

UCLES 2013

5014/11/M/J/13

[Turn over

845

8
4 BALOL
Look at the photograph taken in the Rocky Mountains of Canada.

For
Examiners
Use

(a) Describe the vegetation shown on the photograph.


..........................................................................................................................................
..........................................................................................................................................
..........................................................................................................................................
..........................................................................................................................................
..........................................................................................................................................
...................................................................................................................................... [3]

UCLES 2013

5014/11/M/J/13

846

BALOL
(b) Explain how the types of trees seen on the photograph are adapted to the climate.
..........................................................................................................................................

For
Examiners
Use

..........................................................................................................................................
..........................................................................................................................................
..........................................................................................................................................
..........................................................................................................................................
...................................................................................................................................... [3]
(c) (i)

Describe the distribution of the forest on the photograph.


..................................................................................................................................
.............................................................................................................................. [1]

(ii)

Suggest why there is no vegetation in some parts of the land area shown on the
photograph.
..................................................................................................................................
..................................................................................................................................
..................................................................................................................................
..................................................................................................................................
..................................................................................................................................
.............................................................................................................................. [3]
[Total: 10]

UCLES 2013

5014/11/M/J/13

[Turn over

847

10

BALOL

Section B
Answer both questions.

For
Examiners
Use

(a) Look at the map showing the world distribution of savanna vegetation.

Cancer

A
C

Equator

Capricorn

Key:
Savanna
(i)

Name the continent with the largest area of savanna vegetation.


.............................................................................................................................. [1]

(ii)

Describe the other main features of the distribution of savanna vegetation.


..................................................................................................................................
..................................................................................................................................
..................................................................................................................................
.............................................................................................................................. [2]

(iii)

Name the type of natural vegetation found in the areas marked A and B on the
map.
.............................................................................................................................. [1]

UCLES 2013

5014/11/M/J/13

848

11

BALOL
(b) The savanna climate is tropical with a wet and dry season.

For
Examiners
Use

Summary of climate in northern Nigeria


(area C on the map of savanna vegetation)

Temperature mean monthly temperatures / C


J

22

24

28

31

30

28

26

25

26

27

25

22

Rainfall averages
Wet season (May to September)
Dry season (October to April)
Total annual rainfall

844 mm
26 mm
870 mm

(i)

Plot the mean monthly temperatures on the graph paper using a line graph.

(ii)

What percentage of total annual rainfall falls in the wet season? Circle one answer.

26

(iii)

54

84

[3]

97

[1]

Describe how the data shows that this area of savanna has a tropical climate.
..................................................................................................................................
.............................................................................................................................. [1]

UCLES 2013

5014/11/M/J/13

[Turn over

849

12

BALOL(iv) Describe how the appearance of the natural vegetation changes between wet and
dry seasons in savanna lands.

For
Examiners
Use

..................................................................................................................................
..................................................................................................................................
..................................................................................................................................
.............................................................................................................................. [2]
(v)

Using both the temperature and rainfall data given, describe the advantages and
disadvantages of a savanna climate for farmers growing crops.
..................................................................................................................................
..................................................................................................................................
..................................................................................................................................
..................................................................................................................................
..................................................................................................................................
.............................................................................................................................. [3]

(c) Look at the sketch which shows how tribal groups use traditional subsistence farming in
the savanna lands of West Africa.

(i)

State the two ways in which farmers are making their living here.
1 ........................................................... 2 ........................................................... [1]

UCLES 2013

5014/11/M/J/13

850

13

BALOL(ii) Looking at the sketch, describe how it shows that this is an area of traditional
subsistence farming.

For
Examiners
Use

..................................................................................................................................
..................................................................................................................................
..................................................................................................................................
..................................................................................................................................
.............................................................................................................................. [3]
(iii)

How different would the sketch look if this was an area of modern commercial
farming instead of traditional subsistence farming? Suggest two ways.
..................................................................................................................................
..................................................................................................................................
..................................................................................................................................
.............................................................................................................................. [2]

(d) Population growth is increasing pressure on the land and the risk of soil erosion in many
countries in the savanna lands of West Africa.
(i)

State the evidence from the sketch which shows that this area is at high risk of soil
erosion.
..................................................................................................................................
..................................................................................................................................
..................................................................................................................................
.............................................................................................................................. [2]

UCLES 2013

5014/11/M/J/13

[Turn over

851

14

BALOL(ii) Four strategies of soil conservation are


D tree planting

E dry land farming

F rural development programmes

G community participation

For
Examiners
Use

Choose two of these strategies. For each one, describe how it reduces the risk of
soil erosion and helps with soil conservation.
Letter .......... ...........................................................................................................
..................................................................................................................................
..................................................................................................................................
..................................................................................................................................
Letter .......... ............................................................................................................
..................................................................................................................................
..................................................................................................................................
.............................................................................................................................. [4]
(iii)

Introducing strategies of soil conservation is difficult in areas like the one shown in
the sketch on page 12. Suggest some of the difficulties.
..................................................................................................................................
..................................................................................................................................
..................................................................................................................................
..................................................................................................................................
..................................................................................................................................
.............................................................................................................................. [3]

UCLES 2013

5014/11/M/J/13

852

15

BALOL(iv) Give reasons why population growth continues to be high in developing countries
like those in the savanna land of West Africa.

For
Examiners
Use

..................................................................................................................................
..................................................................................................................................
..................................................................................................................................
..................................................................................................................................
..................................................................................................................................
..................................................................................................................................
..................................................................................................................................
..................................................................................................................................
..................................................................................................................................
.............................................................................................................................. [5]

UCLES 2013

5014/11/M/J/13

[Turn over

853

16

BALOL
(e) Look at the rainfall diagram. It shows rainfall totals for the month of April at Kisumu in
Kenya during a period of 30 years. Kisumu has a savanna climate and April is in the
middle of the wet season.

For
Examiners
Use

Each dot shows a rainfall total for April in one of the 30 years.
How August rainfall totals varied during a period of 30 years
Kisumu Kenya
400

350

300

250
rainfall
/ mm

200

150

100

50

0
(i)

(ii)

Average (mean) monthly rainfall at Kisumu in April is 188 mm.


Show this average value on the diagram using a cross (X).

[1]

What is the size of the difference in the amount of rainfall (in mm) between the
wettest and driest months of April during this 30 year period?

...................................................................................................................... mm. [1]

UCLES 2013

5014/11/M/J/13

854

17

BALOL(iii) How and why do rainfall variations from year to year, like the ones shown in Kisumu,
cause problems for subsistence farmers living in savanna lands?

For
Examiners
Use

Refer to years with both higher and lower than average values.
..................................................................................................................................
..................................................................................................................................
..................................................................................................................................
..................................................................................................................................
..................................................................................................................................
..................................................................................................................................
..................................................................................................................................
.............................................................................................................................. [4]
[Total: 40]

UCLES 2013

5014/11/M/J/13

[Turn over

855

18
6 BALOL
(a) Look at the map of the Pacific Ocean showing the location of destructive plate
boundaries.

For
Examiners
Use

Pacific Ocean

Key:
destructive tectonic
plate boundaries
(i)

Describe the distribution of destructive plate boundaries in the Pacific Ocean.


..................................................................................................................................
..................................................................................................................................
..................................................................................................................................
.............................................................................................................................. [2]

(ii)

State what is happening to the plates at destructive plate boundaries.


..................................................................................................................................
..................................................................................................................................
..................................................................................................................................
..................................................................................................................................
..................................................................................................................................
.............................................................................................................................. [3]

UCLES 2013

5014/11/M/J/13

856

19

BALOL
(b) Look at the map of the Pacific Ocean showing the location of active volcanoes.

For
Examiners
Use

Pacific Ocean
H
Key:
active volcanoes
(i)

Suggest why the distribution of active volcanoes in the Pacific Ocean is known as
The Pacific Ring of Fire.
..................................................................................................................................
..................................................................................................................................
..................................................................................................................................
.............................................................................................................................. [2]

(ii)

Explain how volcanoes are formed along destructive plate boundaries.


..................................................................................................................................
..................................................................................................................................
..................................................................................................................................
..................................................................................................................................
..................................................................................................................................
.............................................................................................................................. [3]

UCLES 2013

5014/11/M/J/13

[Turn over

857

20

BALOL
(c) Read the information about a volcanic eruption in Chile in June 2011 (marked H on the
map of active volcanoes).

For
Examiners
Use

Volcano Puyehue erupts in Chile


Chile is the most volcanic country on Earth with
over 3000 volcanoes, up to 80 of them active.

sore eyes and skin, and breathing problems. They


were advised to stay indoors.

The eruption of the Puyehue volcano in central


Chile on 4th June led to the Authorities evacuating
3,500 people to temporary shelters in safe areas.
Large columns of smoke and ash, mixed with
rocks, were thrown out 10 km into the air. Among
the gases released were high levels of SO2. There
were no reports of any injuries. The previous
eruptions from this volcano were in 1960 and
1921; these eruptions lasted for about two months.

Towns in central Chile, used to making money


from tourists coming to see the stunning volcanic
scenery, were mostly empty. The airport in
Bariloche in Argentina was closed for days. This
stopped high spending Brazilians from coming, at
the worst time, because it was right in the middle
of the main ski season.

The prevailing westerly winds took the ash cloud


east over the Andes mountains into Argentina. One
resident of the ski resort of Bariloche in Argentina
said that Ash was falling like snow. Levels of
air pollution were high and people complained of

(i)

(ii)

By 11th June the ash cloud from Puyehue had


reached Australia and New Zealand, over 6,000 km
away. The cloud hovered in the air between 4,000
and 7000 metres closing Australias two biggest
international airports in Sydney and Melbourne.
As late as 21st June, the ash cloud was still
hanging around.

Name all four emissions from the Puyehue volcano during its June 2011 eruption.
1 ............................................................

2 ...........................................................

3 ............................................................

4 ...........................................................
[1]

Despite this being a major volcanic eruption, no one was killed or even injured.
Suggest three reasons for this.
..................................................................................................................................
..................................................................................................................................
..................................................................................................................................
..................................................................................................................................
..................................................................................................................................
.............................................................................................................................. [3]

UCLES 2013

5014/11/M/J/13

858

21

BALOL(iii) The economic effects of Puyehues eruption were felt not only in Chile and
neighbouring Argentina, but also thousands of kilometres away in Australia.

For
Examiners
Use

Why were the effects from this volcanic eruption international as well as national?
..................................................................................................................................
..................................................................................................................................
..................................................................................................................................
.............................................................................................................................. [2]
(iv)

Were these economic effects greater for the other countries than for Chile? Explain
your views on this.
..................................................................................................................................
..................................................................................................................................
..................................................................................................................................
..................................................................................................................................
..................................................................................................................................
.............................................................................................................................. [3]

(v)

People living in central Chile and Bariloche complained of health problems caused
by the volcano. Explain how these may have been caused.
..................................................................................................................................
..................................................................................................................................
..................................................................................................................................
.............................................................................................................................. [2]

(d) Land close to and around the craters of active volcanoes is often barren wasteland,
places where nothing will grow. One reason is the very acid ground due to frequent
releases of toxic volcanic gases and liquids.
(i)

The pH scale is shown below. Put a tick () in one of the boxes to suggest the pH
of a soil found in areas next to volcanic craters.

pH

10

11

12

13

14

tick one box


[1]

UCLES 2013

5014/11/M/J/13

[Turn over

859

22

BALOL(ii) State and explain another reason why areas on the higher slopes of active
volcanoes cannot usually be used for farming.

For
Examiners
Use

..................................................................................................................................
..................................................................................................................................
..................................................................................................................................
.............................................................................................................................. [2]
(iii)

In areas further away from the crater, volcanic soils are some of the worlds best
soils for growing crops. Many of them have a loam soil texture;
33% sand: 33% clay: 34% silt
Complete the pie graph and key to show these characteristics of a loam soil texture.
Texture of a loam volcanic soil

Key:

Put your answers on the pie chart [3]


(iv)

The pH scale is shown below. Put a tick () in one of the boxes to suggest the pH
of a volcanic soil with a loam texture.

pH

10

11

12

13

14

tick one box


[1]
(v)

Explain why a loam soil texture is good for crop growing.


..................................................................................................................................
..................................................................................................................................
..................................................................................................................................
..................................................................................................................................
..................................................................................................................................
.............................................................................................................................. [3]

UCLES 2013

5014/11/M/J/13

860

23

BALOL
(e) The island of Java in Indonesia was made by volcanoes. Today it still has 45 active
volcanoes. Its rich volcanic soils are some of the worlds best for farming.
It has been suggested that even a stick planted in Javas volcanic soils will grow!
Deposits from erosion on the mountains and from new eruptions are carried to Javas
lowlands, forming thick layers of fertile sediment on the islands plains.
(i)

For
Examiners
Use

Explain why fertile volcanic soils are present almost everywhere in Java.
..................................................................................................................................
..................................................................................................................................
..................................................................................................................................
.............................................................................................................................. [2]

(ii)

Look at the information about the island of Java.


Java in Indonesia

Area

Population

Population density

Birth rate

Death rate

7% of Indonesia

140 million

1025 per km

18 per 1000

7 per 1000

The total population of Indonesia is 235 million. Approximately what percentage of


these live on the island of Java? Circle one answer.

40%

(iii)

50%

60%

70%

[1]

What information shows that Java is a very overcrowded island compared with the
rest of Indonesia?
..................................................................................................................................
.............................................................................................................................. [1]

(iv)

What is the rate of natural increase of population in Java?

.............................................................................................................................. [1]

UCLES 2013

5014/11/M/J/13

[Turn over

861

24

BALOL(v) The government of Indonesia is increasingly worried about environmental damage


in Java as overcrowding forces subsistence farmers to extend cultivation to steep
mountain slopes and forest reserves.

For
Examiners
Use

J, K and L are three ways that have been suggested to tackle the problem.
J
Manage the land

K
Manage population increase

Educate farmers
about strategies of soil
conservation

Put more money into family


planning programmes

L
Promote economic
development
Support growth of industries
in the large cities

In your view, which one of these ways would be the best for solving the problem of
environmental damage in Java? Explain your view.
..................................................................................................................................
..................................................................................................................................
..................................................................................................................................
..................................................................................................................................
..................................................................................................................................
..................................................................................................................................
..................................................................................................................................
.............................................................................................................................. [4]
[Total: 40]

862

BALOL

CAMBRIDGE INTERNATIONAL EXAMINATIONS


GCE Ordinary Level

MARK SCHEME for the May/June 2013 series

5014 ENVIRONMENTAL MANAGEMENT


5014/11

Paper 1, maximum raw mark 120

This mark scheme is published as an aid to teachers and candidates, to indicate the requirements of
the examination. It shows the basis on which Examiners were instructed to award marks. It does not
indicate the details of the discussions that took place at an Examiners meeting before marking began,
which would have considered the acceptability of alternative answers.
Mark schemes should be read in conjunction with the question paper and the Principal Examiner
Report for Teachers.

Cambridge will not enter into discussions about these mark schemes.

Cambridge is publishing the mark schemes for the May/June 2013 series for most IGCSE, GCE
Advanced Level and Advanced Subsidiary Level components and some Ordinary Level components.

863

Page 2

Mark Scheme
GCE O LEVEL May/June 2013

Syllabus
5014

Paper
11

BALOL
General notes
Symbols used in Environmental Management mark schemes.
/

separates alternatives for a marking point other valid ways of expressing the same
idea are also credited

separates points for the award of a mark

[3]

indicates the number of marks available

italic

indicates that this is information about the marking points and is not required to gain
credit
italic text is also used for comments about alternatives that should be accepted, ignored
or rejected

ora

or reverse argument shows that an argument from an alternative viewpoint will be


credited

AW

alternative wording, sometimes called or words to that effect


AW is used when there are many different ways of expressing the same idea

( )

the word / phrase in brackets is not required to gain marks but sets the context of the
response for credit
e.g. (nuclear) waste nuclear is not needed but if it was described as a domestic waste
then no mark is awarded

volcanic

underlined words the answer must contain exactly this word

ecf

error carried forward if an incorrect answer is given to part of a question, and this
answer is subsequently used by a candidate in later parts of the question, this indicates
that the candidates incorrect answer will be used as a starting point for marking the later
parts of the question

864

Page 3

Mark Scheme
GCE O LEVEL May/June 2013

Syllabus
5014

Paper
11

BALOL
Section A
1

(a) (i) three;

[1]

(ii) in rock;
deep;
in a single rock layer;
[2]

(the last statement would score two marks, as it incorporates the first point)
(iii) high level waste;
dangerous;
radioactive;
remains dangerous/radioactive for a very long period of time;
needs to be where it cannot affect people/the environment;
needs to be where it cannot contaminate water sources/rivers etc.;

[3]

(b) earthquake damage/plate movement could cause leaks of radioactive material/damage


power station;
[1]

accept tsunami damage if given


(c) leaks into ocean and moved by ocean currents;
leaks into rivers which pass into another country;
leaks into the atmosphere and wind carries it;
can fall in rain in another country;
therefore soil can become contaminated when rain contains radioactive material;
therefore radioactivity can enter the food chain/contaminate vegetation eaten by
animals;

[3]

[Total:10]

865

Page 4

Mark Scheme
GCE O LEVEL May/June 2013

Syllabus
5014

Paper
11

BALOL
2

(a) (i) it originates in low latitudes/near the Equator;

[1]

(ii) it originats from polar regions/the Arctic/more northerly lattitudes;

[1]

(iii) T, Nanortalik will be colder than the Shetland Islands in winter;

[1]

(b) cooled;
condensation;
decreases;

[3]

(c) (i) the North Atlantic Drift/warm current will keep the sea ice free off Norway; sea ice
will form off Greenland in winter;

[2]

(ii) the Gulf Stream/North Atlantic Drift will carry the waste northwards; the Canary
Current will bring it to the Canary Islands beaches;
if both currents are not mentioned allow the idea that the waste will be moved by
ocean currents for 1 mark.

[2]

[Total:10]
3

(a) (i) bar at 48 million tonnes and exports at 10 million tonnes;

[1]

(ii) the production in 2010 was higher than that in 2000;


it has increased by 1819 mt/from 34/35 to 53;
[2]

or same style of marking for year 20002008;


(iii) explanations of how new techniques increase yields, such as:
irrigation allows crop growth in dry seasons/droughts/where too dry;
fertilisers encourage healthy growth;
pesticides/insecticides kill pests which would harm the crop;
herbicides clear weeds which compete with the crops for nutrients and water;
mechanisation allows greater productivity;
use of new higher yielding varieties/GM seeds;
subsidies/more capital input allows the above to be used;
there is better/more widespread knowledge of good farming techniques;
biological controls reduce pests; (4 1 mark)

[4]

(iv) 2003/2006;

[1]

(b) reward any credible ideas, such as:


yield provides more/less than is needed for the domestic market;
price changes on the world market;
producers can get more/less for it on the world market than if they sold it in the country;
variations in demand because of competition;

[2]

[Total: 10]

866

Page 5

Mark Scheme
GCE O LEVEL May/June 2013

Syllabus
5014

Paper
11

BALOL
4

(a) taiga/coniferous forest;


conical/tall and thin/wedge shaped trees;
dense;
evergreen/uniform;
grass;
some colonising the lake edge;

[3]

(b) allow feature linked to climate, such as;


conical shape allows snow to slide off;
flexible trunk allows sway in strong winds;
shallow roots because only a shallow layer thaws in summer;
thick bark protects from cold;
needle leaves to reduce transpiration;
evergreen because short growing season; (would not allow time for new leaf growth)

[3]

(c) (i) by a lake/on lower slopes (of mountains)/in valley;

[1]

(ii) too cold for growth;


too steep for growth/soil (to form);
bare rock/no soil;
(some parts) snow/ice-covered all year;
Max. 2 for answers without locations on photo

[3]
[Total: 10]

867

BALOL
UNIVERSITY OF CAMBRIDGE INTERNATIONAL EXAMINATIONS
General Certificate of Education Ordinary Level

* 2 6 8 6 7 8 6 0 2 2 *

5014/12

ENVIRONMENTAL MANAGEMENT
Paper 1

May/June 2013
2 hours 15 minutes

Candidates answer on the Question Paper.


Additional Materials:

Ruler

READ THESE INSTRUCTIONS FIRST


Write your Centre number, candidate number and name on all the work you hand in.
Write in dark blue or black pen.
You may use a soft pencil for any diagrams, graphs or rough working.
Do not use staples, paper clips, highlighters, glue or correction fluid.
DO NOT WRITE IN ANY BARCODES.
Electronic calculators may be used.
Answer all questions.
All questions in Section A carry 10 marks.
Both questions in Section B carry 40 marks.
At the end of the examination, fasten all your work securely together.
The number of marks is given in brackets [ ] at the end of each question or
part question.

For Examiners Use


1
2
3
4
5
6
Total

868

BALOL

Section A

For
Examiners
Use

(a) Look at the diagram which shows information about aluminium recycling.
scrap aluminium
(cans, pans etc.)

heated in furnace 1 using only


5% of the energy that is used to
refine aluminium ore

85% aluminium

used to
make cans,
pans

15% ash waste

re-heated in furnace 2
with other substances
producing a chemical reaction

aluminium

(i)

energy

waste used in
concrete and asphalt

State the inputs to the process of recycling aluminium.


..................................................................................................................................
..................................................................................................................................
..................................................................................................................................
.............................................................................................................................. [2]

(ii)

Use the diagram to explain why the production of aluminium by recycling is more
economic than refining aluminium ore (bauxite).
..................................................................................................................................
.............................................................................................................................. [1]

UCLES 2013

5014/12/M/J/13

869

BALOL
(b) Industries, such as aluminium refining which use large amounts of energy, can harm the
environment.
Describe different ways in which factories can harm the environment.

For
Examiners
Use

..........................................................................................................................................
..........................................................................................................................................
..........................................................................................................................................
..........................................................................................................................................
..........................................................................................................................................
..........................................................................................................................................
..........................................................................................................................................
...................................................................................................................................... [4]
(c) Describe ways in which governments can improve protection of the environment from
industrial pollution.
..........................................................................................................................................
..........................................................................................................................................
..........................................................................................................................................
..........................................................................................................................................
..........................................................................................................................................
...................................................................................................................................... [3]
[Total: 10 marks]

UCLES 2013

5014/12/M/J/13

[Turn over

870

4
2 BALOL
(a) Look at the diagram which shows the percentage of total rainfall that runs-off four
different types of land use.

For
Examiners
Use

3HUFHQWDJHRIWRWDOUDLQIDOOWKDWUXQVRIIDUHDV
ZLWKIRXUGLIIHUHQWW\SHVRIODQGXVH
.H\


WUHHVDQGEXVKHV



JUDVV

(i)

SHUFHQWDJHRIUDLQIDOO
WKDWUXQVRII

ZKHDWFURS



EDUHIDOORZ



Complete the bar graph to show the percentage of rainfall that runs-off on land
uses 3. wheat crop and 4. bare fallow.

percentage of rainfall that runs-off

Run-off from four different land uses


60
50
40
30
20
10
0

1
trees
and
bushes

2
grass

3
wheat
crop

4
bare
fallow
[1]

UCLES 2013

5014/12/M/J/13

871

BALOL The diagram shows some processes in the water cycle.


moisture transpires
to the atmosphere

For
Examiners
Use

raindrops
interception

leaf

off

run-

infiltration

plants take up
water through
their roots

Use the information in the diagrams to:


(ii)

explain why more than half the rain that falls on land use 4 runs off.
..................................................................................................................................
..................................................................................................................................
..................................................................................................................................
.............................................................................................................................. [2]

(iii)

explain why land use 2 loses more water in run-off than land use 1.
..................................................................................................................................
..................................................................................................................................
..................................................................................................................................
.............................................................................................................................. [2]

(iv)

suggest why run-off amounts from land use 3 may vary at different times of the year.
..................................................................................................................................
.............................................................................................................................. [1]

(b) Choose the land use where rates of soil erosion are likely to be highest. What can the
farmer do to reduce this risk?
..........................................................................................................................................
..........................................................................................................................................
..........................................................................................................................................
..........................................................................................................................................
..........................................................................................................................................
..........................................................................................................................................
...................................................................................................................................... [4]

UCLES 2013

5014/12/M/J/13

[Total: 10 marks]
[Turn over

872

6
3 BALOL
(a) Look at the map of the distribution of the Equatorial climate.

For
Examiners
Use

Areas with an Equatorial climate

Europe
June 21st
sun overhead

23N
Central America

December 22nd
sun overhead

Africa

South East
Asia
Z

South
America

23S

0
(i)

km 4000

Describe the location of the Equatorial climate shown on the map.


..................................................................................................................................
..................................................................................................................................
..................................................................................................................................
.............................................................................................................................. [2]

(ii)

Describe the features of the Equatorial climate which explain why the forest growth
is dense.
..................................................................................................................................
..................................................................................................................................
..................................................................................................................................
.............................................................................................................................. [2]

UCLES 2013

5014/12/M/J/13

873

BALOL
(b) (i) In area Z some tropical forest has been burnt and replaced with palm oil plantations.
Some people say this will increase global warming. What do you think? Give
reasons for your answer.

For
Examiners
Use

..................................................................................................................................
..................................................................................................................................
..................................................................................................................................
..................................................................................................................................
..................................................................................................................................
.............................................................................................................................. [3]
(ii)

The bar graph shows average amounts of carbon stored in Equatorial rain forest
and in oil palm plantations per hectare.
Carbon storage in
Equatorial rain forest and oil palm plantations
400

300

average amount of
carbon above ground
/ tonnes per hectare

200

100

Equatorial
rain
forest

oil palm
plantation

Compare the amounts of carbon stored above ground in Equatorial rain forest and
oil palm plantations.
..................................................................................................................................
.............................................................................................................................. [1]

UCLES 2013

5014/12/M/J/13

[Turn over

874

BALOL(iii) Some countries in The European Union import palm oil for biogas production in
order to meet their renewable energy targets. Suggest why the following people
might disagree with the removal of areas of equatorial rainforest to grow oil palms.

For
Examiners
Use

a member of the WWF (World Wildlife Fund for Nature);


..................................................................................................................................
..................................................................................................................................
people living in a small settlement in the forest.
..................................................................................................................................
.............................................................................................................................. [2]
[Total: 10 marks]

Look at the photograph which shows animals grazing a savanna area in the dry season.

(a) Describe the features of savanna vegetation shown in the photograph.


..........................................................................................................................................
..........................................................................................................................................
..........................................................................................................................................
..........................................................................................................................................
..........................................................................................................................................
...................................................................................................................................... [3]
UCLES 2013

5014/12/M/J/13

875

BALOL
(b) Use the photograph to illustrate what is meant by the following terms used in studies of
ecosystems:

For
Examiners
Use

population ........................................................................................................................
..........................................................................................................................................
habitat ..............................................................................................................................
..........................................................................................................................................
consumer .........................................................................................................................
..........................................................................................................................................
ecosystem ........................................................................................................................
..........................................................................................................................................
...................................................................................................................................... [4]
(c) Suggest, with reasons, how the animals in the savanna could affect the ecosystem.
..........................................................................................................................................
..........................................................................................................................................
..........................................................................................................................................
..........................................................................................................................................
..........................................................................................................................................
...................................................................................................................................... [3]
[Total: 10 marks]

UCLES 2013

5014/12/M/J/13

[Turn over

876

10

BALOL

Section B

For
Examiners
Use

(a) The table gives average income per head for the worlds six inhabited continents.
average income per head by continent / $US
Africa

(i)

(ii)

690

Asia

2,200

Europe

12,100

Latin America

3,100

North America

26,900

Oceania

13,900

Rank the six continents by average income per head from highest (1) to lowest (6).
1 ........................................................

4 ........................................................

2 ........................................................

5 ........................................................

3 ........................................................

6 ........................................................

[1]

How big is the difference in average income per head (in $US) between the richest
and poorest continents?

.............................................................................................................................. [1]

UCLES 2013

5014/12/M/J/13

877

12

BALOL(iii) Look at the world map of continents. The dividing line between the developed North
and developing South is also shown.

NORTH
AMERICA

For
Examiners
Use

ASIA
EUROPE
ASIA

Equator
LATIN
AMERICA
OCEANIA

r th

South D
evide

AFRICA

No

Key
Average income per head $ US
0 9,999
10,000 19,999
20,000 29,999

On the world map, plot the values shown in the table for each continent using
denser shading or stronger colours for the high values. Complete the key to match
the shading or colours on the map.
[3]
(iv)

Look at the map. How well does the dividing line split the world between the
developed North and developing South?
Answer by
1 describing where the line seems to be a good fit,
..................................................................................................................................
..................................................................................................................................
2 describing where the line seems to be a less good fit,
..................................................................................................................................
..................................................................................................................................
3 writing about how good the fit seems to be overall. ...............................................
..................................................................................................................................
..................................................................................................................................
.............................................................................................................................. [4]

UCLES 2013

5014/12/M/J/13

878

13

BALOL
(b) The table shows measures of poverty and wealth for four Asian countries in 20089.
measures of poverty and wealth
country

average
income per
head ($US)

birth rate
(per 1000)

life
expectancy
(years)

people per
doctor

percentage of
people with
access to safe
(clean) water

China

3,270

13

74

667

89

Japan

38,460

84

476

100

Russia

11,830

11

69

240

96

South Korea

19,120

80

600

98

(i)

For
Examiners
Use

Which one of the four measures is the best example of an economic factor?
.............................................................................................................................. [1]

(ii)

Describe how access to safe (clean) water affects disease levels in a country.
..................................................................................................................................
..................................................................................................................................
..................................................................................................................................
..................................................................................................................................
.............................................................................................................................. [3]

(iii)

Look back to the world map in (a)(iii) on page 11.


Russia and Japan are north of the North-South divide. China and South Korea are
on the south side of the line.
How well do the differences in poverty and wealth between the four countries in the
table support the position of the North-South line in Asia? Explain your answer.
..................................................................................................................................
..................................................................................................................................
..................................................................................................................................
..................................................................................................................................
..................................................................................................................................
..................................................................................................................................
..................................................................................................................................
.............................................................................................................................. [4]

UCLES 2013

5014/12/M/J/13

[Turn over

879

14

BALOL
(c) Unfair world trade is one reason for world inequalities (large differences in wealth
between developing and developed countries).

For
Examiners
Use

Most of the coffee that is traded in the world is grown in developing countries in the
tropics (in the South) and sold in developed countries in temperate lands (in the North).
The table shows how the price of a jar of coffee that sells for $5 in a shop in the USA is
made up.
who takes the money when a jar of coffee is sold in the USA
percentage of
sale price

(i)

coffee farmer in the tropics

10

shipping controlled by transport companies based


in developed countries

20

manufacturing making the jars and roasting the


beans in the USA

45

supermarket selling the coffee

25

Draw a pie graph in the circle below to show the percentages in the table.

Key
coffee farmer
shipping
manufacturing
supermarket
[2]
(ii)

How does the pie graph show that world trade is not fair to developing countries?
..................................................................................................................................
.............................................................................................................................. [1]

UCLES 2013

5014/12/M/J/13

880

15

BALOL(iii) The Fair Trade organisation was set up in the 1990s to give farmers in developing
countries a better deal when exporting their crops. The organisation gives farmers a
guaranteed price, even when world prices fall. It also supports community projects
in areas where the farmers live.

For
Examiners
Use

Read what banana farmers in the Dominican Republic said about Fair Trade. They
live in poor rural areas near the border with Haiti.
comments from banana farmers in the Dominican Republic
... After Hurricane George destroyed our crops in 1998, small farmers
like me with only two hectares of land could not find a big company willing to
take our bananas. We are grateful to the Fair Trade which put us in touch with a
small UK importing company.

... Gone are the days of having to suffer from big changes in world
market prices.

... There was no sanitation in our village until local Fair Trade farmers started
donating outside toilets.

... Fair Trade farmers are now setting up a community canteen where village
people can get a proper meal in the middle of the day at reduced prices.

What is the main advantage of Fair Trade for banana farmers in the Dominican
Republic? Explain why it is a big advantage.
..................................................................................................................................
..................................................................................................................................
..................................................................................................................................
.............................................................................................................................. [2]
(iv)

State one way in which other people who are not banana farmers and who live in
rural areas in the Dominican Republic also benefit from Fair Trade? Explain how it
improves their quality of life.
..................................................................................................................................
..................................................................................................................................
..................................................................................................................................
.............................................................................................................................. [2]

UCLES 2013

5014/12/M/J/13

[Turn over

881

16

BALOL(v) Suggest why some big companies are not interested in changing to Fair Trade.
..................................................................................................................................

For
Examiners
Use

..................................................................................................................................
..................................................................................................................................
.............................................................................................................................. [2]
(d) Aid from rich to poor countries is another way of trying to overcome world inequalities
between rich developed and poor developing countries.
Three types of aid are

governmental (bi-lateral) aid


non-governmental aid (NGO)
food aid

Fill in the table below by


(i)

naming the type of aid from the list which best fits the uses A, B and C (use each
type of aid only once);

(ii)

stating whether the aid is long-term or short-term;

(iii)

suggesting whether the aid is sustainable or unsustainable.


use

(i)
type of aid

(ii)
long-term or
short-term

(iii)
sustainable or
unsustainable

A helps people recover from


a natural hazard
B supports community
projects for water supply
C allows large scale projects
such as large dams
[3]
(iv)

Explain your answer given to C in part (iii) are large scale projects such as large
dams sustainable or unsustainable?
..................................................................................................................................
..................................................................................................................................
..................................................................................................................................
.............................................................................................................................. [2]

UCLES 2013

5014/12/M/J/13

882

17

BALOL(v) State two advantages and two disadvantages of aid for the developing countries
that are receiving the aid.

For
Examiners
Use

advantages ...............................................................................................................
..................................................................................................................................
..................................................................................................................................
disadvantages ..........................................................................................................
..................................................................................................................................
.............................................................................................................................. [4]
(e) A third way to reduce the wealth gap between the developing and developed countries
is to find new sources of income for developing countries. One of these is tourism.
(i)

Name a developing country or an area within a developing country which attracts


many tourist visitors from other countries.
..................................................................................................................................

(ii)

Explain why it attracts tourists and how tourism is being managed for a sustainable
future.
..................................................................................................................................
..................................................................................................................................
..................................................................................................................................
..................................................................................................................................
..................................................................................................................................
..................................................................................................................................
..................................................................................................................................
.............................................................................................................................. [5]
[Total: 40 marks]

UCLES 2013

5014/12/M/J/13

[Turn over

883

19
6 BALOL
(a) A natural hazard is a short-term event that is a threat to life and property.

For
Examiners
Use

list of natural hazards


cyclone
(i)

(ii)

drought

earthquake

flood

volcano

Re-arrange this list of natural hazards to show which are climatic and which are
tectonic.
climatic

tectonic

..............................................................

..............................................................

..............................................................

..............................................................

..............................................................

..............................................................
[1]

In the box below a short, precise definition has been given for drought. Do the
same for the other four natural hazards.
natural hazard

definition

cyclone

..........................................................................................
..........................................................................................

drought

dry period, longer and more severe than

..........................................................................................

normally expected

..........................................................................................

earthquake

..........................................................................................
..........................................................................................

flood

..........................................................................................
..........................................................................................

volcano

..........................................................................................
..........................................................................................
[5]

UCLES 2013

5014/12/M/J/13

[Turn over

884

20

BALOL(iii) Predicting when an area is going to be affected by one of these natural hazards is
more easily done for some than for others.

For
Examiners
Use

Choose one of the five natural hazards, the occurrence of which is more likely to
be predictable by people affected. Describe how such predictions can be done.
Choice .............................
..................................................................................................................................
..................................................................................................................................
..................................................................................................................................
..................................................................................................................................
.............................................................................................................................. [3]
(iv)

Choose one other natural hazard which is much less easy to predict. Explain why
it is more difficult or impossible for people to predict when they are going to be
affected.
Choice .............................
..................................................................................................................................
..................................................................................................................................
..................................................................................................................................
.............................................................................................................................. [2]

(b) (i)

It is estimated that;

90% of the people killed in natural hazards live in developing countries,

98% of the people affected by natural hazards live in developing countries.

Show these percentages by completing two divided bar graphs. Also complete the
key for developing and developed countries.
Percentage of people killed in natural hazards

100%

Key

Percentage of people affected by natural hazards

100%
[2]

UCLES 2013

5014/12/M/J/13

885

21

BALOL(ii) Poor preparation is one reason why the effects of natural hazards are greater in
developing countries.

For
Examiners
Use

Three methods of preparation for natural hazards are listed in the table. Write in
the name of a natural hazard for which the method stated is most useful. Name
three different natural hazards.
method of preparation

natural hazard

build shelters away from the coastline


stocked with emergency supplies of food
and drinking water

........................................................

train emergency rescue teams, with


sniffer dogs, and have heavy lifting
equipment ready for use

........................................................

government controlled store of staple


foodstuffs such as wheat and corn,
enough to last at least 36 months

........................................................
[3]

(iii)

Explain why more people are killed in natural hazards in developing countries than
in developed countries. Refer to at least two different reasons in your answer.
..................................................................................................................................
..................................................................................................................................
..................................................................................................................................
..................................................................................................................................
..................................................................................................................................
..................................................................................................................................
..................................................................................................................................
.............................................................................................................................. [4]

(iv)

Despite the human costs, millions of people in the world continue to live in areas
affected by natural hazards. State some reasons for this.
..................................................................................................................................
..................................................................................................................................
..................................................................................................................................
..................................................................................................................................
..................................................................................................................................
.............................................................................................................................. [3]

UCLES 2013

5014/12/M/J/13

[Turn over

886

22

BALOL
(c) The monsoon rains in Pakistan in summer 2010 were much heavier and longer lasting
than normal. They caused the worst flooding seen in Pakistan since the 1920s.

For
Examiners
Use

The heaviest rains fell in the mountains of the north. Floods began in the Swat Valley
and continued to sweep south for more than 1000 km.
Look at the map and information box about the effects of the flood of 2010.
flooded areas in Pakistan (early August 2010)
N

information box

Swat
Valley

effects of the flood 2010

Ind

us

slives lost
20,000
sPEOPLEAFFECTEDMILLION
sNUMBERMADE
homeless
7 million
sNUMBEROF
villages flooded
11,000

Key
flooded
areas

Sindh
+ARACHI
ARABIAN
Indus
SEA
delta
(i)

300
km

The great flood of 2010 in Pakistan was described as a 1 in 100 year event. Why?
..................................................................................................................................
.............................................................................................................................. [1]

(ii)

Describe what the map shows about the distribution of flooded areas in Pakistan in
August 2010.
..................................................................................................................................
..................................................................................................................................
..................................................................................................................................
..................................................................................................................................
.............................................................................................................................. [3]

(iii)

Why were desert areas in the south such as Sindh, where rainfall was lower, still
affected by the flooding?
..................................................................................................................................
.............................................................................................................................. [1]

UCLES 2013

5014/12/M/J/13

887

23

BALOL(iv) Suggest reasons why the number of lives lost in the floods was so much lower than
the number of people affected by the floods.

For
Examiners
Use

..................................................................................................................................
..................................................................................................................................
..................................................................................................................................
.............................................................................................................................. [2]
(d) Read the report below. It gives more information about the effects on Pakistan of the
floods in 2010.
Nobody in the flooded villages had food. People were hungry.
Transport links with other places were broken. The government
was ill-equipped to respond to the size of the disaster. Aid
relief from outside was slow to reach Pakistan. The shortage
of clean fresh water supplies meant an increased risk of
diseases such as cholera.
Farmers who could go back to their land after flood waters
went down were shocked by what they found. Their green
fields of healthy summer crops had disappeared. They had
been replaced by barren areas of mud with scattered stones,
tree trunks and anything else that the strong flows of flood
water could move. Where could they start?
The Pakistani poor were the worst hit. Without shelter,
food, animals, seeds and tools, people used to surviving on
US$2 per day faced weeks without income, because they could
not grow or sell anything. Many with little before the flood
were left with nothing after it, worsening the rural poverty
trap. Villages lost roads, bridges, clinics, schools, clean
water and power supplies.
For future years, floods leave layers of silt on the land,
renewing soil fertility. The record rains of 2010 will have
helped to fill underground water stores, which have been
going down for years. But will poor farmers still be in
their villages to reap the benefits?
(i)

State one short-term effect, and one likely long-term effect, of the 2010 flood
waters on cropland in Pakistan.
short-term effect .....................................................................................................
..................................................................................................................................
long-term effect ......................................................................................................
.............................................................................................................................. [2]

UCLES 2013

5014/12/M/J/13

[Turn over

888

24

BALOL(ii) How good are the chances of Pakistani farmers and villagers getting out of the
poverty cycle in future years in the rural areas of Pakistan flooded in 2010? Explain
as fully as you can.

For
Examiners
Use

..................................................................................................................................
..................................................................................................................................
..................................................................................................................................
..................................................................................................................................
..................................................................................................................................
..................................................................................................................................
.............................................................................................................................. [4]
(e) Some natural hazards, such as earthquakes, often have more serious effects in urban
than in rural areas.
Other natural hazards, such as drought, often have greater effects in rural than in urban
areas.
Explain why the effects of different natural hazards vary between urban and rural areas.
..........................................................................................................................................
..........................................................................................................................................
..........................................................................................................................................
..........................................................................................................................................
..........................................................................................................................................
..........................................................................................................................................
...................................................................................................................................... [4]
[Total: 40 marks]

889

BALOL
CAMBRIDGE INTERNATIONAL EXAMINATIONS
GCE Ordinary Level

MARK SCHEME for the May/June 2013 series

5014 ENVIRONMENTAL MANAGEMENT


5014/12

Paper 1, maximum raw mark 120

This mark scheme is published as an aid to teachers and candidates, to indicate the requirements of
the examination. It shows the basis on which Examiners were instructed to award marks. It does not
indicate the details of the discussions that took place at an Examiners meeting before marking began,
which would have considered the acceptability of alternative answers.
Mark schemes should be read in conjunction with the question paper and the Principal Examiner
Report for Teachers.

Cambridge will not enter into discussions about these mark schemes.

Cambridge is publishing the mark schemes for the May/June 2013 series for most IGCSE, GCE
Advanced Level and Advanced Subsidiary Level components and some Ordinary Level components.

890

Page 2

Mark Scheme
GCE O LEVEL May/June 2013

Syllabus
5014

Paper
12

BALOL
General notes
Symbols used in Environmental Management mark schemes.
/

separates alternatives for a marking point other valid ways of expressing the same
idea are also credited

separates points for the award of a mark

[3]

indicates the number of marks available

italic

indicates that this is information about the marking points and is not required to gain
credit
italic text is also used for comments about alternatives that should be accepted, ignored
or rejected

ora

or reverse argument shows that an argument from an alternative viewpoint will be


credited

AW

alternative wording, sometimes called or words to that effect


AW is used when there are many different ways of expressing the same idea

( )

the word / phrase in brackets is not required to gain marks but sets the context of the
response for credit
e.g. (nuclear) waste nuclear is not needed but if it was described as a domestic waste
then no mark is awarded

volcanic

underlined words the answer must contain exactly this word

ecf

error carried forward if an incorrect answer is given to part of a question, and this
answer is subsequently used by a candidate in later parts of the question, this indicates
that the candidates incorrect answer will be used as a starting point for marking the later
parts of the question

891

Page 3

Mark Scheme
GCE O LEVEL May/June 2013

Syllabus
5014

Paper
12

BALOL
Section A
1

(a) (i) scrap aluminium (cans, pans etc.);


energy / electricity;
(ii) only 5 % of the energy needed / uses less energy;

[2]
[1]

(b) clearance of land for factories destroys habitats;


gas emissions pollute the atmosphere;
liquid wastes pollute water courses;
solid waste dumps can contaminate water supplies;
piles of waste / chimneys / large factory building look unsightly / are eyesores;
Mention with brief description = 1 mark
Fuller description / use of an example = 2nd mark
Max of 2 marks per way (except in exceptional cases)
Any combination of 1 and 2 mark answers to a max. of 4 marks

[4]

(c) regulations and anti-pollution laws;


monitoring by government officials / agencies;
persecution of offenders with fines / prison terms;
closing down persistent offenders;
government support for renewable / alternative energy sources;
Mention with brief description = 1 mark
Fuller description / use of an example = 2nd mark
Any combination of 1 and 2 marks answers to a max. of 3 marks

[3]
[Total: 10]

892

Page 4

Mark Scheme
GCE O LEVEL May/June 2013

Syllabus
5014

Paper
12

BALOL
2

(a) (i) accurate plots of 26.0 and 50.4;

[1]

(ii) absence of vegetation means;


nothing to intercept the rain;
nothing to slow movement of rainwater downslope;
no plant roots to take up and use rainwater;
bare surface may be baked hard / compacted allowing little rain infiltration;
Any two

[2]

(iii) although the ground vegetation cover in 2 might be denser, the grass is shorter, and has
lower water needs than the trees in 4; also grass leaves are narrower and intercept less
rain;
Some understanding = 1 mark
Clear explanation including direct reference to the grass = 2 marks

[2]

(iv) amount of ground cover varies from none (before and immediately after planting) and
most runoff, to complete cover (before harvesting) and low amount of runoff.
Clearly understood = 1 mark

[1]

(b) best choice is land use 4 (bare fallow); soil conservation measures include;
mixed cropping / crop rotation;
planting trees in the context of acting as windbreaks tree crops (but not just tree planting);
addition of organic matter to maintain soil structure;
possibly contour ploughing and terracing since a slight slope is indicated;
mulching / covering soil with dead plant matter;
Name of method with a little description = 1 mark
Further description about reducing rates of soil erosion = 2nd mark
Any combination of 1 and 2 mark answers up to a maximum of 4 marks
Any other land uses choices mark answer on its merits. In many cases the maximum mark
possible is likely to be only 2, or at best 3 marks.
[4]
[Total: 10]

893

Page 5

Mark Scheme
GCE O LEVEL May/June 2013

Syllabus
5014

Paper
12

BALOL
3

(a) (i) all areas are centred around the Equator;


never extends outside the tropics;
further description of location in terms of extent for individual areas;
any two valid descriptive points

[2]

(ii) hot and wet all year;


further details about either or both of these;

[2]

(b) (i) for an increase in global warming:


burning tropical forests releases carbon dioxide into the atmosphere;
carbon dioxide is a greenhouse gas;
traps in the atmosphere heat radiation from the Earth's surface;
the natural tropical rainforest is a huge carbon store / a larger carbon store than a
palm oil plantation;
against an increase:
both forests and oil palm plantations are carbon neutral;
Three reasons along the lines outlined above.
(ii) rainforest about 3 times as much / 315320 tonnes compared with 100;

[3]
[1]

(iii) WWF member loss of biodiversity / habitats / species;


forest dweller fear of being displaced elsewhere / loss of forest for food and many
other needs;
[2]
[Total: 10]

894

Page 6

Mark Scheme
GCE O LEVEL May/June 2013

Syllabus
5014

Paper
12

BALOL
4

(a) mainly grass with trees / shrubs in background;


further description of relative size of areas covered by each of these;
grass looks brown / dry;
sparse because bare ground / soil is showing in places;
umbrella shaped tree / acacia in centre at back of photo;
some shrubs / bushes look to be without leaves;
Three photo based descriptive points such as these.
3 1 mark

[3]

(b) population total of all the animals of the same species;


habitat home of the animals; here the grassy savanna;
consumer animals eating the grass / vegetation;
ecosystem plants / animals interacting with the natural environment;
4 1 mark

[4]

(c) animals damage vegetation by trampling, eating young shoots and stopping bushes, shrubs
and trees from growing to full size;
some animals such as elephants strip bark off trees / pull down bushes and trees;
herds of animals trample on and compact bare soil, especially in the dry season, increasing
soil erosion and weakening the soil structure, perhaps leading to desertification; also
overgrazing;
vegetation loss can have further effects on the rest of the food chain;
animal wastes fertilise the soil; likewise when they die and decompose, animals help in seed
dispersal;
1 mark for clearly stated reason
2nd mark for more detail about the reason
Any combination of 1 and 2 mark answers to a maximum of 3 marks

[3]
[Total: 10]

895

Page 7

Mark Scheme
GCE O LEVEL May/June 2013

Syllabus
5014

Paper
12

BALOL
Section B
5

(a) (i) 1 North America 2 Oceania 3 Europe 4 Latin America 5 Asia 6 Africa;

[1]

(ii) $26,210;

[1]

(iii) values plotted on map matching the bands of shading;


key completed which matches the colours / shading used on the map;
quality, appearance and value of shading and completed map;

[3]

(iv) 1

Good fit between North and Latin America is best because two shading zones
different; good also between Europe and Africa with one shading zone difference
(and Africa has the lowest value so big difference in actual numbers). Likewise need
to separate Oceania from Asia.

Less good fit Asia is the obvious example because its average value is low, but
some parts such as Russia and Japan are in the developed North. Line splits the
continent in half.

Comment about overall fit many variations possible. Good fit in separating out the
developed continents of North America and Europe from the developing continents
of Africa and Latin America i.e in western half of the map. Line less valuable in the
east of the map where its course is forced to change from running east-west to
north-south to accommodate the high development of Japan and Australia, to some
extent making the North-South world label less valid.

Basically 1 mark for each of parts 1, 2 and 3


4th mark from amount and quality of comment in part 3
It is possible, however, that the 4th mark can be distributed to 1 and 2 as well for a
particularly precise answer.
[4]
(b) (i)

Average income per head;

[1]

(ii) Best answer;


much more likely to suffer from water-borne diseases such as diarrhoea, typhoid and
cholera (i.e. 1 mark for recognising the importance of water-borne diseases), since these
spread as a result of drinking contaminated water, also caught by washing food / pots
and pans / hands and face in dirty water.
Up to the full 3 marks
Related points provided that water-borne recognised; these diseases easily spread to
other people in areas with poor sanitation, young children are most at risk with lower
immune systems, disease reduces work ability of parents / increases poverty with less
chance of affording medical attention.
Maximum 1 mark for these if water-borne not established; otherwise 2 marks possible.
References to water-based diseases such as bilharzia can also be made relevant by
stressing that it is most likely to be caught by people working in polluted waters. Others
such as Guinea worm are caught by people drinking contaminated water.

896

Page 8

Mark Scheme
GCE O LEVEL May/June 2013

Syllabus
5014

Paper
12

BALOL
Up to the full 3 marks, although the marks are harder to earn because the focus must be
kept on access to clean (safe) water.
Water-bred diseases such as malaria are not relevant to this particular question.
3 1 mark for points made, bearing in mind restrictions stated above.

[3]

(iii) Strongest support from values for ChinaJapan separation, particularly the huge
difference between average income per head (over US$ 35,000), but also for birth rate,
life expectancy and access to safe water the relative differences are large;
Average income per head (especially) would suggest that South Korea should now be
placed north of the line, that it no longer belongs in the South;
Russia's position in the North can be questioned life expectancy is well below that of
all the others, even China, despite the very favourable people per doctor ratio;
How well? Individual candidates can hold different overall views, which are capable of
being justified by the data. Perhaps the best view is that the line has its uses for splitting
the world up into a simple two-fold division, but that at the country level there are always
likely to be exceptions;
Quality of explanation based on study of measures in table = 1, 2 or 3 marks
Attempt with support to answer 'How well do the differences ?' = 1 or 2 marks
(c) (i) accurate plot of pie graph;
key completed for shading / colours used in pie graph;

[4]

[2]

(ii) Developing countries are only receiving 10% of final price / only a small amount what
the coffee farmer in the tropics receives is the only source of income for the developing
country;
Or
90% of the final price goes to companies in developed countries for transport,
manufacture and retail;
Understood = 1 mark

[1]

(iii) Guaranteed price to the farmers for their crops, even when world market price falls
farmers know the minimum price they are going to get for their crops, which allows to
survive when world market price falls; No longer at the mercy of world market prices over
which the farmers have no control, and which change wildly;
Or
Helping them to find a market for their produce by putting them in touch with overseas
importing companies from small producers; opening up a market for their produce, again
safeguarding their income;
1 mark for describing the way from the source
2nd mark for explaining the advantage of Fair Trade

[2]

897

Page 9

Mark Scheme
GCE O LEVEL May/June 2013

Syllabus
5014

Paper
12

BALOL
(iv) Ways referred to include community projects / outside toilets leading to improved
sanitation / community canteens.
Explanation depends on choice.
For example, improved sanitation reduces risk of diseases spreading, making people ill
and reducing their work productivity. Community canteens and a proper meal improves
rural health thereby allowing villagers to devote more time to productive activities;
children more likely to grow up healthier with the attendant social and economic benefits.
1 mark for naming supported by only outline explanation
2 marks for fuller explanation related to the named way.

[2]

(v) would need to deal with too many different producers to obtain big amounts needed
(compared with dealing with one large plantation owner);
small producers less able to guarantee reliable quality and consistent quantity;
more interested in profits than in helping local communities and paying farmers more;
Realises what the question is asking and/or identifies the most relevant comment = 1
mark
Understands the question well and make one or more valid suggestions = 2 marks
[2]
(d) (i) A food aid
B non-governmental aid (NGO)
C governmental (bilateral) aid
1 mark
(ii) A short-term
B long-term
C long-term
1 mark
(iii) A unsustainable
B sustainable
C allow either answer; instead mark this answer in part (iv)
1 mark

[3]

(iv) If unsustainable environmental and ecosystem damage for their creation, people
relocated not always well looked for housing and good farmland, are of some relevance;
more important are sediments build up in lake behind dam, its effectiveness is reduced
with time, rivers below dam deprived of water and silt with further environmental
consequences downstream, water often used many kilometres away in channel
irrigation, sometimes leading to soil salinisation.
If sustainable large dam will provide large water supplies / energy for many years,
giving guaranteed water supplies in areas with dry or unreliable climates, allowing
reclamation of desert land for food cultivation, assuring food and clean water supplies,
encouraging economic development benefiting future generations.
Valid explanation for viewpoint chosen 1 or 2 marks according to number of points
made, or for one point well elaborated.
[2]
(v) Advantages
alleviates immediate suffering after a natural disaster;
can be a life saver for poor people badly affected;

898

Page 10

Mark Scheme
GCE O LEVEL May/June 2013

Syllabus
5014

Paper
12

BALOL
well targeted development aid allows improvements that poor villages people could
otherwise not afford;
examples of these such as deep wells, small dams, clinics, schools;
improve infrastructure for longer term economic development;
Points made such as these; 2 1 mark
Disadvantages
fosters dependence on outside help which cannot last for ever;
big money can be spent on large projects which benefit only a few people;
some 'tied aid' does not target the real needs of people or countries;
aid does not always reach remote rural communities where it is most needed;
Points made such as these; 2 1 mark
Allow 3:1 or 1:3 ratios provided that either answer includes reference to a specific
example of a project, or named countries and relevant aid.
[4]
(e) (i) Award 1 mark if the name of a country, or area or resort in a developing country is
sufficiently precise. Country names such as India or Brazil would not be enough, but
holiday islands such as the Maldives, Mauritius and Barbados would be acceptable.
Otherwise hold back the mark at this point; take the name into account when marking
the second part.
(ii) Tourist attractions, either natural or human, and related to the named area
up to 2 further marks with 1 mark already awarded in (i)
up to 3 marks if definitely related to the area named in (i), even though that was
insufficiently precise for the mark in (i)
How managed for a sustainable future?
Controls on destruction of the natural environment, such as National Parks, Forest and
Biosphere reserves; management and maintenance; controlling numbers of tourists;
training guides; building eco-lodges; ecotourism initiatives
up to 3 marks if definitely related to the area named in (i).
Without a valid location named in (i), the maximum mark for an answer of a general
nature is 2 marks, either 1 + 1 mark for attractions and management, or more likely up to
2 marks for management methods used in tourism.
[5]
[Total: 40]

899

Page 11

Mark Scheme
GCE O LEVEL May/June 2013

Syllabus
5014

Paper
12

BALOL
6

(a) (i) climatic cyclone, drought and flood


tectonic earthquake and volcano

[1]

(ii) cyclone tropical storm, with very low pressure, associated with high winds and heavy
rainfall
earthquake shaking of the ground surface; Richter scale measures the strength of the
shaking
flood large amount of surface water covering areas which are usually dry; areas under
water after heavy rains
volcano cone shaped mountain built up by eruptions of magma from the interior of the
Earth.
4 definitions 1 mark
5th mark for high levels of precision, or for the inclusion one or two particularly strong
definitions.
[5]
(iii) Possible choices are all of them, except earthquakes.
Examples;
cyclone done by improved weather forecasting; use of weather satellites to view cloud
patterns and track the course of the storm; possible to give advance notice and estimate
chance of land area being hit. There is time for people to be evacuated to shelters or to
move inland. Areas likely to be affected by cyclones are well known.
volcano set up instruments to measure and monitor temperatures in and around the
crater; use seismographs to register earthquake shocks prior to an eruption; look for
visible signs of impending activity such as increased amounts of steam and gases
coming from the crater.
No mark for the choice; all three marks describing how prediction can be done.
3 points along the lines of those suggested above; 3 1 mark

[3]

(iv) The best choice is earthquake while zones at high risk of earthquakes are well known
since they coincide with plate boundaries, no one has been able to predict accurately
when an earthquake will occur. The big shock is often the first;
the after-shocks are less powerful than the main earthquake, so there are usually no
small warning shakes. Constant plate movement; impossible to know when the tension
will be released
1 mark for choice of earthquake
2nd mark for an explanation
For the others, no mark for choice; 2 marks can be awarded for quality of explanation for
any of the others. The choice will control the likelihood of enough explanation for both
marks.
For example, for volcanoes past performance in eruptions may be no guide to the future;
some do just erupt without notice and much history of doing so. Cyclones can change
course at the last minute, or strengthen suddenly threatening areas that thought they
were safe.
drought needs a longer time frame in which to occur. The annual rains might be late in
arriving, but meteorologists cannot predict whether it will rain weeks or months ahead,
later in the wet season.
Up to 2 marks for quality of the explanation.

[2]

900

Page 12

Mark Scheme
GCE O LEVEL May/June 2013

Syllabus
5014

Paper
12

BALOL
(b) (i) correct plots of 90% and 98% on the bar graphs;
key completed for developed and developing countries which matches what is shown in
the graphs;
[2]
(ii) build shelters cyclone / also tsunami;
train emergency rescue teams earthquake;
store of staple foodstuffs drought;
3 1 mark

[3]

(iii) Possible reasons;


Poverty lack of money so that governments are unable to afford to undertake the
advance precautions, nor do they have the resources to deal with the immediate
aftermath during which lives could be saved. Only the rich can afford to do things for
themselves such as stronger houses.
Poverty cycle people have been unable to recover from previous natural disasters,
most noticeable in droughts; rainfall may have been below average for several years
not giving a chance for food stocks to be built up.
Government attitude many other problems to deal with in developing countries;
hazard protection is a lower priority than in developed countries where the social
role of governments is greater.
Some governments too corrupt to help people.
Infrastructure and organisation places affected are often more difficult to reach in
developing countries making emergency aid arrival slower.
Geographical location some of the natural hazards, noticeably tropical cyclones, affect
latitudes where developing countries are more numerous than developed countries;
droughts are made worse in the tropics by high rates of evaporation
High rates of population increase pressure on the land and family food supplies exist
even without natural hazards. Over-use of farm land, or shanty settlements on steep
slopes, make the effects of some natural hazards worse.
Cumulative effects of repeated / different natural hazards little time for recovery
The most widespread choice is likely to be poverty maximum of 3 marks,
unless two very separate aspects of poverty are included as suggested above.
Otherwise for each reason;
reason identified with outline explanation = 1 mark
more detailed explanation (could be helped by reference to an example) = 2 marks
Any combination of 1 and 2 marks answers up to the mark of 4.

[4]

(iv) some hazards give economic benefits such as volcanoes for fertile soils, minerals,
tourism, and are naturally attractive places to live;
people own land and property in the high risk areas; they cannot give up what may be
their only assets; massive investments in big urban areas, too big to abandon;
attached to the area; have learnt to live with the risk; some can afford precautions;
belief that it won't happen to them; hazard only happens from time to time, and may be
not at all where they live;

901

Page 13

Mark Scheme
GCE O LEVEL May/June 2013

Syllabus
5014

Paper
12

BALOL
Reasons / explanation credit elaboration and use of examples.
Answer could refer to just one natural hazard with full elaboration, which covers different
reasons.
3 valid points made: 3 1 mark
[3]
(c) (i) Big size of flood nothing like it since the 1920s, almost 100 years ago.

[1]

(ii) North- south flood zone around the River Indus, extending the full length of the country
from the northern boundary to the sea, strip of flooded land is 200km or more wide, only
a few areas west and east of the Indus in the north and centre not affected, largest
unaffected area in the south west / to the north and west of Karachi. Estimate of area of
country affected such as about one quarter.
Points such as these taken from study of the map. 3 1 mark

[3]

(iii) Rains in the mountains of the north were so great that the River Indus was still in flood
1000km further south i.e. related to the size of the monsoon rains in 2010.
[1]
(iv) People in the centre and south had plenty of time to move away from the high flood risk
areas on the sides of the rivers it took time for the flood waters to be carried south by
the River Indus. Only people in the mountain valleys of the north such as the Swat Valley
had little warning.
Floods were on such a scale that nobody elsewhere in Pakistan could be unaware of
them, no matter how poor communications might be in rural areas.
River valleys of large rivers such as that of the Indus are very favourable for farming and
settlement that is why so many people were affected. Most people in developing
countries like Pakistan are farmers.
Two different reasons such as these or one reason well elaborated.
2 1 mark

[2]

(d) (i) Short-term effect on cropland one from:


loss of crops; farmland covered by mud / stones / tree trunks; nothing to sell
Long-term effect on cropland one from:
layers of fertile silt deposited, underground water stores filled up with waters making it
more abundant for future crop use
1 + 1 mark

[2]

902

Page 14

Mark Scheme
GCE O LEVEL May/June 2013

Syllabus
5014

Paper
12

BALOL
(ii) Key part of the report refers to the rural poverty trap; 'poor people with little before the
flood were left with nothing after it' ... Also 'Pakistani poor were the worst hit' These,
along with all the further information about losses to crops, animals, belongings and
infrastructure, suggest little chance of getting out of the poverty cycle. This is reinforced
by references in the report to lack of government help and aid from outside.
Great challenges in clearing the farmland of mud, stones, tree trunks and other debris for
poor people without anything.
Only glimmers of hope in the report are the renewal of soil fertility from the silt deposits,
plus more plentiful ground water supplies for irrigation in future years.
Choice and use of content from the report related to the question's poverty cycle theme,
but with limited added explanatory comment 1 or 2 marks
Good choice of content from the report, with the candidate's view about the chances of
getting out of the poverty cycle supported and enhanced by comment 3 or 4 marks [4]
(e) Earthquakes little danger from shaking of the ground in open spaces; plenty of these in
rural areas, but in short supply in cities. Falling debris and collapsing buildings are what do
the damage and kill and injure people in earthquakes.
Droughts most significant for people who rely most upon water supplies from rainfall and
rivers; they are mainly farmers needing water for crops and pastures for livestock. In urban
areas, domestic water use is much lower and water is often piped in from mountains (where
drought is less likely) or from man-made stores such as reservoirs.
Possible for candidates to use examples of other natural hazards in their answers.
Floods the previous example of floods in Pakistan showed the serious effects for farmers
and rural communities
Volcanoes effects often worse in rural areas, closer to the the volcano, and farmland can
be covered by lava and ash; eventually fertile, but it takes time to break down into soils that
can be used.
Cyclones hazard for which effects on urban and rural areas may be equally severe;
coastal areas that take the direct hit can be devastated whether urban or rural.
Answer may be more about the different effects of natural hazards, only weakly related to the
theme of differences between urban and rural areas. Or it may be all about just one natural
hazard such as earthquakes. Overall limited coverage.
1 or 2 marks
Answer better focused on the theme of variations between rural and urban areas, supported
by references to at least two natural hazards and their differing effects.
3 or 4 marks
[4]
[Total: 40]

903

BALOL
UNIVERSITY OF CAMBRIDGE INTERNATIONAL EXAMINATIONS
General Certificate of Education Ordinary Level

* 2 1 0 1 0 2 9 8 4 5 *

5014/11

ENVIRONMENTAL MANAGEMENT
Paper 1

October/November 2013
2 hours 15 minutes

Candidates answer on the Question Paper.


Additional Materials:

Ruler
Protractor

READ THESE INSTRUCTIONS FIRST


Write your Centre number, candidate number and name on all the work you hand in.
Write in dark blue or black pen.
You may use a soft pencil for any diagrams, graphs or rough working.
Do not use staples, paper clips, highlighters, glue or correction fluid.
DO NOT WRITE IN ANY BARCODES.
Answer all questions.
Electronic calculators may be used.
You may lose marks if you do not show your working or if you do not use appropriate units.
Write your answers in the spaces provided on the Question Paper.
All questions in Section A carry 10 marks.
Both questions in Section B carry 40 marks.
At the end of the examination, fasten all your work securely together.
The number of marks is given in brackets [ ] at the end of each question or part
question.

For Examiners Use


1
2
3
4
5
6
Total

904

BALOL

For
Examiners
Use

(a) Look at the graph showing the production of aluminium from 1900 to 2009.
40

40

35

35

30

30

25

25

20

20

15

15

10

10

world production of aluminium / million tonnes per year

world production of aluminium / million tonnes per year

Section A

0
0
1900 1910 1920 1930 1940 1950 1960 1970 1980 1990 2000 2009
year
Describe the main changes in the amounts of aluminium produced since 1900.
..........................................................................................................................................
..........................................................................................................................................
..........................................................................................................................................
..........................................................................................................................................
..........................................................................................................................................
...................................................................................................................................... [3]
UCLES 2013

5014/11/O/N/13

905

BALOL
(b) Aluminium is a very useful metal. It is a strong, soft, long-lasting metal and is very light
in weight. It does not corrode easily.

For
Examiners
Use

Suggest which property of aluminium makes it useful in the manufacture of aeroplanes


and cars. Explain your choice.
(i)

aeroplanes ...............................................................................................................
.............................................................................................................................. [1]

(ii)

drinks cans ...............................................................................................................


.............................................................................................................................. [1]

(c) A large amount of energy is needed for refining aluminium ore (bauxite). Refining is
often done in mountainous regions.
Name the type of power likely to be used there. ..............................................................
...................................................................................................................................... [1]
(d) (i)

Metal ores, such as bauxite, need to be conserved. State three ways of doing this.
1 ...............................................................................................................................
..................................................................................................................................
2 ...............................................................................................................................
..................................................................................................................................
3 ...............................................................................................................................
.............................................................................................................................. [3]

(ii)

The presence of aluminium in soils can increase their acidity. What change would
this cause to the pH of the soil?
..................................................................................................................................
.............................................................................................................................. [1]
[Total: 10]

UCLES 2013

5014/11/O/N/13

[Turn over

906

11 000

11 000

10 000

10 000

9000

9000

8000

8000

7000

7000

6000

6000

5000

5000

4000

4000

3000

3000

2000

2000

1000

1000

For
Examiners
Use

number of cases of cholera

number of cases of cholera

2 BALOL
(a) Look at the bar graph which shows the number of cases of cholera during epidemics in
Sierra Leone between 1990 and 2009.

19

9
19 0
9
19 1
9
19 2
9
19 3
9
19 4
9
19 5
9
19 6
9
19 7
9
19 8
9
20 9
0
20 0
0
20 1
0
20 2
0
20 3
0
20 4
0
20 5
0
20 6
0
20 7
0
20 8
09

(i)

Use the information in the table to complete the bar graph.

year

number of cases of cholera

1994

9700

2007

2200
[2]

(ii)

State the highest number of cases and the year in which they occurred.
number .................................................... year .................................................... [1]

(iii)

The cholera outbreaks shown on the bar graph were described as epidemics.
State one characteristic of an epidemic.
..................................................................................................................................
.............................................................................................................................. [1]

UCLES 2013

5014/11/O/N/13

907

BALOL
(b) (i) Cholera is a water-borne disease. Name one other water-borne disease.
.............................................................................................................................. [1]
(ii)

For
Examiners
Use

What is the difference between a water-borne disease and a water-based disease?


..................................................................................................................................
..................................................................................................................................
..................................................................................................................................
.............................................................................................................................. [2]

(c) Explain the likely impact of a cholera epidemic on the economy of a farming area in
developing countries like Sierra Leone.
..........................................................................................................................................
..........................................................................................................................................
..........................................................................................................................................
..........................................................................................................................................
..........................................................................................................................................
...................................................................................................................................... [3]
[Total: 10]

UCLES 2013

5014/11/O/N/13

[Turn over

908

6
3 BALOL
(a) Name and describe an instrument used at a weather station to measure wind speed.
name ................................................................................................................................

For
Examiners
Use

description ........................................................................................................................
..........................................................................................................................................
..........................................................................................................................................
..........................................................................................................................................
..........................................................................................................................................
..........................................................................................................................................
...................................................................................................................................... [4]
(b) Look at the graph which shows how power produced by a wind turbine in the UK varies
with wind speed.

power
output

(i)

10 20 30 40 50 60
wind speed / metres per second

70

Describe how wind speed affects power output for this wind turbine.
..................................................................................................................................
..................................................................................................................................
..................................................................................................................................
..................................................................................................................................
..................................................................................................................................
.............................................................................................................................. [3]

UCLES 2013

5014/11/O/N/13

909

BALOL(ii) Explain why there is no power output from this wind turbine at certain wind speeds.
..................................................................................................................................

For
Examiners
Use

..................................................................................................................................
..................................................................................................................................
.............................................................................................................................. [2]
(iii)

Wind energy includes an extra cost, known as a standby cost.


Suggest what this is for.
..................................................................................................................................
.............................................................................................................................. [1]
[Total: 10]

UCLES 2013

5014/11/O/N/13

[Turn over

910

8
4 BALOL
(a) Look at the diagram of the nitrogen cycle.

For
Examiners
Use

...................................

nitrate in
chemical
fertiliser

eating by animals

run-off
legumes

animal
waste

dead
organisms
A

absorption by
plant roots

...................................

...................................

nitrate
loss to groundwater
(i)

Complete the boxes in the diagram using the following words:


ammonia

nitrite

nitrogen
[2]

(ii)

Using only the information on the diagram of the nitrogen cycle, describe two ways
in which nitrates can enter water sources.
1 ...............................................................................................................................
..................................................................................................................................
2 ...............................................................................................................................
.............................................................................................................................. [2]

UCLES 2013

5014/11/O/N/13

911

BALOL(iii) Describe the process represented by the arrow labelled A.


..................................................................................................................................

For
Examiners
Use

..................................................................................................................................
..................................................................................................................................
.............................................................................................................................. [2]
(b) (i)

The roots of legumes, such as peas and beans, have nodules that play an important
part in converting nitrogen to a form that plants can use.
What do the nodules contain?
.............................................................................................................................. [1]

(ii)

Nitrates can be added to the soil in fertiliser or by growing legumes.


What are the advantages of using legumes instead of fertiliser for the farmer and
for the ecosystem?
..................................................................................................................................
..................................................................................................................................
..................................................................................................................................
..................................................................................................................................
..................................................................................................................................
.............................................................................................................................. [3]
[Total: 10]

UCLES 2013

5014/11/O/N/13

[Turn over

912

10
5 BALOL
(a) Look at the diagram of natural energy flows and stores for tree and forest growth.

For
Examiners
Use

water is provided by
............................................

............................................ from
the ............................................
leaves take in the

dead leaves fall and

............................................

............................................

............................................

into the soil

from the air

water and nutrients are stored in

............................................ take in

the ............................................

water and .......................................

............................................ store water beneath the soil

(i)

Fill in the spaces to complete the diagram of energy flows and stores for tree and
forest growth.
Write your answers in the spaces on the diagram. [5]

UCLES 2013

5014/11/O/N/13

913

11

BALOL(ii) How and why are trees and other green plants the Earths primary producers?
..................................................................................................................................

For
Examiners
Use

..................................................................................................................................
..................................................................................................................................
..................................................................................................................................
..................................................................................................................................
............................................................................................................................. [3]
(iii)

Trees and other green plants support food chains. For a land based ecosystem
of your choice, complete the diagram below by naming the organisms in the food
chain and describing the natural vegetation.
location of land based ecosystem chosen .......................

...............................................................

...............................................................

...............................................................

producer
description of the natural vegetation

...............................................................
[4]

UCLES 2013

5014/11/O/N/13

[Turn over

914

12

BALOL(iv) What happens to the amount of energy passing along a food chain in a natural
ecosystem? Explain why this happens.

For
Examiners
Use

..................................................................................................................................
..................................................................................................................................
..................................................................................................................................
..................................................................................................................................
............................................................................................................................. [3]
(b) Trees grow in living communities in forest ecosystems. The diagram below shows the
components of a natural forest ecosystem.

climate

.........................

.........................

Key
biotic
abiotic

natural vegetation

On the diagram:
(i)

name two other components of the ecosystem,

(ii)

shade or colour in each of the four components according to whether they are
biotic or abiotic. Shade or colour in the key to match.
Put your answers on the diagram. [2]

(c) Biomes are large scale ecosystems. On a global scale, climate is the most important
component of the ecosystem for determining characteristics of the natural vegetation
and how they change over the Earths surface.
Look at the cross section of natural vegetation from the coast of West Africa (latitude
5 N) to the interior (latitude 20 N) on page 5.
(i)

The annual rainfall totals at the points marked A to E on the section are;
A: 2000 mm B: 1500 mm C: 1000 mm D: 500 mm E: 250 mm.
Plot these rainfall totals as a bar graph on the grid below the section.

(ii)

UCLES 2013

[2]

In the table below the section, describe the natural vegetation between points B
and C, C and D, and D and E, in a similar way to what has already been done for A.
[4]
5014/11/O/N/13

915

13

BALOL
cross section of natural vegetation from the coast of West Africa to the interior
climate
type

equatorial

savanna

For
Examiners
Use

hot desert

natural
vegetation
locations
A
natural
vegetation

mangrove
swamps

temperature of
hottest and coldest
month / C

hottest
coldest

dense
tropical
rain forest
with tall
trees and
five forest
layers

.................. ..................

..................................

.................. ..................

..................................

.................. ..................

..................................

.................. ..................

..................................

.................. ..................

..................................

.................. ..................

..................................

C
31
24
total annual rainfall

A
28
26

2000

E
36
22

1500
total
annual
1000
rainfall
/ mm
500
0
A

(iii)

C
location on section

State which of the two climate factors, temperature or rainfall, is more important for
explaining changes in natural vegetation between 5 and 20 north of the equator
in West Africa. Explain your choice of factor.
..................................................................................................................................
..................................................................................................................................
..................................................................................................................................
..................................................................................................................................
..................................................................................................................................
..................................................................................................................................

UCLES 2013

............................................................................................................................. [4]
5014/11/O/N/13
[Turn over

916

14

BALOL
(d) Today people are often added to diagrams of natural ecosystems. The diagram shows a
forest ecosystem modified by the addition of people.

For
Examiners
Use

climate

natural vegetation

people

(i)

How and why is the role of people different from that of the other components which
make up an ecosystem?
..................................................................................................................................
..................................................................................................................................
..................................................................................................................................
............................................................................................................................. [2]

(ii)

The table shows estimates of the percentage losses of the area of five natural
ecosystems (biomes) up to 2005.
natural ecosystem (biome)

percentage loss

tundra
coniferous forests (taiga)
hot deserts
savanna
tropical rainforest

UCLES 2013

2
5
25
55
40

5014/11/O/N/13

917

15

BALOL

Show these percentages in divided bar graphs in the grid below and complete the
key.

For
Examiners
Use

percentage losses of natural ecosystems up to 2005

tundra

coniferous forest
(taiga)

hot desert

savanna

key

tropical
rainforest

ecosystem
losses
0

10

20

30

40

50

60

70

80

90

100

percentage
[2]
(iii)

Suggest reasons for:


variations in the size of percentage losses between the three tropical ecosystems
the much lower percentage losses in the cold temperate and polar ecosystems.
tropical ecosystems ..................................................................................................
..................................................................................................................................
..................................................................................................................................
..................................................................................................................................
..................................................................................................................................
polar ecosystems .....................................................................................................
..................................................................................................................................
..................................................................................................................................
..................................................................................................................................
............................................................................................................................. [5]

UCLES 2013

5014/11/O/N/13

[Turn over

918

16

BALOL
(e) (i) Describe one sustainable forest management strategy (method) that people can
use to obtain supplies of wood from natural forests.

For
Examiners
Use

..................................................................................................................................
..................................................................................................................................
..................................................................................................................................
..................................................................................................................................
(ii)

Explain why sustainable forest management strategies like this are not used in all
forests.
..................................................................................................................................
..................................................................................................................................
..................................................................................................................................
..................................................................................................................................
[4]
[Total: 40]

UCLES 2013

5014/11/O/N/13

919

17
6 BALOL
(a) Look at the partly completed bar graph showing emissions of carbon dioxide per person
in major world regions.

For
Examiners
Use

carbon dioxide emissions per person in 2000 in major world regions in kilograms
North America
Australasia
Europe
Central America and
Caribbean
Asia
South America
Sub-Saharan Africa
Middle East and
North Africa
0

4000

8000

12000

16000

20000

carbon dioxide per person / kg


Key
mainly developed countries

mainly developing countries

(i)

Emissions of carbon dioxide per person in 2000 in the Middle East and North Africa
were 3900 kilograms. Add this information in the space left at the bottom of the
graph.
[1]

(ii)

On the graph, use different shading or colours to show whether the eight regions
contain mainly developed or mainly developing countries. Fill in the key.
[1]

(iii)

Did you decide that the countries in the Middle East and North Africa are mainly
developed or mainly developing?
Explain your choice of answer.
..................................................................................................................................
..................................................................................................................................
..................................................................................................................................
............................................................................................................................. [2]

UCLES 2013

5014/11/O/N/13

[Turn over

920

18

BALOL(iv) How big is the difference in emissions of carbon dioxide per person between North
America and Sub-Saharan Africa? State your answer in kilograms.

For
Examiners
Use

Space for working.

.................................................. [1]
(v)

According to one environmental group, a person living in the USA is responsible for
seven times more carbon dioxide emissions in a year than a person in Ethiopia is in
a lifetime.
Why are there big differences in carbon dioxide emissions per person between
different countries of the world? Explain your answer as fully as you can.
..................................................................................................................................
..................................................................................................................................
..................................................................................................................................
..................................................................................................................................
..................................................................................................................................
..................................................................................................................................
..................................................................................................................................
............................................................................................................................. [4]

(b) Carbon dioxide is one of the greenhouse gases. It is usually considered to be the most
important greenhouse gas leading to global climate change.
(i)

Name another important greenhouse gas.


............................................................................................................................. [1]

(ii)

Why are they called greenhouse gases?


..................................................................................................................................
..................................................................................................................................
..................................................................................................................................
..................................................................................................................................
..................................................................................................................................
............................................................................................................................. [3]

UCLES 2013

5014/11/O/N/13

921

19

BALOL(iii) Look at the box below which contains statements about global climate change.
global climate change

For
Examiners
Use

LYH[\YLZ VH
W
T
L
[

K
S
D
VY
H]LYHNL^ "* PR LFHWKLQQLQJ
D
XQWDLQ
*
JODFLH QGPHOWLQJ
 

UVU
cutting down forests
for logging, farming and mining

HWUHDW
LQJ

.\RWR
F
WDUJHW OLPDWHFKD
Q
VVHWI
RUFDUE JHFRQIHUH
QF
RQGLR
[LGHU H
HGXFWL
RQV

s
oastal area
c
in
k
is
r
d
ies
higher floo
ing countr
y
-l
w
lo
in
especially

ther events
more extreme wea
ten and stronger
happening more of

OHVVZDWHUIRULUULJDWLRQLQ$VLD
IURPULYHUVVWDUWLQJLQWKH+LPDOD\DV

OZMI\][MWNNW[[QTN]MT
[
NWZMTMK\ZQKQ\aIVL\ZIV
[XWZ

risin
18cm high g sea levels
er than 10
0 years ago

Choose two statements which give physical evidence suggesting the existence of
global warming, and another two statements which are effects of global warming
on people.
Physical evidence for global warming.
1 ...............................................................................................................................
2 ...............................................................................................................................
Effects of global warming on people.
1 ...............................................................................................................................
2 .......................................................................................................................... [2]

UCLES 2013

5014/11/O/N/13

[Turn over

922

20

BALOL(iv) Explain why some countries are more worried about the effects of global warming
than others.

For
Examiners
Use

..................................................................................................................................
..................................................................................................................................
..................................................................................................................................
..................................................................................................................................
..................................................................................................................................
..................................................................................................................................
..................................................................................................................................
............................................................................................................................. [4]
(c) Many people believe that burning fossil fuels causes most of the increased greenhouse
gas emissions and climate change.
Look at the pie graph showing global greenhouse gas emissions from different sources.
peoples waste
heating and lighting
energy supply
farming

forest clearance
manufacturing
industry
key
mainly from the
use of fossil fuels

transport

(i)

UCLES 2013

Show which sources of greenhouse gas emissions are mainly due to burning of
fossil fuels by shading or colouring the sectors and the key of the graph.
[1]

5014/11/O/N/13

923

21

BALOL(ii) What is the approximate total percentage from the use of fossil fuels?

For
Examiners
Use

Show your working.

.................................................. [2]
(iii)

Choose one of the sources you have not shaded in the graph. Describe how human
activities in this sector contribute to the emission of greenhouse gases.
..................................................................................................................................
..................................................................................................................................
..................................................................................................................................
............................................................................................................................. [2]

(iv)

Explain how well the pie graph supports the view that the use of fossil fuels is most
responsible for greenhouse gas emissions and climate change.
..................................................................................................................................
..................................................................................................................................
..................................................................................................................................
............................................................................................................................. [2]

UCLES 2013

5014/11/O/N/13

[Turn over

924

22

BALOL
(d) Many governments are interested in increasing the percentage of energy used from
sustainable alternative sources. Some examples of such alternative energy sources are
named in the diagram.
solar

For
Examiners
Use

wave

sustainable alternative
energy sources

hydro-electric
(i)

geothermal

What do all of these examples have in common that makes them sustainable
sources of energy?
..................................................................................................................................
..................................................................................................................................
..................................................................................................................................
............................................................................................................................. [2]

(ii)

State two different reasons why sustainable alternative energy sources currently
contribute less than 10 percent of global energy consumption.
..................................................................................................................................
..................................................................................................................................
..................................................................................................................................
............................................................................................................................. [2]

UCLES 2013

5014/11/O/N/13

925

23

BALOL(iii) Choose one of the alternative energy sources named in the diagram. Give more
information about where it is used and how people harness the source to provide
energy.

For
Examiners
Use

chosen alternative source .......................................................................................


..................................................................................................................................
..................................................................................................................................
..................................................................................................................................
..................................................................................................................................
..................................................................................................................................
..................................................................................................................................
..................................................................................................................................
............................................................................................................................. [4]
(iv)

Suggest and explain how good the chances are of your chosen alternative energy
source being more widely used in future years.
..................................................................................................................................
..................................................................................................................................
..................................................................................................................................
............................................................................................................................. [2]

(v)

Suggest and explain your view of the chances of a significant increase in the use
of all types of alternative energy sources for world energy supplies during the next
1020 years.
..................................................................................................................................
..................................................................................................................................
..................................................................................................................................
..................................................................................................................................
..................................................................................................................................
..................................................................................................................................
..................................................................................................................................
............................................................................................................................. [4]
[Total: 40]

UCLES 2013

5014/11/O/N/13

926

BALOL
CAMBRIDGE INTERNATIONAL EXAMINATIONS
GCE Ordinary Level

MARK SCHEME for the October/November 2013 series

5014 ENVIRONMENTAL MANAGEMENT


5014/11

Paper 1 , maximum raw mark 120

This mark scheme is published as an aid to teachers and candidates, to indicate the requirements of
the examination. It shows the basis on which Examiners were instructed to award marks. It does not
indicate the details of the discussions that took place at an Examiners meeting before marking began,
which would have considered the acceptability of alternative answers.
Mark schemes should be read in conjunction with the question paper and the Principal Examiner
Report for Teachers.

Cambridge will not enter into discussions about these mark schemes.

Cambridge is publishing the mark schemes for the October/November 2013 series for most IGCSE,
GCE Advanced Level and Advanced Subsidiary Level components and some Ordinary Level
components.

927

Page 2

Mark Scheme
GCE O LEVEL October/November 2013

Syllabus
5014

Paper
11

BALOL
Section A
Notes on application of the mark scheme for Section A

marks are separated by commas. Each line usually represents one mark
oblique lines separate ideas which are alternatives
ideas in brackets are not essential to the answer but anything underlined is
reward any equivalent way of expressing the ideas in the mark scheme
reward any valid answer which is not in the mark scheme

(a) large increase overall,


slow increase to 1940 60 (any in between)
then rapid increase,
fluctuations,
highest in 2008 at 39 600 (accept 39 000 39 900)
credit the use of data to show a major change,
[3]
(b) (i) it is light in weight (so the plane will get off the ground easily)
/it is strong/long lasting (so will be economical),

[1]

(ii) it is non-corrosive (so the drink will not be contaminated)


/it is strong (so the can is less likely to be damaged),
[1]
(c) HEP,

[1]

(d) (i) accept any sensible idea, such as:


recycling,
a substitute/new material can be developed/used,
conservation/use it more efficiently/ prioritise its use,
new technology,
this may use less ore in the production of a material/another metal might be
mixed with it,
(ii) the pH will be lower/e.g. change from 5 to 4 etc.,

[3]
[1]

928

Page 3

Mark Scheme
GCE O LEVEL October/November 2013

Syllabus
5014

Paper
11

BALOL
2

(a) (i) 1994 plot at 9700 (top of bar between the lines),
2007 plot at 2200 (top of line on the correct line),

[2]

(ii) 10 200 to 10 300 in 1995,

[1]

(iii) sporadic/unpredictable,
affects large numbers of people,
fast spreading/develops rapidly,
starts suddenly/stops suddenly,
variable intensity,

[1]

(b) (i) typhoid/dysentery/diarrhoea,


(ii) water-borne is carried in the water but originates elsewhere,
water-based is when the carrier lives in the water,

[1]
[2]

(c) accept ideas, such as:


economy poorer/farmers gain little/no income/remain poor, =1
people become too ill to work,
crops not sown/harvested,
no surplus for sale in the cities,
cannot purchase fertilisers/improved seed etc
businesses that depend on farmers (e.g. shops) suffer,
explanation = 2
3

[3]

(a) anemometer, =1
cups,
rotate,
stand/shaft/pole,
dial/meter/counter, = 3
name + 3 descriptive points =
(b) (i) no power output with wind speeds below 10 m per sec.,
output rises with wind speeds between 10 and 30 m per sec.,
maximum output between 30 and 60m per sec.,
no output over 60 m per sec.,

[4]

[3]

(ii) the wind is not strong enough to turn the blades,


the wind is so strong that it would cause damage (to the blades),
the machines are shut down,

[2]

(iii) the cost of using another type of power when wind power cannot be used,

[1]

929

Page 4

Mark Scheme
GCE O LEVEL October/November 2013

Syllabus
5014

Paper
11

BALOL
4

(a) (i) nitrogen (in atmosphere),


ammonia (top box in soil),
nitrite (middle box in soil),
3 correct = 2
1/2 correct =1

[2]

(ii) in run-off after fertiliser application,


leaching down to the groundwater/in solution to the groundwater,

[2]

(iii) decomposition,
bacteria/fungi/ detritivores etc.,

[2]

(b) (i) (nitrogen-fixing) bacteria,

[1]

(ii) accept ideas, such as:


legumes can be ploughed in after the crop to increase nutrients in the soil,
legumes are a useful supply of beans/peas/animal fodder,
legumes are less expensive/fertilisers are more expensive,
danger to the environment of over-fertilisation/legumes are more environmentally friendly,
eutrophication, leads to death of plants/fish,
[3]
5

(a) (i) light / energy from the Sun


water provided by rainfall / precipitation
dead leaves fall and decay / decompose into the soil
leaves take in the carbon dioxide from the air
water and nutrients are stored in the soil
roots take in water and nutrients (minerals)
permeable rocks / named example store water beneath the soil/ aquifers
9 spaces to be filled in all 9 valid = 5 marks
7 or 8 filled in correctly = 4 marks
5 or 6 = 3 marks
3 or 4 = 2 marks
at least 2 = 1 mark

[5]

(ii) plants manufacture their own food from inorganic / abiotic materials
unique ability to do this makes them the main self-feeders on Earth
make food by the process of photosynthesis
using sunlight, carbon dioxide and water
further detail about photosynthesis such as giving the formula
release oxygen and provide food for herbivores
Three points such as these. 3 @ 1 mark

[3]

930

Page 5

Mark Scheme
GCE O LEVEL October/November 2013

Syllabus
5014

Paper
11

BALOL
(iii) description of vegetation + named example of an herbivore
+ named example of a carnivore + named example of a top carnivore
e.g. in savanna lands of Africa
grassland vegetation + zebra + lion + human / vulture
e.g. in tundra
mosses and lichens + caribou / reindeer + wolves + human / bears
five elements
location + description of vegetation + example of herbivore +
example of carnivore + example of top carnivore.
any four which follow on / fit = 4 marks
any three which follow on / fit = 3 marks
any two which fit and follow on for a location = 2 marks
some idea of a food chain from examples stated = 1 mark

[4]

(iv) energy lost = 1 mark: 90% reduction at each level in the food chain = 1 mark
up to 2 marks
energy lost in life processes in every stage such as breathing, generating heat, growing
and moving around, which is why a smaller number of organisms can be supported at
each level up to 2 marks
[3]
(b) (i) animals and soils in any of the two empty boxes = 1 mark
(ii) climate and soils abiotic; animals and natural vegetation biotic;
and key completed for shading used = 1 mark
(c) (i) length of bars correct = 1 mark
bars kept the same width = 1 mark

[2]

[2]

(ii) B & C description of classic savanna vegetation such as tall grasses with scattered trees;
examples of trees such as acacia and baobab also possible
C & D short grasses (and absence of / reduced number of trees)
D & E thorn scrub / tufted grasses/ more discontinuous cover/ other desert plants
3 @ 1 mark for some valid description.
4th mark for additional detail, most likely for savanna or hot desert

[4]

931

Page 6

Mark Scheme
GCE O LEVEL October/November 2013

Syllabus
5014

Paper
11

BALOL
(iii) rainfall because the decrease in rainfall from 2000 mm to 250 mm from 5N to 20N
coincides with a decrease in vegetation height, biomass and surface cover, as well as a
change towards more drought resistant plants such as thorn scrub and / or to seasonal
variations in the savanna between wet season with trees with leaves and tall grasses
compared with dry season short grasses and trees without leaves
the lowest temperature shown is 22 C, which is hot enough for plants still to grow; the
largest annual range is only 14 C; the conclusion is that rainfall is controlling the type
and amount of vegetation cover, not temperature
temperature based answer is increasing rates of evaporation with increasing
temperatures inland fierce desert heat dries out vegetation
simple statements, but with little elaboration or development = 1 or 2 marks
more complete answer, ideally referring to temperature as well in rainfall based answers
= 3 or 4 marks
[4]
(d) (i) capable of being more disruptive / damaging = 1 mark
2nd mark for further elaboration or greater context; although some indigenous peoples
live like other animals as part of the natural ecosystem, people move in from other
places, in greater numbers, and with the technology to destroy, ability to replace with
different vegetation / crops
[2]
(ii) accurate plots = 1 mark
ecosystem losses consistently identified (by shading / colour) and indicated in
key = 1 mark
[2]
(iii) 1

percentage loss differences reflect human opportunities for settlement and making a
living; shortage of fresh water for crop growing / lack of pastures for livestock rearing
make the hot desert regions the least attractive in the tropics
density of vegetation limits access to tropical rainforests especially for people with
low levels of technology, which applied more strongly in the past than it does today
points made along these lines minimum 2 marks, maximum 3 marks

limiting factor for human settlement and living in cold temperate and polar
environments is the cold / short growing season; much more limited variety and size
of the natural vegetation makes living off what natural ecosystems provide more
difficult; (many indigenous groups in polar lands rely more on ocean than land food
supplies); although lack of rainfall may be a problem in some tropical ecosystems, it
is always hot enough to grow crops and irrigation technology was developed early
by humans
some points made from within these ideas minimum 2 marks, maximum 3 marks
[5]

932

Page 7

Mark Scheme
GCE O LEVEL October/November 2013

Syllabus
5014

Paper
11

BALOL
(e) (i) agro-forestry, community forestry, reforestation, sustainable harvesting of hardwoods,
fuelwood planting and genetic engineering
exemples such as, selective logging under the heading of sustainable harvesting of
hardwoods, reforestation and community forestry
1 mark for naming the method and just enough description to show that they know what
it is
another 2 or 3 marks for fuller description related to question context
minimum 2 marks, maximum 3 marks
(ii) less easy / more expensive than just clearing patches of forest / felling all the trees,
people tend to look in terms of short-term gains rather than long-term sustainability,
difficult to be selective when using modern machinery to cut down the trees, other
advantages to clearing forests to use land for farming / allow economic development,
where relevant lack of laws, no monitoring, corruption
points made along these lines
minimum 2 marks, maximum 3 marks

[4]
[Total: 40]

(a) (i) bar drawn of correct length matching the width of others already drawn

[1]

(ii) mainly developed North America, Australasia, Europe


mainly developing Central America & Caribbean, Asia, South America and SubSaharan Africa
allow Middle East and North Africa in either group (although mainly developing would be
the usual heading)
[1]
(iii) developing: graph evidence is that carbon dioxide emissions are quite a bit lower per
person than in the established developed continents; in general there is a close
relationship between high incomes / industrialisation and high emissions; knowledge that
these countries lie on the southern / developing world side of the North-South divide, or
other relevant knowledge about developing countries such as dependence on farming
and primary activities
or
some countries in the Gulf are among the world's highest for income per head and for
carbon dioxide emissions due to their oil based economies.
credit points to support the choice for up to 2 marks
(iv) 18 10018 300 kilograms

[2]
[1]

933

Page 8

Mark Scheme
GCE O LEVEL October/November 2013

Syllabus
5014

Paper
11

BALOL
(v) most carbon dioxide emissions come from burning fossil fuels, fossil fuels most used in
transport, electricity, and manufacturing industry, much higher individual / domestic
levels of consumption among richer people, whereas in poor countries fossil fuel use is
not always a part of people's everyday lives, more work done manually in farming and
industry, more limited access to electricity and private means of transport, waste of
energy etc. more likely in developed world
Ethiopia is a poor country in sub-Saharan Africa with many subsistence farmers
points made along these lines
12 marks for answers which concentrate only on what happens in rich / developed
countries, or for narrow answers with limited development
34 marks for broader answers, most likely referring to rich and poor, bringing out the
variations in fossil fuel use between them, supported by explanation
[4]
(b) (i) one from methane, CFCs or nitrogen oxides

[1]

(ii) they enhance / increase / speed up / accelerate the natural greenhouse effect, they trap
some of the heat which is radiated from the surface at night, preventing heat loss
beyond the upper atmosphere into space, making the Earth's surface warmer than it
would otherwise be, without 'greenhouse gases' in the atmosphere the Earth would be
about 30C colder
three points made along these lines 3 @ 1 mark

[3]

(iii) physical evidence:


average world temperatures in 1900 and 2000
rising sea levels 18 cm higher than 100 years ago
sea ice melting mountain glaciers retreating
effects:
more extreme weather events happening more often and stronger
higher flood risk in coastal areas especially in low lying countries
risk of less water for irrigation in Asia from rivers starting in the Himalayas
2 @ 1 mark

[2]

934

Page 9

Mark Scheme
GCE O LEVEL October/November 2013

Syllabus
5014

Paper
11

BALOL
(iv) in some cases it depends on location low lying island countries in the Pacific and
Indian Oceans, such as the Maldives, or delta countries such as Bangladesh and the
Netherlands, are at greatest risk of coastal flooding, whereas higher or landlocked
countries will be unaffected; matters less in big countries than small countries
it is also depends on climate type more difficult / marginal climates for people to make
a living such as savanna and monsoon, where many people rely upon the wet season
rains; areas naturally prone to flood and drought such as those near the edges of
deserts, or tropical storms, may find natural disasters more frequent / more intense
poverty / wealth of a country also relevant rich countries better able to prepare and
repair, poor countries have more people trapped in the poverty cycle; less chance to
recover before being affected by the next adverse climatic event
simple statements, limited elaboration, nothing more than passing references to named
examples, narrow coverage concentrating on one aspect without addressing 'some more
than others' 1 or 2 marks
fuller statements, broader coverage, quality may be raised by references to examples of
both more and less affected countries, 'some more than others' part of the question
definitely addressed 3 or 4 marks
[4]
(c) (i) best sectors to shade / colour in for fossil fuel emissions are; energy supply,
manufacturing industry, transport, heating and lighting buildings
[1]
(ii) credit for stating percentages taken from the graph ; in the order of the above in (i);
25 + 19 + 19 + 8 = percentages from graph
total 71% (allow 69 74%) = 2 marks
part answers / answers based on narrow selection in (i) = 1 mark

[2]

(iii) forest clearances: deforestation and burning wood releases carbon dioxide, also
nitrogen oxides; deforestation and decomposition of vegetation releases methane.
credit also references to carbon sinks
farming: rice farming and keeping cattle are examples of types of farming which release
methane; use of chemical fertilisers releases nitrogen oxides
peoples wastes; rotting organic matter on landfill sites releases methane; burning /
incinerating wastes also releases nitrogen oxides
precise description for chosen sector = 2 marks
just named or stated for the sector = 1 mark

[1]

(iv) comment about the big size and dominance of the total percentage in the graph, with
some supporting comment about how the sectors include activities that virtually
everyone participates in / many are considered essential; only farming and forest
clearances involve little fossil fuel use
some comment / understanding shown = 1 mark
fuller understanding of fossil fuel dominance = 2 marks

[2]

935

Page 10

Mark Scheme
GCE O LEVEL October/November 2013

Syllabus
5014

Paper
11

BALOL
(d) (i) all rely upon what nature provides / all are natural sources,
all are sources that will not run out / will always be available in those areas where they
are plentiful
well stated = 2 marks
some understanding shown = 1 mark

[2]

(ii) more expensive than fossil fuels


more recent / newer technology / people not as familiar with their use since they are
natural, not all of them are available everywhere more difficult to harness, transport and
use than just burning oil etc.
unreliable output / limited efficiency
two points such as these 2 @ 1 mark

[2]

(iii) solar: photovoltaic panels for electricity, roof panels for water heating, quite widely used
in tropical countries even in some developing countries, on roofs of buildings in
developed countries, part of going green and reducing carbon dioxide emissions, for
government to meet carbon reduction targets
wave: barrages in places with high tidal range for waves to drive turbines, newer
technology of snakes / tubes on surface oscillating in the waves, at frontiers of
technology so much is experimental and restricted to a few places in developed
countries with high wave energies such as UK
geothermal: heat from the Earth, most in tectonically active places such as Iceland and
New Zealand; water forced down underground where it is heated, steam used to drive
turbines for electricity, hot water may be used in municipal heating systems; well
developed technology but areas for easy use restricted
hydro-electric: most widespread and oldest alternative; most from water storage behind
large dams on the world's big rivers driving giant turbines; plentiful examples in all
inhabited continents; makes use of rainfall and rivers, with sources in mountainous
areas, where steep terrain makes dam building easier
mark awarded will reflect:
number and precision of examples of areas / places where it is used
precision of the information about how it is harnessed and used
any other relevant information such as prospects for wider use
point mark on the basis of amount and precision of information given

[4]

(iv) most depend on new technology and bringing down the price; for example there has
been a big reduction in the price of solar panels made in China; they halved in price in
not much more than a year; more mature sources such as hydro-electric may be more
difficult to expand, likewise geothermal, because of more limited availability of new sites.
answers need to be source specific
content will depend on the chosen source effectively answered = 2 marks,
some progress = 1 mark
[2]

936

Page 11

Mark Scheme
GCE O LEVEL October/November 2013

Syllabus
5014

Paper
11

BALOL
(v) life expectancies of fossil fuels, increasing and high oil prices, but at the same time some
reluctance to give up dependence on fossil fuels, which are so useful especially in
transport; will the push to move away from fossil fuels be stronger than it is now?
optimistic or otherwise about research and development into new and cheaper ways to
harness natural sources, about humans ability to innovate, and create new technology,
especially as there is likely to be more pressure to do so
pressure from green organisations, perhaps noticeably worsening effects of climate
change, perhaps international summits which actually agree on meaningful carbon
emissions reductions
the ease and inertia of using fossil fuels cannot be overcome; there may be big oil and
gas finds in polar lands which ease the pressure to change
simple statements, limited comment directed at the question, may be just one idea that is
restated, candidate's view may not be clear = 1 or 2 marks
broader exploration of the possibilities, clear indication of the chances, answer based on
ideas from within the content in the mark scheme = 3 or 4 marks
[4]
[Total: 40]

937

BALOL
UNIVERSITY OF CAMBRIDGE INTERNATIONAL EXAMINATIONS
General Certificate of Education Ordinary Level

* 0 0 9 5 8 1 3 0 6 8 *

5014/12

ENVIRONMENTAL MANAGEMENT
Paper 1

October/November 2013
2 hours 15 minutes

Candidates answer on the Question Paper.


Additional Materials:

Ruler
Protractor

READ THESE INSTRUCTIONS FIRST


Write your Centre number, candidate number and name on all the work you hand in.
Write in dark blue or black pen.
You may use a soft pencil for any diagrams, graphs or rough working.
Do not use staples, paper clips, highlighters, glue or correction fluid.
DO NOT WRITE IN ANY BARCODES.
Answer all questions.
Electronic calculators may be used.
You may lose marks if you do not show your working or if you do not use appropriate units.
Write your answers in the spaces provided on the Question Paper.
All questions in Section A carry 10 marks.
Both questions in Section B carry 40 marks.
At the end of the examination, fasten all your work securely together.
The number of marks is given in brackets [ ] at the end of each question or
part question.

For Examiners Use


1
2
3
4
5
6
Total

938

BALOL
1

Section A

(a) Look at the graphs showing types of energy consumption in two regions of the world.

For
Examiners
Use

100
90
80
70
Key

60
percentage
of energy
consumed

coal

50

hydro-electricity
nuclear

40

natural gas
30

oil

20
10
0
South and
Central
America
(i)

Middle
East

Describe the differences in the types of energy consumption between the two
regions.
..................................................................................................................................
..................................................................................................................................
..................................................................................................................................
..................................................................................................................................
..................................................................................................................................
.............................................................................................................................. [3]

UCLES 2013

5014/12/O/N/13

939

BALOL(ii) Suggest reasons why the consumption of hydro-electric power in South and Central
America differs from that in the Middle East.

For
Examiners
Use

..................................................................................................................................
..................................................................................................................................
..................................................................................................................................
..................................................................................................................................
..................................................................................................................................
.............................................................................................................................. [3]
(b) Describe the impacts of the consumption of oil and natural gas on:
(i)

the environment
..................................................................................................................................
..................................................................................................................................
..................................................................................................................................
..................................................................................................................................
..................................................................................................................................

(ii)

the economy in the future


..................................................................................................................................
..................................................................................................................................
.............................................................................................................................. [4]
[Total: 10]

UCLES 2013

5014/12/O/N/13

[Turn over

940

4
2 BALOL
(a) Look at the graph showing changes between 1990 and 2008 in the percentages of the
populations with improved sanitation in rural and urban areas of Angola in Africa.
100

100

90

90

80

80

70

70

an
urb

60
percentage
of population
50
using improved
sanitation
40

60

30

30

20

20

percentage
of population
50
using improved
sanitation
40

rural

10
0
1990

(i)

1995

10

2000
year

2005

0
2008

State the percentage of people in rural areas of Angola with access to improved
sanitation in
1990 ...........................................%

(ii)

For
Examiners
Use

2008 ............................................%

[1]

State the difference in the percentage of the population with access to improved
sanitation in urban areas compared with rural areas in 2008.

...............................................% [1]
(iii)

Describe what the graph shows about the differences between the urban and rural
areas between 1990 and 2008.
..................................................................................................................................
..................................................................................................................................
..................................................................................................................................
.............................................................................................................................. [2]

UCLES 2013

5014/12/O/N/13

941

BALOL
(b) Describe different reasons for the difficulties of improving sanitation in:
(i)

For
Examiners
Use

rural areas
..................................................................................................................................
..................................................................................................................................
..................................................................................................................................
..................................................................................................................................

(ii)

squatter settlements on the edge of towns and cities


..................................................................................................................................
..................................................................................................................................
..................................................................................................................................
.............................................................................................................................. [3]

(c) Explain why African countries, such as Angola, have outbreaks of cholera from time to
time.
..........................................................................................................................................
..........................................................................................................................................
..........................................................................................................................................
..........................................................................................................................................
..........................................................................................................................................
...................................................................................................................................... [3]
[Total: 10]

UCLES 2013

5014/12/O/N/13

[Turn over

942

6
3 BALOL
(a) (i)

Name the instrument used to measure rainfall.


.............................................................................................................................. [1]

(ii)

For
Examiners
Use

Why should this instrument be partly buried in the ground where possible?
..................................................................................................................................
.............................................................................................................................. [1]

(iii)

Explain why this instrument is placed away from trees and buildings. Give a different
reason for each.
away from trees ........................................................................................................
..................................................................................................................................
away from buildings ..................................................................................................
.............................................................................................................................. [2]

(b) Look at the bar graph of annual rainfall for a weather station in south east Australia from
2000 to 2010. The long-term average for the weather station is 430 mm a year.
550

550

500

500

450
430

annual
rainfall
/ mm

long - term average rainfall

450
430

400

400

350

350

300
250

300 annual
rainfall
250 / mm

200

200

150

150

100

100

50

50

2000 2001 2002 2003 2004 2005 2006 2007 2008 2009 2010

year

UCLES 2013

5014/12/O/N/13

943

BALOL (i) Complete the graph by plotting the information in the table.

(ii)

year

annual rainfall / mm

2009

349

2010

540

For
Examiners
Use

[1]

Describe how the rainfall during the eleven years from 2000 to 2010 compares with
the long-term average.
..................................................................................................................................
..................................................................................................................................
..................................................................................................................................
.............................................................................................................................. [2]

(iii)

Describe the problems that livestock farmers were likely to face in south east
Australia between 2000 and 2010.
..................................................................................................................................
..................................................................................................................................
..................................................................................................................................
..................................................................................................................................
..................................................................................................................................
.............................................................................................................................. [3]
[Total: 10]

UCLES 2013

5014/12/O/N/13

[Turn over

944

8
4 BALOL
(a) Look at the diagram of part of the carbon cycle.
biosphere

For
Examiners
Use

atmosphere
1. ....................................

2. ....................................

plants

carbon
dioxide

3. ....................................

4.

5.

fossil
fuel

A ..............................................
Complete the diagram by:
(i)

naming the processes numbered 1., 2. and 3.

[3]

(ii)

naming the sphere labelled A.

[1]

(iii)

Name and describe the two processes represented by links 4. and 5.


..................................................................................................................................
..................................................................................................................................
..................................................................................................................................
..................................................................................................................................
..................................................................................................................................
.............................................................................................................................. [3]

UCLES 2013

5014/12/O/N/13

945

BALOL
(b) A number of world biosphere reserves have been set up.

For
Examiners
Use

Describe the benefits of these reserves.


..........................................................................................................................................
..........................................................................................................................................
..........................................................................................................................................
..........................................................................................................................................
..........................................................................................................................................
...................................................................................................................................... [3]
[Total: 10]

UCLES 2013

5014/12/O/N/13

[Turn over

946

10

BALOL

Section B

For
Examiners
Use

Answer both questions


5

(a) The United Nations estimated that by October 31st 2011 total world population had
reached seven billion.
Look at the graph of world population growth from 17002011.
15
human population
10
human population
/ billions
5

0
1700

1800

1900

2000

2100

year
(i)

Around which date did total world population reach 1 billion?


.............................................................................................................................. [1]

(ii)

Describe what the graph shows about population growth since 1700.
..................................................................................................................................
..................................................................................................................................
..................................................................................................................................
..................................................................................................................................
..................................................................................................................................
.............................................................................................................................. [3]

UCLES 2013

5014/12/O/N/13

947

11

BALOL
(b) Look at the graph showing estimates made in 2011 for future population growth.

For
Examiners
Use

15
human population
10
human population
/ billions

estimate A
estimate B
estimate C

0
1700

1800

1900

2000

2100

year
(i)

Describe what each of the three estimates shows about future population growth
up to 2100.
A ...............................................................................................................................
..................................................................................................................................
B ...............................................................................................................................
..................................................................................................................................
C ...............................................................................................................................
.............................................................................................................................. [3]

UCLES 2013

5014/12/O/N/13

[Turn over

948

12

BALOL(ii) Suggest reasons for the wide variations between the three estimates of future world
population growth.

For
Examiners
Use

..................................................................................................................................
..................................................................................................................................
..................................................................................................................................
..................................................................................................................................
..................................................................................................................................
.............................................................................................................................. [3]
(iii)

State which estimate, in your view, is most likely. .....................................................


Explain the reasons for your view. ............................................................................
..................................................................................................................................
.............................................................................................................................. [2]

(c) One reason for world population growth is increasing life expectancy.
Look at the graph showing examples of life expectancy in 2011 for some countries,
world regions and continents.
life expectancy 2011 by country, world region and continent

Sub-Saharan
Cambodia
Africa
Afghanistan
Africa
India
South
Africa
45

(i)

50

55

Latin America
and Caribbean
North
America
world
average
Europe

60
65
life expectancy / years

70

75

80

Japan

85

How big is the difference in years between the countries with the highest and the
lowest life expectancies?

................................................... [1]
UCLES 2013

5014/12/O/N/13

949

13

BALOL(ii) Suggest what is the most important reason for differences in life expectancy
between countries, world regions and continents like the ones shown in the graph.
Explain your answer.

For
Examiners
Use

..................................................................................................................................
..................................................................................................................................
..................................................................................................................................
.............................................................................................................................. [2]
(iii)

Describe other, additional causes of very low life expectancies (below 60) in some
countries.
..................................................................................................................................
..................................................................................................................................
..................................................................................................................................
..................................................................................................................................
..................................................................................................................................
.............................................................................................................................. [3]

UCLES 2013

5014/12/O/N/13

[Turn over

950

14

BALOL
(d) Look at the population pyramids for Nigeria (West Africa) and Japan (East Asia).

For
Examiners
Use

population structure in Nigeria and Japan (2010)


Nigeria

Japan

100 +
95 - 99
85 - 94
80 - 84
75 - 79
70 - 74
65 - 69
60 - 64
age 55 - 59
50 - 54
/ years 45 - 49
40 - 44
35 - 39
30 - 34
25 - 29
20 - 24
15 - 19
10 - 14
5-9
0-4
8

(i)

4
2
2
4
0
percentage of population
in age group

100 +
95 - 99
85 - 94
80 - 84
75 - 79
70 - 74
65 - 69
60 - 64
age 55 - 59
50 - 54
/ years 45 - 49
40 - 44
35 - 39
30 - 34
25 - 29
20 - 24
15 - 19
10 - 14
5-9
0-4
8

4
2
0
2
4
percentage of population
in age group

State two differences in shape between the population pyramids for Nigeria and
Japan.
..................................................................................................................................
..................................................................................................................................
..................................................................................................................................
.............................................................................................................................. [2]

(ii)

UCLES 2013

On the graphs, shade in:

the part of the population under 15 years old in Nigeria

the part of the population 65 years old and over in Japan.

[1]

5014/12/O/N/13

951

15

BALOL(iii) The part of the population aged between 15 and 64 years old makes an important
contribution to a country.

For
Examiners
Use

Explain what is different about this part of the population which enables them to
make an important contribution to a country, compared with the two groups you
shaded on the graph.
..................................................................................................................................
..................................................................................................................................
..................................................................................................................................
.............................................................................................................................. [2]
(iv)

What is the percentage of total population below 15 years old in Nigeria?


Circle one answer.
16%

(v)

22%

31%

42%

[1]

Describe some of the disadvantages and advantages for a country of having a


young population structure, as shown for Nigeria.
..................................................................................................................................
..................................................................................................................................
..................................................................................................................................
..................................................................................................................................
..................................................................................................................................
..................................................................................................................................
..................................................................................................................................
.............................................................................................................................. [4]

(vi)

Explain if, in your view, the disadvantages of having a young population in a country
are greater than the advantages.
your view ..................................................................................................................
explanation ...............................................................................................................
..................................................................................................................................
.............................................................................................................................. [2]

UCLES 2013

5014/12/O/N/13

[Turn over

952

16

BALOL
(e) (i) The pyramid for Japan shows that it has an ageing population. Explain what is
meant by an ageing population.

For
Examiners
Use

..................................................................................................................................
.............................................................................................................................. [1]
(ii)

The table shows information about population structure and fertility in Japan and
the UK for 2010.
population data (2010)
population structure

fertility

under 15

15 60

above 60

children per woman

birth rate per 1000

Japan

13%

64%

23%

1.3

8.2

UK

17%

60%

23%

1.8

12.2

100

Key
old, above 60

percentage

15 65
young, under 15

Complete the divided bar graphs, axes and key on the grid above, to show the
population structures for Japan and the UK.
[3]

UCLES 2013

5014/12/O/N/13

953

17

BALOL(iii) What are the main economic problems for governments in countries with ageing
populations?

For
Examiners
Use

..................................................................................................................................
..................................................................................................................................
..................................................................................................................................
..................................................................................................................................
..................................................................................................................................
.............................................................................................................................. [3]
(iv)

The population data in the table suggests that the economic problems of an ageing
population will be worse in one of the countries than in the other in 15 years time.
Which country is likely to have greater economic problems? Explain your choice.
the country likely to have greater economic problems .............................................
explanation ...............................................................................................................
..................................................................................................................................
..................................................................................................................................
..................................................................................................................................
.............................................................................................................................. [3]
[Total: 40]

UCLES 2013

5014/12/O/N/13

[Turn over

954

18
6 BALOL
(a) Look at the water cycle diagram. Letters D, E, F, G, H, J and K refer to seven water
cycle processes listed below.
condensation

evaporation

precipitation

groundwater flow

surface run-off

For
Examiners
Use

percolation

transpiration

D .........................................

K .........................................
reservoir

E .........................................

dam
lake

J .........................................

F .........................................
wate
r

table

H.........................................

saturated rock
groundwater store

G.........................................
sandstone

(i)

UCLES 2013

In the spaces on the diagram, name the seven water cycle processes shown.

[3]

5014/12/O/N/13

955

19

BALOL(ii) The position of the water table is shown on the diagram.

For
Examiners
Use

Explain why the level of the water table can be important to people.
..................................................................................................................................
..................................................................................................................................
..................................................................................................................................
.............................................................................................................................. [2]
(iii)

State the source of the water which fills the groundwater store.
.............................................................................................................................. [1]

(iv)

Explain why the location chosen for the dam on the diagram was considered to be
the best place for siting a dam and reservoir.
..................................................................................................................................
..................................................................................................................................
..................................................................................................................................
.............................................................................................................................. [2]

(b) There are at least 45,000 large dams in the world. Nearly half of the worlds largest
rivers have at least one large dam on them.
(i)

Name an example of a large dam and state its location.


..................................................................................................................................
.............................................................................................................................. [1]

UCLES 2013

5014/12/O/N/13

[Turn over

956

20

BALOL(ii) Most large dams are described as multi-purpose dams, because they have many
different uses. Look at the diagram below.

For
Examiners
Use

a large dam and some of its uses

Reservoirs can be used for


1 ....................................................
2 ....................................................

delta

In the box on the diagram, write in two different uses of reservoirs for people living
in the area around it.
[2]
(iii)

What is being made at M on the diagram?


.............................................................................................................................. [1]

(iv)

Describe how the water from the reservoir is being used at N.


..................................................................................................................................
..................................................................................................................................
.............................................................................................................................. [2]

UCLES 2013

5014/12/O/N/13

957

22

BALOL
(c) Large dams can also cause problems. Look at the diagram below.

For
Examiners
Use

a large dam and some of its problems


Q
More farming and
settlements higher on
the slopes above the dam.

P
People who lived in the
valley had to move.

R
Eroded soil dropped
in the reservoir.

S
Nutrients washed
into the reservoir.

delta

(i)

Describe some of the social and economic problems caused by the building of the
dam, for the people referred to in box P.
..................................................................................................................................
..................................................................................................................................
..................................................................................................................................
..................................................................................................................................
..................................................................................................................................
.............................................................................................................................. [3]

UCLES 2013

5014/12/O/N/13

958

23

BALOL(ii) One problem can lead to another. Explain how the problem given in box P may lead
to environmental problems caused by the changes stated in boxes Q, R and S.

For
Examiners
Use

..................................................................................................................................
..................................................................................................................................
..................................................................................................................................
..................................................................................................................................
..................................................................................................................................
.............................................................................................................................. [3]
(iii)

Explain why the usefulness of all large dams decreases over the years.
..................................................................................................................................
..................................................................................................................................
..................................................................................................................................
.............................................................................................................................. [2]

UCLES 2013

5014/12/O/N/13

[Turn over

959

24

BALOL
(d) Building large dams on rivers can also cause problems for people living further
downstream in the river delta.

For
Examiners
Use

What has happened in the Indus delta in Pakistan is one example. Over the years more
and more of the water in the river Indus and its tributaries has been trapped behind
dams in northern Pakistan. The water is taken out mainly for agriculture, but also to
supply Pakistans rapidly growing big cities.
Look at the information for a small town located in the Indus delta, comparing what it
was like in 1980 with 2010.
small town in the Indus delta of Pakistan
1980

2010

River Indus

5 km width of river channels

One channel 200 metres wide

view from a
river bridge

Dominated by water the mighty


Indus River

Dominated by sand the Indus


looks like a canal in one small river
channel

land

Fertile silt, renewed every year by


the summer floods

Without fresh silt deposits, the


sea is eroding the delta land and
many soils are now brackish (salt
affected) and useless for farming

economy

Main income from river fishing and


shrimp collecting

River fishing has collapsed; the few


fishermen still working go out to
sea to catch crabs

Also farming bananas, coconuts,


grapes using irrigation water
pumped from the Indus

Collecting firewood is now more


important than farming; river levels
have dropped so low that the old
water pumps cannot work

15 000

3000

population
(i)

Describe the environmental and economic problems caused in this delta town by
so much water being taken out of the River Indus in northern Pakistan.
environmental ............................................................................................................
..................................................................................................................................
..................................................................................................................................
..................................................................................................................................
economic ...................................................................................................................
..................................................................................................................................
..................................................................................................................................
.............................................................................................................................. [4]

UCLES 2013

5014/12/O/N/13

960

25

BALOL(ii) What is the evidence for out-migration from this delta town?
.............................................................................................................................. [1]
(iii)

For
Examiners
Use

Push and pull factors can be used to explain why people migrate from rural to
urban areas in developing countries such as Pakistan.
To explain migration from this town in the Indus delta, which is likely to be stronger
push factors or pull factors? Explain your view.
..................................................................................................................................
..................................................................................................................................
..................................................................................................................................
..................................................................................................................................
..................................................................................................................................
.............................................................................................................................. [3]

(iv)

Suggest how likely it is that the towns population will continue to decline after 2010.
Explain your answer.
..................................................................................................................................
..................................................................................................................................
..................................................................................................................................
.............................................................................................................................. [2]

(e) Most people who migrate from rural areas in Pakistan go to the big cities, such as
Karachi. Karachi is the worlds 10th biggest city with at least 13 million inhabitants. It has
many urban problems, including housing.
(i)

Describe the housing problems found in most big cities of the developing world.
..................................................................................................................................
..................................................................................................................................
..................................................................................................................................
..................................................................................................................................
..................................................................................................................................
.............................................................................................................................. [3]

UCLES 2013

5014/12/O/N/13

[Turn over

961

26

BALOL(ii) Choose one city in the developing world. What strategies are being used to manage
its housing problems and how successful have they been?

For
Examiners
Use

name of city ...............................................................................................................


strategies and their success ......................................................................................
..................................................................................................................................
..................................................................................................................................
..................................................................................................................................
..................................................................................................................................
..................................................................................................................................
..................................................................................................................................
..................................................................................................................................
..................................................................................................................................
.............................................................................................................................. [5]
[Total: 40]

UCLES 2013

5014/12/O/N/13

962

BALOL
CAMBRIDGE INTERNATIONAL EXAMINATIONS
GCE Ordinary Level

MARK SCHEME for the October/November 2013 series

5014 ENVIRONMENTAL MANAGEMENT


5014/12

Paper 1 maximum raw mark 120

This mark scheme is published as an aid to teachers and candidates, to indicate the requirements of
the examination. It shows the basis on which Examiners were instructed to award marks. It does not
indicate the details of the discussions that took place at an Examiners meeting before marking began,
which would have considered the acceptability of alternative answers.
Mark schemes should be read in conjunction with the question paper and the Principal Examiner
Report for Teachers.

Cambridge will not enter into discussions about these mark schemes.

Cambridge is publishing the mark schemes for the October/November 2013 series for most IGCSE,
GCE Advanced Level and Advanced Subsidiary Level components and some Ordinary Level
components.

963

Page 2

Mark Scheme
GCE O LEVEL October/November 2013

Syllabus
5014

Paper
12

BALOL
Section A
1

(a) (i) more types of energy used in SCA (5) than in ME (4),
nuclear only in SCA,
(virtually) all fossil fuel use in ME,
a little more oil used in ME,
but much more natural gas in ME than in SCA,
more coal / HEP use in SCA
use of values which support a valid difference
3 @ 1 mark

[3]

(ii) general reference to hydro-electric power needing specific physical conditions,


further details about these such as plentiful rainfall, big river, steep slopes,
comment about why more likely to exist in SCA than in ME / less likely in ME,
great quantities of oil and gas obtained in ME,
cheaper and easier to use local / one's own natural resources
3 @ 1 mark

[3]

(b) (i) environmental impacts:


air pollution / dirty air results from release of smoke particles,
carbon dioxide emissions increase the greenhouse effect,
sulfur dioxide / oxides of nitrogen cause acid rain
further environmental consequences of any of these,
land destruction /loss of habitats during mining,
pollution from oil spills during transport
min. 1 mark, max. 3 marks
(ii) economy in the future:
exhaustion of finite / non-renewable resources,
industries reliant on them will decline / close down / loss of export income,
alternatives are likely to be more expensive,
also positive view about oil and gas deposits leading to economic development / income
from exporting
min. 1 mark, max. 2 marks
4 @ 1 mark

[4]
[Total: 10]

964

Page 3

Mark Scheme
GCE O LEVEL October/November 2013

Syllabus
5014

Paper
12

BALOL
2

(a) (i) 1990: 6% 2008: 18%


(ii) 68%

[1]
[1]

(iii) little change in the size of the big gap between them up to 1995,
since 1995 the size of the difference between them has been increasing,
showing more marked improvements in urban than in rural areas,
12% increase in rural sanitation compared with 28% for urban
2 @ 1 mark

[2]

(b) (i) rural areas:


remoteness / away from political centre of country,
population more spread out / difficulties of access ,
leading to increased / high costs of improved sanitation,
shortage of water supplies in the area / water needed for other purposes e.g. irrigation,
lack of infrastructure / materials / trained people in rural areas
min. 1 mark, max. 2 marks
(ii) urban squatter settlements:
great size and scale of the problem / problem of numbers,
problem of ever growing numbers / massive and continuing urban growth,
illegal settlements / lack of title to the land / locations keep changing
min. 1 mark, max. 2 marks
3 @ 1 mark

[3]

(c) lack of sanitation means that rivers / lakes are contaminated with dirty water,
cholera is a water-borne disease,
caught by people drinking / washing etc. in contaminated water,
disease easily spreads causing epidemics,
often associated with aftermath of natural / human disasters,
absence of medical facilities to control its spread
three points like these 3 @ 1 mark

[3]
[Total: 10]

965

Page 4

Mark Scheme
GCE O LEVEL October/November 2013

Syllabus
5014

Paper
12

BALOL
3

(a) (i) rain gauge

[1]

(ii) to keep it upright / for stability / reduce water loss from evaporation

[1]

(iii) away from trees; trees can intercept or block rainfall / dripping rainwater can increase
amount
away from buildings; to prevent shelter so that rainfall amount is under-recorded / stop
excess rainfall or rainwater coming off buildings
2 @ 1 mark

[2]

(b) (i) accurate plots for both height and width

[1]

(ii) period of mainly below average rainfall,


in only two of the eleven years (2000 and 2010) was rainfall above average,
greatest different from the average in 2002 (under 270 mm, about 160 mm less)
2 @ 1 mark

[2]

(iii) poor / insufficient pastures in dry years,


several drier than average years together reduce pastures even more,
can lead to overgrazing and soil erosion,
quality and worth of livestock decreases,
forced to sell or kill animals / move animals,
increased expense of needing to buy in additional feed / water for pastures,
dangers of floods in years such as 2010 from excess rains
3 @ 1 mark

[3]
[Total: 10]

(a) (i) 1: photosynthesis


2 and 3: (either order)
respiration / decay / decomposition
combustion / burning of wood / plants (releasing carbon dioxide to the air)
(ii) A Lithosphere

[3]
[1]

(iii) 4: fossilisation of plants as they are buried and turned into carbon, coal and oil
5: is release of carbon dioxide when fossil fuels are burnt
1 mark for each of 4 and 5 = 2 marks
3rd mark for elaboration or precision of the description, most likely for 4 = 1 mark

[3]

966

Page 5

Mark Scheme
GCE O LEVEL October/November 2013

Syllabus
5014

Paper
12

BALOL
(b) protection / conservation of ecosystems / preserving habitats for wildlife,
maintaining biodiversity,
preserving genes for the future,
encourages sustainable use of natural places / eco-tourism,
allows for research / monitoring of environmental issues,
provide education / training about environmental issues
3 @ 1 mark

[3]
[Total: 10]
Section B

(a) (i) 1800

[1]

(ii) persistent but steady increase until 1900, noticeable increase in rate / speed of
population growth between 1900 and 1950,
population take-off from 1950/60 onwards, this very rapid growth continued to 2011 to
take the population to 7 billion,
use of values which support acceptable statements such as from 3 to 7 billion from 1960
to 2011 / more the doubled in 50 years
3 @ 1 mark

[3]

(b) (i) A continuation of current high rates of growth / no let-up in high rates of growth, taking
total population to 15 billion
B continued high rates of growth until about 2050/60, then signs of a slow down so that
the population only reaches 10 billion by 2100
C population peaks around 2030/40 and then goes into decline so that it falls below the
current value / down to 6 billion by 2100.
totals in 2100 given without any supporting description = 1 mark
description of lines without reference to population totals, or
accurate totals with limited descriptive support = max. 2 marks
mixture of description and use of totals = up to 3 marks

[3]

(ii) general point(s) made about them all being estimates and nothing more, no one really
knows what is going to happen to fertility rates in future;
more specific points about the assumptions on which the three estimates are based; A
based on the assumption that the trend of high rates of natural increase seen from 1960
will carry on throughout the next 90 years
B based on the knowledge that there are already signs in some developing countries of
family size coming down, especially those making progress with economic development,
much in the same way that happened previously in developed countries (hence the
stages in the Demographic Transition Model),
C relies upon some change / catastrophe causing a great increase in death rates, such
as crop failures, massive droughts, significant climate change or new infections in the
way that AIDS / HIV once threatened to increase mortality rates significantly

967

Page 6

Mark Scheme
GCE O LEVEL October/November 2013

Syllabus
5014

Paper
12

BALOL
mark on the basis of understanding shown and coverage; simple statements showing
some understanding, perhaps more general than specific, or concentrating on only one
of the estimates without a broader understanding being shown 1 or 2 marks
clear understanding about what these population estimates are showing and that
estimates / opinions vary 3 marks
[3]
(iii) B: because it is more closely related to what has been seen to be happening already; in
global terms fertility rates / birth rates are coming down, and there is even natural
decrease in a few countries in Europe
A: optimistic assumptions about increases in world food supply and technological
breakthroughs to allow so many people to be supported on the Earth's surface by its
natural resources
C: suggestions would be needed for what will force the reversal of all recent previous
trends disasters could be tectonic, climatic, disease related for either crops or people or
both, exhaustion of natural resources.
no marks for choice; both marks for explaining why
(c) (i) 40 years

[2]
[1]

(ii) improved medical treatment; examples of primary and secondary healthcare can be
given to support this, leading to a great reduction in death rates worldwide.
1 mark for choice, 2nd mark for some further explanation
wealth/poverty; medical treatment.

[2]

(iii) if not used in (ii), wealth and poverty which control access to medical care; also wealth
increases access to better diet, clean water and sanitation, and improved living
conditions; where these do not exist, notably in rural areas of poor developing countries,
death rates tend to be high
other factors to explain low life expectancies include:
wars / civil & political unrest, which disrupt normal living, faming, access to clean water
supplies, make worse the effects of natural disasters; e.g. Somalia with civil wars and
drought
diseases for which treatment is not available or cannot be afforded, such as HIV/AIDS,
which has greatly reduced life expectancies in many southern African countries such as
Botswana and South Africa.
special factors such as high rates of alcohol consumption in Russia and some of the
other old Soviet Republics
Points made along these lines; references needed to at least a second cause for all
three marks
[3]

968

Page 7

Mark Scheme
GCE O LEVEL October/November 2013

Syllabus
5014

Paper
12

BALOL
(d) (i) Japan's pyramid is taller / Nigeria's is shorter,
Japan's is more straight up and down / rectangular overall / bottom and top well
represented, Nigeria's is more of a pyramid / has a wide base and narrow top
Descriptions such as these about overall shape, as opposed to variations within the
shape;
two valid differences in shape 2 @ 1 mark
(ii) both males and females needed to be shaded in both graphs.

[2]
[1]

(iii) old and young are the non-working population / they are dependants,
1564 year olds are working population / the independent population,
they provide the revenue / pay taxes to pay for pensions and schools for 65+ and under
15.
some understanding = 1 mark
clearer and fuller understanding of the difference = 2 marks
(iv) 42%

[2]
[1]

(v) disadvantages: young are expensive in terms of their needs for public services, such as
health and education
young people grow up and marry / reach child bearing age; large numbers of them
mean that population increase is likely to continue in the country for many more years,
increasing the pressure of numbers
problem of numbers puts a great strain on a country's economic resources; food output,
services and industry need to increase just to keep the country and its people on the
same economic level as previously; in rural areas large numbers of young people to
feed increases pressure on the land and natural environment
maintains the poverty trap among poor people in rural areas
advantages: today's young are tomorrow's human resources for a country; their effort
and energy will be needed for future economic development; young are more likely to be
willing to change, and adopt new technology as it becomes available; many a country
benefits from remittances from young people reaching working age and going overseas
to work; also possible that might allow a country's empty areas to be settled and their
resources developed
minimum of 1 mark reserved for advantages and disadvantages
otherwise 4 @ 1 mark.

[4]

(vi) disadvantage: finite resources of a country need to be shared between greater numbers
of people; mature population structures in economically developed countries mean a
larger share of the economic cake for all their people and therefore greater wealth
advantage: need for young people to generate the future wealth of a country; a big
problem in developed countries is not enough children to support their ageing
populations by creating the wealth and paying the taxes needed for state pensions;
some governments still believe that big total population numbers increase a country's
international status and power e.g. the big populations in China and India mean that they
cannot be ignored internationally
no mark for the view up to 2 marks for how well the view is supported.

[2]

969

Page 8

Mark Scheme
GCE O LEVEL October/November 2013

Syllabus
5014

Paper
12

BALOL
(e) (i) increasing percentage of old people (65+) in a country/
idea of large numbers / proportion must be included

[1]

(ii) bars divided up correctly for stated percentages 2 @ 1 mark


sectors identified in the bars and key 1 mark

[3]

(iii) tradition in developed countries that people are given pensions once they retire from
work; many pensions are paid by governments; money that governments have mainly
comes from taxes; as the percentage of retired people increases, the percentage of
those working and paying taxes decreases i.e. less money is coming when more needs
to be paid out. health and other care costs increase as people get older and live longer;
for many old people this means reliance on government provision; forecasts about future
population structures in Japan and Europe suggest that future economic problems
are set to worsen;
3 points made along these lines 3 @ 1 mark
[3]
(iv) candidates could sit on a fence, or even argue in favour of the UK, from the point of view
of population structure in 2010; both have the same percentages above 60 and the UK
has 4% fewer of its population of working age
maximum 1 mark for this
the key evidence which supports the answer Japan is the much lower fertility and birth
rates, also lower percentage of under 15s already in 2010 (4% lower); population data
suggests a progressive decrease in the percentage of potentially economically
productive 1560 year olds during the next 15 years, who are the key economic
providers for government costs to support old people
credit comment along these lines in the terms of the future, as well as use of the data in
support of the answer
up to 3 marks for this
[3]
[Total: 40]
6

(a) (i) D condensation


G groundwater flow
J transpiration
all 7 correct = 3 marks
46 correct = 2 marks
23 correct = 1 mark

E precipitation
H surface run-off

F percolation
I evaporation

[3]

(ii) rocks below level of water table are saturated,


may be useful for underground / groundwater sources of water such as aquifers / wells,
supplies clean water for drinking / fresh water for irrigation,
if it touches the surface spring water may reach the surface naturally,
if surface is saturated in lowland areas may lead to surface flooding,
adverse consequences of this for people
2 @ 1 mark

[2]

970

Page 9

Mark Scheme
GCE O LEVEL October/November 2013

Syllabus
5014

Paper
12

BALOL
(iii) rainfall /precipitation

[1]

(iv) steep sided / narrow / mountain valley,


explanation why this is better for dam building and reservoir construction,
steep slopes around it for fast surface run-off of rainfall,
mountain references in terms of higher precipitation / melting snow
two explanatory points 2 @ 1 mark

[2]

(b) (i) name of dam and its location, anywhere in world. Most likely examples include Aswan
High Dam in Egypt, one of the large dams in the west of the USA such as the Hoover
Dam, large dams on the upper Indus in Pakistan such as Tarbela Dam, Three Gorges
dam on the Yangtse in China.
[1]
(ii) source of fresh water supplies / domestic uses,
recreational activities such as sailing / tourist activities,
food source from fishing.
two uses 2 @ 1 mark
(iii) hydro-electric power / electricity

[2]
[1]

(iv) for irrigation / growing crops,


with further details about it such as reference to the canal and regular pattern
of water channels leading the water into the fields
2 @ 1 mark

[2]

(c) (i) lost their farmland and thus their source of income / livelihoods,
often given replacement land elsewhere which is of inferior quality,
land on valley floors is often the flattest and most fertile land in the valley,
living community of people broken up / destroyed / forced to move,
may be forced to move a long distance from known people / places
minimum of 1 mark reserved for each of economic and social problems
otherwise 3 points made along these lines 3 @ 1 mark

[3]

(ii) loss of land in valley floor / on lower slopes will lead to greater use of land higher up the
valley sides, this is steeper / less productive land naturally more at risk from soil erosion,
pressure of people looking for farmland in the same area will lead to over-use, overcultivation / overgrazing / clearance of vegetation all speed up soil erosion, soil sediment
/ pollution waste / fertilisers will enter mountain streams and be transferred downslope to
the reservoir.
three points such as these showing understanding of the possible knock-on effects
3 @ 1 mark

[3]

971

Page 10

Mark Scheme
GCE O LEVEL October/November 2013

Syllabus
5014

Paper
12

BALOL
(iii) the clue to answering is given in boxes R & S about the dam being a solid wall traps all
the sediment behind it, whereas previously it was progressively carried downstream by
the river and deposited in the sea. The dams interfere with natural river processes.
Accumulation of sediments behind the dam, resulting reductions with time in the volume
of water that can be held in the reservoir, life expectancy reduced even faster if
catchment areas for the streams are over-used by people leading to increased surface
run-off.
understood (however expressed) and well explained = 2 marks
some understanding, but probably with a great dependence on the content of boxes
R & S= 1 mark
[2]
(d) (i) environmental:
many river channels / water everywhere (total width 5 km) in 1980, compared with just
one water channel in 2010 (only 300 m wide, many times less); water replaced by
deposits of sand where river channels used to be; soil characteristics have changed; silt
soils, kept fertile by constant renewal, have been replaced by brackish soils; falling water
levels in the soil.
economic:
livelihoods of delta dwellers have disappeared; no more income from river fishing; only
chance is to go out to sea to fish; water no longer available for crop farming, not enough
river water and water levels too low for pumps to work; most crops will not grow in salty
soils; forced into less profitable alternatives such as collecting firewood
marking:
incomplete / part answer; relies on direct use made of information given in the table;
content selected not always successfully arranged under the headings environmental
and economic; 1 or 2 marks
complete answer in terms of use of what is in the table and correctly arranged under the
headings environmental and economic; 3 marks
not only complete, but better described in terms of using headings or language which
helps to emphasis the scale and range of the problems caused; 4 marks
[4]
(ii) population decrease (from 15 000 to 3000)

[1]

(iii) the clearest push factor is the loss of employment with the disappearance of jobs in river
fishing, and difficulties if farming after the loss in water supplies, future for farming looks
gloomy after loss of regular fertile silt deposits and soils being affected by higher salt
content, collecting firewood is a sign of how bad things are / not likely to be a sustainable
activity and all the consequences that follow from loss of livelihoods
there are always going to be the pull factors of the cities which can offer things that the
countryside cannot such as better services (education / medical), more chances of
electricity and access to clean water, more variety and greater range of job types
general answers or answers limited to only one push factor = 1 mark
answers specific to this example including fuller references to push factors = 2 marks
answers which support the view that push factors are likely to be dominant in this
example, backed up strongly or in a balanced way with comment; comment may also
refer to increasing attractiveness of the city = 3 marks
[3]

972

Page 11

Mark Scheme
GCE O LEVEL October/November 2013

Syllabus
5014

Paper
12

BALOL
(iv) the easiest view to support is highly likely due to further knock-on effects such as
deteriorating soils, falling water supplies, over-use of firewood resources
also since the dams are in place much further north outside the region there doesn't
seems much chance that more water will be released into the Indus seems more
chance that there might be even less
once the community disappears, more likely to leave; some will now have relatives and
friends in the cities to make the move easier; community itself likely to be ageing after
people of working age have migrated
support for unlikely to decline further is less obvious; but the 3000 might represent the
number that can be supported by current resources and activities like sea fishing and
collecting firewood; all those with any desire to migrate have already done so
view expressed but weakly supported = 1 mark
view expressed and more strongly supported = 2 marks

[2]

(e) (i) the obvious focus is on squatter settlements / slum housing / shanty towns, self-help
housing using mixed 'building materials', high density / haphazard layout, lacking in
essential public services such as water supply, overcrowded / easy spread of disease,
located in inferior locations not already used for formal housing.
possible also to refer to the high costs of modern housing such as apartment blocks
supposedly built for the poor;
points made along these lines; credit any references to specific examples;
3 @ 1 mark

[3]

973

Page 12

Mark Scheme
GCE O LEVEL October/November 2013

Syllabus
5014

Paper
12

BALOL
(ii) example from a big city in the country where the candidate lives, examples such as
Chennai and the Housing and Slum Clearance Boards, the Orangi Pilot Project in
Karachi involving the communities in self-help schemes, and the 'new settlement' policy
in Greater Cairo
most strategies for improving housing are of the ASH (aided self-help) type, giving legal
titles to the land, providing essential services, offering loans and providing technical help.
Less successful are those that involve clearance and housing redevelopment (usually
high-rise) in another part of the city, often some distance away. Even worse of bulldozing
without any new provision
answer about strategies for managing housing problems; many will be general in nature
and weakly related to a city even when one is named
maximum 3 marks for these answers, with the mark within the range dependent on the
dependent on amount given or range of points made; general answers
answers that contain direct and identifiable references to a named city
up to 5 marks with adequate supporting detail; city specific answers

[5]
[Total: 40]

974

Cambridge International Examinations


Cambridge Ordinary Level

5014/11

ENVIRONMENTAL MANAGEMENT
Paper 1

May/June 2014

INSERT
2 hours 15 minutes

This Insert contains the Photograph needed for Question 6(c).


DO NOT WRITE ON THIS INSERT.

www.onlineexamhelp.com

*5252131643-I*

READ THESE INSTRUCTIONS FIRST

This document consists of 2 printed pages.


DC (AC/KN) 91289/1
UCLES 2014

[Turn over

www.onlineexamhelp.com

www.onlineexamhelp.com

Photograph for Question 6(c)

Copyright Acknowledgements:
Question 6c

John Pallister UCLES

Permission to reproduce items where third-party owned material protected by copyright is included has been sought and cleared where possible. Every
reasonable effort has been made by the publisher (UCLES) to trace copyright holders, but if any items requiring clearance have unwittingly been included, the
publisher will be pleased to make amends at the earliest possible opportunity.
Cambridge International Examinations is part of the Cambridge Assessment Group. Cambridge Assessment is the brand name of University of Cambridge Local
Examinations Syndicate (UCLES), which is itself a department of the University of Cambridge.

UCLES 2014

5014/11/INSERT/M/J/14

www.onlineexamhelp.com

Cambridge International Examinations


Cambridge Ordinary Level

Paper 1

May/June 2014
2 hours 15 minutes

Candidates answer on the Question Paper.


Additional Materials:

Ruler
Insert

READ THESE INSTRUCTIONS FIRST


www.onlineexamhelp.com

* 5 2 5 2 1 3 1 6 4 3 *

5014/11

ENVIRONMENTAL MANAGEMENT

Write your Centre number, candidate number and name on all the work you hand in.
Write in dark blue or black pen.
You may use an HB pencil for any diagrams or graphs.
Do not use staples, paper clips, glue or correction fluid.
DO NOT WRITE IN ANY BARCODES.
Answer all questions.
Electronic calculators may be used.
You may lose marks if you do not show your working or if you do not use appropriate units.
Write your answers in the spaces provided on the Question Paper.
All questions in Section A carry 10 marks.
Both questions in Section B carry 40 marks.
At the end of the examination, fasten all your work securely together.
The number of marks is given in brackets [ ] at the end of each question or part question.

This document consists of 23 printed pages, 1 blank page and 1 insert.


DC (SJF/CGW) 80754/3
UCLES 2014

[Turn over

www.onlineexamhelp.com

2
Section A
Answer all the questions.

(a) Look at the diagrams of the mineral content of three different soils, A, B and C.
soil A

soil B

grains
0.02 0.05 mm
in diameter

soil C

flat particles
too small to be
visible

grains
0.05 2.00 mm
in diameter

(not to scale)
(i)

Place a tick in the table below for the correct answer to the following questions. The first
has been done for you as an example.
Which of the three soils
A

has the largest particles?


would be the most waterlogged after rain?
will dry fastest after rain?
is the easiest for plant roots to grow into?
(ii)

[3]

Identify the three soils shown in the diagrams. Choose from clayey, sandy, silty.
A ..........................................................................
B ..........................................................................
C ..........................................................................

UCLES 2014

5014/11/M/J/14

[2]

3
(b) Give one reason why each of the following is important for good soil:
(i)

organic content ..................................................................................................................


...........................................................................................................................................
.......................................................................................................................................[1]

(ii)

soil organisms ...................................................................................................................


...........................................................................................................................................
.......................................................................................................................................[1]

(iii)

air ......................................................................................................................................
...........................................................................................................................................
.......................................................................................................................................[1]

(iv)

nutrients ............................................................................................................................
...........................................................................................................................................
.......................................................................................................................................[1]

(c) Look at the diagrams below of soils D and E.


Use evidence in the diagrams to suggest why soil D may be less suitable for plant growth than
soil E.
soil D

soil E
soil

soil

rock

rock

...................................................................................................................................................
...............................................................................................................................................[1]
[Total: 10]

UCLES 2014

5014/11/M/J/14

[Turn over

4
(a) Look at the map which shows the worlds largest marine protected areas (MPAs).
Countries that used to send fishing boats to the Chagos Islands MPA before fishing was
banned, are also shown.

France

Spain
Papahnaumokuakea
362 074
Atlantic
Ocean
Pacific
Ocean

Japan
Mariana
Taiwan Trench
246 608
Pacific Ocean
Chagos
Islands
545 000

Phoenix Islands
408 250
Motu
Matiro Hiva
150 000

Indian Ocean

Prince Edward Islands


180 000

Great
Barrier
Reef
345 400

Macquarie Island
162 000

Key
marine protected area (MPA)
362 074

area of MPA in kilometres square

France

country fishing in the Chagos Islands

(i)

Name the worlds largest MPA.


.......................................................................................................................................[1]

(ii)

How many of the eight largest MPAs are in the Pacific Ocean?
.......................................................................................................................................[1]

(iii)

Using the map, suggest what is common about the locations of the four countries that
send fishing vessels to the Chagos Islands MPA.
...........................................................................................................................................
.......................................................................................................................................[1]

UCLES 2014

5014/11/M/J/14

www.onlineexamhelp.com

www.onlineexamhelp.com

5
(b) In the past, the annual catch of tuna fish taken from the Chagos Islands to Spain and France
varied from less than 100 tonnes to about 24 000 tonnes.
Suggest reasons why annual fish catches vary.
...................................................................................................................................................
...................................................................................................................................................
...................................................................................................................................................
...................................................................................................................................................
...................................................................................................................................................
...................................................................................................................................................
...................................................................................................................................................
...............................................................................................................................................[4]
(c) Explain why areas where fishing is banned, have benefits for the environment but
disadvantages for people.
benefits for the environment .....................................................................................................
...................................................................................................................................................
...................................................................................................................................................
...................................................................................................................................................
disadvantages for people ..........................................................................................................
...................................................................................................................................................
...................................................................................................................................................
...............................................................................................................................................[3]
[Total: 10]

UCLES 2014

5014/11/M/J/14

[Turn over

6
3

(a) Look at the photograph of an instrument that records an element of weather.

A
B
C

(i)

Name the weather instrument shown on the photograph.


.......................................................................................................................................[1]

(ii)

From which direction was the wind blowing when the photograph was taken?
Circle one answer.
north

(iii)

south

east

west

[1]

Describe the function of the three parts that make up the weather instrument shown in
the photograph.
A ........................................................................................................................................
B ........................................................................................................................................
C ....................................................................................................................................[3]

(iv)

Describe the position of this weather instrument.


...........................................................................................................................................
.......................................................................................................................................[1]

UCLES 2014

5014/11/M/J/14

7
(v)

Suggest why the position of this weather instrument would give correct readings of the
wind direction.
...........................................................................................................................................
.......................................................................................................................................[1]

(b) The wind rose below shows wind directions for most of a month in a temperate area.
N

Key
0 1 2

days
calm

(i)

Complete the wind rose to show that four days had winds from the east.

[1]

(ii)

Suggest how this information could be used by farmers to position windbreaks.


...........................................................................................................................................
...........................................................................................................................................
...........................................................................................................................................
.......................................................................................................................................[2]
[Total: 10]

UCLES 2014

5014/11/M/J/14

[Turn over

8
4

(a) Three types of soil preparation for crops are used in a country.
type of soil preparation

description of the method

normal tilling

ploughing breaks up the soil

mulch tilling

a dry land farming method where the ploughed soil is


covered with a mulch

no tilling

a dry land farming method where crops are sown or


planted in soil that has not been ploughed

Look at the graph showing how the use of these methods changed over a ten year period.
40

n orm
al
t

percentage of planted hectares

35

i ll

30

ti
lch
u
m

25

ll

20
no

15

till

10
5
0

10

years
(i)

What percentage of planted hectares was prepared by normal tilling in year 4?


...................................................... % [1]

(ii)

Describe the trends in normal tilling over the 10 year period.


...........................................................................................................................................
...........................................................................................................................................
...........................................................................................................................................
.......................................................................................................................................[2]

(iii)

If the trends shown on the graph continued, which method would be most likely to have
been used in year 12 to prepare the most land?
.......................................................................................................................................[1]

UCLES 2014

5014/11/M/J/14

9
(b) Suggest why the recent trend in normal tilling may have been influenced by:
(i)

the increased cost of energy


...........................................................................................................................................
.......................................................................................................................................[1]

(ii)

a series of drier years


...........................................................................................................................................
.......................................................................................................................................[1]

(c) The no till and mulch till methods leave at least 30% of the soil covered. Explain why this
helps to prevent soil erosion.
...................................................................................................................................................
...................................................................................................................................................
...................................................................................................................................................
...................................................................................................................................................
...................................................................................................................................................
...................................................................................................................................................
...................................................................................................................................................
...............................................................................................................................................[3]
(d) Describe one method used to reduce soil erosion by water.
...................................................................................................................................................
...............................................................................................................................................[1]
[Total: 10]

UCLES 2014

5014/11/M/J/14

[Turn over

10
Section B
Answer both questions.
5

(a) Very large cities with populations over 10 million are called mega-cities. Look at the maps
showing the distribution of mega-cities in 1975 and 2010.
mega-cities in 1975
North
America

Europe

Asia

Africa
Latin
America

Australasia

mega-cities in 2010

(i)

Look at the table giving a summary of the distribution of mega-cities in 1975.

date

continents

total

ratio

Europe

Africa

Asia

Australasia
(Oceania)

North
America

Latin
America

1975

2010

......

......

......

......

......

......

20

......

......

Complete the table using the map for 2010.

UCLES 2014

5014/11/M/J/14

developing : developed
countries
countries

[3]

11
(ii)

When comparing the world map of mega-cities in 2010 with that in 1975, three important
differences can be seen.
One difference is stated below. State two other important differences.

(iii)

Mega-cities are increasingly concentrated in developing countries.

....................................................................................................................................

....................................................................................................................................
[2]

One cause of the increasing concentration of mega-cities in developing countries is the


higher rate of natural increase of population than in developed countries.
State and explain two different reasons for the higher rate of natural increase in
developing countries.
1 ........................................................................................................................................
...........................................................................................................................................
...........................................................................................................................................
...........................................................................................................................................
2 ........................................................................................................................................
...........................................................................................................................................
...........................................................................................................................................
...........................................................................................................................................
[4]

(iv)

Why are rates of rural to urban migration much higher in developing countries than in
developed countries?
...........................................................................................................................................
...........................................................................................................................................
...........................................................................................................................................
.......................................................................................................................................[2]

UCLES 2014

5014/11/M/J/14

[Turn over

12
(b) A survey was carried out among people living in an informal settlement (shanty town)
in Mexico City. These people had recently migrated from rural areas and were asked the
following question. What was the main reason why you migrated from the countryside?
The results are shown below.
reason

(i)

percentage (%) of
total answers

to find work

33

to live near relatives

17

for more and better public services

12

to get away from poverty

10

food shortages

better chance of schooling for the children

other reasons

12

Use the percentages to complete and label the divided bar graph below.
summary of answers

better schooling

10

20

30

40

50

60

70

80

90

100

Key
other
reasons
[3]
(ii)

List the letters on the lines below to show which are the push and which are the pull
factors.
push factors
...........................................................................................................................................
pull factors
...........................................................................................................................................
[3]

UCLES 2014

5014/11/M/J/14

13
(iii)

State a different reason that some migrants might have given, which could have been
included among the answers for other reasons.
...........................................................................................................................................
.......................................................................................................................................[1]

(iv)

What do the answers given by these migrants suggest about the relative importance of
push and pull factors when they decided to migrate from the countryside to the city? Use
the data in the table to support your answer.
...........................................................................................................................................
...........................................................................................................................................
...........................................................................................................................................
.......................................................................................................................................[2]

(c) Many recent migrants from rural to urban areas live in poor housing areas in the big cities of
developing countries (known as informal settlements, shanty towns or squatter settlements).
Kibera in Nairobi, the capital city of Kenya, is an example of a shanty town.
Read the information about Kibera.
Kibera is home to between 700 000 and 1 million people. It is the largest shanty town in sub-Saharan
Africa. Crowded, noisy and polluted, most families lack even the most basic amenities.
Even so, it is a place of great activity, full of people on the move, working, selling goods on the
street, sorting through rubbish and looking after livestock.
(i)

Name an example of a basic amenity (service) that is likely to be lacking in shanty town
areas like Kibera.
.......................................................................................................................................[1]

(ii)

Explain why the lack of this amenity can have serious consequences for the people in
the shanty town.
...........................................................................................................................................
...........................................................................................................................................
.......................................................................................................................................[2]

UCLES 2014

5014/11/M/J/14

[Turn over

14
(iii)

Use your knowledge of shanty towns in cities of developing countries to describe fully:
1

characteristic features of the houses in which most people live

...........................................................................................................................................
...........................................................................................................................................
...........................................................................................................................................
...........................................................................................................................................
2

types of places where shanty towns are located

...........................................................................................................................................
...........................................................................................................................................
...........................................................................................................................................
...........................................................................................................................................
[4]
(iv)

Suggest how people living in shanty towns might improve their standard of living.
...........................................................................................................................................
...........................................................................................................................................
...........................................................................................................................................
.......................................................................................................................................[2]

(v)

Suggest how city authorities can improve shanty towns.


Explain your answer as fully as you can.
...........................................................................................................................................
...........................................................................................................................................
...........................................................................................................................................
...........................................................................................................................................
...........................................................................................................................................
...........................................................................................................................................
...........................................................................................................................................
.......................................................................................................................................[4]

UCLES 2014

5014/11/M/J/14

15
(d) (i)

Poor housing is just one of the many problems in the worlds big cities.
Look at the spider diagram showing some problems of big cities.
big city problems
poor housing

pollution

slum houses packed together,

.................................................

most lacking basic amenities

.................................................

big city
problems

congestion

loss of agricultural land

.................................................

.................................................

.................................................

.................................................

In the spaces on the spider diagram, write short descriptions about congestion, pollution
and loss of agricultural land.
[3]
(ii)

Suggest reasons why these urban problems tend to be worse in cities in developing
countries rather than developed countries.
...........................................................................................................................................
...........................................................................................................................................
...........................................................................................................................................
...........................................................................................................................................
...........................................................................................................................................
...........................................................................................................................................
...........................................................................................................................................
.......................................................................................................................................[4]
[Total: 40]

UCLES 2014

5014/11/M/J/14

[Turn over

16
6

(a) Look at the world map showing the location of land with a tundra climate.
location of land with a tundra climate
Greenland

60 N

23.5 N
0
23.5 S

60 S
Key
tundra climate
(i)

Describe the location of land with a tundra climate in the northern hemisphere.
...........................................................................................................................................
...........................................................................................................................................
...........................................................................................................................................
...........................................................................................................................................
.......................................................................................................................................[3]

(ii)

Why are there no large areas of tundra in the southern hemisphere?


...........................................................................................................................................
.......................................................................................................................................[1]

UCLES 2014

5014/11/M/J/14

17
(b) The climate graph below is for the small coastal settlement of Qaanaaq in north west Greenland.

10

10

15

15

20

20

25

25

30

30

20

20

15

15

10

10

0
(i)

Mean monthly temperatures for Qaanaaq are:

22

24

23

18

+2

+5

+3

15

22

Complete the climate graph for Qaanaaq.


(ii)

temperature
/ C

10

precipitation
/ mm

10

precipitation
/ mm

temperature
/ C

tundra climate: Qaanaaq, Greenland (76N)

[2]

Calculate the annual range of temperature in Qaanaaq.

............................C [1]

UCLES 2014

5014/11/M/J/14

[Turn over

18
(iii)

Tundra lands are often described as cold deserts. Look at the climate graph for Qaanaaq
on page 17. What evidence is there to support this description?
...........................................................................................................................................
...........................................................................................................................................
...........................................................................................................................................
...........................................................................................................................................
...........................................................................................................................................
.......................................................................................................................................[3]

(iv)

Look at the diagram below of insolation. It can be used to explain why the tundra is so
cold.
insolation

60
Suns
rays
equator

atmosphere

Add labels to the diagram to explain why it is much colder in the tundra than at the
equator.
[3]

UCLES 2014

5014/11/M/J/14

19
(c) Look at the Insert photograph taken in summer near the coast in western Greenland.
(i)

Describe what the photograph shows about tundra vegetation and its distribution in this
area.
...........................................................................................................................................
...........................................................................................................................................
...........................................................................................................................................
...........................................................................................................................................
...........................................................................................................................................
.......................................................................................................................................[3]

(ii)

In western Greenland, the traditional way to make a living is from the sea, by fishing,
hunting seals and whaling.
Look at both the climate data on page 17 and the Insert photograph. Explain why it is not
possible to make a living from farming the land in western Greenland.
...........................................................................................................................................
...........................................................................................................................................
...........................................................................................................................................
...........................................................................................................................................
...........................................................................................................................................
.......................................................................................................................................[3]

UCLES 2014

5014/11/M/J/14

[Turn over

20
(d) The diagram below shows part of a food web in the Arctic.
a food web in the Arctic
people

polar bears

arctic foxes

seals
caribou

fish

lemmings

insects

zooplankton
phytoplankton
energy from the sun
(i)

Complete the food web by writing in the box.

(ii)

Use the food web above to complete the food chain below.

[1]

people
................................

................................

................................
[2]

(iii)

Describe and explain the effects of a decline in fish numbers on the food web above.
...........................................................................................................................................
...........................................................................................................................................
...........................................................................................................................................
...........................................................................................................................................
...........................................................................................................................................
.......................................................................................................................................[3]

UCLES 2014

5014/11/M/J/14

21
(e) For many years, oil companies showed little interest in searching for oil under the sea bed in
the Arctic Ocean off the coast of western Greenland.
(i)

One reason for this was the high cost of searching for oil here.
Suggest reasons why searching for oil off the coast of Greenland is difficult and
expensive.
...........................................................................................................................................
...........................................................................................................................................
...........................................................................................................................................
.......................................................................................................................................[2]

(ii)

The rush for oil exploration in the Arctic since 2005 is being driven by the world price of
crude oil. The bar graph below shows average price of oil on world markets 19992012.

120
110
100

average price of crude oil


/ $US per barrel

90
80
70
60
50
40
30
20
10
0

1999 2000 2001 2002 2003 2004 2005 2006 2007 2008 2009 2010 2011 2012
year
How many times higher was the average price of oil in 2012 than in 1999?
Circle one answer.
45 times

67 times

89 times

1012 times
[1]

UCLES 2014

5014/11/M/J/14

[Turn over

22
(iii)

Oil company interest in searching for oil under the Arctic Ocean has greatly increased
since 2005. Use data from the bar graph on page 21 to explain why.
...........................................................................................................................................
...........................................................................................................................................
...........................................................................................................................................
.......................................................................................................................................[2]

(f)

People in Greenland have strong views about oil companies coming to the Arctic Ocean to
search for oil. Some typical views are shown below.
We have been waiting for something
like this to happen for years. I hope it
will provide income for Greenland so
that we have the money to become a
more independent nation.

Most people here think


that developing an oil
industry can only be good
for Greenland.

We are very dependent on


fishing, a small amount of
tourism and one working gold
mine. Without the $US 500
million Denmark gives us every
year, we would not be able to
afford our modern health and
education services.
(i)

Environmental groups
are not popular here
because they are
opposed to the oil
industry in Greenland.

What is the general opinion of the people of Greenland about developing an oil industry
off the coast of Greenland? Explain why most people in Greenland have this opinion.
opinion ...............................................................................................................................
...........................................................................................................................................
explanation ........................................................................................................................
...........................................................................................................................................
...........................................................................................................................................
...........................................................................................................................................
...........................................................................................................................................
.......................................................................................................................................[4]

UCLES 2014

5014/11/M/J/14

23
(ii)

Environmental groups are against any development in the Arctic. The Arctic is one of the
worlds last remaining wilderness regions, largely untouched by humans.
Explain the problems for the environment of developing an oil industry in the Arctic.
...........................................................................................................................................
...........................................................................................................................................
...........................................................................................................................................
...........................................................................................................................................
.......................................................................................................................................[3]

(iii)

Should the people in Greenland take more seriously the arguments made by
environmental groups? Explain your views.
...........................................................................................................................................
...........................................................................................................................................
...........................................................................................................................................
...........................................................................................................................................
.......................................................................................................................................[3]
[Total: 40]

UCLES 2014

5014/11/M/J/14

24
BLANK PAGE

Copyright Acknowledgements:
Question 3a
Insert

M A Fretwell @ UCLES
John Pallister UCLES

Permission to reproduce items where third-party owned material protected by copyright is included has been sought and cleared where possible. Every
reasonable effort has been made by the publisher (UCLES) to trace copyright holders, but if any items requiring clearance have unwittingly been included, the
publisher will be pleased to make amends at the earliest possible opportunity.
Cambridge International Examinations is part of the Cambridge Assessment Group. Cambridge Assessment is the brand name of University of Cambridge Local
Examinations Syndicate (UCLES), which is itself a department of the University of Cambridge.

UCLES 2014

5014/11/M/J/14

CAMBRIDGE INTERNATIONAL EXAMINATIONS


GCE Ordinary Level

MARK SCHEME for the May/June 2014 series

5014 ENVIRONMENTAL MANAGEMENT


5014/11

Paper 1, maximum raw mark 120

Mark schemes should be read in conjunction with the question paper and the Principal Examiner
Report for Teachers.

Cambridge will not enter into discussions about these mark schemes.

Cambridge is publishing the mark schemes for the May/June 2014 series for most IGCSE, GCE
Advanced Level and Advanced Subsidiary Level components and some Ordinary Level components.

www.onlineexamhelp.com

www.onlineexamhelp.com

This mark scheme is published as an aid to teachers and candidates, to indicate the requirements of
the examination. It shows the basis on which Examiners were instructed to award marks. It does not
indicate the details of the discussions that took place at an Examiners meeting before marking began,
which would have considered the acceptability of alternative answers.

Page 2

Mark Scheme
GCE O LEVEL May/June 2014

Syllabus
5014

Paper
11

Section A
1

(a) (i)
A

has the largest particles?


would be the most waterlogged after rain?

will dry fastest after rain?

is the easiest for plant roots to grow into?


[3]

(ii) A silty
B clayey
C sandy
Two or three correct for two marks, one correct for one mark.

[2]

(b) (i) organic content: provides nutrients / helps bind soil particles together / is important for soil
stability; (clay humus complex) keeps nutrients in the soil / prevents leaching; helps to
retain moisture; source of nutrients;
[1]
(ii) soil organisms: earthworms burrow and provide air / root spaces / improve drainage; mix
plant matter with the minerals; fauna digest / breakdown / decompose plant matter / help
humus formation;
[1]

(iii) air: needed for soil fauna / plants to respire;

[1]

(iv) nutrients: provide food for plants;

[1]

(c) it is shallower so insufficient depth for deep roots;


too shallow to plough;

[1]

(a) (i) Chagos (Islands);

[1]

(ii) five (accept six);

[1]

(iii) all a long distance away from the fishing area / Chagos Islands;
all northern hemisphere;
all coastal;
all MEDCs / developed countries;

[1]

Cambridge International Examinations 2014

Page 3

Mark Scheme
GCE O LEVEL May/June 2014

Syllabus
5014

Paper
11

(b) Any relevant reasons such as:


number / size of fishing vessels;
overfishing / under-fishing;
reduces / increases reproduction of fish;
quotas changed;
net sizes changed;
fishing methods changed;
change in fishing season length;
reversal of ocean currents / change in sea conditions;
reduction / increase in food for fish;
reduction / increase in predators;

[4]

(c) Any relevant reasons such as:


there will be more fish of breeding age;
it will allow the fish population to increase;
beneficial impact on the food chain;
unemployment / loss of income (of fishermen or processing factory workers etc.);
increase in fish prices;
(may lead to) less protein in diets;
At least one benefit and one disadvantage.
3

[3]

(a) (i) wind vane;

[1]

(ii) south;

[1]

(iii) A (arrow / blade) points to the direction from which the wind has come / indicates the
wind direction;
B (shaft / pole) raises the arrow above the roof / reduces the effect of friction;
C (compass / cardinal points) identifies the direction;
(iv) on roof / high (above ground);

[1]

(v) no obstructions to deflect the wind;

[1]

(b) (i) drawn length shows four days from the east;
(ii) prevailing wind / most frequent wind direction can be identified;
wind break at right angle to the prevailing wind direction;
4

[3]

(a) (i) 37;

[1]
[2]
[1]

(ii) decline to year 2 / in first year;


increase next 2 years / year 2 to year 4;
decrease since year 4 / year 4 to year 10;
(iii) no till;
(b) (i) it saves the cost of the fuel needed for ploughs / harrows;

Cambridge International Examinations 2014

[2]
[1]
[1]

Page 4

Mark Scheme
GCE O LEVEL May/June 2014

Syllabus
5014

Paper
11

(ii) moisture evaporates more quickly from a tilled soil / increased drought makes the soil
drier especially if tilled;
[1]
(c) (i) Credit any valid idea, such as:
shades the soil so it does not dry out so quickly, so it is less likely to be moved;
moist soil is heavier than dry soil;
moist soil particles stick together / dry soil crumbles;
mulch protects the soil from the wind;
mulch protects the soil from raindrop impact;
mulch encourages rain to infiltrate / reduces run-off;

(d) Any valid method by which soil erosion by water can be reduced, such as:
terracing;
contour ploughing;
covering the soil with a mulch;
strip cropping;
cover cropping;
planting trees in gullies;
check dams across gullies;
Etc.;

Cambridge International Examinations 2014

www.onlineexamhelp.com

[1]

www.onlineexamhelp.com

[3]

Page 5

Mark Scheme
GCE O LEVEL May/June 2014

Syllabus
5014

Paper
11

Section B
5

(a) (i) Table infilling from left to right:

2 : 2 : 10 : 0 : 2 : 4 : 20 : 16 : 4
or
1 : 4
1 : 2 : 11 : 0 : 2 : 4 :
17 : 3

or
All correct for three marks
or
all six correct continents for two marks,
at least three correct for one mark,
ratio correct for one mark.

[3]

(ii) more megacities / great increase in numbers;


more further south / in lower latitudes;
more in the tropics than in temperate lands;
many more / most in Asia;
more widespread throughout;
Europe / Africa now have mega-cities;
none in Australia;

[2]

(iii) less use / availability of methods of contraception;


greater economic need for and value of children;
absence of / weak national population policies / government attitude;
religious reasons;
poor / absence of education especially for women;
tradition of large families;
large proportion of women are of child-bearing age;
all in death rate explained;
Remember that the question focus is upon developing countries.
Reason for one mark.
Further development / elaboration for one mark.
(iv) already much urbanisation in developed world countries;
less difference in services between rural and urban areas in developed countries;
greater poverty and absence of modern facilities in rural areas;
natural and human disasters may drive people out such as flood, wars;
industrialisation in developing / jobs in urban areas;
in developed countries people wish to escape crime / pollution / noise of cities;
(b) (i) plots complete and accurate on the divided bar for two marks
at least two plotted correctly for one mark
labelling for one mark

[4]

[2]

[3]

(ii) push factors D E


pull factors A B C F
All correct for three marks, 4 or 5 correct for two marks, 2 or 3 correct for one mark. [3]

Cambridge International Examinations 2014

Page 6

Mark Scheme
GCE O LEVEL May/June 2014

Syllabus
5014

Paper
11

(iii) driven out by a natural disaster / drought;


jobs only in farming in rural areas / more variety in urban;
better paid jobs in urban;
shortage of land / land not owned / declining soil fertility;
in the poverty trap;
bright lights syndrome;
war / conflict;
No credit for public services.

[1]

(iv) 4:2 ratio of pull : push factors;


70% : 18%;
Pull more important than push (reserve one mark);
Both important, but pull more so;
(c) (i) piped water supply;
sewerage / sanitation;
electricity;
health care;
education;
rubbish collection;
transport;
etc.;

[2]

[1]

(ii) piped water or sanitation


disease / health problems;
electricity;
no light crime rates high / no fridge food goes off;
health care;
too sick to work / high death rates;
Credit other relevant correct amenity.
Valid consequence identified for one mark

Further development for second mark

[2]

(iii) Characteristics of slum housing:


self-built / haphazard collection of building materials / examples of building materials used
/ poorly built;
lack running water / toilets / power / etc.;
unplanned / very high density / narrow alleyways between small houses;
most with only one storey and just one or two rooms;
Types of locations for poor houses:
unused land / often unused for a good reason such as liable to flood;
cheap land on steep (unstable) slopes;
edge of cities;
undeveloped site;
along main transport links
next to a source of pollution such as a factory;
next to rivers and on swampy land; (which increases disease risks)
Two marks for each of the two parts of the question.

Cambridge International Examinations 2014

[4]

Page 7

Mark Scheme
GCE O LEVEL May/June 2014

Syllabus
5014

(iv) cooperating to install water / sanitation;


self-help schemes;
from income on formal or informal sector;
petitioning government / protests;

Paper
11

[2]

(v) providing essential services such as water, electricity and roads;


so that they are linked into the rest of the urban area;
community participation schemes;
such as ASH (Aided Self-Help) for obtaining building materials and technical help,
sometimes with small loans;
affordable housing;
basic but attractive to people in need of cheap housing;
providing security of tenures; so people are willing to invest in / improve their homes /
services;
site and service schemes;
Give full credit for references to a relevant named example.
One line of explanation followed, without significant development or reference to an
example. Or a general answer describing improvements lacking in detailed explanation
for one or two marks.
Fuller and more precise explanation; has some width and / or depth to it for three or four
marks.
[4]
(d) (i) Pollution:
air, water and land pollution;
air pollution from factories and traffic;
impact of pollution;
Congestion:
traffic congestion from cars and / or lorries;
increasing number of vehicles;
old road / narrow streets;
lack of transport planning;
Loss of agricultural land:
sprawl / growth of the city into surrounding rural areas / land built on;
unplanned / uncontrolled;
less food produced;
non-stop growth from in-migration;
Three at one mark each.

Cambridge International Examinations 2014

[3]

Page 8

Mark Scheme
GCE O LEVEL May/June 2014

Syllabus
5014

Paper
11

(ii) traffic congestion in large cities;


old city areas not suitable modern traffic;
worse in developing, no money to deal with problem;
lack of knowledge / technology;
lack of government will / leadership to enforce regulation;
speed of development in newly developing;
many migrants putting pressure on existing services;
pressure on space little resistance to urban sprawl;
Credit use of specific city references.
General answers, more stating than giving reasons. Perhaps deals adequately with only
one of the question themes. One point about each question without much elaboration for
either for one or two marks.
More precise answers, based upon giving reasons for two themes. Development of
these reasons shows a higher level of understanding about both question themes. May
include something specific such as a city example for one part, which helps to offset the
outline answer given to the other for three or four marks.
[4]

Cambridge International Examinations 2014

Page 9
6

Mark Scheme
GCE O LEVEL May/June 2014

Syllabus
5014

Paper
11

(a) (i) in the extreme north of the continents / land masses;


mostly confined to polar lands north of 60 / high latitudes;
extend for the full width across the continents of North America and Europe;
narrow strip around the coastal edges of Greenland;
further south in west and / or east in North America / in east in Asia;
covers the islands off northern Canada / Asia;
coastal;
(ii) continents without land south of 60 owtte;
the southern continent of Antarctica is too far south / covered by
ice and snow;
(b) (i) accurate plots for two marks
2 correct for one mark
(ii) 29 C

[3]

[1]

[2]
[1]

(iii) 9 months below freezing point / cold most of the year;


highest summer temperature only +5 C / even summers are cold;
temperatures below 20 C make it extremely cold;
average temp. 11 C;
low average annual precipitation total (6570 mm);
not more than 5 mm of rain in 9 of the months;
max monthly rainfall only 13 mm;
under 250 mm per year is the definition for a desert climate;
Give marks for the use of values from the graph / table which support the answer.
Maximum one mark for answers without supporting use of direct evidence.
Maximum two marks for answers that are either all temperature or all precipitation.

[3]

(iv) Labels
at tundra latitudes
Suns rays have a large area of Earth's surface to heat up;
longer journey through the Earth's atmosphere / more sunlight reflected at Equator;
Suns rays shorter journey through Earths atmosphere;
smaller area of the Earth's surface to heat up;

[3]

(c) (i) vegetation is visible on only a few areas;


(short) grass / flowers;
many bare rocks without visible vegetation;
bare ground also where rock is loose as in foreground / steep slopes in the background;
marked absence of any taller bushes and trees / short vegetation;
[3]
low-lying bushes / shrubs;

Cambridge International Examinations 2014

Page 10

Mark Scheme
GCE O LEVEL May/June 2014

Syllabus
5014

Paper
11

(ii) soil is too thin to allow ploughing / digging for crop growing / not much soil;
not much grass and vegetation shown on the photograph for grazing animals;
too large a proportion of the area is covered by bare or loose rock;
summer temperatures are never warm enough for cultivation / too cold for farming;
maximum temperature is only 5 C in July;
frost-free period too short with below freezing average temperatures from September to
May;
little precipitation available / a lot of it may fall as snow;
Two marks max. photo only.
Two marks max. climate only.
Three explanatory points at one mark each, placed in the context of farming.
(d) (i) plants / grass / named arctic plant;
(ii) phytoplankton zooplankton fish (people)
organisms correct;
arrows correct;

[3]
[1]

[2]

(iii) less fish for seals so fewer seals;


fewer fish for people so they eat more caribou;
fewer caribou so more plants for lemmings and insects;
caribou decline so more plants = more lemmings;
more lemmings and insects so more food for foxes;
less fish so more zooplankton;
more zooplankton less phytoplankton;
fewer seals for polar bears;
Must indicate decline or increase.
(e) (i) difficult remote / takes long time to get there;
extreme cold / workers welfare;
sea ice makes searching from ships difficult;
limited services / facilities on land;
have to pay high wages;
expensive- cost of getting there;
supporting workforce at base / on ship;

[3]

[2]

(ii) 67 times circled;

[1]

(iii) rising world oil prices;


makes Greenland more cost effective / covers costs;
figures quoted from graph;

[2]

Cambridge International Examinations 2014

Page 11

Mark Scheme
GCE O LEVEL May/June 2014

Syllabus
5014

Paper
11

(f) (i) Opinion


Greenlanders are (overwhelmingly) in favour of exploration going ahead;
Explanation
provide a new source of income;
development of new income sources;
previously dependent on a narrow range of activities (fishing, tourism, gold mining);
increases their independence / lowers their dependence on Denmark's subsidy;
further economic development possible / jobs health education;

[4]

(ii) Problems
risks of pollution from leaks and spills;
examples like the Exxon Valdez in Alaska;
cold climate means that environmental damage takes much longer to be naturally
repaired;
marginal areas for wildlife;
fewer wilderness areas remain, those that do are under increasing pressure from
humans;
One mark per problem briefly stated to a max. of two marks.
Development of problems for further marks.

[3]

(iii) aware of the environment and extra regulations placed on the oil companies;
adequate safety and measures to protect environment from problems stated;
Greenland should not be denied the chance to exploit its own resources just because
wilderness areas have disappeared elsewhere for economic growth;
global warming if applied to Greenland;
yes, because could threaten fish stocks / way of life;
could threaten tourist trade;
cost-benefit applied;
[3]
[Total: 120]

Cambridge International Examinations 2014

Cambridge International Examinations


Cambridge Ordinary Level

Paper 1

May/June 2014
2 hours 15 minutes

Candidates answer on the Question Paper.


Additional Materials:

Ruler

READ THESE INSTRUCTIONS FIRST


Write your Centre number, candidate number and name on all the work you hand in.
Write in dark blue or black pen.
You may use an HB pencil for any diagrams or graphs.
Do not use staples, paper clips, glue or correction fluid.
DO NOT WRITE IN ANY BARCODES.

www.onlineexamhelp.com

* 9 4 1 9 5 5 9 4 2 5 *

5014/12

ENVIRONMENTAL MANAGEMENT

Answer all questions.


Electronic calculators may be used.
You may lose marks if you do not show your working or if you do not use appropriate units.
Write your answers in the spaces provided on the Question Paper.
All questions in Section A carry 10 marks.
Both questions in Section B carry 40 marks.
At the end of the examination, fasten all your work securely together.
The number of marks is given in brackets [ ] at the end of each question or part question.

This document consists of 25 printed pages and 3 blank pages.


DC (SJF/CGW) 80757/3
UCLES 2014

[Turn over

www.onlineexamhelp.com

2
Section A
Answer all the questions.
1

(a) Look at the diagram of a plate boundary.

ocean

(not to scale)
(i)

On the diagram label:


the oceanic plate
the magma reservoir
the crater of the volcano

(ii)

[3]

The diagram shows part of one of the layers of the Earth.


Circle which layer is shown.
crust

(iii)

inner core

mantle

outer core

[1]

Name the type of plate boundary shown in the diagram.


.......................................................................................................................................[1]

(iv)

Explain how magma forms at this type of plate boundary.


...........................................................................................................................................
...........................................................................................................................................
...........................................................................................................................................
.......................................................................................................................................[2]

UCLES 2014

5014/12/M/J/14

3
(b) Suggest three hazards of living on the slopes of volcanoes in areas such as the one in the
diagram.
1 ................................................................................................................................................
...................................................................................................................................................
2 ................................................................................................................................................
...................................................................................................................................................
3 ................................................................................................................................................
...............................................................................................................................................[3]
[Total: 10]

UCLES 2014

5014/12/M/J/14

[Turn over

4
2

(a) Look at the diagrams which show the seas around the Galapagos Islands in a normal year
and in an El Nio year.
a normal year
little rain

west

east
strong trade
winds

Pacific Ocean

Key
Galapagos
Islands
warm ocean
current
upwelling
cold water
containing
nutrients

an El Nio year
rain clouds

west

east

weak trade
winds

Pacific Ocean

(not to scale)
(i)

Use the diagrams to explain the causes of an El Nio event.


...........................................................................................................................................
...........................................................................................................................................
...........................................................................................................................................
...........................................................................................................................................
...........................................................................................................................................
.......................................................................................................................................[3]

UCLES 2014

5014/12/M/J/14

www.onlineexamhelp.com

www.onlineexamhelp.com

trade winds

5
(ii)

Suggest why El Nio years are better for most land-based animals in the Galapagos
Islands.
...........................................................................................................................................
...........................................................................................................................................
...........................................................................................................................................
...........................................................................................................................................
...........................................................................................................................................
...........................................................................................................................................
...........................................................................................................................................
.......................................................................................................................................[4]

(b) The diagram shows one food chain for the Galapagos Islands.
seaweed (algae) marine iguana

Galapagos hawk

Seaweed needs nutrients.


Explain why thousands of marine iguana die during an El Nio event.
...................................................................................................................................................
...................................................................................................................................................
...................................................................................................................................................
...................................................................................................................................................
...................................................................................................................................................
...............................................................................................................................................[3]
[Total: 10]

UCLES 2014

5014/12/M/J/14

[Turn over

6
3

(a) Look at the bar graph which shows the number of deaths in four countries caused by some
cyclones between 1900 and 2010.
750
700
650

1919

600
550

1991

deaths in cyclones
/ thousands

500
450
400
350
300
250

1970
1922

200
150
100

1975
2008

50

1977
1942

0
Bangladesh

China

Myanmar

India

country
(i)

Complete the graph by adding the cyclone in Bangladesh in 1912 that killed 40 000
people.
[1]

(ii)

Compare the effect of these cyclones before and after 1950.


...........................................................................................................................................
...........................................................................................................................................
...........................................................................................................................................
.......................................................................................................................................[2]

UCLES 2014

5014/12/M/J/14

7
(b) State two characteristics of the weather of cyclones that cause loss of life.
1 ................................................................................................................................................
2 ............................................................................................................................................[2]
(c) Explain why cyclones of the same strength kill very different numbers of people, using the
following headings:
the type of land and its location
...................................................................................................................................................
...................................................................................................................................................
...................................................................................................................................................
...................................................................................................................................................
...................................................................................................................................................
...................................................................................................................................................
human and economic reasons
...................................................................................................................................................
...................................................................................................................................................
...................................................................................................................................................
...................................................................................................................................................
...................................................................................................................................................
...............................................................................................................................................[5]
[Total: 10]

UCLES 2014

5014/12/M/J/14

[Turn over

8
4

(a) Look at the graph which shows the number of hectares used for genetically engineered crops
between 1996 and 2011.
160

150

140

Others

140

130

India

130

120

Brazil

120

110
100
90
80

110

Canada

100

Argentina

90

USA

80

70

70

60

60

50

50

40

40

30

30

20

20

10

10

area under genetically engineered crops


/ million hectares

area under genetically engineered crops


/ million hectares

150

160
Key

0
1996 1997 1998 1999 2000 2001 2002 2003 2004 2005 2006 2007 2008 2009 2010 2011
year

(i)

Complete the graph to show that the total area of land under genetically engineered
crops in the world was 160 million hectares in 2011.
[1]

(ii)

Approximately what proportion of the worlds genetically engineered crops was grown in
the USA in 2011? Circle the correct answer below.
20%

(iii)

35%

45%

65%

[1]

In which year did Brazil start growing genetically engineered crops?


.......................................................................................................................................[1]

UCLES 2014

5014/12/M/J/14

9
(b) (i)

Some genetically engineered crops are resistant to insect pests.


Suggest two advantages of this.
1 ........................................................................................................................................
...........................................................................................................................................
2 ........................................................................................................................................
.......................................................................................................................................[2]

(ii)

Some genetically engineered crops have increased protein content.


Suggest one advantage of this.
...........................................................................................................................................
.......................................................................................................................................[1]

(iii)

Give reasons why some people are concerned about the growth of genetically engineered
crops.
...........................................................................................................................................
...........................................................................................................................................
...........................................................................................................................................
...........................................................................................................................................
...........................................................................................................................................
...........................................................................................................................................
...........................................................................................................................................
.......................................................................................................................................[4]
[Total: 10]

UCLES 2014

5014/12/M/J/14

[Turn over

10
Section B
Answer both questions.
5

(a) Look at the map showing the world distributions of two biomes, tropical rainforest and taiga
forests.
location of tropical rainforest and taiga forest biomes

60N

23

23

60S
Key
taiga forest
tropical rainforest
(i)

Describe where tropical rainforests are located.


...........................................................................................................................................
...........................................................................................................................................
...........................................................................................................................................
...................................................................................................................................... [2]

(ii)

Describe where taiga forests are located.


...........................................................................................................................................
...........................................................................................................................................
...........................................................................................................................................
...................................................................................................................................... [2]

UCLES 2014

5014/12/M/J/14

11
(iii)

State the most important difference between the distribution of taiga forest and tropical
rainforest.
...........................................................................................................................................
...................................................................................................................................... [1]

(b) Some of the characteristics of the vegetation in the two biomes are given in the table.
Complete the table by filling in the empty boxes.
tropical rainforests
A

taiga forest

forest characteristics
1

biodiversity

many species of tree in


50 hectares

two or three species of


tree in 50 hectares

forest layers

4 or 5

...............................................

tree shape

1. tall and slim trunks,

1. ...........................................
...........................................

2. branches and leaves


mainly at the top

2. ...........................................
...........................................

leaf characteristic

example of named tree

...............................................

...............................................

...............................................

...............................................

mahogany

...............................................
[6]

UCLES 2014

5014/12/M/J/14

[Turn over

12
(c) Look at the table showing a summary of the climates for tropical rainforests and taiga forests.
temperature / C
mean monthly temperature

precipitation / mm

climate

highest

lowest

mean
annual total

equatorial for
tropical rainforest

28

26

2000

270

150

cool temperate
interior for taiga
forest

14

12

500

70

20

(i)

wettest
month

driest
month

Calculate the annual range of temperature in taiga forest.


Space for working.

..................................................... C [1]
(ii)

Use the climate data to explain two differences in vegetation characteristics between
tropical rainforest and taiga forest.
difference ...........................................................................................................................
explanation ........................................................................................................................
...........................................................................................................................................
difference ...........................................................................................................................
explanation ........................................................................................................................
...................................................................................................................................... [3]

UCLES 2014

5014/12/M/J/14

13
(iii)

The table below shows the percentage of tropical rainforest and taiga forest cleared by
people.
tropical rainforest
45% already cleared

taiga forest
less than 5% cleared

Suggest how the climates of the two biomes explain this difference.
...........................................................................................................................................
...........................................................................................................................................
...........................................................................................................................................
...........................................................................................................................................
...........................................................................................................................................
...................................................................................................................................... [3]

UCLES 2014

5014/12/M/J/14

[Turn over

14
(d) Environmental groups are concerned about tropical rainforest losses in Brazil.
Look at the bar graph showing estimates of forest clearances from 1999 to 2010.
Brazil: rainforest losses 1999 2010
30 000

25 000

20 000

rainforest
losses
/ km2 per year

15 000

10 000

5000

UCLES 2014

1999 2000 2001 2002 2003 2004 2005 2006 2007 2008 2009 2010
year

5014/12/M/J/14

15
(i)

Describe the trends shown by the graph on page 14.


...........................................................................................................................................
...........................................................................................................................................
...........................................................................................................................................
...........................................................................................................................................
...........................................................................................................................................
...................................................................................................................................... [3]

(ii)

Discuss the extent to which the graph shows that the environmental groups should be
concerned about the clearance of the tropical rainforest in Brazil.
...........................................................................................................................................
...........................................................................................................................................
...........................................................................................................................................
...........................................................................................................................................
...........................................................................................................................................
...................................................................................................................................... [3]

UCLES 2014

5014/12/M/J/14

[Turn over

16
(e) One strategy for tropical rainforest conservation is to create a biosphere reserve. One
example is the Maya biosphere reserve in Guatemala. It was created in 1990 to protect the
largest remaining area of tropical rainforest in Central America. Map A shows its location and
map B shows the zones within it.
map A
location of Maya biosphere reserve
N

Flores
BELIZE

MEXICO

GUATEMALA

Chichicastenango
Antigua

Guatemala City

Pacific
Ocean

40 km

Key
Maya biosphere
reserve
mountain
region
capital
city
other main cities
and towns

UCLES 2014

5014/12/M/J/14

17
map B
zones inside the Maya biosphere reserve
N

NP

30 km

NR

NP

Carmetita
NR

NP

San
Miguel
NR

NP

Tikal
NP

Lake
Flores
Petn Itz

buffer zone
NP national parks
NR nature reserve
multiple use zone
outside the reserve

(i)

Name the two land uses where conservation of the rainforest will be greatest.
...................................................................................................................................... [1]

(ii)

How does this give protection to the rainforest?


...........................................................................................................................................
...................................................................................................................................... [1]

(iii)

Describe where the buffer zone is located and suggest its purpose.
...........................................................................................................................................
...........................................................................................................................................
...........................................................................................................................................
...................................................................................................................................... [2]

UCLES 2014

5014/12/M/J/14

[Turn over

18
(iv)

People living in the multiple use zone are encouraged to follow new economic activities,
such as the sustainable harvesting of forest products rather than slash and burn
subsistence farming.
A Fair Trade group has been organised to buy and export products.
Explain how and why these new economic activities can increase the chances of
remaining areas of forest being conserved by the local people.
...........................................................................................................................................
...........................................................................................................................................
...........................................................................................................................................
...........................................................................................................................................
...........................................................................................................................................
...................................................................................................................................... [3]

(v)

Tikal, with its Mayan pyramids, attracts 200 000 visitors a year many of them from
overseas.
Suggest what can be done to try to ensure that local people can share in some of the
economic benefits from tourism.
...........................................................................................................................................
...........................................................................................................................................
...........................................................................................................................................
...........................................................................................................................................
...........................................................................................................................................
...................................................................................................................................... [3]

UCLES 2014

5014/12/M/J/14

19
(vi)

Government figures show that the Maya biosphere reserve has lost 21% of its forest
cover since being declared a protected zone in 1990. Some losses are due to illegal
activities, such as production of drugs and forest clearance for airstrips to export the
drugs.
Suggest why the government find it difficult to protect the forest from clearance.
...........................................................................................................................................
...........................................................................................................................................
...........................................................................................................................................
...................................................................................................................................... [2]

(vii)

Many biosphere reserves are located in countries where average incomes are low, such
as Guatemala.
To what extent is the strategy of creating biosphere reserves likely to be successful?
Explain your answer.
...........................................................................................................................................
...........................................................................................................................................
...........................................................................................................................................
...........................................................................................................................................
...........................................................................................................................................
...........................................................................................................................................
...........................................................................................................................................
...................................................................................................................................... [4]
[Total: 40]

UCLES 2014

5014/12/M/J/14

[Turn over

20
6

(a) The bar graph shows the results of a survey in 2011, which identified the five cities with the
worlds highest urban air pollution. It was based on the measurement of particulates (such as
smoke particles) in the atmosphere.
the worlds top 5 cities for air pollution in 2011

particulates / arbitrary units

400

300

200

100

0
Ahwaz
Iran
(i)

Ulan Bator Sanandaj


Mongolia
Iran

Ludhiana
India

Quetta
Pakistan

The World Health Organisation (WHO) recommends 20 arbitrary units as a safe limit for
particulates.
On the graph above, draw a line to show this safe limit.

(ii)

[1]

Approximately how many times higher were the particulates in Quetta than the WHO
safe limit?
Circle one answer.
5 times

(iii)

9 times

12 times

18 times

[1]

Describe how high levels of air pollution can affect economic activity in a city.
...........................................................................................................................................
...........................................................................................................................................
...........................................................................................................................................
...........................................................................................................................................
...........................................................................................................................................
...................................................................................................................................... [3]

UCLES 2014

5014/12/M/J/14

21
(iv)

One of the reasons for the high levels of air pollution in these cities is the burning of fuel
oil, firewood and dried dung.
Suggest reasons why it is difficult to reduce the amount of particulates going into the
atmosphere in cities such as these.
...........................................................................................................................................
...........................................................................................................................................
...........................................................................................................................................
...........................................................................................................................................
...........................................................................................................................................
...................................................................................................................................... [3]

(b) (i)

Particulate amounts in four of the worlds cities are given in the table.
city

country

particulates
/ arbitrary units

Beijing

China

121

Mexico City

Mexico

30

Los Angeles

USA

25

New York

USA

21

On the grid below, draw a bar graph of particulate amounts in Beijing, Mexico City, Los
Angeles and New York.
air pollution in four of the worlds cities

[3]
UCLES 2014

5014/12/M/J/14

[Turn over

22
(ii)

There are certain physical characteristics which increase the chances of high levels of
air pollution in cities. Three of these are named in the spider diagram.
high air pressure

calm conditions

physical characteristics that can lead to


the build up of high levels of air pollution

valley surrounded by steep sided hills


Explain how these physical characteristics can lead to the build up of high levels of air
pollution.
...........................................................................................................................................
...........................................................................................................................................
...........................................................................................................................................
...........................................................................................................................................
...........................................................................................................................................
...................................................................................................................................... [3]
(iii)

Describe strategies that can be used to improve the air quality in cities.
...........................................................................................................................................
...........................................................................................................................................
...........................................................................................................................................
...........................................................................................................................................
...........................................................................................................................................
...........................................................................................................................................
...........................................................................................................................................
...........................................................................................................................................
...........................................................................................................................................
...................................................................................................................................... [5]

UCLES 2014

5014/12/M/J/14

23
(iv)

Explain the problems that governments have in implementing strategies to reduce air
pollution in cities.
...........................................................................................................................................
...........................................................................................................................................
...........................................................................................................................................
...........................................................................................................................................
...........................................................................................................................................
...................................................................................................................................... [3]

(c) Read the newspaper report about an air pollution accident in Bhopal, India in December 1984.

Disaster in Bhopal
A gas leak from a pesticide factory has killed more than 25 000 people. A large cloud of
poisonous gas covered the shanty towns around the factory. The health of about 250 000
people living further away is thought to be at risk. People in Bhopal had little idea about the
dangers of living next to a factory producing toxic chemicals.
The accident was caused by design faults in the factorys safety systems and by poor
maintenance.
In the United States of America, factories making such toxic chemicals must be located at
least 80 kilometres away from the nearest settlement.
(i)

State two ways in which the factorys managers could have reduced the risk of a gas leak
from this factory in Bhopal.
1 .......................................................................................................................................
...........................................................................................................................................
2 .......................................................................................................................................
...................................................................................................................................... [2]

UCLES 2014

5014/12/M/J/14

[Turn over

24
(ii)

Suggest why there was a greater loss of life in India, a developing country, than there
would have been if the accident had happened in the United States of America, a
developed country.
...........................................................................................................................................
...........................................................................................................................................
...........................................................................................................................................
...........................................................................................................................................
...........................................................................................................................................
...................................................................................................................................... [3]

(d) Read this newspaper report written in 2009.

25 years later; the effects of the disaster are still being felt by the people of Bhopal
More than 25 000 people have died and at least 500 000 have serious health problems, such as
cancer, anaemia, infertility and birth defects. Large areas around the factory have been labelled a
global toxic hot-spot, unfit for any kind of use.
The factory site remains closed and abandoned, with thousands of tonnes of toxic wastes stored
there. In the monsoon season, the rains wash these chemicals into the groundwater, contaminating
local wells. These contain mercury concentrations thousands of times higher than WHO (World
Health Organisation) recommended limits. An estimated 90 per cent of local people have to use
this well water. They complain that not enough water is being piped in by the authorities.
(i)

Describe and explain the long-term effects of the Bhopal disaster.


...........................................................................................................................................
...........................................................................................................................................
...........................................................................................................................................
...........................................................................................................................................
...........................................................................................................................................
...........................................................................................................................................
...........................................................................................................................................
...................................................................................................................................... [4]

UCLES 2014

5014/12/M/J/14

25
(ii)

Give three reasons why people could still be suffering from the effects of this disaster in
another 25 years time.
...........................................................................................................................................
...........................................................................................................................................
...........................................................................................................................................
...........................................................................................................................................
...........................................................................................................................................
...................................................................................................................................... [3]

(e) Human activities damage environments in many ways, such as the impact of mineral
exploitation.
Discuss what measures could be taken to restore such damaged environments.
...................................................................................................................................................
...................................................................................................................................................
...................................................................................................................................................
...................................................................................................................................................
...................................................................................................................................................
...................................................................................................................................................
...................................................................................................................................................
...................................................................................................................................................
...................................................................................................................................................
...................................................................................................................................................
...................................................................................................................................................
.............................................................................................................................................. [6]
[Total: 40]

UCLES 2014

5014/12/M/J/14

26
BLANK PAGE

UCLES 2014

5014/12/M/J/14

27
BLANK PAGE

UCLES 2014

5014/12/M/J/14

28
BLANK PAGE

Permission to reproduce items where third-party owned material protected by copyright is included has been sought and cleared where possible. Every
reasonable effort has been made by the publisher (UCLES) to trace copyright holders, but if any items requiring clearance have unwittingly been included, the
publisher will be pleased to make amends at the earliest possible opportunity.
Cambridge International Examinations is part of the Cambridge Assessment Group. Cambridge Assessment is the brand name of University of Cambridge Local
Examinations Syndicate (UCLES), which is itself a department of the University of Cambridge.

UCLES 2014

5014/12/M/J/14

CAMBRIDGE INTERNATIONAL EXAMINATIONS


GCE Ordinary Level

MARK SCHEME for the May/June 2014 series

5014 ENVIRONMENTAL MANAGEMENT


5014/12

Paper 1, maximum raw mark 120

Mark schemes should be read in conjunction with the question paper and the Principal Examiner
Report for Teachers.

Cambridge will not enter into discussions about these mark schemes.

Cambridge is publishing the mark schemes for the May/June 2014 series for most IGCSE, GCE
Advanced Level and Advanced Subsidiary Level components and some Ordinary Level components.

www.onlineexamhelp.com

www.onlineexamhelp.com

This mark scheme is published as an aid to teachers and candidates, to indicate the requirements of
the examination. It shows the basis on which Examiners were instructed to award marks. It does not
indicate the details of the discussions that took place at an Examiners meeting before marking began,
which would have considered the acceptability of alternative answers.

Page 2

Mark Scheme
GCE O LEVEL May/June 2014

Syllabus
5014

Paper
12

Section A
1

(a) (i) oceanic plate;


magma reservoir;
crater;
Accept labels if there is no ambiguity. One mark per correct label.

[3]

(ii) crust;

[1]

(iii) destructive / convergent/collision;

[1]

(iv) oceanic plate moves beneath continental plate/subduction;


friction creates heat;
rocks melt;

[2]

(b) eruptions of lava / lava flows;


volcanic bombs;
eruptions of hot / poisonous gases;
eruptions of ash;
earthquakes;
mud flows / lahars
landslides;
pyroclastic flows / nues ardentes;
Must relate to a volcanic hazard, so not unspecified death, loss of houses, loss of farmland,
etc.
[3]
2

(a) (i) trade winds weaken;


allows warm water / ocean current to move eastwards / into the area;
prevents cold water reaching the surface;
(ii) Any sensible reasons, such as:
(more) rainfall / water;
replenishing fresh water supplies;
growth of vegetation;
food for herbivores;
more prey for carnivores;
healthier so more survive;
more reproduction;
dead fish washed up as food source;

Cambridge International Examinations 2014

[3]

[4]

Page 3

Mark Scheme
GCE O LEVEL May/June 2014

Syllabus
5014

Paper
12

(b) nutrient rich cold waters do not reach the surface;


warm current poor in nutrients;
seaweed / algae deprived of nutrients;
seaweed / algae die;
lack of food for marine iguanas;
weakened animals die / only the fittest survive;
weakened animals more susceptible to predators;

[3]

(a) (i) addition of 1912 and a bar representing 40 000 for Bangladesh;

[1]

(ii) Credit ideas, such as:


more deaths after 1950;
except in India;
no deaths in Myanmar before 1950;
more deaths in Bangladesh China (Myanmar) pre-1950;
(b) very strong winds;
very heavy rainfall;
storm surge / flood;

[2]

[2]

(c) Allow any sensible factor with development, such as:


the type of land and its location
more deaths if:
flat land;
lowland;
delta;
at the coast;
head of a bay;
Allow development points, such as:
so area easily flooded;
so prone to storm surges / tidal waves;
so nothing to break the force of the wind;
so water cannot drain quickly away;
etc.;
Allow converse for fewer deaths but do not credit the converse ideas for both.
human and economic reasons
population growth over time;
densely populated / large population;
flimsy/poorly built buildings;
poor transport for rapid evacuation;
inadequate warning systems;
inadequate post cyclone relief systems;
inadequate numbers of cyclone shelters;
poverty of developing countries;
disease (e.g. cholera);
famine;
etc.;
Max. three marks on either part.
Cambridge International Examinations 2014

[5]

Mark Scheme
GCE O LEVEL May/June 2014

Syllabus
5014

Paper
12

(a) (i) completion of graph line for total to 160 million hectares;

[1]

(ii) 45%;

[1]

(iii) 2001 or 2002;

[1]

(b) (i) increased yields / reduced consumption of crops by insects;


reduced transmissions of disease to crops by insects;
less need for pesticide;
improves farmers finances;

[2]

(ii) improved diet / better health / growth and repair;


commands higher price;

[1]

(iii) Allow any sensible suggestion, such as:


pollen / genetic material may escape into the natural vegetation around,
leading to a change in the natural vegetation;
could out-compete the natural vegetation;
could lead to loss of varieties;
some believe it could lead to illness / health concerns;
such as cancer / other relevant named illness;
people may be allergic to the products;
some people object on moral grounds / believe we should not interfere with nature;
resistant to change / new ideas;

[4]

Cambridge International Examinations 2014

www.onlineexamhelp.com

www.onlineexamhelp.com

Page 4

Page 5

Mark Scheme
GCE O LEVEL May/June 2014

Syllabus
5014

Paper
12

Section B
5

(a) (i) around the Equator / both sides of the Equator / mainly in the tropics;
names of the continents / countries;
some ides of the relative size / extent within / between continents;

[2]

(ii) (most) in cold temperate / southern edge of polar latitudes;


more detailed description in relation to latitude 60N;
some idea of the relative size / extent within / between continents;
names of the continents / countries;

[2]

(iii) tropical compared with temperate;


tropical north and south of the Equator compared with taiga northern hemisphere only;
longer more continuous unbroken extent for taiga across the continents / vice-versa for
tropical rainforest;
[1]
tropical more spread out / dispersed;
(b) Forest layers taiga
Tree shape taiga

1 or 2

one mark

two of:
conical shape;
downward sloping branches;
branches all the way up the stem;

Leaf characteristics

two marks

tropical rainforest one of:


drip tips (pointed ends);
large / broad leaves / leathery;
network of veins;

one mark

taiga one of:


needle leaves;
small / hard;
waxy;
dark colour;
Examples of named types of trees

taiga
pine, spruce, larch

one mark
one mark

Cambridge International Examinations 2014

[6]

Page 6

Mark Scheme
GCE O LEVEL May/June 2014

Syllabus
5014

Paper
12

(c) (i) 26 C

[1]

(ii) The following factors


temperature high and constant in tropical rainforest fluctuates to very low in winter in
taiga
rainfall tropical rainforest receives rain all year round / tropical rainforest has higher
rainfall taiga has long dry spells
No marks for difference as question is about explaining differences in vegetation that
result.
Used to explain the chosen differences in biodiversity, forest layers, tree shape, leaf
characteristics, etc., e.g.:
conical trees to shed snow in taiga, no snow in tropical rainforest;
evergreen leaves in taiga so can start photosynthesis as soon as warm enough; and not
waste energy growing new leaves, whereas no energy shortage in tropical rainforest;
drip tip leaves in tropical rainforest to remove high rainfall; needle leaves in taiga to
reduce water losses as so dry, etc.;
[3]
(iii) the high temperature and rainfall in tropical rainforest result in:
more opportunities for using the deforested land;
fast /all year crop growth;
2 or 3 crops per year can be grown;
wide range of different crops can be grown;
faster nutrient cycling;
more population in tropical rainforest;
timber from tropical rainforest more valuable;
ORA for taiga.

[3]

(d) (i) rises initially;


slow rise to 2001;
then quicker rise;
followed by fall;
peaks at 2004 / 27 100 27 500 km2;
then decreases;
decrease steeper than increase;
falls lower than it was in 1999;
small rise again in 2008;
Max. one mark for data quoted.

Cambridge International Examinations 2014

[3]

Page 7

Mark Scheme
GCE O LEVEL May/June 2014

Syllabus
5014

Paper
12

(ii) Positive points:


the amount deforested has declined fast;
the annual clearance by 2010 is only about a quarter of what it was at the peak in
2004;
by 2006 it had fallen to levels not previously seen between 1999 and 2004;
Negative points:
the forest clearances are still continuing;
there was a slight increase from 2007 to 2008;
therefore reductions in amount cleared cannot be taken for granted;
45% already cleared;
Max. one mark for relevant data.
(e) (i) National Parks and nature reserves; (accept NP and NR)

[3]
[1]

(ii) legal protection to natural environments / prevent exploitation;


international recognition / reputation / show importance of rainforest to people;
funds for research;
ecotourism is maintained / money from ecotourists;

[1]

(iii) buffer zone stretches the full width / across the southern edge of the reserve;
to limit the amount of human activity;
so that it conserves the core;
separates the core from the areas outside the reserve without protection;

[2]

(iv) the new activities are a source of income for local people;
conservation is encouraged when they can make a living by sustainably harvesting
forest products;
sustainable harvesting preserves biodiversity (for future use);
a fair trade organisation helps them to market overseas;
a fair trade organisation guarantees prices / market outlets / adds money for use in
community projects;
Max. two marks on either how or why.

[3]

(v) employ local people:


as tourist guides;
in maintenance and management of the parks;
in tourist facilities;
to make tourist souvenirs;
government channelling money back into infrastructure / local facilities;
Max. two marks for list. One idea explained well can get all three marks.

Cambridge International Examinations 2014

[3]

Page 8

Mark Scheme
GCE O LEVEL May/June 2014

Syllabus
5014

Paper
12

(vi) distant from main cities / law enforcement authorities;


large area to police;
expensive to police effectively;
forest difficult to travel through;
people desperate for food / employment;
growing population puts pressure on land;
corruption;
drug cartels threaten government / weak government;

[2]

(vii) poor chance:


government not well developed;
average income per head shows that Guatemala is a developing country;
government have greater priorities for spending its limited economic resources;
people poor so will risk illegal activity;
if locals dont cooperate;
high cost of creating /enforcing;
good chance;
world biosphere reserves are internationally recognised and supported;
easier for governments in poor countries to attract funding;
and outside expertise;
ecotourism can provide jobs / income;
Max. three marks if only one side addressed.

Cambridge International Examinations 2014

[4]

Page 9
6

Mark Scheme
GCE O LEVEL May/June 2014

Syllabus
5014

Paper
12

(a) (i) line drawn across the graph at 20;

[1]

(ii) 12 times circled or otherwise clearly indicated;

[1]

(iii) health issues leads to loss of work days;


wealthy / wealth creators move away;
e.g. of health issue (bronchitis / asthma / irritates eyes / skin irritation / breathing
difficulties);
transport issues caused by fog / smog;
costs to industry of permits to pollute / cleaning effects of pollution;

[3]

(iv) satisfying essential domestic needs for energy;


people / industries using the cheapest energy source / cost implications;
people using the one most readily available to them;
too engaged in survival to consider environmental effects;
cannot afford alternative fuels;
increasing wealth of population leads to greater fuel use (cars);
(cleaner) alternative fuels not available;
high density / concentrated population;

[3]

(b) (i) plotted accurately for the scale used


Four correct for two marks, two or three correct for one mark.
with axes numbered and labelled;

[3]

(ii) high air pressure sinking air so that pollutants are trapped in the lower atmosphere;
high pressure associated with low wind speeds / calm weather;
calm conditions increasing temperature with height stops air rising and dispersing;
pollutants not dispersed by winds;
steep sided hills pollutants are trapped in the basin between steep sided mountains;
less able to be dispersed by winds;
[3]
(iii) banning cars from city centres;
according to registration numbers;
compulsory fitting of catalytic converters on vehicle exhausts;
petrol and diesel replaced by cleaner fuels / or named (natural gas, CNG / CBG);
fitting diesel vehicles with particulate filters;
facilitating electric powered vehicles;
encouraging greater use of public transport / bikes;
laws on emissions from vehicles;
laws on emissions from industry / power stations;
relocating industrial areas to downwind side of city;
alternative fuels (geothermal, solar, wind, etc.);
sulfur scrubbing;
planting trees to filter particulates;
Max. three marks for a list. Must describe how it will improve air quality for
development marks.
[5]

Cambridge International Examinations 2014

Page 10

Mark Scheme
GCE O LEVEL May/June 2014

Syllabus
5014

(iv) difficulty of monitoring;


problems catching offenders / weak law enforcement;
cost implications;
people difficult to convince / citizens ignore;
not a priority;
inadequate legislation;
businesses put pressure on governments;
(c) (i) by remedying the design faults;
increasing safety measures;
with examples such as double skinned tanks, computerised monitoring, etc.;
by not allowing maintenance standards to decline over the years;
locating factory away from built-up areas;

Paper
12

[3]

[2]

(ii) similar chemical factory would be located 80 km away from the nearest settlement in the
USA;
in India factory surrounded by slums / places where many people live;
in India no zoning of land uses / no urban planning;
comment on attitude of the authorities / enforcement between developed and developing
countries;
comment about slums and their associated high densities of population;
poor people wanting and needing to live close to places of work;
better health care in USA;
better evacuation procedures in USA;
[3]
(d) (i) chronic health problems are still affecting lots of / at least 500 000 people;
examples of health problems such as cancers;
groundwater supplies remain contaminated;
large areas around the factory are cannot be used by people;
toxicity passed to offspring resulting in birth defects;
global toxic hot spot;
tonnes of toxic waster stored;
chemicals washed into water supplies;
which people have to use;
wage earners died / too ill to work;
so families in poverty / malnourished / etc.;
Two marks for description, two marks for explanation.

[4]

(ii) thousands of tonnes of waste still stored there;


long term nature of the groundwater contamination from persistent seepage;
poor slum dwellers often have nowhere else to go, and with such poverty and bad health
now, great improvement cannot be expected in the next 25 years;
lack of water piped in;
50 000 serious health problems will still exist;
birth defects still present in 25 years;
mercury has long term effect;
lack of government action;
[3]
little compensation / help from factory owners;

Cambridge International Examinations 2014

Page 11

Mark Scheme
GCE O LEVEL May/June 2014

Syllabus
5014

Paper
12

(e) keep people out / fence off;


landscaping;
draining contaminated water;
removing waste to dispose of safely elsewhere;
removing contaminated soil and treating it;
sealing contaminated areas with clay so water cannot take toxins into groundwater / streams;
government fines for illegal dumping;
top soil added;
acid / alkali added to soil to neutralise;
fertilisers added so it can be used for farming / recreation / forest;
mention of problems of restoration;
create nature reserves;
create land / lakes for recreational use;
Max. three marks if just brief points listed.

[6]
[Total: 120]

Cambridge International Examinations 2014

Вам также может понравиться